Está en la página 1de 115

SECCIÓN 11

GASTROENTEROLOGIA

Coordinadoras:
Hairé Manzano Cortés
Brenda Patricia Rendón Martínez
SECCIÓN 11
GASTROENTEROLOGÍA

Casos clínicos
Capítulo 11.1 Estructura del esófago, síntomas y anomalías
Capítulo 11.2 Trastornos motores del esófago
Capítulo 11.3 Otros trastornos esofágicos
Capítulo 11.4 Tumores esofágicos
Capítulo 11.5 Gastritis: aguda erosiva y crónica
Capítulo 11.6 Enfermedad ulcerosa péptica
Capítulo 11.7 Tumores gástricos
Capítulo 11.8 Síndrome diarreico
Capítulo 11.9 Malabsorción
Capítulo 11.10 Enfermedad inflamatoria intestinal
Capítulo 11.11 Síndrome del intestino irritable
Capítulo 11.12 Estudio del paciente con enfermedad
hepatobiliar
Capítulo 11.13 Hepatitis víricas
Capítulo 11.14 Fármacos e hígado
Capítulo 11.15 Hepatitis crónicas
Capítulo 11.16 Cirrosis
Capítulo 11.17 Complicaciones de la cirrosis
Capítulo 11.18 Colestasis crónicas
Capítulo 11.19 Enfermedades hepáticas de causas metabólica
y cardiaca
Capítulo 11.20 Abscesos hepáticos
Capítulo 11.21 Pancreatitis aguda
Capítulo 11.22 Pancreatitis crónica
Respuestas a casos clínicos
SECCIÓN 11 GASTROENTEROLOGrA @
CASOS CLÍNICOS
SECCIÓN 11. GASTROENTEROLOGÍA

► CASO CLÍNICO 1 FIGURA 1. MANOMETRÍA ESOFÁGICA CONVENCIONAL


DEL CASO CLÍNICO 4.
Masculino recién nacido, obtenido por vía abdominal G1 C1,
cuenta con el antecedente ultrasonográfico de polihidram­
nios. Dentro de sus primeros días de vida extrauterina, pre­
senta sialorrea, regurgitación e intolerancia a la vía oral. ¿Cuál
es la sospecha diagnóstica?
A. Atresia esofágica.
B. Fístula traqueoesofágica.
C. Estenosis esofágica.
D. Estenosis congénita del píloro.
E. Sólo Ay B.

► CASO CLÍNICO 2

Mujer de 45 años de edad, la cual acude a consulta por pre­


sentar disfagia a sólidos de 2 años de evolución. Ella modifi­
có su dieta pero no ha obtenido respuestay durante los últi­
mos meses también rechaza los líquidos, con baja ponderal tría esofágica convencional de cuatro canales, que reporta
de 13 kg. ¿Cuál sería el primer estudio a realizar? relajación incompleta del esfínter esofágico inferiory contrac­
A. Esofagograma. ciones esofágicas no peristálticas, como se observa en la fi­
B. Radiografía. gura 1. De acuerdo con los hallazgos manométricos, ¿cuál es
C. Endoscopia. la posibilidad diagnóstica?

1
D. Tomografía. A. Acalasia clásica.
B. Acalasia vigirosa.
► CASO CLÍNICO 3 C. Espasmo esofágico difuso.
D. Dismotilidad esofágica grave.
Hombre de 24 años de edad acude a la consulta externa de
Gastroenterología, procedente de su unidad de medicina fa­
► CASO CLÍNICO 5
miliar, por pérdida ponderal, disfagia progresiva de presenta­
ción retroesternal o subxifoidea de seis meses de evolución, A la consulta externa acude un hombre de 24 años de edad
con regurgitación ortostática y pirosis ocasional. Al realizar con antecedentes de tos crónica y múltiples infecciones de
un estudio baritado (serie esófago-gastroduodenal) se obser­ vías respiratorias altas. Es evaluado por otorrinolaringólogo,
va ensanchamiento mediastinal. ¿Cuál de las siguientes afir­ quien realiza una laringoscopiay encuentra eritema laríngeo
maciones es correcta? y edema de la epiglotis. El paciente fuma entre dos y cinco
A. El paciente padece enfermedad por reflujo gastroesofá­ cigarrillos al día desde hace un año. Refiere que los síntomas
gico complicada. se acentúan en las nochesy percibe pirosisy malestar torá­
B. El paciente tiene esófago de Barrett. cico. Su médico tratante le prescribe omeprazol 20 mg al día;
C. El cuadro clínico del paciente es compatible con trastor­ sin embargo, no ha obtenido mejoría de los síntomasy acu­
no motor del esófago. de a consulta para obtener una segunda opinión. Se le diag­
D. El paciente tiene estenosis maligna por cáncer esofágico. nostica una probable enfermedad por reflujo gastroesofági­
co con manifestaciones extraesofágicas.
¿Cuál de los siguientes estudios se solicitarían de prime­
► CASO CLÍNICO 4
ra intención para evaluar la enfermedad?
Mujer de 32 años de edad con historia de dolor torácico y A. Monitoreo con pHmetría de 24 horas con impedancia.
disfagia progresiva por lo general a líquidos. Se le realiza es­ B. Manometría esofágica de perfusión.
tudio endoscópico en donde sólo se observa dilatación esofá­ C. Serie esofagogastroduodenal.
gicay abundantes restos alimentarios. Se le realiza manome- D. Endoscopia alta.

681
e MANUAL PARA EL EXAMEN NACIONAL DE RESIDENCIAS MÉDICAS

► CASO CLÍNICO 6 A. Esofagitis por VHS.


B. Esofagitis por Candida.
Mujer de 40 años de edad, sin antecedentes heredofami­ C. Esofagitis por CMV.
liares de importancia, con diagnóstico de ERGE desde hace D. Estenosis péptica.
cuatro años, laringitis crónica y síntomas nocturnos relevan­
tes; se encuentra en tratamiento con omeprazol 40 mg cada
doce horas, requiere el uso de procinéticos y, como medidas ► CASO CLÍNICO 9
de rescate, consume antiácidos líquidos. Su endoscopia alta
es normal, por lo cual se le realizó un monitoreo ambulatorio Mujer de 23 años de edad ingresa a urgencias durante la no­
del pH intraesofágico que mostró reflujo patológico. Su estu­ che por ingestión de limpiador doméstico líquido a base de
dio de manometría esofágica muestra motilidad del cuerpo hidróxido de sodio como intento suicida. La paciente tiene
esofágico normal, con presión normal en reposo y relajación agitación psicomotora, disnea y taquicardia. De primera ins­
adecuada del esfínter esofágico inferior. ¿Cuál sería la reco­ tancia para su abordaje, ¿qué estudios solicitaría?
mendación que se le haría a la paciente?
A. Endoscopia alta con ultrasonido endoscópico.
A. Aumentar el tratamiento con omeprazol. B. Rx PA de tórax y endoscopia.
B. Medidas higiénico-dietéticas. C. Rx PA de tórax y TC toracoabdominal simple y contras­
C. Cirugía antirreflujo: funduplicatura de Nissen. tada.
D. Intensificar el tratamiento médico y las medidas higiénico­ D. Rx PA de tórax y esofagograma con medio hidrosoluble.
dietéticas.

► CASO CLÍNICO 10
► CASO CLÍNICO 7
Hombre de 3 años de edad ingiere de manera accidental lim­
Hombre de 49 años de edad con antecedentes de VIH+, piador de metales con ácido oxálico; ingresa a urgencias con
SIDA C2, CD4 150, con síndrome de consumo secundario, signos vitales dentro de parámetros y sistema cardiopulmo­
hace dos meses presentó disfagia, dolor retroesternal y ma­ nar sin compromiso. Muestra edema y eritema en cavidad oral
lestar general. Se realizó endoscopia alta y se encontraron y laringe. Rx tórax se reporta sin afección en campos pulmo­
lesiones blanquecinas, en placas y confluentes en toda la nares. Ingresa a sala de endoscopia para evaluar grado y ex­
circunferencia y el cuerpo esofágico, con borde eritematoso tensión de las lesiones. De las siguientes aseveraciones, ¿cuál
y sangrado fácil a la toma de biopsia, probable candidosis es incorrecta?
esofágica, corroborada por biopsias y cultivo. Se dio trata­
miento con fluconazol 200 mg como dosis inicial y 100 mg/ A. Los álcalis y los ácidos producen la misma lesión en el
día como dosis de mantenimiento por 28 días. Hay mejoría esófago.
de la disfagia; sin embargo, a la fecha persiste y continúa B. La lesión causada por ácidos produce coagulación y ne­
agravándose, ahora con dolor torácico. Se realiza nueva en­ crosis.
doscopia y se encuentran lesiones ulceradas, eritematosas C. La lesión causada por álcalis causa licuefacción y necro­
y alternadas con áreas de mucosa sana. es transferido para sis con saponificación.
tratamiento. ¿Cuál es el diagnóstico más probable? D. La lesión causada por cáusticos depende de si son álca­
lis o ácidos.
A. Esofagitis por Candida.
B. Esofagitis por complejo avium intrace/lulare (CAi).
C. Esofagitis por citomegalovirus (CMV). ► CASO CLÍNICO 11
D. Esofagitis herpética.
Hombre de 67 años de edad, con antecedente de disfagia y
reflujo de 3 meses de evolución, halitosis y en ocasiones sen­
► CASO CLÍNICO 8 sación de masa en hemicuello izquierdo. Niega pérdida de
peso, hemorragia digestiva, tabaquismo y datos de reflujo.
Mujer de 78 años de edad con artritis reumatoide activa, en 1. El primer estudio a realizar en un paciente con disfagia
tratamiento con prednisona y metotrexato por persistencia alta es:
de flogosis y actividad de la AR; en fechas recientes se ha
introducido infliximab. Desde hace una semana presenta do­ A. Endoscopia digestiva alta.
lor retroesternal, malestar general, odinofagia y disfagia a só­ B. Videofluoroscopia con bario.
lidos. En la exploración física se detecta cavidad oral sana, C. Tomografía computarizada.
eritema y placas blanquecinas en orofaringe. ¿Cuál sería el D. Manometría esofágica.
diagnóstico más probable? E. pHmetría con impedancia intraluminal.

682
SECCIÓN 11 GASTROENTEROLOGÍA @
FIGURA 2. VISIÓN ANTEROPOSTERIOR DE UN FIGURA 3. ENDOSCOPIA DIGESTIVA ALTA DEL CASO
ESOFAGOGRAMA CON BARIO QUE MUESTRA UN CLÍNICO 13.
PEQUEÑO DIVERTÍCULO DE ZENKER (FLECHA) EN LA CARA
LATERAL DE LA FARINGE POSTERIOR (CASO CLÍNICO 11).

2. Dentro del protocolo de estudio se realiza un esofago­


grama con bario y se observa una evaginación de la pared de
aspecto sacular a nivel del tercio proximal del esófago (figu­
Se propone como candidato a cirugía antirreflujo. Se decide
ra 11-3-1). La imagen radiológica es compatible con el siguien­
realizar esofagogastroduodenoscopia, en la que se identi­
te diagnóstico:
fica en la maniobra de retroflexión la imagen de la figura 3.
A. Carcinoma epidermoide de esófago. La condición que se reporta es una hernia deslizante tipo 1.

1
B. Divertículo epifrénico. ¿Cuál es el mecanismo de la formación de este tipo de her­
C. Divertículo faringoesofágico o de Zenker. nias?
D. Hernia hiatal tipo 1 o por deslizamiento.
A. Deslizamiento de la parte alta del estómago al mediasti­
no por el hiato esofágico.
► CASO CLÍNICO 12 B. Deslizamiento del estómago a través del orificio de Boch­
dalek.
Hombre de 45 años de edad originario de Nicaragua, cuenta C. Deslizamiento del cuerpo gástrico a través del orificio de
con antecedente de diabetes mellitus tipo 2 y tuberculosis Morgagni.
pulmonar, presenta disfagia progresiva y regurgitación des­ D. Deslizamiento de antro gástrico por el hiato esofágico.
de hace tres meses. Dentro del protocolo de estudio se rea­
liza esofagograma con bario y se identifica divertículo en ter­
► CASO CLÍNICO 14
cio medio esofágico. El diagnóstico del paciente sería:

A. Divertículo por pulsión o de Zenker. Mujer de 29 años de edad, sin antecedentes personales pa­
B. Divertículo epifrénico. tológicos, cursa con un embarazo de 10 semanas de ges­
C. Divertículo de Meckel. tación. Desde hace una semana presenta vómito persistente
D. Divertículo por tracción o epibrónquico. y en el último día cinco vómitos de contenido gastroalimen­
E. Sarcoidosis. tario. Refiere un episodio de hematemesis de alrededor de
250 ce, sin inestabilidad hemodinámica. Niega episodios pre­
vios y uso de AINE. Después del manejo médico inicial se
► CASO CLÍNICO 13 indica estudio endoscópico. El diagnóstico más probable en
el contexto de la paciente es:
Hombre de 28 años de edad, con antecedente de tabaquis­
mo y alcoholismo ocasional; enfermedad por reflujo gastro­ A. Gastropatía erosiva por AINE.
esofágico de tres años de evolución, manejada con omepra­ B. Hemorragia variceal.
zol 20 mg 15 minutos antes del desayuno y cena desde hace C. Úlcera duodenal.
seis meses, con mejoría de los síntomas de reflujo en 80%. D. Desgarro de Mallory-Weiss.

683
e MANUAL PARA EL EXAMEN NACIONAL DE RESIDENCIAS MÉDICAS

► CASO CLÍNICO 15 A. Anillo de Schatzki.


B. Síndrome de Plummer-Vinson.
Hombre de 40 años de edad, sin antecedentes de interés, C. Membranas esofágicas.
consulta por cuadro brusco de dolor epigástrico irradiado a D. Esofagitis eosinofílica.
región retroesternal, acompañado de sudoración y vómito de
contenido biliar y hemático. En la exploración física se detec­
► CASO CLÍNICO 18
ta que el paciente está agitado, sudoroso y hemodinámica­
mente estable. Se palpa crepitación en cuello y disminución Mujer de 75 años de edad que después de la ingesta de un
del murmullo vesicular en bases pulmonares. La explora­ trozo de carne inicia con disfagia, dolor torácico y sialorrea.
ción abdominal muestra defensa y ausencia de peristaltismo. Acude a urgencias, donde se le encuentra con los siguien­
1. Una vez tomado en consideración el ABC del paciente, tes signos vitales: tensión arterial: 120/70; frecuencia cardia­
¿cuál estudio sería de utilidad en el protocolo diagnóstico? ca: 85 por minuto; frecuencia respiratoria: 24 por minuto, sin
A. Ultrasonido abdominal. datos de dificultad respiratoria pero persiste con disfagia. Por
B. Endoscopia digestiva alta. este motivo se le realiza un estudio endoscópico, donde se
C. Radiografías de tórax y abdomen. encuentra bolo alimenticio impactado en tercio inferior del
D. Esofagograma con bario. esófago. Se realiza la extracción del alimento impactado. ¿qué
porcentaje de los cuerpos extraños digestivos transitan de
2. Ante la presencia de la tríada de Mackler (vómito, dolor forma espontánea sin producir daño?
torácico y enfisema) que presenta el paciente, ¿cuál sería el
diagnóstico? A. Menos de 10%.
B. 40%.
A. Desgarro de Mallory-Weiss.
B. Úlcera péptica perforada.
c. 60%.
D. Entre 80 y 90%.
C. Vólvulo gástrico.
D. Hernia hiatal complicada.
E. Síndrome de Boerhaave. ► CASO CLÍNICO 19

Mujer de 44 años de edad con antecedente de ingesta de


► CASO CLÍNICO 16
cáusticos en la infancia y aparente tratamiento con dilata­
Mujer de 67 años de edad, hipertensa, diabética y usuaria ción esofágica, sin ninguna complicación posterior, inicia su
de aspirina en dosis anticoagulante, acude al servicio de ur­ cuadro aproximadamente de dos meses con disfagia pro­
gencias por un cuadro de inicio súbito de dolor retroesternal gresiva a sólidos y en la actualidad a líquidos; baja ponderal
acompañado de disfagia, con un episodio posterior de hema­ de 15 kg.
temesis escasa. Electrocardiograma reportado como normal, 1. Ante el cuadro clínico, ¿cuál es el método diagnóstico
radiografía de tórax sin evidencia de neumomediastino. El más adecuado?
cuadro clínico, al ser súbito, es compatible con el siguiente A. Esofagograma.
diagnóstico: B. Endoscopia.
A. Infarto agudo del miocardio. C. TC.
B. Disección aórtica. D. Ultrasonido endoscópico.
C. Estenosis péptica. 2. De acuerdo con el antecedente, ¿cuál sería la sospe­
D. Hematoma intramural esofágico. cha diagnóstica?
A. Adenocarcinoma.
► CASO CLÍNICO 17
B. Leiomioma.
Hombre de 65 años de edad, con antecedente de dermati­ C. Carcinoma de células escamosas.
tis atópica. Inicia su padecimiento hace tres meses con dis­ D. Lipoma.
fagia a sólidos. Recibió tratamiento con inhibidor de bomba
de protones durante ocho semanas sin mejoría, niega da­ ► CASO CLÍNICO 20
tos de hemorragia gastrointestinal. Registra hemoglobina de
13 g/dl. Ante la persistencia de los síntomas se le realiza eso­ Mujer de 65 años de edad con náusea y vómito recurrentes
fagograma, sin encontrar alteraciones relevantes, por lo cual desde hace tres meses, acompañados de epigastralgia uren­
se le realiza panendoscopia y se encuentra un esófago ani­ te que no se alivia con el vómito. Ha presentado pérdida de
llado en tercio medio y distal. ¿Cuál es su diagnóstico más peso de 6 kg en los últimos dos meses. En la exploración fí­
probable? sica sólo destacan hipotrofia muscular generalizada y abdo-

684
SECCIÓN 11 GASTROENTEROLOGÍA @
men sin masas ni megalias. Los análisis de laboratorio repor­ ► CASO CLÍNICO 23
tan Hb de 13.2 g/dL, hematocrito de 39%, plaquetas 250 000
y leucocitos 4 800. Sangre oculta en heces negativa. La en­ Mujer de 65 años de edad con antecedentes de diabetes me­
doscopia con toma de biopsias mostró datos de inflamación llitus tipo 2 desde hace 13 años, en tratamiento con hipoglu­
aguda con presencia de microorganismos compatibles con cemiantes orales, tiene obesidad Grado 1, presenta ingesta
H. py/ori. Se descartó neoplasia gástrica. ¿Cuál de las siguien­ de diclofenaco 100 mg cada 12 horas desde hace tres sema­
tes afirmaciones es verdadera? nas por gonartralgias, al momento sin protocolo de estudio.
Inicia con epigastralgia urente de tres días de evolución, y
A. Debe repetirse endoscopia por la baja ponderal.
cuatro días antes de su revisión médica presenta hemate­
B. Revisión de biopsias por segundo patólogo.
mesis con pérdida del estado de conciencia. A su llegada al
C. Los hallazgos de endoscopia no correlacionan con los ha­
servicio de urgencias tiene presión arterial de 90/60, frecuen­
llazgos clínicos, lo cual puede ser normal.
cia cardiaca de 120 1pm, frecuencia respiratoria 20 rpm, tem­
D. Repetir biometría hemática.
peratura 36.5 ºC, glucemia capilar de 80 mg/dL, desorien­
tada en tiempo, orientada en persona y lugar, con mucosas
► CASO CLÍNICO 21 pálidas, regular estado de hidratación, cardiorrespiratorio sin
compromiso, abdomen con dolor a la palpación en cuadran­
Mujer de 56 años de edad con epigastralgia de larga evolu­
tes superiores, sin rebote, miembros normales. Los análisis ini­
ción, con vómito precedido por náusea de manera intermi­
ciales revelaron hemoglobina de 8 g/dL, hematocrito de 30%,
tente, palpitaciones, astenia y adinamia progresivas, dificul­
con plaquetas 240 000 y 13 200 leucocitos totales. ¿qué ries­
tad para deglutir alimentos y dolor en la boca. Se realizaron
go de morbimortalidad se daría a esta paciente y cómo in­
análisis generales de laboratorio previo a su consulta, que
fluiría en la decisión terapéutica?
revelaron hemoglobina de 7.6 g/dL, hematocrito de 22%,
plaquetas 140 000 y leucocitos de 4 500 con macrocitosis. A. Riesgo bajo, manejo sintomático.
Por la sintomatología y los hallazgos de laboratorio se sos­ B. Riesgo alto sin necesidad de endoscopia urgente (en las
pecha: primeras 24 horas).
C. Riesgo alto con necesidad de endoscopia urgente.
A. Gastritis linfocítica.
D. Riesgo intermedio, sólo dar tratamiento médico y normar
B. Gastritis por enfermedad de Crohn.
según conducta.
C. Gastritis de tipo autoinmunitario con anemia perniciosa
asociada.

1
D. Déficit de niacina. ► CASO CLÍNICO 24
E. Anemia por deficiencia de hierro. Hombre de 40 años de edad acude a facultativo por presen­
tar dolor abdominal en epigastrio de tipo urente, de tres me­
► CASO CLÍNICO 22 ses de evolución, que mejora de forma parcial con ingesta
de omeprazol; además refiere saciedad temprana y niega
Hombre de 38 años de edad, con antecedente de tabaquis­ pérdida de peso. Se realizan estudios de laboratorio con BH
mo positivo desde los 16 años, cuatro cigarrillos al día, y al­ normal y sangre oculta en heces negativo. El médico decide
coholismo los fines de semana, sin llegar a la embriaguez. realizar estudio endoscópico y se reporta el siguiente hallaz­
Inicia con epigastralgia urente de un mes de evolución, des­ go: lesión localizada en antro gástrico, de aspecto polipoide
encadenada con la ingesta de alimentos ácidos y colecistoqui­ de alrededor de 20 mm, con pedículo corto y único. Se toman
néticos. A las dos semanas de iniciado su cuadro, el paciente biopsias. Por el hallazgo reportado el diagnóstico correcto
presenta citofobia y evacuaciones melénicas sin datos de puede ser:
compromiso hemodinámico. El dolor continúa presente, es
intenso e interrumpe el descanso nocturno. En la exploración A. Adenocarcinoma gástrico l.
física el paciente está consciente, orientado, cardiorrespira­ B. Gastritis antral foveolar.
torio sin compromiso, abdomen globoso por panículo adipo­ C. Tumor tipo MALT.
so, con dolor a la palpación media y profunda en epigastrio, D. Pólipo gástrico antral.
peristalsis presente normal. Los análisis de laboratorio reve­ E. Lipoma gástrico antral.
lan hemoglobina de 10.4 g/dl. ¿Cuál estudio es obligatorio
realizar en el paciente dados los hallazgos clínicos y de la­ ► CASO CLÍNICO 25
boratorio? Mujer de 67 años de edad que tiene antecedente de taba­
A. Ultrasonido abdominal. quismo de larga evolución, acude a valoración médica por
B. Endoscopia digestiva alta. presentar episodios de evacuaciones melénicas en tres oca­
C. Serie esofagogastroduodenal. siones en los últimos dos meses. Además tiene pérdida de
D. Sangre oculta en heces. peso no cuantificada. Estudios de laboratorio con Hb 10 g/dL.

685
e MANUAL PARA EL EXAMEN NACIONAL DE RESIDENCIAS MÉDICAS

Se realiza endoscopia con siguiente hallazgo: en curvatura das de consistencia, fétidas, espumosas, con presencia de
menor se observa una úlcera mal delimitada de alrededor de restos de alimentos y grasa. Al interrogatorio dirigido refiere
40 mm, bordes irregulares, eritema, cubierta de fibrina y res­ pérdida de 8 kilogramos de peso en los últimos seis meses.
tos necróticos, friable a toma de biopsias. El diagnóstico más Biometría hemática: Hb: 10.6 g/dL, leucocitos 4 300 mm3 ,
probable, por la descripción endoscópica, es: plaquetas 257 000 mm3 • Coprológico y coproparasitoscópi­
co negativos. Panendoscopia: duodenitis crónica con aplana­
A. Úlcera gástrica benigna Sakita A1.
miento de válvulas conniventes. Biopsia de duodeno: atrofia
B. Gastritis hemorrágica.
subtotal de las vellosidades. Carotenos: 37 mg/dL, o-xilosa:
C. Úlcera gástrica crónica.
2.4 gramos/5 horas. ¿Cuál de las siguientes afirmaciones es
D. Adenocarcinoma gástrico
verdadera?

► CASO CLÍNICO 26 A. La distensión abdominal es la manifestación clínica pivo­


te para el diagnóstico de malabsorción.
Hombre de 72 años de edad presenta cuadro de dos días B. La anemia se debe a malabsorción de vitamina 8 12 por
de evolución caracterizado por dolor abdominal, evacuacio­ atrofia subtotal de las vellosidades duodenales.
nes diarreicas líquidas con moco, sin sangre, en número de C. Los niveles séricos de carotenos son el estándar de oro
10 a 12 diarias, escasas. Presenta pujo y tenesmo leves. En la para el diagnóstico de malabsorción.
exploración física se encuentra con temperatura de 38.2 ºC D. La determinación de o-xilosa ayuda a diferenciar entre
y dolor en cuadrante inferior izquierdo, sin datos de irrita­ una alteración de la pared o de la luz intestinal.
ción peritoneal. Su biometría hemática muestra 15 200 leu­
cocitos, 82% neutrófilos y 6 neutrófilos en banda. De acuer­ ► CASO CLÍNICO 29
do con los síntomas que presenta este paciente, ¿cuál es el
mecanismo fisiopatológico relacionado con la diarrea? Mujer de 54 años de edad, diabética con seis años de evo­
lución y en tratamiento con hipoglucemiante oral (glibencla­
A. Diarrea osmótica. mida), con adecuado control metabólico, refiere iniciar hace
B. Diarrea facticia. tres meses con evacuaciones disminuidas de consistencia,
C. Diarrea inflamatoria. líquidas, explosivas, con presencia de restos de alimento y
D. Diarrea malabsortiva. grasa. Desde hace un mes presenta erupciones cutáneas
E. Diarrea por tránsito intestinal acelerado. pruriginosas en manos, hombros y espalda, por lo que acude
al médico especialista quien diagnostica dermatitis herpeti­
► CASO CLÍNICO 27 forme. El tránsito intestinal es acelerado con espiculaciones,
moldeamiento y floculación. Coprológico y coproparasitos­
Hombre de 48 años de edad, marino mercante, acude por cópico negativos. Se realiza panendoscopia para aspirado
presentar evacuaciones disminuidas en consistencia en nú­ duodenal y resulta negativo a parásitos y a crecimiento micro­
mero de seis a siete, abundantes, color verdeamarillento, sin biano, así como biopsias de duodeno con atrofia parcial de
moco, sin sangre, desde hace ocho semanas. Además tiene las vellosidades, con hiperplasia críptica y linfocitos intraepi­
distensión abdominal y borborigmos. Las evacuaciones ce­ teliales. El anticuerpo antitransglutaminasa tisular es positi­
den con el ayuno. Refiere pérdida de peso de 6 kg. En la vo. ¿Cuál de los siguientes es el diagnóstico más probable?
exploración física llama la atención la distensión abdominal
grave. No se palpan tumores ni hepatoesplenomegalia. Sus A. Enfermedad de Whipple.
exámenes de laboratorio demuestran Hb 11.1 g/dL (normal 13 B. Enfermedad de Crohn.
a 15 g/dL), VGM 102 fL (normal hasta 95 fL), ácido fólico inde­ C. Enfermedad celiaca.
tectable. ¿A cuál patología orientan los datos clínicos, la ex­ D. Sobrecrecimiento bacteriano.
ploración física y los exámenes de laboratorio?
► CASO CLÍNICO 30
A. Enfermedad celiaca.
B. Esprúe tropical. Hombre de 29 años de edad, cursa con cuadro de un año
C. Sobrepoblación bacteriana. de evolución con evacuaciones disminuidas de consistencia
D. Colitis ulcerativa crónica inespecífica. acompañadas de moco y sangre con duración de tres a cua­
E. Enfermedad de Crohn. tro días; modificó dieta sin mejoría y en ocasiones aumenta
la frecuencia de evacuaciones hasta cuatro a seis al día y
► CASO CLÍNICO 28 requiere antidiarreicos. Ha tenido tratamientos con antipa­
rasitarios y antimicrobianos, con mejoría parcial de los sínto­
Mujer de 65 años de edad refiere que hace cuatro meses mas. Ahora las evacuaciones se han incrementado a seis a
inició con distensión abdominal de predominio posprandial ocho en 24 horas, con hemorragia evidente leve y dolor tipo
tardío, acompañada de flatulencias y evacuaciones disminuí- cólico localizado en hipogastrio. En la evaluación clínica pre-

686
SECCIÓN 11 GASTROENTEROLOGÍA @
senta TA 100/70 mm Hg, FC 90 1pm, temperatura 37 ºC, sin 1. ¿Cuál es el diagnóstico más probable?
datos de irritación peritoneal, con lesiones eritematosas in­
A. Enfermedad de Crohn.
duradas y dolorosas en miembros pélvicos.
B. Enfermedad celiaca.
1. El diagnóstico probable es:
C. Esprúe tropical.
A. Enfermedad hemorroidal. D. Giardiosis.
B. Colitis amebiana.
2. Ante la sospecha diagnóstica, ¿cuál es el estudio que
C. Colitis ulcerativa crónica inespecífica.
se debe realizar?
D. Síndrome de intestino irritable con predomino de diarrea.
A. Tránsito intestinal.
2. Al paciente se le realizan análisis de laboratorio que re­
B. Colonoscopia con ileoscopia.
portan PCR 45, VSG 33, hemoglobina 11.0 g/dL, Hto 32%, pla­
C. Enteroscopia.
quetas 600 000; con estos hallazgos, ¿qué estudio se debe
D. Cápsula endoscópica.
realizar?
3. Se realizó endoscopia �I paciente mencionado y se
A. Ultrasonido.
detectaron úlceras profundas serpiginosas en ciego y par­
B. Colon por enema.
te de colon ascendente, así como úlceras aftoides en íleon
C. Colonoscopia con ileoscopia.
terminal. Se maneja con prednisona oral y mesalazina de ma­
D. TC.
nera ambulatoria. Hay disminución de evacuaciones, pero
3. Al paciente se le realizó una colonoscopia, la cual re­ reingresa a las dos semanas con dolor intenso a la evacua­
portó úlceras, algunas profundas, y sangrado espontáneo ción y continuo en glúteo derecho, donde se encuentra abs­
desde el recto hasta el colon descendente; según los hallaz­ ceso de alrededor de 6 cm. ¿Cuál es el tratamiento indicado
gos clínicos y de laboratorio, ¿cuál es la conducta a seguir? en abscesos?
A. Mesalazina en enemas y manejo ambulatorio. A. Drenaje quirúrgico y antibióticos.
B. Esteroides vía oral y mesalazina en supositorios con ma­ B. Terapia biológica.
nejo ambulatorio. C. Azatioprina.
C. Hospitalización y se inicia con esteroides y mesalazina o D. Se incrementa la dosis de esteroides.
terapia biológica.
4. De acuerdo con el caso clínico el paciente tiene activi­
D. Cirugía.
dad grave con múltiples factores de mal pronóstico. ¿Cuál se­
4. De las siguientes aseveraciones, ¿cuál es la verdadera ría el tratamiento de elección para este paciente?

1
para este caso?
A. Tratamiento convencional con esteroides e inmunomodu-
A. Los análogos de las purinas es el tratamiento de elección ladores.
para inducir la remisión. B. Resección quirúrgica ileocolónica.
B. Los esteroides son el tratamiento de elección para el C. Terapia biológica.
mantenimiento de la remisión. D. Antibióticos y mesalazina.
C. El metotrexato es de elección en pacientes con CUCI
grave.
► CASO CLÍNICO 32
D. La terapia biológica se utiliza para inducir y mantener la
remisión.
Acude a consulta externa una mujer de 41 años de edad con
molestias gastrointestinales y diarrea de 10 meses de dura­
► CASO CLÍNICO 31 ción. Niega pérdida de peso u otros síntomas constitucio­
nales. No ha visto sangre en las heces. No ha recibido trata­
Hombre de 29 años de edad tiene antecedente de absceso
miento previo y ahora el problema es intermitente, pues se
en glúteo derecho hace cuatro años y requirió drenaje qui­
alternan periodos de estreñimiento con periodos de diarrea.
rúrgico. T iene tres años con diagnóstico de colon irritable
¿Cuál de los siguientes es el diagnóstico más probable?
por dolor abdominal y toma antiespasmódicos, cuadro de un
año con episodios intermitentes de diarrea fétida, en ocasio­ A. Parasitosis intestinal.
nes abundante con lientería, tratado con metronidazol, qui­ B. Colitis microscópica.
nolonas y antiparasitarios, baja ponderal de 10 kg y fatiga C. Intolerancia a carbohidratos.
leve. Desde hace un mes observa grasa en evacuaciones, tie­ D. Síndrome de intestino irritable.
ne distensión abdominal, el dolor no cede con el tratamiento
establecido y ha tenido incremento de temperatura que ha ► CASO CLÍNICO 33
requerido paracetamol. Los análisis de laboratorio reportan
hemoglobina 10.0 g/dL, VSG 35, PCR 40, coprológico con es­ Mujer de 62 años de edad con dolor abdominal que se en­
teatorrea, creatorrea y sangre oculta en heces positiva. cuentra presente la mayoría de los días de la semana durante

687
e MANUAL PARA EL EXAMEN NACIONAL DE RESIDENCIAS MÉDICAS

los últimos ocho meses. El dolor varía en localización e in­ ► CASO CLÍNICO 35
tensidad. Identifica al estrés y la ingesta de alimentos como
Se evalúa a un adolescente masculino que es traído por sus
factores agravantes del dolor y presenta mejoría con la defe­
padres porque "siempre lo ven pálido" desde que entró a la
cación. Seis meses previos el dolor se asociaba con disten­
preparatoria. Al interrogar extensamente no hay anteceden­
sión y flatulencias, pero en las últimas semanas ha presentado
tes patológicos de relevancia. Además tiene un adecuado
cuadros de diarrea alternada con estreñimiento. ¿qué inter­
desempeño escolar y un desarrollo físico y psicomotor nor­
vención es apropiada en este momento?
mal. Refiere que sus hábitos han cambiado desde que entró
A. Iniciar con antiespasmódicos. a la preparatoria y en ocasiones no completa las tres comidas
B. Iniciar con antidepresivos. al día, obviamente ha disminuido sus horas de sueño y con­
C. Colonoscopia. sidera que ha sido un tanto estresante para él mantener un
D. Iniciar con antibióticos. buen promedio; refiere que tiene cefaleas cada tercer día que
ceden con una tableta de paracetamol. En la exploración lla­
ma la atención un muy discreto tinte ictérico en escleras, re­
► CASO CLÍNICO 34 fiere que hoy no ha desayunado y el resto de la exploración
es normal. Sus padres presentan las siguientes pruebas de
Acude a consulta una mujer de 24 años de edad que refiere
laboratorio de un check-up: Hb 14.5 g/dL, Hto 43%, leucoci­
tener diagnóstico de lupus eritematoso sistémico. Comenta
tos 5 000, PLT 240 000, Cr 0.5 mg/dL, nitrógeno ureico 23
que desde la semana pasada tiene debilidad franca genera­
mg/dL, 8T 3.5 mg/dL, 8D 1 mg/dL, 81 2.5 mg/dL, AST 10 U/L,
lizada, imposibilidad para llevar a cabo ejercicio físico leve,
ALT 10 UI/L, FA 30 U/L, DHL 180 UI/L y GGT 8 UI/L (todas las
ha tenido exantema facial y artritis de las manos. En la explo­
pruebas de función hepática son normales excepto las bili­
ración se mira una palidez generalizada grave en piel y mu­
rrubinas).
cosas, los signos vitales con FC de 110 1pm persistente y so­
1. ¿Cuál es el diagnóstico más probable?
plo holosistólico leve. Su estado de hidratación es normal,
no hay fiebre o edema. Exámenes de laboratorio: Hb: 6.5 g/ A. Hepatitis viral de tipo C.
dL, Hto: 20%, plaquetas 150 000, bilirrubinas totales de 4.5 B. Hepatitis tóxica por medicamentos.
mg/dL, bilirrubina directa de 1.5 mg/dL, bilirrubina indirecta C. Trastorno de la conjugación biliar.
de 3 mg/dL, DHL 2 000 U/L (normal de 115 a 221 U/L), FA de D. Hemólisis de causa autoinmunitaria.
77 U/L (normal de 33 a 96 U/L). Desde hace una semana ha 2. ¿Dónde se encuentra la falla que provoca esta enfer­
incrementado la dosis de prednisona de 5 mg VO al día a 10 medad?
mg VO al día.
A. Falla en la glucuronización hepática.
1. ¿Cuál es la razón más probable de las alteraciones en
B. Destrucción del hepatocito de grado leve.
Hb y DHL de esta paciente?
C. Falla en la biliverdina reductasa.
A. Hemólisis secundaria a prednisona. D. Destrucción periférica de eritrocitos.
B. Hemólisis autoinmunitaria.
C. Hepatitis autoinmunitaria. ► CASO CLÍNICO 36
D. Fragmentación mecánica de eritrocitos.
Hombre de 17 años de edad, residente del estado de More­
2. ¿Cuántas veces puede elevarse la DHL en los eventos los, consume alimentos en la calle. inicia su padecimiento con
de hemólisis? anorexia, fatiga, fiebre de 38 ºC, diarrea, artralgias, dolor ab­
dominal y vómito; a la semana se asocia ictericia ascendente,
A. Dos veces. coluria, acolia y prurito. En la exploración física se encuentra
B. Diez veces. con ictericia de mucosas y tegumentos++ . Urticaria disemi­
C. Veinte veces. nada; hepatomegalia dolorosa. Exámenes de laboratorio: ALT
D. Cien veces. 1 568 UI/L, AST 1 215 UI/L, FA 96 UI/L, 8T 14.2 mg/dL, 8D 8.7
3. ¿Cuál de los siguientes enunciados es correcto? mg/dL y 81 5.5 mg/dL. ¿Cuál es el diagnóstico más probable?
A. Hepatitis por virus A.
A. Disminuirá la bilirrubina en orina porque la bilirrubina di­
B. Hepatitis por virus 8.
recta es hidrosoluble. C. Hepatitis por virus D.
B. Aumentará la bilirrubina en orina porque la bilirrubina in­ D. Hepatitis por virus E.
directa directa es liposoluble.
C. La bilirrubina conjugada se encuentra unida fuertemente
► CASO CLÍNICO 37
a albúmina.
D. La glucuronización es el paso en que la bilirrubina se Mujer de 47 años de edad con antecedente de aborto a los
vuelve directa. 23 años _de edad. Requirió transfusión de tres unidades de

688
SECCIÓN 11 GASTROENTEROLOGÍA @
concentrados eritrocitarios. Tiene una pareja sexual, niega ► CASO CLÍNICO 40
alcoholismo, tabaquismo, tatuajes y acupuntura. Niega ante­
cedentes de hepatitis o cuadros de ictericia. Por requerir de Mujer de 19 años de edad, sin antecedentes de importancia,
colecistectomía, en estudios preoperatorios se encuentra inicia con cuadro de ictericia, motivo de la consulta. En la bio­
FA 98 UI/L, ALT 135 UI/L, AST 76 UI/L, GGT 89 UI/L y BT 0.98 química presenta daño hepático de predominio hepatocelu­
mg/dl. Por sus antecedentes: lar con BT 4 mg/dl, BD 3.1 mg/dl, AST 650 UI/L, ALT 211 UI/L,
1. ¿Cuál sería la hepatitis viral más probable? GGT 460 UI/L, FA 330 UI/L; gammaglobulina 1.8 g/dl e lgG
1 700 mg/dl, anticuerpos antinucleares 1/80. El ultrasonido no
A. A.
muestra alteraciones de relevancia. Se realiza biopsia hepá­
B. B.
tica percutánea.
e.e. 1. ¿Qué hallazgos se espera encontrar según la conside­
D. D.
ración diagnóstica?
2. ¿Qué estudios se solicitan para dirigir el diagnóstico
A. Hepatitis de interfase e infiltrado leucocitario agudo.
diferencial?
B. Esteatosis de gota gruesa.
A. lgG anti-VHA. C. Cobre en hepatocitos.
B. lgG anti-VHE. D. Granulomas.
C. Agc VHB. E. Imagen en capas de cebolla.
D. Anti-VHC. 2. Se decide iniciar corticoterapia. ¿Por cuánto tiempo se
E. Anti-VHD. debe manejar la prednisona?
A. Seis meses y revalorar.
► CASO CLÍNICO 38 B. Un mes y posterior reducción gradual.
Mujer en la sexta década de la vida, con antecedentes de C. Hasta la remisión de la enfermedad.
anorexia, depresión y abuso de ingesta de alcohol más va­ D. De por vida.
rios intentos suicidas. Se presenta en unidad de urgencias E. No está indicada la terapia con esteroides.
con náusea, vómito y dolor en hipocondrio derecho. En la
exploración física se encuentra con signos vitales normales ► CASO CLÍNICO 41
e hipersensibilidad en hipocondrio derecho.
Hombre de 38 años de edad, con antecedentes de tabaquis­
1. ¿De qué medicamento es más probable que la pacien­

1
mo y etilismo, acude por cuadro de inicio súbito con desorien­
te haya abusado?
tación, fiebre no cuantificada, ictericia y aumento de volumen
A. Halotano. abdominal. Bioquímica mente con leucocitosis de 18 000, BT
B. lsoniazida. 12 mg/dl, ALT 80 UI/L, AST 310 UI/L, GGT 110 UI/L, FA 480
C. Metotrexato. UI/L, TP 80%, INR 1.1, panel viral negativo y ultrasonido repor­
D. Paracetamol. ta hepatomegalia. Se considera hepatitis alcohólica de alto
2. ¿Qué exámenes paraclínicos son de utilidad en el riesgo.
caso anterior? 1. ¿Cuál es el manejo inicial más adecuado?
A. Pruebas de funcionamiento hepático y pruebas de coa- A. Pentoxifilina 400 mg cada ocho horas por un mes.
gulación. B. Prednisona 40 mg al día + pentoxifilina por un mes.
B. Biometría hemática. C. Prednisona 40 mg al día por una semana y revaloración.
C. Examen general de orina. D. Trasplante hepático temprano.
D. Perfil de lípidos. E. Antibioticoterapia y medidas anti-amonio.
2. ¿En qué escenarios se justifica el uso de pentoxifilina
► CASO CLÍNICO 39 en la HA grave?
A. En caso de no respuesta a esteroides.
Hombre de 48 años de edad, con antecedente de colecistec­
B. No está indicado el uso de pentoxifilina en estos casos.
tomía hace 10 días. Durante la cirugía se utilizó halotano, ci­
C. En sepsis, hemorragia digestiva o síndrome hepatorrenal.
profloxacino y metamizol. Se presenta a la consulta con ele­
D. En caso de persistir las toxicomanías.
vación de transaminasas >5 veces el valor normal e ictericia.
E. En caso de encefalopatía hepática.
¿Cuál podría ser la causa de su ictericia?

A. Hepatotoxicidad por halotano. ► CASO CLÍNICO 42


B. Uso de cefalosporina.
C. Uso de quinolona. Mujer de 51 años de edad acude a la consulta porque desde
D. Lesión de vía biliar. hace tres meses presenta petequias y hematomas que se

689
e MANUAL PARA EL EXAMEN NACIONAL DE RESIDENCIAS MÉDICAS

desarrollan con facilidad, sin traumatismos. Desde hace un de tórax, aumento del perímetro abdominal, con signo de la
mes se agrega epistaxis ocasional, sin ninguna causa justi­ ola positivo pero blando, sin tensión, sin datos de irritación
ficada, por lo que acudió a revisión con su médico familiar, peritoneal y con edema leve de extremidades. Exámenes de
quien encontró trombocitopenia de 41 000 en análisis de la­ laboratorio con biometría hemática, con leucocitos de 4 500,
boratorio. La paciente se refiere asintomática, pero al reali­ hemoglobina de 11.5 g/dL, plaquetas de 110 000. PFH con al­
zar interrogatorio dirigido presenta astenia y adinamia que se búmina de 3.2 g/dL, BT de 2.0 mg/dL, con BD de 1.8 mg/dL,
han exacerbado desde hace un año; niega otra sintomatolo­ ALT 66 UI/L, AST 82 UI/L, FA 50 UI/L, DHL 220 UI/L, TP 14 s,
gía. Dentro de sus antecedentes de importancia sólo desta­ con INR 1.4.
ca diagnóstico de Diabetes Mellitus tipo 2 desde hace tres 1. ¿Cuál es la causa más probable de cirrosis hepática de
años, en tratamiento irregular con metformina y glibenclami­ este paciente?
da sin lograr un control adecuado; no lleva dieta y es obesa A. Esteatohepatitis.
desde los 28 años, con un peso actual de 120 kg y talla de B. Etilismo crónico.
1.56 m. En la exploración física se detectan petequias en ex­ C.Obesidad.
tremidades inferiores, obesidad central y resto de explora­ D.Hepatitis autoinmunitaria.
ción normal. Exámenes: BH con leucocitos de 7 200, Hb 15.9 E.Hepatitis por VHC.
g/dL, plaquetas de 52 000, BT 0.7 mg/dL, BD 0.3 mg/dl, PT
6.9, albúmina 3.6 g/dL, globulina de 3.3 g/dL, ALT 79 UI/L, 2. ¿Qué estudios iniciales se solicitarían?
AST 46 UI/L, FA 190 UI/L, DHL 184 UI/L, TP 13.3, INR 1.0. A.PFH.
1. ¿Cuál de las siguientes es la respuesta correcta? B.BH, QS y PFH.
A. La paciente no presenta datos compatibles con hepato­ C.BH, QS, ES, PFH, TP, TC de abdomen y endoscopia supe­
patía crónica del tipo de la cirrosis. rior.
B.No requiere estudios, sólo seguimiento de la trombocito­ D.BH, QS, ES, PFH, TP y ECO de abdomen.
penia. E.Todo lo referido en la respuesta D, además de pruebas
C.Dentro de las posibilidades diagnósticas se debe descar­ específicas para determinar la etiología y endoscopia.
tar cirrosis hepática. 3. ¿Qué grado de insuficiencia hepática presenta el pa­
D.Se debe descartar problema hematológico como causa ciente, según la clasificación de Child-Pugh?
de la trombocitopenia. A.A.
E.Respuestas C y D son correctas. B.B.
2. En caso de sospecha de cirrosis hepática, ¿cuál sería e.e.
el siguiente estudio diagnóstico recomendado? D. No se puede determinar con la información proporcio­
A. Frotis de sangre periférica y aspirado de médula ósea. nada.
B.Marcadores virales. E.Ninguna de las anteriores.
C. Ultrasonido Doppler hepatoesplénico.
D.Anticuerpos antimitocondriales y antinucleares. ► CASO CLÍNICO 44
E. Endoscopia superior diagnóstica.
Hombre de 60 años de edad, con antecedente de alcoholis­
3. ¿Cuál es la etiología de la cirrosis más probable en esta
mo crónico, suspendido hace dos años, acude a la consulta
paciente?
por presentar evacuaciones melénicas intermitentes des­
A.Hepatitis autoinmunitaria. de hace varias semanas. En la exploración física el paciente
B.Hepatitis virales. presenta estigmas de hepatopatía crónica. Se le solicita bio­
C. Esteatohepatitis no alcohólica. metría hemática y los resultados son los siguientes: hemo­
D.CBP. globina 10.5 g/dL, VCM 79 fL, CMH 27 pg, plaquetas 90 000,
E.ldiopática. leucocitos 4 500. ¿Cuál es la causa más probable del san­
grado?
► CASO CLÍNICO 43
A. Las plaquetas por debajo de 100 000.
Hombre de 57 años de edad con eventos de etilismo inten­ B.La ingesta de alcohol.
so hasta la embriaguez cada dos semanas desde los 15 años C. Gastropatía hipertensiva portal.
y desde los 42 años cada tercer día. Acude por presentar D. Ectasia vascular antral.
evacuaciones melénicas en dos ocasiones de moderada can­
tidad hace dos días, autolimitadas; desde hace un mes se ► CASO CLÍNICO 45
agrega astenia, adinamia, hiporexia y aumento del perímetro
abdominal. En la exploración física presenta leve tinte ictéri­ Al paciente del caso anterior se le solicita ultrasonido hepato­
co, hipertrofia parotídea, telangiectasias en pared anterior biliar por sospecha de hepatopatía crónica, el cual mostró:

690
SECCIÓN 11 GASTROENTEROLOGÍA @
hígado disminuido de tamaño, de parénquima con ecoestruc­ A. Cirrosis hepática por causa desconocida.
tura "burda" de bordes lobulados; diámetro de la porta de 13 B. Hepatitis autoinmunitaria en estadio temprano.
mm. ¿Cuál es el mejor indicador de la presencia de várices C. Hemocromatosis.
esofágicas? D. Cirrosis biliar primaria en fase avanzada.
A. Recuento plaquetario por debajo de 100 000/ml. E. Hiperbilirrubinemia no conjugada.
B. Diámetro de la porta superior a 13 mm.
c. Clases B y C de Child-Pugh. ► CASO CLÍNICO 48
D. Visualización directa mediante endoscopia.
Hombre de 54 años de edad, sin antecedentes personales
► CASO CLÍNICO 46 patológicos, niega cirugías y transfusiones; en ocasiones in­
giere algún analgésico. Desde hace cuatro meses refiere
Mujer de 48 años de edad presenta pruebas de funciona­ dolor en hipocondrio derecho, de intensidad leve pero cons­
miento hepático (PFH) anormales al acudir a donar sangre y tante; en dos ocasiones presentó fiebre y escalofríos que
seis meses después acude a su médico. Niega ser portado­ fueron manejados con antibiótico sin documentar foco infec­
ra de alguna enfermedad, no toma medicamentos y su madre cioso. Desde hace 20 días presenta coluria, escalofríos in­
padece esclerodermia. Refiere casi un año con episodios de termitentes, ictericia que ha sido progresiva, debilidad y fa­
prurito manejados como cuadros de "alergia" con antihista­ tiga, por lo que acude al médico, quien bajo sospecha de
mínicos, tiene fatiga y ha notado discreto oscurecimiento de hepatitis solicitó ultrasonido de abdomen, que reporta discre­
su piel. Desde hace dos meses presenta sequedad de la ca­ ta hepatomegalia sin dilatación de la vía biliar, vesicula bi­
vidad oral. En la exploración física se detectan huellas de ras­ liar normal y esplenomegalia. Los exámenes de laboratorio
cado en el tronco y las extremidades, sin estigmas que su­ reportan: leucocitos 15 300, Hb 12.0 g/dl, plaquetas 110 000
gieran enfermedad hepática crónica. Sus nuevos exámenes mm3, ALT 98 UI/L, TGO 92 UI/L, BT 12.6 mg/dl, BD 8.6 mg/dl,
reportan alaninoaminotransferasa (ALT) 90 UI/L, aspartato­ 81 4.0 mg/dl, FA 382 UI/L, LDH 150 UI/L, GGT 278 UI/L, pro­
aminotransferasa (AS T) 78 UI/L, fosfatasa alcalina 485 UI/L, teínas totales 7.6 g/dl, albúmina 3.0 g/dl, TP 17" (testigo 13",
gamma-glutamiltranspeptidasa (GGT) 310 UI/L, bilirrubinas to­ paneles virales para VHB y VHC negativos. Al tener en cuen­
tales 1.0 mg/dl, ultrasonido (US) de abdomen normal, pane­ ta el cuadro clínico y los resultados, ¿cuál considera que es
les virales para hepatitis B y C negativos. ¿Cuál es la sos­ el siguiente paso para establecer un diagnóstico?
pecha clínica en esta paciente?

1
A. Solicitar una tomografía computarizada de abdomen.
A. Hígado graso. B. Solicitar un gammagrama hepatoesplénico.
B. Toxicidad a los antihistamínicos. C. Solicitar ANA, AMA, ANCA y SMA.
C. Enfermedad colestásica autoinmunitaria. D. Realizar biopsia hepática.
D. Colecistitis crónica alitiásica. E. Solicitar una colangiorresonancia o CPRE.
E. Enfermedad dermatológica.

► CASO CLÍNICO 49
► CASO CLÍNICO 47
Hombre de 42 años de edad a quien hace ocho años se le
Mujer de 59 años de edad presenta ictericia progresiva de diagnosticó colitis ulcerativa, en la actualidad en tratamiento
tres meses de evolución, con fatiga, adinamia, prurito ge­ y remisión con mesalazina y asintomático desde hace seis
neralizado, intenso e incontrolable, por lo que acude al ser­ meses. En sus exámenes de seguimiento se documentó fo­
vicio de urgencias. En la exploración física está consciente y fatasa alcalina 278 UI/L, GGT 180 UI/L, ALT 60 UI/L, TGO 48
orientada, con ictericia en escleras y piel +++; hay estigmas UI/L, por lo cual se solicitó US de abdomen que se reportó
de hepatopatía crónica, huellas de rascado generalizadas, normal, serologías para VHB y VHC negativas, fosfatasa al­
algunas con equimosis, abdomen plano sin visceromegalias, calina 246 UI/L, GGT 167 UI/L, ALT 62 UI/L, TGO 48 UI/L, LDH
edema en miembros inferiores +. Sus exámenes de labora­ 160 UI/L, p ANCA +.
torio y gabinete son: ALT 120 UI/L, AST 90 UI/L, FA 780 UI/L, 1. ¿Cuál es el posible diagnóstico en este caso?
GGT 529 UI/L, bilirrubinas totales 15 mg/dl, B indirecta 6.7
mg/dl, B directa 8.3 mg/dl, proteínas totales 7.0 g/dl, albú­ A. Colestasis secundaria a medicamentos.
mina 3.0 g/dl, colesterol 350 mg/dl, INR 1.8, anticuerpos an­ B. Hepatitis viral sobreagregada a la colitis ulcerosa.
tinucleares (ANA) negativos, anticuerpos antimitocondriales C. Colecistitis.
(AMA) positivos, serología viral VHB y VHC negativa. Ultraso­ D. Colitis ulcerosa activa con manifestaciones extrahepá­
nido abdominal con datos de daño hepático difuso, sin dilata­ ticas.
ción de la vía biliar intra y extrahepática ni lesiones ocupati­ E. Colitis ulcerosa asociada con colangitis esclerosante pri­
vas. ¿Cuál es su principal diagnóstico? maria.

1 691
e MANUAL PARA EL EXAMEN NACIONAL DE RESIDENCIAS MÉDICAS

2. Para complementar el diagnóstico se solicitó una co­ AMA, ANA, ASMA negativos. lgG 1 790 mg/dl. Colesterol
langiorresonancia, la cual se reportó normal. ¿Cuál sería el 300 mg/dl, HDL 30 mg/dl. Exploración física: TA 130/90 mm
paso a seguir? Hg, IMC 35 kg/m 2 , sin otros datos relevantes. Se determina
HOMA 3.5. Ultrasonido hepático con datos de enfermedad
A. Vigilancia cada mes con pruebas de funcionamiento he­ hepática difusa y aumento de la ecogenicidad. Se realizó
pático. biopsia hepática: esteatohepatitis con fibrosis F2, motivo por
B. Determinación de CA 19-9 cada año a partir de este mo- el cual se decidió iniciar pioglitazona como terapéutica. La
mento. paciente está en control, con descenso de las enzimas he­
C. Realizar CPRE para toma de biopsias o citología. páticas. ¿Cuál es su diagnóstico para esta paciente?
D. Biopsia hepática.
E. Realizar estudios de complementación hasta que el pa­ A. Hígado graso alcohólico.
ciente esté asintomático. B. Hígado graso no alcohólico.
C. Esteatohepatitis alcohólica.
D. Esteatohepatitis no alcohólica.
► CASO CLÍNICO SO E. Hepatitis por tóxicos.

Hombre de 53 años de edad, diabético de diez años de evo­


lución, sin antecedentes familiares de importancia, es ofici­ ► CASO CLÍNICO 52
nista y consume dos cervezas por semana. Inicia su padeci­
miento actual hace seis meses caracterizado por debilidad, Hombre de 45 años de edad, con etilismo intenso, se presen­
fatiga, artralgias y dolor en hipocondrio derecho no específi­ ta en el servicio de urgencias por cuadro clínico de ocho
co. En la exploración física se reporta IMC 32 kg/m2 , hiperpig­ días de evolución caracterizado por malestar general, fiebre
mentación de tegumentos, hepatoesplenomegalia, sin otros y dolor en hipocondrio derecho. En la clínica con hepatome­
datos específicos. Se realizaron análisis bioquímicos de ruti­ galia 3 cm por debajo de reborde costal, sin datos de irrita­
na, de los cuales destacan ALT 120 U/L, AST 75 U/L y perfil ción peritoneal En los análisis de laboratorio presenta leu­
viral para VHA, VHB, VHC negativos. AMA, ANA, ASMA ne­ cocitosis de 13 000 con predominio de neutrófilos, elevación
gativos. Saturación de transferrina 50%, ferritina 1 000 ng/ml. de transaminasas al doble del valor normal y bilirrubinas nor­
Ultrasonido hepático con aumento de la ecogenicidad. Se males.
realizó estudio invasivo que demuestra concentración de hie­ 1. ¿Cuál es la sospecha diagnóstica?
rro hepático 2 µmol/L. ¿Cuál es el diagnóstico y qué elemen­
A. Colecistitis crónica litiásica agudizada.
tos diagnósticos se encuentran en este paciente?
B. Pancreatitis crónica.
A. Esteatohepatitis alcohólica. Elementos diagnósticos: in­ C. Absceso hepático.
gesta de alcohol, IMC 32 kg/m 2 . ALT 120 U/L, ferritina D. Hepatitis viral aguda.
>1 000 ng/ml.
2. ¿Cuál sería el estudio de imagen más accesible para
B. Esteatohepatitis no alcohólica. Elementos diagnósticos:
llegar al diagnóstico?
IMC 32 kg/m 2 , hepatomegalia. ALT 120 U/L, ferritina
>1 000 ng/ml. A. Tomografía computarizada de abdomen.
C. Enfermedad de Wilson. Elementos diagnósticos: diabe­ B. Ultrasonido de abdomen superior.
tes mellitus tipo 2, IMC 32 kg/m2 . ALT 120 U/L, ferritina C. Resonancia magnética.
>1 000 ng/ml. D. Tele de tórax.
D. Hemocromatosis. Elementos diagnósticos: hiperpigmen­
3. En el ultrasonido se observa imagen anecoica, única,
tación de tegumentos, transferrina >50%, ferritina >1 000
mayor a 12 cm de diámetro, localizada en lóbulo izquierdo.
ng/ml, hierro hepático 2 en biopsia hepática.
¿Cuál es la conducta a seguir?

A. Vigilancia ambulatoria con laboratorios cada semana.


► CASO CLÍNICO 51
B. Manejo intravenoso con fluoroquinolonas durante siete a
Mujer de 44 años de edad, sin antecedentes de importancia, diez días.
consume una cerveza por semana. Inicia su padecimiento C. Drenaje percutáneo.
con cansancio inespecífico, no incapacitante. Referida para D. Resección quirúrgica de segmentos implicados.
estudio por persistencia de elevación de enzimas hepáticas
en los últimos diez meses; exámenes bioquímicos: ALT 633 ► CASO CLÍNICO 53
U/L, AST 544 U/L, FA 100 U/L, BT 0.51 mg/dl, Alb 3.8 g/dl,
PT 8.5 g/dl, TP 14 segundos, INR 1.0, Hb 15.1 mg/dl y plaque­ Hombre de 65 años de edad acude al servicio de urgencias
tas 221 000/mm3 • Perfil VHC y VHB negativo. Anticuerpos con dolor abdominal epigástrico intenso que irradia hacia la

692
SECCIÓN 11 GASTROENTEROLOGÍA @
espalda. Antecedentes médicos de úlcera péptica y vascu­ 2. ¿Cuál de las siguientes afirmaciones es cierta respec­
lopatía periférica. En la exploración física se detecta taquicar­ to de la amilasa sérica?
dia, taquipnea, hipotensión ortostática e hipersensibilidad ab­
dominal a la palpación, sin signos peritoneales. Las pruebas A. Es más específica que la lipasa para la pancreatitis.
de laboratorio incluyen: recuento de leucocitos 15 000, ami­ B. En la insuficiencia renal crónica, los niveles no suelen es­
lasa 240 UI, lipasa 180 UI, perfil hepático normal, BUN 32 tar elevados más del doble del valor normal.
mg/dL, creatinina 1.6 mg/dL. Radiografía simple de abdomen C. Puede ser normal en la pancreatitis aguda.
muestra patrón compatible con íleo y ausencia de aire libre. D. Tiende a permanecer elevada más tiempo que la lipasa.
1. ¿Cuál de las siguientes medidas se consideraría a con­
tinuación? ► CASO CLÍNICO 55

A. Realizar una colangiopancreatografía retrógrada endos- Hombre de 47 años de edad, referido por dolor abdominal
cópica (CPRE) con esfinterotomía. recurrente de diez años de evolución; antecedente de taba­
B. Solicitar una ecografía abdominal. quismo y etilismo durante veinte años. Ha presentado pan­
C. Solicitar una TC abdominal. creatitis aguda recurrente sin datos de respuesta inflamato­
D. Recomendar laparotomía. ria sistémica o complicaciones locales. Hace cuatro años se
E. Solicitar un tránsito digestivo superior. realizó colecistectomía por presentar lodo biliar. Sin embar­
2. ¿Cuál de los siguientes procesos no suele asociarse go, el dolor ha persistido y en la actualidad se lo controla de
con pancreatitis aguda? manera parcial con analgésicos narcóticos. Niega esteatorrea
y los niveles de glucemia en ayuno son de 90 mg/dl. En re­
A. Litiasis biliar. lación con lo anterior,
B. Consumo de alcohol. 1. ¿Cuál se considera la causa más probable de pancrea­
C. Herencia. titis crónica en este paciente?
D. Hipertrigliceridemia.
E. Fármacos. A. Este paciente sólo tiene pancreatitis aguda.
B. Lito residual.
C. Etilismo y tabaquismo.
► CASO CLÍNICO 54 D. Mutaciones genéticas.
Hombre de 40 años de edad que ingresa al servicio de urgen­ 2. El paciente acude ocho meses después por presentar

1
cias por presentar cuadro de pancreatitis alcohólica secunda­ evacuaciones pastosas, explosivas, "que flotan y dejan un
ria al alcohol. Al principio evoluciona de forma satisfactoria halo grasoso". Ha perdido 10 kg de peso en este periodo y
pero siete días después del ingreso desarrolla dolor abdomi­ presenta ictericia. Las enzimas pancreáticas se encuentran
nal cada vez más intenso, fiebre y leucocitosis. Una TC esta­ normales y la glucemia es de 220 mg/L. Tiene una serie de
blece una clasificación E de Balthazar (colecciones con gas malestares: diabetes, salida de grasa en las evacuaciones,
peripancreático). diarrea crónica. ¿Cuál es la explicación que se da al paciente
1. ¿Cuál de las siguientes estrategias se recomendaría? respecto de este nuevo cuadro clínico?
A. Instaurar octreótido intravenoso. A. Insuficiencia endocrina y exocrina del páncreas.
B. Realizar una aspiración del páncreas con aguja fina. B. Asociación de trastorno funcional digestivo y síndrome
C. Obtener cultivos de sangre y esperar resultados. metabólico.
D. Instaurar cefalosporina intravenosa de manera empírica. C. Efecto secundario del consumo crónico de AINE.
E. Repetir la TC al cabo de dos días. D. Efecto secundario del consumo crónico de opiáceos.

693
e MANUAL PARA EL EXAMEN NACIONAL DE RESIDENCIAS MÉDICAS

CAPÍTULO 11.1
ESTRUCTURA DEL ESÓFAGO, SÍNTOMAS Y ANOMALÍAS
Hairé Manzano Cortés

ESTRUCTURA Y FUNCIÓN DE BOCA 6. ¿Qué es la pirosis funcional?


Y ESÓFAGO Episodios de pirosis o dolor retroesternal, sin evidencia de da­
ño esofágico o reflujo anormal; es más frecuente en mujeres y
1. ¿Cómo se lleva a cabo la secreción de la saliva? en protocolo de estudio con pHmetría de 24 horas presenta
Cada 24 horas se secretan 1 500 mL de saliva, la cual se produ­ reflujo fisiológico normal.
ce en las glándulas parótidas, submaxilares y sublinguales. Es­ Existen dos grupos: esófago hipersensible y la pirosis fun­
ta secreción está bajo control nervioso y del péptido intestinal cional.
vasoactivo.
a) Esófago hipersensible: pacientes que tienen reflujo fisiológi­
co normal por pHmetría, pero con un indice de síntomas
2. ¿Cuáles son los componentes de la saliva?
�50%. El tratamiento es con inhibidores de bomba de pro­
La saliva contiene enzimas (lipasa lingual y la amilasa-alfa sa­
tones (IBP) a doble dosis y antidepresivos tricíclicos.
lival llamada ptialina), lgA y concentraciones de Na+, K+, cr
b) Pirosis funcional: pacientes con reflujo fisiológico normal
y HCO 3, similares a las del plasma.
pero con un índice de síntomas <50% (no relacionados con
reflujo ácido). Tratamiento con antidepresivos tricíclicos o
3. ¿Cuál es la longitud del esófago y cómo se divide? inhibidores de la recaptura de serotonina.
El esófago mide 25 cm de longitud y se le ha dividido en cuatro
segmentos: faringoesofágico, cervical, torácico y abdominal. 7. ¿Qué es el dolor torácico (esofágico)?
Se extiende desde la sexta o séptima vértebra cervical hasta la
El dolor torácico puede ser de causa cardiaca y no cardiaca.
décima primera vértebra torácica. A través del mismo pasan
La causa más común de dolor torácico no cardiaco es ERGE.
los alimentos de la faringe al estómago. La palabra esófago se
En trastornos de la motilidad es intermitente, y de estos tras­
deriva del latin esophagus, que deriva de la palabra griega oiso­
tornos el esófago en cascanueces es la causa más común.
phagos, literalmente "entrar por alimentos". El esfínter esofá­
gico superior, que está formado por el músculo cricofaríngeo,
ANOMALÍAS DEL DESARROLLO
lo delimita de la faringe y el esfínter esofágico inferior lo deli­
mita del estómago. Discurre por el cuello y por el mediastino
8. ¿Cuáles son las malformaciones asociadas con
posterior hasta introducirse en el abdomen a través del dia­
atresia esofágica (AE) y fístula traqueoesofágica
fragma. Presenta tres zonas de estrechamiento: constricción
(FTE)?
cervical, que es la más angosta y mide 14 mm de diámetro;
constricción broncoaórtica, de 15 a 17 mm, y constricción dia­ Véase cuadro 11-1-1.
fragmática, de 16 a 19 mm.

4. ¿Qué es el esfínter esofágico inferior? CUADRO 11-1-1.


MALFORMACIONES ASOCIADAS CON AE Y FTE
Es una zona de mayor presión que mide de 3 a 5 cm en el seg­
mento distal del esófago, que sirve de "barrera" contra la regur­ SISTEMA INVOLUCRADO MALFORMACIÓN
gitación normal de contenido gástrico hacia el esófago, cuya
Musculoesquelético Hemivértebras, displasia radial o amelía,
presión es de 10 a 45 mm Hg. polidactilia, sindactilia, escoliosis, malfor­
maciones de las costillas, defectos en
SÍNTOMAS ESOFÁGICOS las extremidades inferiores

Digestivo Ano imperforado, atresia intestinal, diver­


5. ¿Qué es la pirosis? tículo de Meckel, páncreas anular, mal­
a) Sensación de dolor, ardor o quemazón en el esófago a nivel rotación
retroesternal. Cardiovascular Defectos del tabique ventricular, ductus
b) Puede iniciar en epigastrio y ascender; aparece entre 30 y arteriosus, tetralogía de Fallo!, defectos
60 minutos después de la ingesta. del tabique auricular, arco aórtico des­
plazado
e) Suele asociarse con enfermedad por reflujo gastroesofági­
co (ERGE) y es su síntoma más común (75%); sin embar­ Genitourinario Agénesis o displasia renal, riñones en he­
go, su presencia o ausencia no confirma el diagnóstico de rradura, riñón poliquístico y malformacio­
nes uretrales y en uréteres, hipospadias
ERGE.

694
SECCIÓN 11 GASTROENTEROLOGÍA @
CUADRO 11-1-2. cirugía correctora debido a múltiples factores: reflujo gastro­
ENFERMEDAD O SÍNDROME ASOCIADO esofágico, aspiración de contenido gástrico, traqueomalacia,
CON AE Y FTE motilidad esofágica anormal, estenosis esofágica o recurrencia
de la FfE. Sin embargo, se considera que la complicación más
Síndrome óculo-dígito-esofágico-duodenal
frecuente es el reflujo gastroesofágico por alteración en la mo­
Anoftalmía y atresia esofágica
tilidad y aclaramiento esofágico.
Epidermólisis ampollosa letal con atresia pilórica
Hipertelorismo con anormalidades esofágicas e hipospadias DISFAGIA
Asociación VATER
13. ¿Qué es la disfagia?
Síndrome de McKusick-Kaufman
La disfagia es un síntoma que expresa la existencia de una al­
Síndrome de Antley-Bixler teración orgánica o funcional en el trayecto del bolo desde la
Síndrome de Apert boca hasta el estómago. Hay que diferenciar disfagia de odino­
Anormalidades gastrointestinales míltiples fagia y globo histérico.

14. ¿Qué es la odinofagia y el globo histérico?


La odinofagia es el dolor a la deglución y puede asociarse o no
9. ¿Cuáles son las enfermedades o síndromes aso­ con un retraso en el vaciamiento esofágico. El globo histérico
ciados con AE y FTE? es la sensación de nudo o cuerpo extraño referido a nivel de la
Véase cuadro 11-1-2. unión faringoesofágica.

10. ¿Cuáles son los síntomas que orientan al diag­ 15. ¿Cuál es la fisiología de la deglución?
nóstico de estenosis esofágica y atresia esofágica? El proceso de la deglución comienza con una fase voluntaria
Los signos clínicos más típicos para el diagnóstico son sialo­ (bucal) durante la cual el bolo alimentario es impulsado por la
rrea, crisis de tos y cianosis; la clinica es más evidente si se ini­ contracción lingual hacia el interior de la faringe, lo cual acti­
cia la alimentación oral. va los receptores sensoriales orofaríngeos que inician la fase
Si una estenosis esofágica es grave se desarrollará disfagia involuntaria (faríngea y esofágica) o el reflejo de la deglución.
para líquidos en los primeros días/semanas de vida, infeccio­ Este reflejo favorece la contracción del músculo constrictor
nes respiratorias recurrentes y sialorrea. En el caso de una es­ de la faringe superior contra el paladar blando, lo que gene­

1
tenosis congénita de píloro, el síntoma cardinal es el vómito, ra una contracción peristáltica o peristalsis primaria, seguida
abundante y de contenido exclusivamente gástrico; aparece en por una contracción secuencial del esófago o peristalsis secun­
la segunda a tercera semanas de vida. daria. El esófago tiene contracciones denominadas terciarias,
El esófago y la tráquea derivan de un tubo común pero duran­ pero no son peristálticas ya que se producen de manera simul­
te la cuarta a quinta semanas de gestación, si se presenta una tánea, como respuesta a la deglución o distensión esofágica, o
separación incompleta de la parte dorsal del tubo esofágico pri­ de forma espontánea.
mitivo, condiciona atresia esofágica y en la mayoría de los casos El transporte normal del bolo alimenticio depende del ta­
el esófago posterior no se separa por completo de la tráquea, maño del bolo, el diámetro de la luz esofágica, la contracción
lo que da lugar a las variedades de fístula traqueoesofágica. peristáltica y la inhibición deglutoria, así como de la relajación
de los esfínteres esofágicos durante la deglución.
11. Según la clasificación de Gross, ¿cuál es el tipo 16. ¿Cómo se clasifican las disfagias?
más común de AE? Se denomina disfagia mecánica si es ocasionada por un bolo de
La clasificación anatómica de Gross agrupa a la AE en cinco gran tamaño o por estrechamiento de la luz. La disfagia moto­
tipos y la tipo C asocia atresia esofágica y fístula traqueoeso­ ra o neuromuscular se debe a la falta de coordinación, a debi­
fágica. Es la más frecuente en 90 a 95% de los casos. lidad de las contracciones peristálticas o a una inhibición de­
glutoria alterada.
12. ¿Qué complicación tardía se espera en el pacien­ La disfagia motora orofaríngea altera la deglución; por lo
te operado de atresia esofágica y fístula traqueoeso­ general se debe a trastornos neuromusculares que ocasionan
fágica? parálisis muscular (miastenia grave, EVC).
La mayoría de los pacientes con AE y FfE presenta morbili­ Las enfermedades de la musculatura lisa afectan a la porción
dad significativa. Las complicaciones más frecuentes que ocu­ torácica del esófago y al esfínter esofágico inferior (EEI). Apa­
rren en los pacientes con AE/FfE luego de la reparación qui­ recen cuando las contracciones peristálticas son débiles (escle­
rúrgica son de tipo respiratorio y digestivo, que condicionan un rodermia) o no son peristálticas o cuando el esfínter inferior
deterioro de la calidad de vida y alta tasa de hospitalización. deja de abrirse de forma normal (defecto en inervación vagal
Estos problemas suelen persistir aun después de realizada la inhibitoria; acalasia).

695
e MANUAL PARA EL EXAMEN NACIONAL DE RESIDENCIAS MÉDICAS

CUADRO 11-1-3. a) La disfagia es para alimentos sólidos, líquidos o ambos: en­


CAUSAS DE LA DISFAGIA fermedades de la mucosa esofágica distal y del mediastino
producen disfagia a sólidos por estrechamiento de la luz y
a) Lesión bilateral de la neurona motora superior:
mínimo obstáculo a líquidos. Alteración del peristaltismo
, Disfagia pseudobulbar por parálisis muscular
espástica por afectación del músculo liso y su inervación es para am­
b) Daño medular: bos. En obstrucción esofágica por lo general se percibe que
, Esclerosis lateral amiotrófica el material deglutido se detiene en un punto que está por
, Enfermedad de la neurona motora
encima o a nivel de la lesión causante de la obstrucción.
• Siringobulbia
• Poliomielitis b) Evolución intermitente o progresiva: en pacientes con un ani­
c) Debilidad del músculo estriado: llo mucoso esofágico inferior (Schatzki) suele ser intermiten­
• Miastenia grave te y no progresiva, con episodios breves y autolimitados de
• Dermatomiositis disfagia para alimentos sólidos, por lo general asociados con
• Distrofia miotónica
d) Obstrucciones mecánicas:
comidas copiosas (carne), en un restaurante o una reunión
• Neoplasias social ("síndrome de bistec"). Los episodios pueden estar
• Divertículo faríngeo separados por semanas, meses o años y el paciente no suele
• Obstrucción cricofaríngea y acalasia presentar dificultad en la deglución entre dichos episodios.
e) Inflamación:
Por el contrario, la estenosis esofágica es progresiva en fre­
• Tonsilitis
• Candidosis cuencia y gravedad. Si es una estenosis benigna, la pro­
• Herpes simple gresión es lenta e insidiosa (periodos de meses a años) y
f) Psicológicas: la pérdida de peso es mínima, pero si es maligna se produce
• Globo histérico disfagia rápidamente progresiva (periodo de semanas a me­
• Depresión
ses), donde la pérdida de peso puede ser importante.
Esofágica a) Enfermedades neurológicas: e) Historia de pirosis crónica: al ser síntoma cardinal de la en­
• Acalasia y enfermedad de Chagas
• Espasmo esofágico difuso
fermedad por reflujo gastroesofágico (ERGE) sugiere este­
• Hipertonía del esfínter esofágico inferior nosis péptica.
• Neuropatía autonómica (por ejemplo, diabetes) d) Ingesta de medicamentos: AINE, quinidina, alendronato y
• Esclerosis múltiple doxiciclina pueden producir esofagitis.
b) Enfermedades musculares:
e) Enfermedades colágeno-vasculares: afectan esófago distal y
• Esclerosis sistémica (esclerodermia)
• Lupus eritematoso sistémico alteran la motilidad, como la esclerodermia, la artritis reu­
• Distrofia miotónica matoide o el lupus eritematoso sistémico.
c) Obstrucciones mecánicas: f) Inmunodepresión: las esofagitis infecciosas ocurren con fre­
• Anillos: disfagia sideropénica
cuencia en pacientes con compromiso inmunológico grave
• Estenosis fibrosas: reflujo gastroesofágico, anillo
de Schatzki, ingestión de fármacos o sustancias debido a infección por el virus de la inmunodeficiencia hu­
corrosivas, enfermedad de Behc;et, radioterapia mana (VlH), neoplasias terminales, trasplante o fármacos
• Cuerpos extraños: en particular en niños y enfer­ inmunosupresores. Se estima que entre 30 y 40% de los pa­
mos mentales cientes con infección por el VlH desarrolla síntomas de
• Neoplasias: carcinoma de esófago o cardias,
leiomioma
enfermedad esofágica. La mayoría de las infecciones esofá­
• Compresiones extrínsecas: carcinoma bronquial, gicas son causadas por Candida, citomegalovirus y virus her­
adenopatías mediastínicas, bocio torácico, aneu­ pes simple.
risma de aorta, poscirugía (funduplicatura) g) Síntomas acompañantes: vómito y regurgitación, tos, odino­
• Divertículos esofágicos
fagia, hipo, eructos, etcétera.
d) Enfermedades inflamatorias:
• Esofagitis por reflujo gastroesofágico h) Síntomas de alarma: epigastralgia grave, hemorragia y pér­
• Disfagia sideropénica dida ponderal.
• Candidosis
• Herpes simple
BIBLIOGRAFÍA
Hógenauer C, Hammer HF. Sleisenger & Fortran's Gastrointestinal and Liver
Según el sitio anatómico la disfagia puede ser esofágica u Disease. 9th ed. New York: Saunders; 2010.
orofaríngea, cuyos síntomas son regurgitación nasal, tos al de­ Jakubson SL. Atresia esofágica y fistula traqueoesofágica. Evolución y com­
glutir, intentos repetidos de deglución y residuo oral posdeglu­ plicaciones postquirúrgicas. Rev Chil Pediatr. 20l0;81(4):339-46.
ción. Cada una tiene diferentes causas. Véase cuadro 11-1-3. Kovesi T, Rubin S. Long-term complications of congenital esophageal atresia
and/or tracheoesophageal fistula. Chest. 2004 Sep;126(3):9l5-25.
17. ¿Cómo se elabora el diagnóstico de disfagia? Loannides SA, Copp JA. Embryology of oesophageal atresia. Semin Pediatr
Surg. 2009 February; 18( 1 ):2-11.
La historia clínica permite determinar el sitio anatómico to­ Spitz L. Esophageal atresia and tracheoesophageal fistula in children. Curr
mando en cuenta lo siguiente: Opin Pediatr. 1993 Jun;5(3):347-52.

696
SECCIÓN 11 GASTROENTEROLOGÍA @
CAPÍTULO 11.2
TRASTORNOS MOTORES DEL ESÓFAGO
Jony Cerna Cardona • Aleida Bautista Santos

ACALASIA zadas, cirugía o la inyección intraesfinteriana de toxina botulí­


nica.
1. ¿Cuál es la patogénesis de la acalasia?
Se considera una enfermedad motora del esófago, caracteri­ ESPASMO ESOFÁGICO DIFUSO
zada por ausencia de peristalsis del cuerpo esofágico y dis­
función del EEI (esfínter esofágico inferior) que se muestra 6. ¿Qué es el espasmo esofágico difuso?
incapaz de relajarse tras la deglución y puede ser primaria o Los trastornos motores esofágicos primarios por incoordina­
secundaria. La pérdida del peristaltismo del cuerpo esofági­ ción se consideran alteraciones de la coordinación de las con­
co y la incapacidad de relajación del EEI ocasionan retención tracciones no peristálticas (segmentarías, simultáneas y re­
de los alimentos en la luz del esófago y su progresiva dilatación. trógradas) del cuerpo del esófago. Esta es una alteración que
Su causa es desconocida, pero se debe a falta de inervación del afecta ante todo los dos tercios inferiores del cuerpo esofágico.
músculo liso del esófago, con reducción de células gangliona­
res del plexo mientérico de Auerbach. 7. ¿Cuál es la patogénesis del espasmo esofágico
difuso (EED)?
2. ¿Cuál es la incidencia anual de la acalasia? Es un trastorno idiopático primario o secundario que puede ser
La incidencia anual en Estados Unidos y Europa oscila entre ocasionado por enfermedad por reflujo gastroesofágico, obs­
el 0.8 y 2 por 200 000 habitantes por año; es decir, diez casos trucción del cardias o neuropatía periférica. El EED idiopáti­
por cada 100 000 habitantes. co se ha relacionado con distintas alteraciones en el músculo
esofágico y de elementos nerviosos. Las alteraciones en el ple­
3. Mencione los criterios manométricos de la acala­ xo de Auerbach, las fibras vagales y el músculo liso son míni­
sia: mas, aunque se ha descrito con frecuencia la existencia de in­
El principal criterio manométrico es la aperistalsis y se con­ filtrado inflamatorio en la pared esofágica.
sidera un criterio de tipo obligado. El resto de los criterios

1
pueden ser opcionales. Véase cuadro 11-2-1. CUADRO 11-2-2.
PROCESOS QUE CURSAN CON ALTERACIONES
4. ¿Cómo se establece el diagnóstico diferencial de MOTORAS TIPO ACALASIA
la acalasia?
Acalasia primaria (idiopática)
Debe realizarse con el resto de los trastornos motores esofá­ Acalasia clásica
gicos primarios, con la acalasia secundaria a otros procesos Acalasia vigorosa
(pseudoacalasia) y con las estenosis orgánicas (cáncer de car­
Acalasia secundaria
dias, estenosis pépticas). Véase cuadro 11-2-2. Infección por Trypanosoma cruzi(enfermedad de Chagas)
Afectando la región del EEI
5. ¿Cuál es el tratamiento para los trastornos moto­ A distancia del EEI
res primarios del esófago? Cáncer(pseudoacalasia)
Se dispone de una amplia variedad de opciones terapéuticas: Enfermedades infiltrativas del EEI
Amiloidosis
pueden utilizarse medidas farmacológicas, dilataciones for-
Enfermedad de Fabry
Infiltración eosinofílica
CUADRO 11-2-1. Sarcoidosis
CRITERIOS MANOMÉTRICOS Asociación con enfermedades sistémicas
Diabetes mellitus
Presión de reposo del EEI normal o elevada" Insuficiencia suprarrenal y alacrimación
Relajaciones del EEI incompletas(< 80% o inexistentes") Síndrome de Sicca con hiposecreción gástrica
Presión basal del cuerpo esofágico elevada''(igual o superior a la Neoplasias endocrinas múltiples tipo 2b
gástrica) Múltiples defectos congénitos con acalasia
Aperistalsis' Asociación con alteraciones de la motilidad intestinal
Comportamiento vigoroso del cuerpo esofágico'" Pseudoobstrucción intestinal
Test colinérgico positivo" Acalasia y piloroespasmo
Cirugía de la unión esofagogástrica
'Criterio obligado; "criterios opcionales; "'específico de acalasia vigorosa.

697
e MANUAL PARA EL EXAMEN NACIONAL DE RESIDENCIAS MÉDICAS

FIGURA 11-2-1. HALLAZGOS RADIOLÓGICOS DEL 9. ¿Cuáles son los criterios manométricos de diag­
ESPASMO ESOFÁGICO DIFUSO. nóstico del EED?
El hallazgo manométrico más constante es la presencia de
ondas simultáneas junto con ondas peristálticas de caracte­
rísticas normales. El porcentaje mínimo de las contraccio­
nes simultáneas es de 30%, amplitud de ondas simultáneas
> 30 mm Hg, ondas peristálticas normales, ondas de gran am­
plitud, ondas de gran duración, ondas de triple pico, activi­
dad espontánea, hipertonía del EEI y relajación anormal del
EEI.

ESÓFAGO EN CASCANUECES

10. ¿Qué es el esófago en cascanueces?


Los trastornos motores por hipercontractilidad se caracterizan
por contracciones de gran amplitud, duración prolongada o
ambas que afectan a todo el cuerpo esofágico o a un segmento
determinado. El esófago en cascanueces también es conocido
como peristalsis esofágica sintomática. La amplitud promedio
de las ondas contráctiles es de 180 mm Hg, con una duración
media de siete segundos.
8. ¿Cómo se realiza el diagnóstico radiológico de
EED? 11. ¿Cuáles son los hallazgos manométricos median­
Las anomalías radiológicas más frecuentes consisten en la apa­ te la manometría de alta resolución en el esófago en
rición, tras la deglución del medio de contraste, de contraccio­ cascanueces?
nes anómalas en la mitad inferior del esófago. La morfología Los hallazgos manométricos mediante esta herramienta de
puede variar desde una imagen en sacacorcho hasta escotadu­ alta resolución son comportamiento hipercontráctil de predo­
ras simétricas, profundas y segmentarias (espasmo segmenta­ minio distal con integral de contracción distal (DCI) y prome­
rio). Véase figura 11-2-1. dio superior a 5 000 mm Hg/cm/segundo (figura 11-2-2).

FIGURA 11-2-2. MANOMETRÍA ESOFÁGICA.

Esfínter
rl '" ceposo

EES
Presión intraesofágica

esofá gico P resión a l ta -- Rel aj a ción tra nsito ria del EES

-
O - -- - - -
;i;��f
superior -o -------- - - - - - - - -- --- - - --- - ----------- ·
·········· ..-'····· ·�
(EES) ····
� ....•··
....� _�
�8 :
tra r
�� �����
__ ___ ____ r á
��:� �� ��� ��::�
8
·.
· ·· . .. · ·
. . .. ·····
·
:f e ::-:-::-:-:: º
.. - ····
····
/ Rel ajación
·
·· • ···· EEI

.. ...·· P resión a l ta
--------... del EEI

Dia fr a gm a
. . •· ·�o -------o
------.......,;. Rel aj a ción receptiva
/ del estóm a go

J
698
SECCIÓN 11 GASTROENTEROLOGÍA @
TRASTORNOS MOTORES SECUNDARIOS CUADRO 11-2-3.
(TMS) TRASTORNOS MOTORES ESOFÁGICOS SECUNDARIOS

12. ¿Qué son los trastornos motores secundarios? Enfermedades del sistema nervioso central
Patología vascular
La actividad motora del esófago puede alterarse por diferentes Accidentes cerebrovasculares
enfermedades sistémicas (metabólicas, neuromusculares) y por Parálisis pseudobulbar
múltiples procesos infiltrativos, inflamatorios u obstructivos Patología degenerativa
(estenosis benignas o malignas, posquirúrgicas del propio esó­ Enfermedad de Parkinson
fago; véase cuadro 11-2-3). Para su clasificación, los TMS suelen Enfermedades desmielinizantes
agruparse según el nivel en el que se localiza la alteración de Esclerosis múltiple
la dinámica esofágica. Esclerosis lateral amiotrófica
Miscelánea
Tumores intracraneales
ENFERMEDAD POR REFLUJO Alteraciones congénitas
GASTROESOFÁGICO Neuropatía periférica
Enfermedades de las neuronas intramurales
13. Ante la presencia de síntomas de ERGE, sin datos Reducción del número de células ganglionares
de alarma, ¿cuál es el tratamiento de elección? Acalasia (ldiopática)
Enfermedad de Chagas (es infecciosa no tóxica)
El tratamiento con inhibidor de bomba de protones (IBP) cada
Destrucción neoplásica
12 horas por dos semanas y evaluar si existe una disminución "Irritación" de las células ganglionares
de sintomas de al menos 50%. El diagnóstico de ERGE tiene Reflujo gastroesofágico
una sensibilidad de 68% y una especificidad de 83%. Véase fi­ lngesta de cáusticos
gura 11-2-3. Alteraciones de la placa motora
Miastenia grave
Alteraciones musculares
14. Ante la presencia de esofagitis por ERGE, ¿cuál es
Músculo esquelético (polimiositis, dermatomiositis)
la estadificación endoscópica correcta? Músculo liso (esclerodermia, miopatía visceral, etcétera)
La más utilizada y recomendada es la clasificación de Los Án­ Miopatías metabólicas (tirotoxicosis)
geles; sin embargo, también existe la de Savary-Miller. Ambas
permiten la estadificación de la esofagitis.

1
19. ¿Cuál es la función de los esteroides en el trata­
15. ¿Cuál es la indicación de realización de pHme­ miento para las úlceras esofágicas por VIH?
tría de 24 horas en un paciente con ERGE? Este tipo de lesiones ulceradas se presentan en pacientes con
Pacientes con ERGE no erosiva, los que no responden a dosis VIH indetectable y CD4 > 250, bajo tratamiento con antirre­
altas de IBP, aquellos que serán protocolizados para cirugía an­ trovirales (ARV), y responden de forma adecuada a dosis de
tirreflujo, los persistentes con síntomas después de cirugía. prednisona de 40 mg/día por cuatro semanas.

16. ¿Cuáles son las complicaciones esofágicas de la ESOFAGITIS POR CÁUSTICOS


ERGE?
20. ¿Cuál es el manejo agudo de las quemaduras
Hemorragia, estenosis péptica y esófago de Barrett. esofágicas por cáusticos?
Hidratación parenteral, analgesia, Rx de tórax, TC de tórax y
ESOFAGITIS INFECCIOSA endoscopia alta. El uso de inhibidores de la bomba de proto­
nes es controvertido. Los antibióticos se consideran útiles en
17. ¿Cuáles son las opciones de tratamiento tópico la profilaxis infecciosa, cuando existe necrosis esofágica (lesio­
para la esofagitis por Candida? nes grado 3.A); su uso está recomendado en casos de lesión
Clotrirnazol crema 10 mg disuelta en la boca cinco veces al esofágica transmural, necrosis extensa y neumomediastino, y
día o nistatina 200 000 UI disueltas YO cuatro o cinco veces para la prevención de mediastinitis y sepsis.
al día, por 14 días. La cirugía urgente está indicada en la perforación esofági­
ca; sus complicaciones son coagulación intravascular disemi­
18. ¿Qué tratamiento se recomienda para la esofagi­ nada, falla renal aguda, acidosis metabólica y hemorragia ma­
tis por herpes? siva. La cirugía urgente tiene como fin preservar la vida y se
Aciclovir 250 mg N por metro de superficie corporal cada realiza limpieza, hemostasis y esofagectomía, con cierre de oro­
ocho horas hasta que el paciente tolere la vía oral; más adelan­ faringe, gastrectomía parcial y yeyunostomía para alimentación
te 200 mg VO cada seis horas por siete a diez días. del paciente.

699
e MANUAL PARA EL EXAMEN NACIONAL DE RESIDENCIAS MÉDICAS

FIGURA 11-2-3. EVALUACIÓN Y TRATAMIENTO DEL PACIENTE CON ERGE.

Aumentar
No
dosis de IBP

Continuar con
">------Sí------� terapia de
mantenimiento

No

Confirmar diagnóstico por


endoscopia y pH-metría,
manometría e impedancia,
individualizado para cada caso

Sí Referir a cirugía

No

Tomado de la Guía de práctica clínica. Diagnóstico y tratamiento quirúrgico de la enfermedad por reflujo gastroesofágico en el adulto. México:
CENETEC; Secretaría de Salud. 2012.

21. ¿Cuáles parámetros y escala endoscópica se uti­ 23. ¿Cuál es la terapéutica para la estenosis esofá­
lizan para estadificar el daño a la mucosa? gica secundaria?
Se utiliza la escala de Zargar. Véase cuadro 11-2-4. La estenosis esofágica se presenta a partir de la tercera semana
después de la quemadura. Las lesiones 2B a 3 presentan este­
22. ¿Qué función tiene el empleo de esteroides en nosis en 71 a 100% de los casos, siendo más propensas las lesio­
las quemaduras esofágicas por cáusticos? nes por álcalis que por ácidos. Las lesiones provocan disminu­
La administración de esteroides se recomienda a partir de las ción en la peristalsis esofágica y en la presión en reposo del
lesiones grado 3, aunque su uso tanto N como intralesional es esfínter esofágico inferior, lo que expone al esófago dañado a
un tema de debate. Su objetivo teórico es la disminución del mayor reflujo ácido y a lesiones por esofagitis péptica. En esta
proceso inflamatorio y de la cicatrización, que promueven es­ etapa se recomienda la terapéutica con IBP a dosis altas. En
tenosis esofágica y dismotilidad del cuerpo esofágico, beneficio cuanto a la estenosis esofágica, la terapia con dilatadores tipo
que aún no se ha probado. vela (Savary) puede íniciarse desde la tercera semana posterior

700
SECCIÓN 11 GASTROENTEROLOGÍA @

-
CUADRO 11·2·4. reflujo esofágico en el adulto. México: CENETEC; 2012. Disponible en:
CLASIFICACIÓN DE ZARGAR DE ESOFAGITIS http//:www.cenetec.salud.gob.mx. Consultado el 11 de enero de 2016.
POR CÁUSTICOS Contini S, Scarpignato C. Caustic injury ofthe upper gastrointestinal tract: A
comprehensive review. World J Gastroenterol. 2013; 19(25):3918-30.
CARACTERISTICAS Díaz-Rubio M, Díaz Rubio E. Trastornos motores del aparato digestivo. 3a.
Grado O Normal ed. Madrid: Médica Panamericana; 20!0.
Katzka DA. Esophageal Disorders caused by Medications, Trauma and Infec­
Grado 1 Edema y eritema superficial de la mucosa
tion. En: Feldman M, Friedman LS, Brandt LJ (ed). Sleisenger and For­
Grado 2 Ulceración mucosa y submucosa tran's Gastrointestinal and Liver Disease Textbook. 9th ed. New York:
Grado 2.A Ulceración mucosa con exudados Saunders Elsevier; 201O. pp. 922-1960.
Grado 2.8 Ulceración circunferencial profunda Richter JE, Friedenberg FK. Gastroesophageal Reflux Disease. En: Feldman
M, Friedman LS, Brandt LJ (ed). Sleisenger and Fortran's Gastrointesti­
Grado 3 Ulceración transmural con necrosis
nal and Liver Disease Textbook. 9th ed. New York: Saunders Elsevier;
Grado 3.A Necrosis focal 20!0. pp. 922-1960.
Grado 3.B Necrosis extensa Ruiz de León A, Pérez de la Serna J. P74 LES pseudo-relaxations in oesopha­
Grado 4 Perforaciones geal achalasia evaluated by high-resolution manometry. J Clin Gastroente­
rol. 2006;40:S2l4-5.
Spechler SJ, Castell DO. Classífication of esophageal motility abnormalities.
Gut. 2001;49: 145-51.
a la quemadura y tienen menor riesgo de perforación esofági­
Spechler SJ, Castell DO. The Esophagus. 4th ed. Philadelphia: Lippincott;
ca que los dilatadores neumáticos. 2003. pp, 262-74.

BIBLIOGRAFÍA
Centro Nacional de Excelencia Tecnológica en Salud. Guía de práctica clí­
nica para el diagnóstico y tratamiento quirúrgico de la enfermedad por

CAPÍTULO 11.3
OTROS TRASTORNOS ESOFÁGICOS
Brenda Patricia Rendón Martínez
1
DIVERTÍCULOS ESOFÁGICOS ticas de los pacientes, el tamaño del divertículo y la experien­
cia del centro donde se realiza el procedimiento.
1. En un adulto mayor con disfagia alta, ¿cuáles son
los diagnósticos diferenciales a estudiar?
HERNIA HIATAL
La disfagia alta es ocasionada por múltiples patologías. Algu­
nas de ellas incluyen divertículo de Zenker, neoplasias, osteo­ 3. ¿Cuál es el estudio diagnóstico de primera elec­
fitos, esclerosis múltiple, miastenia grave y polimiositis, entre ción para el diagnóstico de la hernia hiatal?
otras. Véase cuadro 11-3-1. El estudio baritado esofagogastroduodenal es la técnica diag­
nóstica idónea para el abordaje de la hernia hiatal, sin tomar
2. ¿Cuál es el tratamiento para los divertículos eso­ en cuenta el tipo. El paciente ingiere medio de contraste ba­
fágicos? ritado simple en posición prona, con distensión máxima del
Los divertículos asintomáticos del esófago no requieren trata­ esófago con el fin de poner en evidencia un anillo esofágico
miento. Sólo los pacientes con síntomas claramente relacio­ inferior suprahiatal. En la examinación con doble medio de
nados con el divertículo precisan tratamiento. Se recomienda contraste la mucosa gástrica podría reconocerse en la región
una endoscopia y una manometría esofágica preoperatoria pa­ torácica. A menudo las hernias se reconocen por la presencia
ra descartar trastorno motor esofágico. Existen dos modalida­ de pliegues mucosos gástricos en su interior que pueden cau­
des de tratamiento: la diverticulectomía por cirugía abierta o sar deformidad del esófago y del fondo gástrico. El esófago tor­
laparoscópica y las diversas modalidades endoscópicas. La tuoso presenta una unión de forma excéntrica con el borde de
elección de la técnica quirúrgica dependerá de las caracterís- la hernia. Véase figura 11-3-1.

701
e MANUAL PARA EL EXAMEN NACIONAL DE RESIDENCIAS MÉDICAS

CUADRO 11-3-1. FIGURA 11-3-1. HERNIA HIATAL PARAESOFÁGICA GRANDE.


DIAGNÓSTICO DIFERENCIAL DE DISFAGIA ALTA

ESTRUCTURAL PROPULSIVA
Divertículo de Zenker NEUROGÉNICA MIOGÉNICA
Neoplasias Evento Miastenia grave
cerebrovascular
Barra cricofaríngea Enfermedad de Par- Polimiositis
kinson
Osteofitos Esclerosis lateral Enfermedad mixta del
amiotrófica tejido conectivo
Anormalidades con­ Tumor de Brainsteim Distrofia muscular
génitas
Posterior a cirugía de Síndrome de Síndrome paraneo-
cabeza y cuello Guillain-Barré plásico
Lesión por cáusticos Corea de Huntington Distrofia miotónica
Infección Esclerosis múltiple Sarcoidosis

4. ¿Cuál es el tipo de hernia que ocurre cuando el


estómago protruye por el hiato esofágico pegado al
esófago?
La hernia hiatal tipo 2 o paraesofágica (cuadro 11-3-2) ocurre
cuando el estómago protruye a través del hiato esofágico pega­
do al esófago. La unión esofagogástrica suele mantenerse en
la posición normal, a la altura del diafragma, ya que conserva
el ligamento frenoesofágico posterior que fija esta unión. La
mitad de estos pacientes refiere reflujo gastroesofágico. Otros
síntomas comprenden disfagia, dolor torácico, molestias pos­
prandiales vagas y disnea. La principal complicación es la he­
morragia y la presencia de vólvulo gástrico. Los grados de her­
nia hiatal se muestran en la figura 11-3-2.

5. ¿Qué debe tomarse en cuenta en el tratamiento


de las hernias hiatales?
Las hernias tipo 1 gigantes, mayores de 5 cm, sintomáticas, pa­
raesofágicas y mixtas deben someterse a tratamiento quirúrgi­
co. Muchos expertos consideran que la cirugía debe ofrecerse
a todos los pacientes con hernias paraesofágicas, pues si no se CUADRO 11-3-2.
tratan 30% acaba con complicaciones. Existen varias opciones CLASIFICACIÓN DE LA HERNIA HIATAL
quirúrgicas de funduplicatura total o parcial que pueden ser Por deslizamiento o tipo 1 Donde se presenta desplazamiento
realizadas por vía transabdominal (funduplicatura de Nissen, superior de la unión esofagogástrica
Hill, Toupet) o transtorácica (funduplicatura de Nissen o Belsey­ hacia el mediastino posterior. Se
Mark IV), ya sea en forma abierta o por cirugía mínimamente denomina hernia "por deslizamiento"
ya que tiene un saco de peritoneo
invasiva, pero que en general cumplen los mismos principios parietal parcial, cuya pared posterior
quirúrgicos que son: reducción de la hernia hiatal, cierre del está formada por el estómago
hiato esofágico, restablecer la función del EEI, reposicionar el Paraesofágica o tipo 2 Desplazamiento superior del fondo gás-
esófago intraabdominal y crear un mecanismo de válvula an­ trico, anterior y lateral al esófago, con
tirreflujo. En aquellos casos de acortamiento esofágico debe la unión esofagogástrica localizada en
realizarse una gastroplastia de Collis para alargar el esófago y su posición intraabdominal normal
posteriormente la funduplicatura total o parcial, sin tensión Mixta o tipo 3 Desplazamiento superior tanto de la
sobre el esófago. Cuando la indicación quirúrgica es por RGE, unión esofagogástrica como del fondo
gástrico
todos estos procedimientos ofrecen un alivio de los síntomas

702
SECCIÓN 11 GASTROENTEROLOGÍA @
de reflujo en 80 a 100% de los pacientes, incluso en seguimien­ alimentación como la bulimia, aunque en ocasiones puede de­
to a largo plazo (entre 80 y 90% a diez años). berse a un toser con extremada fuerza, así como en otras situa­
ciones como una obstrucción importante por alimentos. Pue­
de causar neumomediastino y mediastinítis, por lo que es una
patología potencialmente grave.
SÍNDROME DE MALLORY-WEISS

6. ¿En qué porcentaje está presente el antecedente


de vómito en los pacientes con desgarro de Mallory­
10. Al ingreso de un paciente con sospecha de sín­
Weiss?
drome de Boerhaave, ¿cuáles son los diagnósticos
Aunque existe la idea de que el desgarro de Mallory-Weiss se diferenciales?
debe al esfuerzo del vómito, este antecedente sólo está presen­
te en 29% de los pacientes. Existen factores predisponentes: El diagnóstico diferencial de la perforación esofágica es muy
hábitos alcohólicos, desórdenes alimentarios, ingestión de sa­ amplio; el primer paso para realizarlo es considerar la posibi­
licilatos y la presencia de hernia hiatal y otras lesiones gastro­ lidad de su existencia como causa de dolor torácico potencial­
intestinales que causen vómito o náusea. Factores desencade­ mente mortal, sobre todo si se descartan otros procesos más
nantes: situaciones que aumenten la presión intraabdominal, frecuentes. Debe diferenciarse en las primeras horas de otras
como náusea, vómito, tos, convulsiones epilépticas, masaje car­ entidades como úlcera perforada, infarto agudo del miocardio,
diaco externo, trauma abdominal o esfuerzo al defecar. disección aórtica, pancreatitis aguda, pericarditis, neumotórax
espontáneo, hernia diafragmática y embolismo pulmonar, en­
tre otros.
7. ¿En qué porcentaje se autolimita la hemorragia di­ El método diagnóstico ideal para la perforación esofágica
gestiva alta asociada con desgarro de Mallory-Weiss? espontánea es el esofagograma con material hidrosoluble, que
El sangrado por desgarro de Mallory-Weiss se autolimita en 80 es positivo en 75 a 90% de los casos con este padecimiento.
a 90% de los casos. Menos de 5% de los pacientes presenta nue­
vo episodio de sangrado (sobre todo aquellos con anteceden­
tes de diátesis hemorrágica). En la mayoría de los casos la he­ 11. ¿Cuál es el tratamiento de elección para un pa­
morragia se detiene de forma espontánea después de 24 a 48 ciente con perforación del tercio inferior del esófa­
horas y los desgarros suelen cicatrizar en diez a doce días sin go y neumomediastino, confirmados por esofago­
ningún tratamiento especial. grama?
El tratamiento de la perforación esofágica espontánea es con-
8. ¿Cuál es el tratamiento para el desgarro de Ma­ trovertido, respecto a la elección de la técnica quirúrgica; el 1
llory-Weiss sin evidencia de sangrado activo? tiempo transcurrido desde el inicio del cuadro es uno de los
factores decisivos. Cuando el diagnóstico se establece en las
En la mayoría de los casos la hemorragia se detiene de forma
primeras 24 horas se recomienda el cierre primario de la le-
espontánea después de 24 a 48 horas y los desgarros suelen
sión, acompañado de drenaje mediastínico. Si la intervención
cicatrizar en diez a doce días sin ningún tratamiento especial.
no se efectúa en este periodo la existencia de edema, necrosis
Pueden administrarse inhibidores de la secreción gástrica (en
tisular e infección mediastínica dificultan en gran medida la
particular inhibidores de la bomba de protones). Si la hemo­
cirugía. La complicación más importante del cierre primario
rragia persiste se hace necesaria la transfusión sanguínea y el
es la aparición de fístulas, que pueden afectar hasta a 39% de
tratamiento endoscópico incluye métodos térmicos (cauteriza­
los pacientes intervenidos. Con el fin de disminuir su inciden-
ción, fotocoagulación), la inyección de epinefrina o hemoclips.
cia se han desarrollado las técnicas de cierre reforzado con
En muy raras ocasiones se utiliza la embolización de las arte­
pleura, diafragma, pericardio o fundus gástrico.
rias que irrigan la zona afectada. El tratamiento quirúrgico es
Otras técnicas que se han utilizado son la exclusión esofá­
excepcional y puede presentarse cuando el desgarro es mayor
gica, la implantación de una endoprótesis y la resección eso­
de 2 centímetros, ya que puede causar irritación del mediastino
fágica, indicada cuando el estado del esófago o la existencia de
y derrame pleural. Sin evidencia de sangrado por endoscopia
patologías subyacentes permiten anticipar que el cierre de la
se da tratamiento de soporte al paciente y puede egresarse de
perforación será difícil. Puede intentarse un tratamiento con­
forma temprana.
servador en perforaciones pequeñas, con sintomatología míni­
ma y lesiones limitadas al mediastino. Se han descrito casos
ROTURA ESOFÁGICA (PERFORACIÓN anecdóticos de perforaciones graves en las que el paciente so­
ESOFÁGICA O SÍNDROME DE BOERHAAVE) brevivió sin cirugía. Además de la corrección quirúrgica, el tra­
tamiento de soporte con alimentación parenteral, sonda na­
9. ¿Qué es el síndrome de Boerhaave? sogástrica, antibióticos de amplio espectro e inhibidores de la
El síndrome de Boerhaave es una rotura de la pared esofágica, secreción ácida gástrica desempeña una función importante
por lo general causada por vómito excesivo en trastornos de la en estos pacientes.

703
e MANUAL PARA EL EXAMEN NACIONAL DE RESIDENCIAS MÉDICAS

HEMATOMA INTRAMURAL ESOFÁGICO 17. ¿Cuál es la diferencia entre membranas y anillos


esofágicos?
12. ¿Cuál es la etiología del hematoma intramural eso­
Las membranas no suelen circunscribir toda la luz esofágica,
fágico en la mayoría de los pacientes? sino que se extienden de forma lateral sin llegar hasta la pared
Los hematomas intramurales esofágicos asociados con trastor­ posterior. Los anillos esofágicos son una banda ancha y simé­
nos de la coagulación ocurren en pacientes que poseen condi­ trica que constriñe la luz esofágica.
ciones de base que predisponen el sangrado, como es el caso
de pacientes hemofílicos, con leucemia, trombocitopenia o en­ 18. ¿Cuáles estudios son útiles para elaborar el diag­
fermedad renal o por uso de medicamentos que afecten la he­ nóstico?
mostasia, como son los anticoagulantes, antiplaquetarios y
agentes trombolíticos, lo cual se reporta en más de un tercio Panendoscopia y esofagograma.
de los casos.
19. ¿Cuál es el tratamiento de los anillos y las mem­
branas esofágicas cuando provocan síntomas?
13. ¿Cuál es el estudio de elección para confirmar la
sospecha de hematoma intramural esofágico?
Dilatación endoscópica con dilatadores de diámetro creciente
(50 F).
La tomografía computarizada es el examen de elección para
la evaluación de la pared esofágica, la aorta y otras estructuras
CUERPOS EXTRAÑOS
del mediastino. Es un examen ampliamente disponible, rápido,
no invasivo, que permite diferenciar la patología esofágica de 20. ¿Qué medida (longitud) tienen los objetos extra­
otros procesos torácicos. Además permite definir características ños que no atraviesan el píloro?
del hematoma, como el compromiso concéntrico o excéntrico de
Se recomienda extraer por endoscopia los cuerpos extraños
la pared y definir bien los limites, que por lo regular se extienden
que tienen una longitud mayor o igual a 6 centímetros y un
hacia la pared posterior, además de definir su extensión.
diámetro mayor a 2.5 centímetros por el riesgo de que no atra­
viesen el píloro.
14. ¿Cuál es el tratamiento recomendado para el he­
matoma parietal esofágico? 21. ¿Cuáles son las complicaciones secundarias a la
El manejo conservador con observación continua y soporte es ingesta de cuerpos extraños?
la recomendación actual. Al inicio se suministrará una dieta El atrapamiento de asas se presenta con la ingesta de imanes;
líquida y a las 72 horas se cambiará por dieta blanda. En caso la absorción de narcóticos que se degluten para tráfico ilegal
de no tolerancia a la vía oral se iniciará alimentación parente­ es una indicación de extracción quirúrgica. La necrosis por
ral. Los medicamentos procinéticos no se recomiendan y los coagulación se relaciona con ingesta de ácidos. En pacientes
inhibidores de la secreción gástrica se han sugerido para dismi­ en quienes se documente ingesta de baterías, cuando éstas se
nuir el riesgo de ulceraciones esofágicas. En pacientes con al­ encuentran en el esófago, debe realizarse extracción por en­
teraciones de la coagulación, éstas deben corregirse. No son doscopia ya que generan necrosis por licuefacción.
necesarios los controles endoscópicos posteriores y, cuando
se han realizado, mostraron curación completa de la pared eso­ 22. ¿Cuáles son las indicaciones de endoscopia
fágica, ya que los hematomas se resuelven en un promedio de cuando hay ingestión de cuerpos extraños?
diez a catorce días. La necesidad de manejo quirúrgico es rara
Véase cuadro 11-3-3.
y se requiere en menos de 15% de los casos. Puede ser necesa­
rio en caso de perforación o sangrado incontrolable, lo cual es BIBLIOGRAFÍA
poco frecuente en la evolución.
Achkar E. Esophageal diverticula. Gastroenterol Hepatol (NY). 2008 (10):4.
Evan S, Dellon MD etal. ACG clinical guideline: Evidenced, Based Approach
to the Diagnosis and Management of Esophageal Eosinofilia and Eosi­
MEMBRANAS Y ANILLOS ESOFÁGICOS
nophilic Esophagitis. Am J Gastroenterol. 2013; 108:679-92.
Feldman Metal. Gastrointestinal and Liver Disease. Elsevier. 2008(1): 465-9.
15. ¿Cuál es la localización más frecuente del anillo Gawrieh S. Treatment of actively bleeding Mallory-Weiss syndrome: epine­
de Schatzki y cuál es su frecuencia? phrine injection or band ligation? Curr Gastroenterology Rep. 2005 Jun;
Se localiza en la unión del vestíbulo y el cardias y su frecuen­ 7(3): 175.
Gutiérrez A etal. Perforación esofágica espontánea o síndrome de Boerhaave.
cia se observa en 6 a 14% de los casos. Una causa de dolor torácico potencialmente mortal. Emergencias. 1998;
( 10): 196-9.
16. ¿Cuáles son los componentes del anillo de Higuchi N. Endoscopic band ligation therapy upper gastrointestinal bleeding
to Mallory-Weiss syndrome. Surg Endose. 2006 Sep; 20(9): 1431-4.
Schatzki? Kahrilas P, IQm H. Pandolfino J. Approaches to the Diagnosis and Grading
Mucosa y submucosa. of Hiatal Hernia. Bes\ Pract Res Clin Gastroenterol. 2008:(22)4;601-16.

704
SECCIÓN 11 GASTROENTEROLOGÍA @
CUADRO 11-3-3. Long JO, Orlando RC. Anatomía, histología, embriología y anomalías del
INDICACIONES DE ENDOSCOPIA ANTE LA INGESTIÓN desarrollo del esófago. En: Feldman M, Friedman LS, Brand! LJ (eds.).
Sleisenger y Fortran's Gastrointestinal y la enfermedad hepática, 9a. ed.
DE CUERPOS EXTRA�OS
Philadelphia, Pa: Saunders Elsevier; 2012. pp. 40.
IMfoMfüii
Pacientes con
URGENTE NO URGENTE Madriz W et al. Diagnóstico radiológico de la hernia hiatal. Revista Médica
de Costa Rica y Centroamérica. 2008:(583): 103-8.
Objetos no Monedas en el esófago pue­
obstrucción puntiagudos en den ser observadas por 12 Martínez J et al. Síndrome de Boerhaave. Reporte de un caso y revisión de la
esofágica esófago a 24 horas antes de ser re­ literatura. Rev. Gastroenterol Mex. 2002;(67):190-4.
Baterías de Obstrucción movidas por endoscopia, Meléndez H. Síndrome Boerhaave. Una causa infrecuente de perforación esofá­
disco en esofágica en pacientes asintomáticos gica. Revista médica de Costa Rica y Centroamérica. 2012;(603):367-71.
esófago incompleta de Objetos en el estómago con Menachem B. Management of ingested foreign bodies and food impactions.
Objetos pun- alimento diámetro mayor a 2.5 cm Gastrointestinal Endoscopy. 2011;73: 1085-91.
tiagudos Objetos puntiagudos Baterías de disco y cilíndri­ Mitchell S. lnitial management UGIB. Med Clin N Am. 2008(92):491-509.
en esófago en estómago de cas que estén en el estó­ Pardo J et al. Hematoma masivo esofágico con compromiso supraglótico: re­
longitud mayor a mago se pueden observar porte de caso y revisión de la literatura. Rev Otorrinolaringol Cir Cabeza
6cm durante 48 horas y, si per­ Cuello. 2010;70:237-44.
Imanes manecen ahí, se deben Pfau PR, Ginsberg GG. Cuerpos extraños y bezoares. En: Feldman M, Friedman
extraer LS, Brand! LJ (ed). Sleisenger and Fortran's Gastrointestinal and Liver
Disease Textbook. 9th ed. Philadelphia, Pa: Saunders Elsevier; 2012. pp. 23.
Tamatey M et al. Boerhaave's syndrome: diagnosis and successful primary re­
Karelovic S, Liberen B, Mijac N, Badilla N, Wipe B. Hematoma intramural pair one month after the oesophageal perforation. Ghana Medica! Journal.
esofágico: Caso clínico. Gastr Latinoam. 2007;18:319-22. 2013;(47):53-5.
Lombardi J et al. Hematoma intramural esofágico. Rev. Chilena de Cirugía. Yamada T et al. Atlas of Gastroenterology. 4th ed. Philadelphia: Lippinco­
2012;(64):68-71. tt-Raven; 2009. pp. 749-60.

CAPÍTULO 11.4 1
TUMORES ESOFÁGICOS
Hairé Manzano Cortés

TUMORES ESOFÁGICOS BENIGNOS CUADRO 11-4-1.


CLASIFICACIÓN DE LOS TUMORES ESOFÁGICOS
1. ¿Como se dividen los tumores esofágicos?
TUMORES EPITELIALES TUMORES NO EPITELIALES
Los tumores esofágicos constituyen lesiones benignas o malig­
Malignos Malignos
nas (cuadro 11-4-1) que por lo regular se manifiestan como ma­
Carcinoma de células escamosas Linfoma
sas intraluminales que pueden reducir la luz esofágica, así co­ Adenocarcinoma Sarcoma, incluyendo GIST
mo invadir estructuras mediastínicas. Adenocarcinoma de la unión Carcinoma metastásico
gastroesofágica
Carcinoma verrucoso
2. ¿Qué tipos de tumores esofágicos existen según
Carcinosarcoma
su origen histológico? Carcinoma de células pequeñas
Véase cuadro 11-4-2. Melanoma
Benignos Benignos
3. ¿Cómo se presentan los tumores benignos? Papiloma escamoso GIST
Adenoma Leiomioma
Los tumores benignos son poco frecuentes y la mayoría son Pólipo fibroide inflamatorio Tumor de células granulares
asintomáticos por su tamaño pequeño. Se originan de las es­ Tumor fibrovascular
tructuras de la pared esofágica ( cuadro 11-4-1) y comprome­ Hemangioma
Hamartoma
ten la luz esofágica y/o la peristalsis. La mayoría es asintomá-

705
e MANUAL PARA EL EXAMEN NACIONAL DE RESIDENCIAS MÉDICAS

CUADRO 11-4-2. de tejido conectivo vascular, cubierto de epitelio escamoso es­


ORIGEN HISTOLÓGICO DE LOS TUMORES ESOFÁGICOS tratificado. Se relaciona con ERGE y virus del papiloma hu­
mano (VPH). Estos papilomas presentan síntomas tardíos por
Epitelial Papilomas, pólipos adenomatosos,
crecimiento lento, pero ante disfagia o imposibilidad para ex­
pseudopólipos inflamatorios
cluir malignidad el tratamiento es la extirpación endoscópica
Muscular Leiomiomas (más frecuentes)
Rabdomiomas en el tercio superior
o quirúrgica.
Tejidos conjuntivos Pólipos fibrovasculares. hemangiomas,
tumores de células granulosas (tumor de 8. ¿Cuáles son las características del leiomioma?
Abrikossoff), GIST (tumor estroma! El leiomioma es el tumor esofágico más frecuente; 90% se lo­
gastrointestinal)
caliza en tercio inferior del esófago; pueden ser únicos o múl­
Sistema nervioso Schwannomas, neurinomas tiples; son tumores intramurales por lo general submucosos,
recubiertos de epitelio escamoso y se presentan con mayor
frecuencia en la tercera a quinta décadas de la vida.
tico y según el tamaño y la localización los síntomas pueden
Su tamaño puede ser hasta de 10 cm. La mayoría es asinto­
ser dolor esofágico, malestar retroesternal, disfagia y asfixia en
mático por años y en casos sintomáticos puede provocar disfa­
casos de obstrucción de vía aérea; con menor frecuencia pre­
gia y dolor retroesternal en 50% de los casos, así como regur­
sentan hemorragia que se manifiesta como melena o hema­
gitación, alteraciones en la motilidad, pseudoacalasia y reflujo.
temesis.
Las biopsias no suelen ser diagnósticas, por lo que el ultraso­
nido endoscópico es el mejor método diagnóstico. El trata­
4. ¿Cuáles son los tipos de cáncer de esófago más miento ante la presencia de síntomas es quirúrgico.
frecuentes?
Los tumores malignos constituyen la mayoría de los tumores 9. ¿Cuáles son las características del tumor de célu­
esofágicos; los dos tipos de carcinomas más frecuentes son el las granulosas (tumor de Abrikosov)?
cáncer de células escamosas y el adenocarcinoma.
El tumor de células granulosas se origina en la submucosa y su
histogénesis no está bien precisada. Se considera que su ori­
5. ¿Cuáles son las manifestaciones o forma de pre­
gen estaría en las células de Schwann por su estructura y dar
sentación de los tumores esofágicos?
reacción con la proteína S-100 y la enolasa neurona específica.
En un inicio los tumores de esófago producen escasas manifes­ Afecta la región del cuello y la cabeza, con la lengua como
taciones clínicas. El síntoma inicial más frecuente es la disfa­ uno de los órganos más afectados. En esófago afecta a ambos
gia, que suele ser rápidamente progresiva, tanto a sólidos co­ sexos y la ubicación más frecuente es el tercio inferior. Son tu­
mo a liquidos. Este síntoma es consecuencia de la reducción mores pequeños de alrededor de 1 cm de diámetro. Son de cre­
del calibre de la luz esofágica y puede ocasionar episodios de cimiento lento y muchas veces se mantienen estacionarios o
impactación alimentaria y regurgitación. Otras manifestacio­ son diagnosticados por hallazgo; un síntoma tardío es la dis­
nes clínicas incluyen odinofagia, anorexia y pérdida de peso y, fagia y su aspecto endoscópico es un pólipo sésil cubierto por
con menor frecuencia, tos, náusea, vómito y hemorragia diges­ mucosa pálida blanquecina. La ecoendoscopia permite la vi­
tiva. El dolor retroesternal y referido a la región dorsal suele sualización y su origen en la subsubmucosa. El tratamiento es
indicar invasión mediastínica. endoscópico en lesiones pequeñas y si son lesiones mayores
es quirúrgico.
6. ¿Cuáles son los estudios diagnósticos iniciales an­
te sospecha de tumor esofágico? 10. ¿Cuáles son las características de los pólipos fi­
El diagnóstico incluye esofagograma, sobre todo en casos de brovasculares?
disfagia o en contraindicación de endoscopia; sin embargo, el Por lo general ocurren en hombres y se presentan en tercio su­
estudio endoscópico permite la visualización del tumor, así co­ perior esofágico por debajo del cricofaríngeo, de tamaño varia­
mo la toma de biopsias; más adelante debe realizarse ultraso­ ble; es una formación polipoide alargada con pedículo extenso
nido endoscópico para valorar la localización y extensión en que permite su movilidad tanto a cavidad bucal como a esófa­
la pared esofágica. En lesiones pequeñas puede realizarse tomo­ go y vías respiratorias produciendo obstrucción.
grafía y, si son vasculares, angiografía. El tratamiento depende La mucosa que lo recubre puede presentar erosiones o ul­
de los síntomas y puede ser endoscópico o quirúrgico. ceraciones, sobre todo cuando las lesiones llegan al cardias y
existe reflujo gastroesofágico. Histológicamente es una forma­
7. ¿Cuáles son las características del papiloma de cé­ ción recubierta por mucosa y en su interior hay tejido conjun­
lulas escamosas? tivo, adiposo y vasos sanguíneos.
Los papilomas de células escamosas son tumores pequeños, El esofagograma muestra dilatación del esófago y formación
blancos o rosáceos, sésiles o polipoides, con un núcleo central intraluminal que cambia de posición y puede mostrar el pe-

706
SECCIÓN 11 GASTROENTEROLOGÍA @
dículo y su implantación; la endoscopia con toma de biopsias rragia digestiva. El dolor retroesternal y referido a la región dor­
en algunos casos no puede realizarse por la lesión, pero la to­ sal suele indicar invasión mediastinica.
mografía permite valorar su localización. La mitad de los pacientes diagnosticados con cáncer de esó­
El tratamiento puede ser por vía endoscópica o quirúrgica. fago presentan metástasis a distancia o tumores irresecables
en el momento del diagnóstico.
11. ¿Cuáles son las características de un hemangioma?
Tumores poco frecuentes que se originan en la submucosa y 15. ¿Cuál es el riesgo anual de cáncer esofágico?
protruyen hacia la luz dando lugar a pseudopólipos. La disfa­ Alrededor de 0.25% para pacientes sin displasia y 6% para pa­
gia es tardía porque son tumores blandos que no llegan a obs­ cientes con displasia de alto grado.
truir la luz pero pueden provocar hemorragias.
La endoscopia muestra una formación azul grisácea, fácil­ CARCINOMA EPIDERMOIDE DE ESÓFAGO
mente compresible; se contraindica la biopsia, por lo que debe
realizarse ultrasonido endoscópico para valorar su localiza­ 16. ¿Cuáles son las características del carcinoma epi­
ción y precisar el origen vascular; la TC contrastada también dermoide de esófago?
determina su vascularidad. Tumor esofágico maligno más frecuente en raza negra, se pre­
El tratamiento es la resección quirúrgica en la mayoría de senta más en hombres que en mujeres.
los casos por riesgo de hemorragia.
17. ¿Cuáles son los factores de riesgo para carcino­
12. ¿Cuáles son las características de los lipomas? ma escamoso?
Son muy raros y pueden ubicarse en la submucosa y en la capa Entre los factores de riesgo para carcinoma escamoso se
muscular y son sésiles o pediculados, rara vez son pequeños y encuentran tabaco, alcohol, nitrosaminas, ingesta de cáusti­
causan síntomas como disfagia o hemorragia. cos, cáncer de cabeza y cuello, acalasia, tilosis, síndrome de
El diagnóstico se sospecha con rayos X al ver una formación Plummer-Vinson, anemia de Fanconi, esclerodermia, asbesto­
clara, por lo general delimitada, o en la tomografía que mues­ sis, historia de cáncer de mama tratado con radioterapia, nive­
tra una formación con la densidad del tejido adiposo, lo que lo les disminuidos de selenio y deficiencia de folatos.

1
diferencia de otros tipos de tumores, pero el diagnóstico de
elección es el ultrasonido endoscópico. 18. ¿Qué es la tilosis?
El tratamiento es quirúrgico si son sintomáticos. Enfermedad hereditaria de carácter autosómico dominante cu-
ya anormalidad se localiza en el cromosoma 17q25; también
TUMORES ESOFÁGICOS MALIGNOS llamada queratoderma palmoplantar no epidermolitica que se
caracteriza por hiperqueratosis de palmas y plantas y engrosa-
13. ¿Cuáles son los tumores esofágicos malignos más miento de la mucosa oral. Los pacientes con esta patología
y menos frecuentes? requieren vigilancia endoscópica, ya que 95% desarrolla cán-
La mayoría de los tumores esofágicos son malignos, y de éstos cer epidermoide a los 65 a 70 años de edad.
los más frecuentes son el adenocarcinoma y el carcinoma es­
camoso; sin embargo, de estos dos tumores el más frecuente es 19. ¿Qué es el síndrome de Plummer-Vinson?
el carcinoma escamoso (epidermoide), aunque en los últimos Presencia de membranas en esófago superior, suele incluir ane­
años se ha incrementado la incidencia de adenocarcinoma. mia y glositis. Uno de cada diez pacientes presenta cáncer
Existen otros tumores malignos que se presentan en raras epidermoide.
ocasiones como el leiomiosarcoma, el sarcoma de Kaposi, los
tumores de células pequeñas, el linfoma y el melanoma. Tam­ 20. ¿Cuáles son los síntomas más frecuentes de este
bién se pueden presentar tumores metastásicos (mamario, pul­ síndrome?
món, melanoma). Disfagia y síndrome de consumo; con menor frecuencia odino­
fagia, fístula traqueoesofágica (tos) y disfonía (afección del ner­
14. ¿Cuál es la presentación clínica de los tumores vio laríngeo recurrente).
malignos de esófago?
De manera inicial las manifestaciones clinicas son mínimas, 21. ¿De qué porción se deriva con mayor frecuencia
pero el sintoma inicial más frecuente es la disfagia, que suele ser el carcinoma de células escamosas?
rápidamente progresiva, tanto a sólidos como a liquidos, como Del esófago medio con invasión temprana de la submucosa,
consecuencia de la reducción del calibre esofágico y puede oca­ así como de los ganglios linfáticos regionales. La invasión di­
sionar episodios de impactación alimentaria y regurgitación. recta puede producir hemorragia masiva cuando involucra
Otras manifestaciones incluyen odinofagia, anorexia y pérdida la aorta o aspiración por desarrollo de fístula traqueoesofá­
de peso y, con menor frecuencia, tos, náusea, vómito y hemo- gica.

707
e MANUAL PARA EL EXAMEN NACIONAL DE RESIDENCIAS MÉDICAS

ADENOCARCINOMA DE ESÓFAGO ticos es temprana y puede haber invasión directa de estructu­


ras adyacentes.
22. ¿Cuál es la incidencia del adenocarcinoma de
esófago? 28. ¿Cuáles son los genes relacionados en el desa­
Es más frecuente en hombres con edad pico en la sexta déca­ rrollo del adenocarcinoma?
da de la vida. En los últimos años y en algunos países ha incre­ De manera temprana puede haber una serie de mutaciones en
mentado su incidencia hasta ser similar a la del cáncer epider­ el gen pl6 que codifica un inhibidor de la progresión del ciclo
moide. celular y mutaciones en el genp53 (antes de la malignización),
el cual codifica una proteína supresora de tumores involucra­
23. ¿Cuál es el principal factor de riesgo? da en el ciclo celular y en la reparación del ADN. La aneuploi­
El esófago de Barrett se considera el principal factor y por ello día es frecuente en el adenocarcinoma esofágico y puede incluir
se requiere vigilancia endoscópica periódica. la mutación del gen K-RAS, así como la expresión de factores
de crecimiento endotelial.
24. ¿Cuáles son los factores de riesgo para cáncer
29. ¿Cuál es la presentación clínica característica del
en pacientes con esófago de Barrett?
adenocarcinoma?
ERGE crónico, hernia hiatal, edad avanzada, sexo masculino,
raza blanca, tabaquismo y obesidad con distribución central. Historia de síntomas de reflujo seguido de disfagia progresiva
y pérdida ponderal.
25. ¿Qué otros factores de riesgo pueden condicio­
30. ¿Cuál es el procedimiento diagnóstico de elec­
nar adenocarcinoma?
ción para el cáncer esofágico?
ERGE sintomático y crónico, obesidad, tabaco e historia de
Los estudios baritados son los iniciales en caso de disfagia; sín
cáncer de mama tratado con RT.
embargo, la esofagogastroduodenoscopia (EGD) con biopsia
es el procedimiento de elección para identificar lesiones tem­
26. ¿Cuál es la relación que tiene el adenocarcino­
pranas y determinar la estirpe histológica.
ma con He/icobacter py/ori? El ultrasonido endoscópico (USE) es el mejor estudio para
Se considera una relación inversa; aunque aún hay controver­ etapificar la enfermedad local y regional y determinar la profun­
sia, se considera que la presencia de Helicobacter pylori reduce didad de la invasión y la afección de los ganglios linfáticos, en
el riesgo de adenocarcinoma. Este efecto protector se deriva especial de los ganglios celiacos (más frecuente en adenocar­
de la disminución de ácido gástrico en la infección por Helico­ cinoma). Si se combina con aspiración con aguja fina (AAF),
bacter, lo cual reduce la frecuencia de ERGE y minimiza la la sensibilidad para detectar metástasis regionales es superior
formación subsecuente de metaplasia de Barrett y adenocarci­ a 85%.
noma. La tomografía puede etapificar estadios avanzados o detec­
tar estenosis de alto grado o fístulas, pero puede pasar por alto
27. ¿De cuál porción se deriva con mayor frecuencia cánceres tempranos. En casos de sospecha de enfermedad avan­
el adenocarcinoma de esófago? zada y sin evidencia de estadio III-IV (véase cuadro 11-4-3) por
Deriva del esófago distal y tiene características de tipo intesti­ USE, la tomografía por emisión de positrones (PET) es útil
nal con producción de mucina. La metástasis a ganglios linfa- para valorar la resecabilidad.

CUADRO 11-4-3.
ESTADIFICACIÓN DE CÁNCER ESOFÁGICO
TUMOR (T)
TO: sin tumor primario
GANGLIOS (N)
NO: sin ganglios
METÁSTASIS (M)
MO: sin evidencia de metástasis
IHiMi·
o
I COMPONENTES DEL ESTADIO
Tis, NO, MO

Tis: carcinoma in situ N1: evidencia de afección de M1: evidencia de metástasis T1, NO, MO
ganglios linfáticos

T1: invade lámina propia o la NX: ganglios linfáticos no MX: metástasis no valoradas 2a T2-3, NO, MO
submucosa valorados

T2: invade muscularis propia 2b T3, N1, MO

T3: invade adventicia 3 T3, N1, MO


T4, cualquier N, MO

T4: invade estructuras 4 Cualquier T y N, M1


adyacentes

708
SECCIÓN 11 GASTROENTEROLOGÍA @
31. ¿Cuál es el tratamiento de elección para el cán­ BIBLIOGRAFÍA
cer de esófago? Galindo, Marzano, De la Torre. Tumores benignos de esófago. Cirugía Diges­
tiva, F. Galindo. 2009;1( 180): 1-8. Disponible en: http://www. sacd.org.ar/
Para los estadios I y II, tanto del adenocarcinoma como del
uochenta.pdf. Consultado el 16 de enero de 2016.
carcinoma de células escamosas, el tratamiento de elección es Hammoud GM, Hammad H, Ibdah JA. Endoscopic assessment and manage­
la cirugía radical y la selección para realizarla depende de la ment of early esophageal adenocarcinoma. World J Gastrointest Oncol.
localización del tumor y si hubo cirugía o radioterapia previa. 2014 Aug l5;6(8):2752-88. doi: I0.4251/wjgo.v6.i8.275.
Inclusive si no hay evidencia de diseminación se requiere una Kruszewski WJ. Endoscopic methods in the treatment of early-stage esopha­
geal cancer. Wideochir lnne Tech Malo lnwazyjne. 2014 Jun;9(2): 125-130.
evaluación cuidadosa, ya que la mortalidad y la morbilidad pos­
doi: 10.5114/wiitm.2014.41620. Epub 2014 Mar 24.
operatorias son altas: 10 y. 75%, respectivamente. En cáncer Sobrino Cosío. Adenocarcinoma de esófago. Rev Gastroenterol Mex. 2008;
esofágico temprano, si está disponible, puede realizarse la di­ (73)2: 13-20.
sección de la mucosa y también es una alternativa si existen Sun F, Chen T, Han J, Ye P, Hu J. Staging accuracy of endoscopic ultrasound
contraindicaciones quirúrgicas. for esophageal cancer after neoadjuvant chemotherapy: a meta-analysis
and systematic review. Dis Esophagus. 2014 Aug 29. doi: 10.111 !/dote.
La quimiorradiación neoadyuvante puede mejorar la sobre­
12274. [Epub ahead of print]
vida a corto plazo sobre la cirugía en estadios II-IV. La mayo­ van Rossum PS, van Lier AL, Lips IM, Meijer GJ, Reerink O, van Vulpen M
ría de los pacientes no es candidato quirúrgico y sólo requiere et al. lmaging of oesophageal cancer with FDG-PET/CT and MRI. Cliní­
medidas paliativas. El cisplatino y el fluorouracilo constituyen cal Radiology. Clin Radio!. 2015 Jan;70( 1):81-95.
la terapia paliativa estándar en enfermedad irresecable. Wang AH et al. Epidemiological studies of esophageal cancer in the era of
genome-wide association studies. World J Gastrointest Pathophysiol.
Las estenosis esofágicas pueden tratarse con stent metálico
2014 Aug 15;5(3):335-343. doi: 10.429l/wjgp.v5.i3.335.
autoexpandible cubierto con una membrana o dilataciones eso­
fágicas repetidas.
Existen terapias adicionales que permiten disminuir el volu­
men del tumor, como la terapia fotodinámica, la radioterapia,
la terapia endoscópica con láser y la coagulación bipolar.

CAPÍTULO 11.5
GASTRITIS: AGUDA EROSIVA Y CRÓNICA
1
Axe/ Mena Quintero

GASTRITIS AGUDA EROSIVA 4. ¿Qué fármacos son más efectivos para el manejo
agudo de estos pacientes y para prevenir las recu­
1. ¿Cuáles son los tres principales factores de riesgo rrencias?
para la gastritis aguda erosiva? Tanto en el manejo agudo como en la prevención de recurren­
El uso de AINE; la edad avanzada y el antecedente de úlcera cias los inhibidores de bomba ( omeprazol, lanzoprazol, seo­
péptica. meprazol) son más eficaces que los bloqueadores H 2 .

2. ¿Cuál es el estudio de elección para abordar una '5) http://www.cenetec.salud.gob.mx/descargas/gpc/


hemorragia de tubo digestivo alto? CatalogoMaestro/516_GPC_Gastritisagudaerosiva/GPC_EyR_
GASTRITIS_EROSIVA.pdf
El estudio de elección para la hemorragia del tubo digestivo
alto es la esofagogastroduodenoscopia.
GASTRITIS CRÓNICA
3. ¿Qué pacientes deben tratarse de urgencia?
Aquellos que tienen sangrado activo manifestado como mele­ 5. ¿Cómo se define la gastritis por tiempo?
na o hematemesis dentro de las primeras 24 horas de presen­ La gastritis es la inflamación de la mucosa gástrica; en general
tación y un puntaje de Rockall mayor de 2. Véase cuadro 11- se acepta el término de aguda para aquella de inicio abrupto.
6-2 en el capítulo siguiente. El espectro de la gastritis crónica es más difícil de definir por

709
e MANUAL PARA EL EXAMEN NACIONAL DE RESIDENCIAS MÉDICAS

la clínica y en general se refiere a hallazgos histológicos, de CUADRO 11-5-1.


manera que el infiltrado inflamatorio agudo suele caracterizar­ CAUSAS DE LA GASTRITIS
se por neutrófilos y el infiltrado inflamatorio crónico por célu­
AGUDA CRÓNICA
las mononucleares.
Fármacos Hipertensión portal
Alcohol Uremia
6. ¿Cuáles son los síntomas de la gastritis? Isquemia Helicobacter pylori
Náusea, dolor en epigastrio y vómito es lo más frecuente. Los Infecciones por hongos o virus, Autoinmunitaria
síntomas no correlacionan con los hallazgos endoscópicos. Helicobacter pylori Biliar
Radiación AINE
Alergias Linfocítica
7. ¿Cuáles son las causas de la gastritis? Estrés Granulomatosa (tuberculosis, en­
Véase cuadro 11-5-1. Trauma fermedad de Crohn)
AINE Eosinofílica
Isquemia
8. ¿Cómo se clasifica la gastritis según topografía e
inmunología?
• Tipo A: gastritis corporal o autoinmunitaria que conlleva el En caso de sospechar autoinmunidad se pueden ordenar
desarrollo de anemia perniciosa. pruebas serológicas con anticuerpos antifactor intrínseco, an­
• Tipo B: gastritis antral o por reflujo duodenogástrico. ticélula parietal, prueba de Schilling o niveles séricos de coba­
• Tipo C: gastritis inducida por fármacos. lamina.

9. ¿Cuáles son los factores protectores y agresores 14. ¿Cuál lesión se asocia con la gastritis atrófica au­
de la mucosa gástrica? toinmunitaria?
Véase cuadro 11-5-2. Con carcinoides gástricos por hiperplasia de células enterocro­
mafines.
10. ¿Qué es la gastritis autoinmunitaria?
15. ¿Con qué tipo de lesiones se asocia la gastritis
Corresponde a la antes llamada gastritis atrófica o tipo A, en
la cual se evidencia la presencia de anticuerpos dirigidos con­
atrófica de antro?
tra las células parietales. Hay hiperplasia de células enterocro­ Con metaplasia intestinal, la cual a su vez lleva al desarrollo de
mafines y un riesgo elevado de carcinoma gástrico. displasia y finalmente a cáncer gástrico de tipo intestinal.

11. ¿Por qué se asocia la anemia perniciosa con la 16. ¿Cuál es el tratamiento para la gastritis?
gastritis autoinmunitaria? En general el uso de inhibidor de bomba de protones es lo más
Por la ausencia de producción de factor intrínseco por las cé­ aceptado para la gastritis; sin embargo, las causas específi­
lulas parietales, lo cual disminuye la absorción de cianocoba­ cas, como la gastritis autoinmunitaria, pueden requerir suple­
lamina o vitamina B 12 . Es preciso decir que en la pangastritis mentos de cobalamina por vía parenteral. Para la gastritis por
crónica por H. pylori no existe esta deficiencia vitamínica. La gastroparesia o biliar los procinéticos desempeñan una fun­
presencia de anticuerpos dirigidos contra la célula parietal o ción importante en el tratamiento. Las causas infecciosas tie­
contra el propio factor intrínseco explica su deficiencia. nen un tratamiento encaminado a combatir al microorganis­
mo en cuestión.
12. ¿Qué sintomatología puede indicar una gastritis
BIBLIOGRAFÍA
autoinmunitaria?
Zimaity HE. Gastritis and gastric atrophy. Curr Opin Gastroenterol. 2008;
Básicamente tres tipos de síntomas según su origen:
24:682-6.
a) Por gastritis: epigastralgia urente, náusea y vómito.
b) Por anemia: vértigo o mareo, acúfenos, palpitaciones, ane­ CUADRO 11-5-2.
mia, trombocitopenia, datos de falla cardiaca. FACTORES PROTECTORES Y AGRESORES
e) Por déficit de cobalamina: glositis, pérdida de peso, entu­ DE LA MUCOSA GÁSTRICA
mecimiento, parestesias, irritabilidad, demencia o psicosis.
PROTECTORES AGRESORES

13. ¿Cómo se diagnostica la gastritis atrófica? Producción de prostaglandinas Fármacos (AINE)


Producción de moco Quimioterapia o radioterapia
Mediante la visión directa de la atrofia en biopsias de cuerpo y Reproducción de células de la Producción aumentada de ácido
fundus tomadas durante endoscopia o mediante pruebas in­ mucosa Estrés
directas, como son la medición de pepsinógeno l disminuido Flujo vascular adecuado Isquemia
Infección por Helicobacter pylori
( <20 ng/dL) o relación pepsinógeno 1/II disminuida.

710
SECCIÓN 11 GASTROENTEROLOGÍA @
CAPÍTULO 11.6
ENFERMEDAD ULCEROSA PÉPTICA
Carlos Eduardo Samaniego Chávez • Brenda Arace/y Herrera Chi

ÚLCERA PÉPTICA al paciente. En caso de hemoglobina menor a 7 g/dL se reque­


rirá transfusión. En caso de pacientes con enfermedad coro­
1. ¿Cómo se define una úlcera péptica? naria conocida o EPOC se deben alcanzar cifras mayores de
Se define como una solución de continuidad en la mucosa del hemoglobina para evitar complicaciones debidas al síndrome
estómago, con medida mayor a 5 mm (lesiones menores se con­ anémico.
sideran erosiones), que penetra la muscular de la mucosa.
8. ¿Bajo qué condiciones clínicas se clasifica al pa­
2. ¿Cuáles son los factores de riesgo más comunes ciente como de bajo riesgo y puede considerarse
de una úlcera péptica? manejo ambulatorio?
Infección por H pylori, ingesta de AINE, tabaquismo, alcoholis­ En caso de BUN (nitrógeno ureico) < 18.2 g/dL, hemoglobina
mo, uso de corticosteroides (en terapia conjunta con AINE), > 13 g/dL en hombres y> 12 g/dL en mujeres, tensión arterial
EPOC, cirrosis, uremia y estrés (quemaduras, posquirúrgico, sistólica mayor de 100 mm Hg, pulso menor de 100 1pm y au­
etcétera). sencia de sincope, melena, falla cardiaca o cirrosis puede con­
siderarse realizar una endoscopia programada, pues menos de
3. ¿Cuáles son las presentaciones de la úlcera péptica? 1 % de los pacientes requerirá intervención endoscópica o qui­
Como úlcera gástrica y úlcera duodenal. rúrgica para la resolución de su cuadro.

4. ¿Qué factores protegen a la mucosa gástrica en 9. ¿Cuál es la utilidad del lavado gástrico por sonda
un estado normal, para la prevención de formación nasogástrica en los pacientes con sospecha de san­
de úlcera péptica? grado por úlcera péptica?
La barrera mucosa (capa de moco y bicarbonato), células mu­ Aunque la presencia de sangre fresca o posos de café en el la­
cosas gástricas y caliciformes duodenales, el flujo sanguíneo vado aumenta la probabilidad de visualización de lesión en la

1
submucoso, la bicapa lipídica de células epiteliales y las pros­ endoscopia, hasta 18% de los lavados en que se obtiene conte­
taglandinas. nido biliar presenta una lesión con sangrado activo. El lava­
do con sondas de tamaño convencional no mejora la visuali­
5. ¿Cuál es la epidemiología de la úlcera péptica? zación durante la endoscopia y tampoco logra la detención
La úlcera gástrica predomina en hombres, mientras que la re­ del sangrado. En la actualidad el lavado gástrico no es una
lación hombre-mujer es igual en la úlcera duodenal. Es la cau­ herramienta útil para el sangrado digestivo alto.
sa más común de sangrado del tracto digestivo alto (hasta 50%
del total y entre 80 y 90% si se excluye al sangrado de várices 10. ¿Cómo debe realizarse el diagnóstico de la úlce­
esofágicas). La mayor parte de los casos (68%) se presenta en ra péptica?
personas mayores de 60 años de edad. Aunque antes se usaban estudios baritados, en la actualidad
el estándar de oro es la endoscopia digestiva alta, la cual ofre­
6. ¿Cuál es el cuadro clínico de la úlcera péptica? ce la posibilidad de clasificar a la úlcera según topografía, ta­
Para la úlcera gástrica el cuadro característico es epigastralgia maño y características (benigna o maligna), además de aplicar
que empeora con la ingesta de alimentos, con irradiación ha­ tratamiento endoscópico inmediato (inyección de sustancias
cia la espalda, náusea y/o vómito y anemia ferropénica. esclerosantes, colocación de hemoclips o métodos de coagula­
En el caso de la úlcera duodenal, epigastralgia no irradia­ ción) en caso necesario.
da, que inicia dos a tres horas después de la ingesta de alimen­
tos, que mejora con la ingesta de alimentos, antiácido y lác­ 11. ¿Cuál es el momento ideal para realizar la endos­
teos; 60% de los casos presenta dolor por las noches. copia en un sangrado con úlcera péptica?
Se considera que la endoscopia deberá realizarse dentro de
7. ¿Cuál es el abordaje inicial en un paciente con las primeras 24 horas desde que se establece el diagnóstico.
sospecha de úlcera péptica? En pacientes de alto riesgo (datos de hipotensión con tensión
Independientemente de que se sospeche sangrado activo o no, sistólica inicial menor de 100 mm Hg y pulso> 100 1pm, lava­
el estado hemodinámico debe ser valorado para descartar do gástrico positivo a sangre fresca y/o escala de Blatchford
choque hipovolémico y, en el caso de presentarlo, estabilizar mayor a doce puntos), la endoscopia debe realizarse en las

I 711
e MANUAL PARA EL EXAMEN NACIONAL DE RESIDENCIAS MÉDICAS

primeras doce horas para disminuir la morbimortalidad en el CUADRO 11-6-1.


paciente. ESCALA DE BLATCHFORD

Nitrógeno ureico en sangre (BUN mg/dl)


12. ¿Cuáles son las escalas pronósticas que existen 18.2 a 22.4 2
para evaluar al paciente? 22.4 a 28 3
La escala de Blatchford y la de Rockall (en su parte clínica y 28 a 70 4
Más de 70 6
luego con complementación endoscópica) son las más cono­
cidas. Hemoglobina para hombres (g/dl)

La escala de Blatchford es un sistema pronóstico para es­ 12 a 13 1


tratificación de riesgo que utiliza parámetros clínicos, de labo­ 10 a 12 3
Menos de 10 6
ratorio y hemodinámicos para identificar a los pacientes con
riesgo de intervención quirúrgica; así, los pacientes con tres Hemoglobina para mujeres (g/dl)

puntos o menos tienen un riesgo de 6% de requerir una in­ 10 a 12 1


tervención quirúrgica. Los pacientes con un puntaje mayor a Menos de 10 6

6 tienen un riesgo superior a 50% de requerir intervención Presión arterial sistólica (mm Hg)
quirúrgica para el control del sangrado. Véanse cuadros 11-6-1 100 a 109 1
y 11-6-2. 90 a 99 2
El sistema de puntuación de Rockall se diferencia de la es­ Menos de 90 3

cala de Blatchford en que contiene parámetros propios a los Frecuencia cardiaca mayor a 100 1pm
hallazgos endoscópicos en el paciente; sin embargo, el ejerci­ Presentación con melena
cio de sumar la puntuación previa a la endoscopia también es Presentación con síncope 2
útil para una calificación inicial. Puntajes mayores o iguales a
Enfermedad hepática 2
2 indican paciente de bajo riesgo.
Historia de falla cardiaca 2

13. ¿Cuál clasificación endoscópica es útil para me­


dir riesgos de resangrado, cirugía y mortalidad en un 14. ¿Cuál es el tratamiento médico para la úlcera pép­
paciente con úlcera péptica? tica con sangrado activo?
La clasificación de Forrest es la más utilízada, con lesiones El tratamiento de elección son los inhibidores de bomba de
activas o de alto riesgo clasificadas como Forrest 1-a hasta Fo­ protones o IBP (omeprazol, pantoprazol, etcétera), con bolo
rrest 11-b, y con lesiones de bajo riesgo como Forrest 11-c y inicial de 80 mg y luego infusión de 8 mg/hora por 72 horas
Forrest III. Véase cuadro 11-6-3. que disminuyen el riesgo de resangrado (número necesario a

-
CUADRO 11-6-2.
ESCALA DE ROCKALL

PUNTAJE
Variable o 2 3

Edad.años <60 60 a 79 �80

Shock No shock Taquicardia Hipotensión


PAS >100 mm Hg PAS >100 mm Hg PAS <100 mm Hg
Pulso <100 latidos/min Pulso >100 latidos/min

Comorbilidad Ninguna Falla cardiaca, cardiopatía Falla renal, insuficiencia


isquémica, cualquier hepática, neoplasia
comorbilidad mayor diseminada

Diagnóstico Desgarro de Mallory-Weiss, Todos los demás Malignidad del tracto


ausencia de lesiones y no diagnósticos gastrointestinal superior
EHR

EHR mayor Ninguno o puntillado oscuro Sangre en el tracto


gastrointestinal superior,
coágulo adherido, vaso visible
o con sangrado activo
PAS = presión arterial sistólica; EHR = estigmas de hemorragia reciente.
Profilaxis, diagnóstico y tratamiento de la gastritis aguda (erosiva) en adultos en los tres niveles de atención. MÉXiCO 2011; CENETEC.

712
SECCIÓN 11 GASTROENTEROLOGÍA @
CUADRO 11-6-3. 19. ¿Cuál es la epidemiología de Helicobacter py/ori
CLASIFICACIÓN DE FORREST en México y el mundo?
Hemorragia activa
En México 80% de los adultos mayores de 26 años de edad
está infectado en su forma crónica. En el mundo se calcula que
Forrest la Hemorragia arterial en chorro
Forrest lb Hemorragia en capa
50% de la población se encuentra infectada. Es mucho más fre­
cuente en países en vías de desarrollo, lo cual se ha relaciona­
Signos de hemorragia reciente
do con sistemas de suministro de agua deficientes, mala higie­
Forrest lla Vaso visible ne y hacinamiento.
Forrest llb Vaso con coágulo centinela o coágulo adherido
Forrest lle Lesión cubierta por fibrina
20. ¿Cuáles son los porcentajes por patología de
Lesiones sin estigma de hemorragia
pacientes infectados con Helicobacter py/ori?
Forrest 111 No existen estigmas de hemorragia
El organismo se puede identificar en 90% de los pacientes con
cáncer gástrico, en 80 a 90% de los pacientes con úlcera pép­
tica y en 51 a 80% de los pacientes con dispepsia funcional.
tratar, NNT = 12), cirugía (NNT = 28) y mortalidad (NNT =
45). Esto seguido por una terapia de seis a ocho semanas con
IBP oral a monodosis. 21. ¿Cómo se transmite Helicobacter pylori?
Por vía oral-oral o por vía fecal-oral.
15. ¿Cómo actúan los inhibidores de la bomba de pro­
tones en la úlcera péptica? 22. ¿Cuáles son los patrones de infección por Helico­
Neutralizan el pH gástrico a las tres horas de iniciada su admi­ bacter pylori?
nistración, lo cual promueve la estabilización del coágulo y, por a) Para infección aguda: pangastritis (fondo, cuerpo y antro)
ende, la hemostasia y la cicatrización. con disminución de secreción de somatostatina, disminu­
ción de gastrina y, por ende, hipoclorhidria.
16. ¿Por cuánto tiempo se administra el tratamiento b) Para infección crónica existen dos patrones básicos:
oral con inhibidor de bomba de protones para la úl­ • Gastritis antral: 15% con disminución de la secreción
cera gástrica y la duodenal? de somatostatina, pero con aumento de gastrina e hiper­
Para la úlcera gástrica dura seis a ocho semanas y para la duo­ clorhidria; es la gastritis asociada con úlcera péptica gás­

1
denal cuatro a seis semanas, con lo cual se obtienen tasas de trica y duodenal.
curación de hasta 70%. En los estados hipersecretores se • Pangastritis: 85% de los casos, con inflamación crónica,
recomiendan tratamientos más prolongados, de ocho a doce asociada con atrofia y, a la larga, con lesiones preneoplá­
semanas. sicas (metaplasia intestinal y displasia) y cáncer gástrico.

17. ¿Cuáles son las particularidades del tratamiento 23. ¿Cómo daña la bacteria al estómago?
con inhibidor de bomba de protones? H. pylori tiene los siguientes factores de virulencia:
Los inhibidores de bomba de protones son superiores a los
a) Ureasa: desdobla la urea en amoniaco y dióxido de carbo­
anti-H 2 (ranitidina, famotidina y otros) en cuanto a la efectivi­
no, neutralizando el ácido. El amoniaco es citotóxico para
dad en la cicatrización y la prevención de recurrencia de las
el epitelio gástrico y estimula la producción de autoanticuer­
úlceras, además de tener la ventaja de no presentar fenómeno
pos.
de tolerancia al ser administrados a largo plazo. Los IBP deben
b) Flagelos: explican su motilidad.
suministrarse entre 30 y 60 minutos antes de los alimentos pa­
e) Moléculas de adhesión.
ra inactivar el mayor número posible de bombas de protones
d) Proteasa: digiere el moco.
activas.
e) Sustancias tóxicas.
INFECCIÓN POR HEL/COBACTER PYLOR/
24. ¿Cuáles proteínas de Helicobacter pylori se han
18. ¿Qué es Helicobacter pylori? descrito para gravedad?
Es una bacteria gramnegativa y microaerofílica con cinco fla­ La citotoxina VacA, vista en las llamadas cepas Tox+, es una
gelos que produce inflamación aguda y crónica de la mucosa proteína que secreta la bacteria y produce daño directo a la mu­
gástrica. Es la causa más frecuente de úlcera péptica a nivel cosa. Se ha relacionado con enfermedades gástricas graves,
mundial y un factor de riesgo reconocido para adenocarcino­ incluso úlceras complicadas y cáncer gástrico. Esta proteína
ma gástrico y MALToma (linfoma de tejido linfoide asociado daña de manera directa al epitelio mediante la producción de
con mucosas). canales liberadores de bicarbonato, lo cual asegura la supervi-

713
e MANUAL PARA EL EXAMEN NACIONAL DE RESIDENCIAS MÉDICAS

vencía de la bacteria y el daño a la membrana rnitocondrial que + tinidazol 1 gramo cada doce horas + tetraciclina 500 mg
termina en apoptosis celular. VO cada seis horas + sales de bismuto 565 mg cada seis
La proteína CagA es un antígeno de la bacteria que se pre­ horas por catorce días.
senta hasta en 80% de los casos de úlcera duodenal.
29. ¿Cómo se comprueba la erradicación de Helico­
25. ¿Con qué enfermedades se asocia Helicobacter bacter py/ori?
pylori?
Debe realizarse cuatro semanas después del tratamiento. De
a) Definitivamente asociadas: gastritis atrófica, úlcera gástrica manera ideal, en caso de no requerir endoscopia de seguimien­
y duodenal, MALToma y cáncer gástrico. to, las pruebas no invasivas son el método de elección. La más
b) Probablemente asociadas: linfoma no Hodgkin gástrico pri­ sensible es la prueba de ureasa en aliento (sensibilidad de 94%
mario, dispepsia, púrpura trombocitopénica idiopática y y especificidad de 95%) con la antigenemia fecal.
anemia por deficiencia de hierro.

26. ¿Cómo se detecta la presencia de la bacteria en TRATAMIENTO QUIRÚRGICO


el cuerpo? DE LA ÚLCERA GASTRODUODENAL
Mediante dos tipos de pruebas:
30. ¿Cuáles son las complicaciones de la úlcera gas­
a) Invasivas: troduodenal que con más frecuencia requieren tra­
• Prueba rápida de ureasa, realizada con biopsias gástri­ tamiento quirúrgico?
cas, tinción con Giemsa en biopsias gástricas y cultivo. El sangrado se presenta en 60% de los pacientes con enfer­
b) No invasivas: medad ulcerosa; sin embargo, sólo entre 5 y 10% de estos pa­
• Anticuerpos séricos, no útil para valorar erradicación. cientes requiere un procedimiento quirúrgico. La perforación
Útil para evaluar posibles falsos negativos en pacientes tiene una incidencia de siete a diez casos/ 100 000 adultos por
con uso reciente de inhibidor de bomba de protones o año, se presenta con mayor frecuencia entre los 40 y 60 años
antibioticoterapia reciente. de edad y están localizadas en la cara anterior del duodeno
• Antigenemia fecal. (60%), el antro gástrico (20%) y la curvatura menor del estó­
• Prueba de aliento con urea. mago (20%).
27. ¿Cuáles son las indicaciones de tratamiento para
31. ¿Cuál ha sido el comportamiento epidemiológico
Helicobacter py/ori?
en el siglo XXI de los pacientes con complicaciones
a) Absolutas: úlcera péptica activa o no, MALToma, gastritis
de úlcera gastroduodenal?
atrófica o folicular.
b) Relativas: dispepsia familiar, familiares de primer grado de Estas complicaciones suelen presentarse entre la séptima y la
pacientes con cáncer gástrico, postresección de cáncer gás­ octava décadas de la vida y son 1.5 veces más frecuentes en
trico, uso crónico de AINE asociado con infección por He­ hombres que en mujeres. Ocurren en 2 a 10% de los pacientes
licobacter pylori, deseo del paciente de ser tratado, uso cró­ con enfermedad ulcerosa péptica y provocan 70% de las muer­
nico de inhibidores de bomba de protones asociado con tes asociadas con este padecimiento.
infección por Helicobacter pylori.
32. ¿Cuáles son las manifestaciones clínicas de los
28. ¿Cuáles y cómo son los tratamientos para Heli­ pacientes que presentan sangrado ulceroso péptico?
cobacter pylori? Por lo regular se presentan con hematemesis, melena o ambas.
Los tratamientos avalados en México para Helicobacter pylori En los casos de sangrado masivo puede haber hematoquecia.
se mencionan a continuación. Debido a la alta prevalencia de Muchos pacientes ingresan a los servicio de urgencias ya con
resistencia a metronidazol (hasta 80%) se considera: deterioro hemodinámico o pérdida significativa de volumen, in­
cluso en shock. En ocasiones reportan una historia de sincope
a) Esquema de primera elección (esquema triple): inhibidores
previo a la primera manifestación clínica, lo cual sugiere pér­
de la bomba de protones cada doce horas + claritromicina
dida masiva de sangre.
500 mg cada doce horas + amoxiclina 1 gramo VO cada
doce horas por catorce días.
b) En caso de alergia a amoxicilina: inhibidores de la bomba de 33. ¿Cuál es el tratamiento quirúrgico que se indica
protones cada doce horas + tetraciclina 500 mg VO cada con más frecuencia para pacientes con sangrado ul­
seis horas o metronidazol 250 mg VO cada seis horas + sa­ ceroso péptico?
les de bismuto 565 mg cada seis horas por catorce días. Excisión de la úlcera y reparación del defecto gastroduodenal
e) Esquema de segunda elección en México ( esquema cuádru­ resultante. La excisión es importante, ya que entre 4 y 5% de
ple): inhibidores de la bomba de protones cada doce horas las úlceras pueden ser malignas. El abordaje estándar para el

714
SECCIÓN 11 GASTROENTEROLOGÍA @
sangrado gastroduodenal es la realización de una duodenoto­ por hemorragia secundaria a úlcera gastroduode­
mía extendida a través del píloro hasta el estómago distal, liga­ nal?
dura del vaso sangrante, que a menudo es la arteria gastroduo­ La recurrencia del sangrado ocurre en 6 a 17% de los pacien­
denal, con una sutura en 8 en la parte alta y el fondo del cráter tes; en estos casos la terapia endoscópica no es una opción si
ulceroso y una tercera sutura en U por debajo de las anteriores la duodenotomía es reciente, por lo cual la mayoría de los ci­
para el control de las ramas pancreáticas transversas. Se reali­ rujanos prefiere realizar una cirugía más extensa, por lo regu­
za el cierre transverso de la incisión longitudinal previamente lar la gastrectomía distal con o sin vagotomía; sin embargo, en
realizada al duodeno y al antro gástrico para construir una pi­ los hospitales que cuenten con la posibilidad de realizar una
loroplastia Heineke-Mikulicz. embolización arterial transcatéter, puede ser una buena op­
ción para evitar un nuevo procedimiento quirúrgico.
34. ¿Cómo se realiza el diagnóstico de un paciente
con úlcera péptica perforada?
Son pacientes con una edad promedio de 48 años, con histo­ 37. ¿Cuáles son las complicaciones posquirúrgicas
ria de enfermedad ulcerosa péptica en 29% de los casos y con que suelen observarse en la cirugía por úlcera gas­
ingesta crónica de antiinflamatorios no esteroideos en 20% de troduodenal?
los casos. Se presentan con dolor espontáneo punzante locali­ La complicación observada con más frecuencia es la neumo­
zado en epigastrio, en ocasiones irradiado a ambos hombros nía, con una incidencia de 3.6 a 30% de los casos, seguida por
(lo que indica aire libre subdiafragmático). Náusea y vómito la infección del sitio quirúrgico en 10 a 17%; dehiscencia
se presentan en 50% de los casos. Hay taquicardia por arriba de de las suturas de 2 a 16%; y formación de absceso de O a 9%.
90 pulsaciones por minuto y entre 5 y 10% de los casos presen­ La posibilidad de reintervención va de 2 hasta 9%, y 2.5% de
ta una presión arterial media de 80 mm Hg. La pérdida de la los pacientes desarrolla sepsis.
matidez hepática se presenta en 37% de los casos, por lo que
como herramienta diagnóstica tiene sus limitaciones. En los 38. ¿Qué factores de riesgo influyen en el resultado
análisis sanguineos puede encontrarse una leucocitosis mode­ del procedimiento?
rada; sin embargo, éstos se solicitan con la finalidad de descar­ La mortalidad después de la cirugía oscila entre 6 y 10%, y
tar pancreatitis. La placa simple de abdomen y tórax revela aire existen cuatro factores de mal pronóstico que incrementan
libre subdiafragmático en 80 a 85% de los casos. el riesgo de muerte en estos pacientes: edad mayor a 60 años,
retraso en el tratamiento mayor a 24 horas, estado de shock
35. ¿Cuál es el manejo quirúrgico actual de la úlcera

1
al momento del ingreso a urgencias, definido como una pre­
péptica perforada?
sión sistólica menor a 100 mm Hg, y enfermedades concomi­
Una vez realizado el diagnóstico se inicia resucitación con cris­ tantes.
taloides, se coloca una sonda nasogástrica y se administran an­
tibióticos de amplio espectro. De manera convencional la ciru­
gía inicia con una incisión media supraumbilical, se identifica ÚLCERAS ASOCIADAS CON
el sitio de la perforación, se toma una biopsia de los bordes pa­ ANTIINFLAMATORIOS NO ESTEROIDEOS
ra descartar cáncer gástrico y se realiza un cierre simple de la
perforación, la cual puede efectuarse con o sin un parche pe­ 39. ¿Cómo actúan los antiinflamatorios no esteroides
diculado de epiplón encima de la reparación, o simplemente (AINE) a nivel gástrico?
se coloca el parche pediculado de epiplón para sellar el defec­ Son ácidos débiles que, al estar en contacto con la mucosa,
to, procedimiento conocido como parche de Graham. El lava­ atraviesan membranas lipídicas y entran en las células epitelia­
do de la cavidad peritoneal con seis a diez litros de solución les, liberando hidrogeniones que quedan atrapados dentro de
salina es una rutina histórica intraoperatoria que carece de evi­ las células. Con ello:
dencia científica que la soporte. La colocación de drenajes no
a) Disminuyen la producción de prostaglandinas, lo cual con­
reduce la incidencia de colecciones o abscesos intraabdomina­
diciona la reducción de flujo sanguíneo a mucosa gástrica,
les; no obstante, se ha demostrado que aumenta la infección
disminución de producción de bicarbonato y moco y aumen­
en el sitio de introducción del drenaje y puede causar obstruc­
to de la secreción ácida gástrica.
ción intestinal, por lo cual no está indicada en este tipo de pro­
b) Disminuyen la fosforilación oxidativa y glucólisis dentro de
cedimientos.
la célula, lo cual causa daño metabólico directo.
e) Promueven daño a la mucosa inducido por neutrófilos.
COMPLICACIONES DE LA CIRUGÍA
DE ÚLCERA PÉPTICA
40. ¿Cómo actúan los AINE a nivel sistémico?
36. ¿Con qué frecuencia los pacientes presentan Bloquean a la ciclooxigenasa tipo 1 (constitutiva) y tipo 2 (in­
nuevos episodios de sangrado posterior a la cirugía ducible), lo cual condiciona daño a la mucosa gástrica, inhibí-

715
e MANUAL PARA EL EXAMEN NACIONAL DE RESIDENCIAS MÉDICAS

ción de la agregación plaquetaria y disminución del dolor, la bargo, los números necesarios a tratar son muy altos. Ha caí­
fiebre y la inflamación. do en desuso debido a la alta prevalencia de efectos adversos
(diarrea).
41. ¿Cuáles son los factores de riesgo para el desa­
rrollo de úlcera por AINE? 49. ¿Los inhibidores de la COX-2 aumentan el riesgo
Edad >65 años, evento ulceroso gastrointestinal previo, uso de úlcera péptica?
concomitante de esteroides o anticoagulantes, dosis elevadas Aunque en un grado mucho menor que los AINE no selectivos,
o uso de múltiples AINE, cardiopatía isquémica, diabete me­ los inhibidores selectivos de COX-2 también elevan el riesgo
llitus tipo 2, enfermedad pulmonar obstructiva crónica y cirro­ de úlcera péptica.
sis hepática.
50. ¿Cuál es la utilidad de los antihistamínicos anti­
42. ¿Cuál es el porcentaje de alteraciones endoscó­ H 2 en la úlcera por AINE?
picas gástricas y duodenales asociadas con AINE? Aunque útiles en la curación de la úlcera, una vez formada la
Entre 40 y 60% de los pacientes que usan AINE presenta ero­ misma no son útiles para la prevención de desarrollo en perso­
siones en la mucosa; entre 10 y 40% presenta úlceras gástricas, nas que utilizan AINE.
asintomáticas o no, a diferencia de 4 a 15% para úlceras duo­
denales. DISPEPSIA

43. ¿Cuál es la incidencia de eventos gastrointesti­ 51. ¿Qué es la dispepsia?


nales complicados por úlcera por AINE?
El término dispepsia es amplio y abarca el dolor, malestar o
De 1 a 1.5 eventos complicados al año (sangrado, perforación molestia en abdomen superior, asociado con distensión post­
u obstrucción). prandial, sensación de plenitud gástrica, náusea, anorexia o
reflujo. Así, una dispepsia no estudiada puede incluir una gran
44. ¿Cuál es la localización de la úlcera gástrica más variedad de diagnósticos diferenciales, como enfermedad por re­
asociada con AINE? flujo gastroesofágico, úlcera péptica por H. pylori o por AINE,
El antro gástrico. gastritis aguda o crónica y gastroparesia, entre otras patologías.

45. ¿Qué opciones existen para disminuir la inciden­ 52. ¿Qué se conoce como dispepsia funcional?
cia de úlcera gástrica por AINE? Según los criterios de Roma III, la dispepsia funcional debe
Utilizar inhibidores selectivos de la ciclooxigenasa COX-2 (ce­ incluir uno o más de los siguientes síntomas:
lecoxib, rofecoxib) o paracetamol para el control del dolor;
en personas mayores utilizar la mitad de la dosis tradicional a) Plenitud postprandial que incomoda.
del AINE y aplicar tratamiento profiláctico con inhibidor de b) Saciedad temprana.
bomba de protones a monodosis. e) Dolor epigástrico.
d) Sensación de quemazón epigástrica.
46. ¿Las dosis de aspirina profiláctica para eventos Se debe excluir evidencia de enfermedad gastrointestinal.
cardiovasculares (75 a 150 mg/día) son perjudiciales Dichos criterios deben cumplirse en los últimos tres meses,
para el estómago? con inicio de las molestias al menos seis meses antes del diag­
Sí lo son; de hecho, sumar aspirina a dosis bajas a cualquier nóstico. En la actualidad se engloban dos patologías en la dis­
otro AINE por causa diversa aumenta la incidencia de compli­ pepsia funcional: el síndrome de dolor epigástrico y el síndro­
caciones de úlcera gástrica. me de dolor postprandial.

47. ¿Cuál es el tratamiento médico de las úlceras por 53. ¿Cuáles son los datos de alarma en un paciente
AINE? con dispepsia?
Es primordial retirar el AINE en cuestión y sustituirlo, de ser a) Inicio en mayor de 55 años de edad.
posible, por un inhibidor selectivo de la COX-2 o paracetamol. b) Historia de cáncer gástrico en la familia.
Además se debe administrar un inhibidor de bomba de pro­ e) Pérdida de peso no intencionada.
tones. d) Datos de sangrado de tubo digestivo.
e) Disfagia progresiva u odinofagia.
48. ¿Cuál es la utilidad del misoprostol en úlcera por f) Anemia por deficiencia de hierro.
AINE? g) Vómito persistente.
El misoprostol, un análogo de las prostaglandinas, es útil para h) Masa palpable o linfadenopatía.
disminuir las complicaciones de la úlcera por AINE; sin em- i) Desarrollo de ictericia.

716
SECCIÓN 11 GASTROENTEROLOGÍA @
54. ¿Cuál es el abordaje diagnóstico de la dispepsia? BIBLIOGRAFÍA
El abordaje inicial es la minuciosa historia clínica y la explo­ Abdo-Francis JM, Uscanga-Domínguez L. III consenso mexicano sobre H.
ración física. En pacientes con datos de alarma, el estudio de pylori. Rev Gastroenterol Mex. 2007;72: 136-53.
Bertleff M, Lange J. Perforated peptic ulcer dísease: a review of history and
elección es la endoscopia gastrointestinal alta. En ciertos gru­
treatment. Dig Surg. 20!0;27: 161-9.
pos de personas el tamizaje para infección por H pylori puede Blatchford O, Murray WR, Blatchford M. A risk score to predict need for
ser útil. treatment for treatment for upper-gastrointestinal haemorrage. Lancet.
2000;356(9238): 1310-21.
55. ¿Qué otros estudios puede utilizar el clínico? Boparia V, Rajagopalan J, Triadafilopoulos G. Guide to the use of proton
pump inhibitors in adult patients. Drugs. 2008;68:925-47.
Los estudios baritados aún son de utilidad en ciertos lugares
Drossman DA. The functional gastrointestinal disorders and the Rome III
en donde la accesibilidad a endoscopia no es fácil o económi­ process. Gastroenterology. 2006 Apr; 130(5): 1377-90.
camente posible. En estados hipersecretores se pueden hacer Gisbert JP. Tratamiento farmacológico de la hemorragia digestiva por úlcera
estudios de la secreción ácida gástrica. Son útiles el ultrasoni­ péptica. Med Clin (Barc). 2006;127:66-75.
do, la tomografía y la resonancia magnética cuando se sospe­ Graham DY, Rugge M. Clinical practice: diagnosis and evaluation of dyspep­
sia. J Clin Gastroenterol. 2010 Mar;44(3):167-72.
cha patología pancreática.
Gralnek IM, Barkun AN, Bardou M. Management of acule bleeding from a
peptic ulcer. N Engl J Med. 2008 Aug 28;359(9):928-37.
56. ¿Cuál es el tratamiento de la dispepsia? Laine L, Jensen DM. Management of Patients With Ulcer Bleeding. Am J
Depende del diagnóstico definitivo. En caso de ser una dispep­ Gastroenterol. 2012; 107:345-60.
sia funcional no existe un tratamiento eficaz sino que debe Laine L. Nonsteroidal anti-inflammatory drug gastropathy. Gastrointest En­
dose Clin N Am.1996;6:489-504.
individualizarse. Puede abarcar: Laine L. The role of proton pump inhibitors in NSAJD-associated gastropathy
a) Medidas higiénico-dietéticas: evitar tabaquismo, alcohol, and upper gastrointestinal symptoms. Rev Gastroenterol Disord. 2003;
3(Suppl 4):S30-9.
fármacos antiinflamatorios y alimentos grasos.
Lanas Al, Nerín J, Esteva F, Sainz R. Non-steroidal anti-inflammatory drugs
b) lnhibidores de secreción ácida gástrica: en general anti-H2 and prostaglandin effects on pepsinogen secretion by dispersed human
e inhibidores de bomba de protones en dispepsia ulcerosa. peptic cells. Gut. l995;36:657-63.
e) Agentes promotilidad: útiles en alteraciones de la motilidad Lee C, Sarosi G. Emergency Ulcer Surgery. Surg Clin N Am. 2011 ;91: 1001-
gástrica. 13.
Malfertheiner P, Megraud F, O'Morain C et al. Current concepts in the mana­
d) Erradicación de Helicobacter pylori. gement of Helicobacter pylori infection: the Maastricht 111 Consensus
e) Agentes que actúan sobre la percepción visceral: antagonis­ Report. Gut. 2007;56:772-81.
tas 5-HT 3 y agonistas parciales 5-HT 4 . Además, los antide­ Nirula R. GastroduodenaJ perforation. Surg Clin N Am. 2014;94:3l -4.

1
presivos tricíclicos como inhibidores de la recaptación de
serotonina resultan de utilidad.

CAPÍTULO 11.7
TUMORES GÁSTRICOS
Paulo Roberto Rojas Macuil

BENIGNOS 3. ¿Qué porcentaje representan los tumores gástri­


cos de tipo benigno?
1. Mencione cinco tipos de tumores benignos gástri­ Corresponden a 1 a 5% de todos los tumores gástricos.
cos:
Los pólipos hiperplásicos, también llamados regenerativos, son 4. ¿Cuál es la manifestación más frecuente de los tu­
los más frecuentes en estómago (75%); lipoma de tipo submu­ mores gástricos benignos?
coso con presentación antral 3%; los tumores de naturaleza Por lo general los tumores gástricos benignos cursan de forma
neurogénica a nivel gástrico son raros; los tumores más comu­ asintomática; es decir, es común que se encuentren como ha­
nes son schwannomas y neurofibromas, y el páncreas ectópico llazgo endoscópico en búsqueda de otra patología. Los sínto­
de tipo submucoso con presentación en antro. mas dependen del tamaño, la localización o la presencia o no
de sangrado digestivo. Los tumores en antro que dificultan el
2. ¿Qué tipo de origen embriológico tienen los tumo­ orificio de salida del tracto gástrico, los de tamaño gigante o
res gástricos benignos? los que muestran datos de sangrado digestivo son los que re­
Tejido epitelial, mesenquimático y neural. quieren terapia de tipo quirúrgico o endoscópico.

717
e MANUAL PARA EL EXAMEN NACIONAL DE RESIDENCIAS MÉDICAS

ADENOCARCINOMA GÁSTRICO Desafortunadamente 60% de los casos se encuentra en etapas


avanzadas, en los cuales el tratamiento se reduce a terapia pa­
5. ¿Cuál es el tumor gástrico maligno más frecuente? liativa. La radioterapia no ha demostrado ningún beneficio en
Es el adenocarcinoma gástrico en 90% de los casos. reducción tumoral ni en sobrevida. La quimioterapia sólo
ha reducido el tamaño tumoral en 15% de los casos, sin tener
6. ¿Cuál es la localización más frecuente del adeno­ un efecto en la sobrevida.
carcinoma gástrico?
LINFOMA NO HODGKIN GÁSTRICO
La localización es más frecuente en el estómago distal y el an­
tro es el lugar más afectado. Los tumores de localización pro­
12. ¿Cuál es la localización más común de linfoma no
ximal tienen un curso más rápido y agresivo.
Hodking?
7. Mencione los factores de riesgo para el desarrollo La localización más común del linfoma no Hodgkin es el estó­
de adenocarcinoma gástrico: mago y corresponde a 5 a 10% de los tumores gástricos.
a) Dietético: dieta alta en productos salados o con nitratos y 13. Mencione los factores de riesgo para el desarro­
baja en frutas y verduras. llo de linfoma gástrico.
b) Tabaquismo.
Infección por Helicobacter pylori, enfermedades autoínrnunita­
e) Cirugía gástrica: los pacientes con cirugía gástrica tipo Bill­
rias, síndromes de inmunodeficiencia o terapia ínmunodepre­
roth II tienen más riesgo de desarrollar cáncer gástrico; el
siva prolongada.
riesgo aumenta después de quince años del procedimiento
quirúrgico. 14. ¿Existen síntomas característicos para la sospe­
d) Infeccioso: la presencia de Helicobacter pylori, por ser un
cha de linfoma gástrico?
agente carcinógeno tipo l.
e) Miscelánea: anemia perniciosa, enfermedad de Ménétrier, Por desgracia son síntomas inespecificos vagos, con la presen­
pólipos adenomatosos. cia de dolor abdominal, sensación de llenura, dispepsia o san­
grado digestivo alto en etapas avanzadas. Se requiere estudio
8. Mencione las manifestaciones clínicas más fre­ endoscópico con toma de biopsias y análisis con inmunohisto­
cuentes de adenocarcinoma gástrico: química para su confirmación histológica.
Los síntomas son vagos en etapas tempranas y se manifiestan 15. Mencione los sistemas de etapificación para lin­
como dolor en epigastrio inespecífico, náusea y síntomas dis­ foma no Hodking:
pépticos; la presencia de llenado posprandial inmediato, vómi­
• Etapa l. Limitado a estómago.
to, pérdida de peso, anemia o hemorragia digestiva alta son
• Etapa 11. Estómago y ganglios adyacentes y limitados a un
síntomas de alarma y corresponden a enfermedad avanzada.
solo lado del diafragma.
• Etapa 111. Ganglios presentes en ambos lados del diafragma.
9. ¿Cuáles son las manifestaciones extradigestivas
• Etapa IV. Enfermedad diseminada.
del carcinoma gástrico?
Las manifestaciones extradigestivas más frecuentes de cáncer 16. ¿Cuál es la sobrevida para los pacientes de linfo­
gástrico son: ganglio supraclavicular (Virchow), nódulo axilar ma?
izquierdo (lrish), nódulo periumbilical (hermana Mary Joseph) Depende del estadio y la etapa de la enfermedad. En los pa­
o la presencia de síndromes paraneoplásicos ( acantosis pigmen­ cientes en etapa temprana 1-11 y con tamaño menor de 5 cm la
taria, dermatomiositis, trombosis venosa o queratosis seborrei­ sobrevida a diez años es mayor de 80%; sín embargo, en etapas
ca en tórax, signo Leser-Trélat). El sitio más frecuente de me­ avanzadas 111 y IV es menor de 50% a cinco años.
tástasis es hepático, pulmonar u óseo.
TUMORES DEL ESTROMA
10. ¿Cuál es el estudio de elección para su diagnós­
tico? 17. ¿Cuál es el origen de los tumores de tipo estroma!
El estudio de mayor utilidad para el diagnóstico es la realización o GIST?
de endoscopia con toma de biopsia de la lesión sospechosa. Son tumores de tipo mesenquimatoso y su origen está en célu­
las madres pluripotentes y en las células de Cajal.
11. ¿Cuál es el tratamiento de elección para el carci­
noma gástrico? 18. ¿Cuál es la localización más frecuente de los tu­
El tratamiento con finalidad curativa es el quirúrgico; sin em­ mores estromales o GIST?
bargo, el tratamiento a elegir depende de muchos factores; por El estómago en 60%, seguido por el intestino delgado en 35%
ejemplo: tamaño, localización, extensión e invasión tumoral. de los casos.

718
SECCIÓN 11 GASTROENTEROLOGÍA @
19. ¿Cómo se diagnostica este tipo de tumor gástrico? MALT
Por lo general los tumores estromales o GIST se diagnostican
por un hallazgo endoscópico o quirúrgico, pues la sintomato­ 23. ¿Cuál es el germen asociado con el linterna tipo
logía es vaga e inespecífica. Una vez que es encontrado en es­ MALT?
tudio endoscópico, se toma biopsia y la técnica de inmunohis­ Es Helicobacter pylori.
toquímica CD 117 positiva es el diagnóstico de certeza de este
tumor. 24. ¿Qué porcentaje de curación se obtiene en el lin­
terna tipo MALT con el tratamiento de erradicación
20. ¿Es de utilidad la nueva tecnología en modalidad para H. py/ori?
ecoendoscópica para el diagnóstico de este tipo de Se obtiene en 70 a 85% de casos cuando el linfoma es de bajo
tumores gástricos? grado.
Sí, las nuevas modalidades diagnósticas endoscópicas son
de utilidad. El desarrollo del estudio ecoendoscópico permite 25. ¿Cuál es el triple esquema de erradicación para
diferenciar el sitio de origen de este tipo de tumor. Con frecuen­ H. pylori?
cia se encuentra en la capa muscular (cuarta capa), se toma El esquema de prímera linea de erradicación para H pylori es
biopsia por aspiración con aguja fina para la toma precisa de el triple a base de amoxicilina 1 g VO cada doce horas, clari­
la lesión tumoral y con el análisis posterior con técnica inmu­ tromicina 500 mg VO cada doce horas y omeprazol 20 mg VO
nohistoquímica se obtiene el diagnóstico en 100% de los casos. cada doce horas durante catorce días.

21. ¿Cuáles son las modalidades de terapia para BIBLIOGRAFÍA


este tipo de tumores? Francis DL. Gastric neoplasms and gastroenteropancreatic neuroendocrine
En general existen dos modalidades: endoscópica para tumo­ tumors. En: Hauser S, Pardi D, Poterucha J et al. ( ed). Mayo Clinic gas­
res menores de 20 mm, bien delimitados, bien diferenciados y troenterology and hepatology board review. 3 r<1 ed. Rochester, MN: Mayo
Clinic Scientific Press; 2008. pp. 77-96.
sin presencia de metástasis; y quirúrgica para tumores de ma­ Galindo F, Chiesa D, Rubio H. Tumores benignos del estómago, Cirugía Di­
yor tamaño. gestiva. F. Galindo, www.sacd.org.ar, 2009; II-220, pág. 1-13.

22. ¿Cuál es el lugar de metástasis más frecuente de


los GIST?
Es el hígado, seguido en frecuencia por los pulmones y los
huesos.
1
CAPÍTULO 11.8
SÍNDROME DIARREICO
José María Remes Troche

DIARREA AGUDA tino delgado y el colon, además de ocasionar inflamación (ma­


nifestada por prurito, tenesmo, fiebre, etcétera), existen otras
1. ¿Cuál es la definición objetiva de diarrea aguda? condiciones, como las intolerancias a los alimentos, que pro­
La diarrea aguda se define como la presencia de más de tres ducen una diarrea osmótica caracterizada por ceder con el ayu­
evacuaciones al día o la evacuación de más de 200 mL de agua no. Los tipos de diarrea y sus mecanismos asociados se mues­
en 24 horas. La diarrea se denomina aguda si tiene una presen­ tran en el cuadro 11-8-1.
tación igual o menor de catorce días.
3. ¿Cuál es la causa más común de la diarrea agu­
2. ¿Cómo se clasifica la diarrea de acuerdo con sus da?
mecanismos fisiopatológicos? Las infecciones son la causa más frecuente de diarrea aguda y
Existen diversos mecanismos generadores de diarrea. Mientras un amplio espectro de agentes puede causar diarrea en el hu­
los virus, bacterias y protozoarios pueden incrementar la per­ mano: virus, bacterias, parásitos y hongos. Los agentes más co­
meabilidad y alterar la función digestiva y absortiva del intes- munes se muestran en el cuadro 11-8-2.

719
e MANUAL PARA EL EXAMEN NACIONAL DE RESIDENCIAS MÉDICAS

-
CUADRO 11-8-1.
TIPOS DE DIARREA Y SUS MECANISMOS ASOCIADOS

:;;;; MECANISMOS ASOCIADOS AGENTE CAUSAL CARACTERÍSTICAS CLÍNICAS


• Osmótica Aumento en la osmolaridad Intolerancia a la lactosa, azúcares no Cede con el ayuno, irritación perianal
digeribles, etcétera.
• Inflamatoria Daño directo epitelial, incremento Bacterias (gramnegativas), protozoarios Pujo, tenesmo, moco, a veces sangre, fiebre,
de la permeabilidad intestinal (E. hystolitica), radiación, isquemia, leucocitosis fecal
enfermedad inflamatoria
• Malabsortiva Disminución de la superficie de Atrofia de vellosidades y aplanamiento Borborigmos, distensión abdominal,
absorción inducido por rotavirus y giardiosis flatulencias excesivas
• Secretora Secreción excesiva de fluidos en Toxinas A y B de C. difficile, Vibrio No cede con el ayuno, puede ser nocturna,
la luz intestinal cholerae, E. coli, enterotoxigénicas desequilibrio hidroelectrolítico
• Motora Alteraciones de la motilidad Síndrome de intestino irritable con Asociada con dolor abdominal, sin datos de
predominio de diarrea alarma, frecuente ansiedad y somatización

4. De los agentes infecciosos, ¿cuáles son los que ficiente incrementar el aporte de liquidos que contengan agua,
causan diarrea aguda con más frecuencia? sal y glucosa por vía oral. Se recomienda evitar bebidas carbo­
En general las infecciones virales son las más frecuentes y au­ natadas para evitar la distensión abdominal; la cafeína, ya que
tolimitadas. Los dos agentes virales más importantes son los puede estimular la secreción intestinal; y la lactosa, debido a
norovirus y rotavirus. Los norovirus son calicivirus que produ­ que la deficiencia de lactasa acompaña con frecuencia a las in­
cen más de 90% de los brotes de diarrea viral. fecciones entéricas.

5. ¿Cómo se diferencia una diarrea originada en el 9. ¿Cómo se deben rehidratar los pacientes con dia­
intestino delgado de una originada en el colon? rrea aguda?
Las evacuaciones abundantes pero no tan frecuentes, sin ur­ Los casos moderados deben recibir soluciones con electroli­
gencia ni tenesmo, hacen pensar en el intestino delgado como tos orales, como la que recomienda la Organización Mundial
principal segmento afectado. Por el contrario, las evacuaciones de la Salud y que contiene 2.6 g de cloruro de sodio, 2.5 g de
escasas pero frecuentes, con sangre y moco, con urgencia y te­ bicarbonato de sodio, 1.5 g de cloruro de potasio y 13.5 g
nesmo, sugieren involucro del colon distal y recto. de glucosa. Existen múltiples soluciones comerciales disponi­
bles y en México la Secretaría de Salud promueve y distribu-
6. ¿Cuáles son los patógenos que originan la mayo­
ría de los cuadros de diarrea inflamatoria? CUADRO 11-8-2.
Los microbios que producen estos cuadros suelen afectar al co­ CAUSAS MÁS COMUNES DE DIARREA AGUDA
lon invadiendo la mucosa o produciendo citotoxinas que gene­
ran la reacción inflamatoria local y sistémica. Los agentes cau­ a) Infecciones
• Virales
sales más frecuentes incluyen Salmonella, Shigella, Yersinia,
• Bacterianas
Campylobacter, E. coli enterohemorrágica y Entamoeba histoly­ • Parasitarias
tica. b) Relacionada con alimentos
• Alergias
7. ¿En qué pacientes deben realizarse exámenes? • Intolerancias a carbohidratos
• Aditivos y suplementos nutricionales
La mayoría de los pacientes con diarrea aguda presenta cua­
e) Agudización de enfermedades crónicas
dros relativamente leves que se autolimitan en 24 a 48 horas, • Funcionales (por ejemplo, síndrome de intestino irritable}
por lo que no es necesario realizar exámenes. Aquellos enfer­ • Orgánicas (por ejemplo, enfermedad celiaca, enfermedad
mos con fiebre, deshidratación grave, dolor abdominal intenso, inflamatoria intestinal)
hematoquecia o disentería, inmunocompromiso, enfermeda­ d) Medicamentos
des graves debilitantes y quienes no responden al tratamien­ • Laxantes
• Procinéticos
to de sostén o empeoran luego de tres o cuatro días deben ser
• Antiácidos
sometidos a exámenes. • Antiinflamatorios no esteroideos
• lnhibidores de la absorción (orlistat, acarbosa, etcétera}
8. ¿Cuál es el primer paso en el manejo de la diarrea • Oncológicos
aguda? • Promotores de la secreción intestinal (misoprostol)
• Causantes de carga osmótica (alimentos enterales}
La intervención terapéutica más importante en el paciente con
• Diversos (colchicina, sales de oro, antiarrítmicos}
diarrea aguda es la rehidratación. En casos leves suele ser su-

720
SECCIÓN 11 GASTROENTEROLOGÍA @
ye ampliamente los sobres de electrolitos "Vida Suero Oral". CUADRO 11-8-3.
Todos aquellos enfermos con deshidratación grave, náusea y TRATAMIENTO FARMACOLÓGICO DE LOS DIFERENTES
vómito incoercible y con enfermedades crónicas debilitantes PATÓGENOS
deben ser rehidratados por vía endovenosa. Los niveles de elec­
trolitos séricos deben vigilarse en forma estrecha y todas las MMM9fü·M RECOMENDACIÓN PARA ADULTOS

deficiencias deben corregirse. Shigelfo sp. Ciprofloxacino 500 mg cada 12 horas durante uno
a tres días
� Guía de Práctica Clínica. Atención, diagnóstico y tratamiento Alternativa: trimetoprim/sulfametoxazol 800/160 mg
de la Diarrea Aguda en Adultos en el Primer nivel de Atención, cada 12 horas
http://bvs.insp.mx/local/File/SSA_106_08_EyR.pdf Solmonelfo Trimetoprim/sulfametoxazol 800/160 mg cada doce
sp. especies horas o ciprofloxacino 500 mg cada doce horas
10. ¿Cuál es la recomendación respecto de la alimen­ no typhi durante cinco a siete días

tación durante los episodios de diarrea aguda? E. coli Ciprofloxacino 500 mg cada doce horas durante
uno a tres días
La alimentación debe continuar durante todas las etapas de la Alternativa: trimetoprim/sulfametoxazol 800/160 mg
enfermedad según lo máximo posible y debe incrementarse du­ cada 24 horas por siete días
rante el periodo de convalecencia para evitar efectos perjudi­ Yersinio sp. Doxiciclina 300 mg y aminoglucósidos
ciales al estado nutricional. La modificación a la dieta es una Alternativa: trimetoprim/sulfametoxazol
medida efectiva para el control sintomático en gran número
Vibrio Dosis única de 300 mg de doxiciclina. Tetraciclinas
de pacientes con intolerancia a elementos específicos de la die­ choleroe 500 mg cuatro veces al día durante tres días.
ta, como el sorbitol y la fructosa. La administración de lactasa Alternativa: ciprofloxacino 500 mg dosis única
puede ser útil en pacientes intolerantes a la lactosa, en quienes C. difficile Metronidazol 250 mg cuatro veces al día hasta 500
resulta difícil apegarse a una dieta libre de este azúcar. mg tres veces al día por diez días
Giordio sp. Metronidazol 250 a 500 mg tres veces al día por
11. ¿Es recomendable el uso de antidiarreicos? siete a diez días

Aunque su uso es controversia!, el agente de elección es la lo­ E. hystolytico Metronidazol 500 mg tres veces al día durante
peramida, 4 mg por dosis, vía oral; pueden administrarse pos­ cinco a diez días más iodoquinol 650 mg tres
veces al día durante diez días
teriormente 2 mg después de cada evacuación, con un máxi­
mo de 16 mg al día. Sin embargo, su uso está contraindicado So/monello Ciprofloxacino 500 mg VO cada doce horas por
typhi diez días
en diarrea asociada con toxinas, colitis ulcerativa, íleo, disen­

1
Ceftriaxona 1 a 2 g al día por diez días Subsalicilato
tería, enfermedad hepática y diarrea infecciosa. de bismuto en suspensión se sugiere en diarreas
agudas leves a moderadas no complicadas a
dosis de 10 ml VO cada cuatro horas y 10 ml
12. ¿Cuándo se deben usar antimicrobianos?
adicionales posterior a cada evacuación
El uso empírico de antibióticos se acepta en los casos con fie­
bre elevada y escalofrío, disentería, diarrea grave o que se ha
prolongado por más de una semana, sujetos inmunocomprome­ 14. ¿Qué son y para qué se usan los probióticos en
tidos y aquellos con diarrea profusa del viajero, en quienes el la diarrea aguda?
cuadro clínico puede acortarse. Las quinolonas han sido los
Los probióticos son microorganismos que proporcionan efec­
antibióticos empleados con más frecuencia, pero el desarrollo
tos benéficos para el huésped. Son efectivos para reducir la
de resistencia bacteriana ha llevado a utilizar otras opciones,
duración de la diarrea, en la prevención de la diarrea del viaje­
como la azitromicina. Los antibióticos están contraindicados
ro y la diarrea asociada con el uso de antibióticos. Ejemplos
en pacientes con diarrea sanguinolenta y dolor abdominal que
de probióticos son: Lactobacillus rhamnosus GG, Lactobacillus
no presenten fiebre hasta que la infección por E. coli entero­
reuteri y Bifidobacterium animalis lactis.
toxigénica haya sido excluida.
DIARREA CRÓNICA
13. ¿Cuál es el tratamiento farmacológico específi­
co con antimicrobianos para el manejo de la diarrea 15. ¿Cuál es la definición de diarrea crónica?
aguda? Aunque no hay consenso para definir su cronicidad, la mayo­
Los antibióticos específicos para cada patógeno se muestran ría de los expertos acepta que si la diarrea dura cuatro o más
en el cuadro 11-8-3. semanas debe considerarse crónica.
� Adaptado de Guía de práctica clínica. Atención, diagnóstico
y tratamiento de la diarrea aguda en adultos en el primer 16. Con base en su localización, ¿cuál es la clasifica­
nivel de atención (http://bvs.insp.mx/1oca1/Fi1e/SSA_106_ ción de la diarrea crónica?
08_EyR.pdf).
Puede sustentarse como se muestra en el cuadro 11-8-4.

721
e MANUAL PARA EL EXAMEN NACIONAL DE RESIDENCIAS MÉDICAS

CUADRO 11-8-4. CUADRO 11-8-5.


CAUSAS DE DIARREA CRÓNICA CON BASE EN SU DATOS PARA ESTABLECER EL DIAGNÓSTICO ENTRE
LOCALIZACIÓN DIARREA ORGÁNICA Y DIARREA FUNCIONAL

NEOPLASIAS DIARREA ORGÁNICA DIARREA FUNCIONAL


Intestinal Enfermedad inflamatoria • Historia familiar de enfermedad • Edad < 50 años
intestinal inflamatoria intestinal o enfermedad • Trastornos alimentarios
Colitis microscópicas celiaca • Consumo de cafeína
• Hospitalizaciones • Uso de productos que
Trastornos funcionales
, Viajes recientes contienen sorbitol y otros
digestivos
, Antecedentes de diabetes mellitus, edulcorantes
Intestino corto distiroidismo, aterosclerosis, • Alteraciones psicológicas
Enfermedad celiaca enfermedades de tejido conectivo
, Edad >50 años
Enfermedad de Crohn
, Alcoholismo
Otras enteropatías Enfermedad de Whipple , Uso reciente de medicamentos
Esprúe tropical {quimioterapia, laxantes, antibióti­
Amiloidosis cos, antidepresivos, inhibidores de
Linfangiectasias la producción de ácido)
intestinales , Pérdida de peso (>10% del peso
Ácidos biliares habitual)
• Sangrado {melena o hematoquecia)
Deficiencia de
disacaridasas • Fiebre
• Esteatorrea
Sobrepoblación
• Diarrea nocturna
bacteriana
Isquemia mesentérica
Enteritis posradioterapia rrea crónica se puede dividir en tres grupos: diarrea con grasa,
Linfoma diarrea con sangre y diarrea acuosa.
Giardiosis a) Diarrea con grasa: la información generada por el coproló­
Extraintestinal Pancreática Pancreatitis crónica gico lo hace un estudio de primera elección en todo sujeto
Insuficiencia pancreática con diarrea crónica. Debe tenerse en cuenta que la inexis­
Cáncer pancreático
Fibrosis quística
tencia de grasa no excluye la posibilidad de esteatorrea, que
puede investigarse con otros métodos como la cuantifi­
Endocrina Hipertiroidismo
Diabetes cación de grasa en heces (prueba ideal de diagnóstico) o la
Hipoparatiroidismo cuantificación de carotenos en sangre. La presencia de es­
Tumores teatorrea confirma el diagnóstico de síndrome de absorción
neuroendocrinos intestinal deficiente (SAID), que puede ser secundario a un
Otras causas Diarrea facticia problema de digestión (insuficiencia pancreática o hepáti­
Causas quirúrgicas ca) o de absorción (enteropatía o trastornos de transporte).
Medicamentos
Alcohol
En estos casos la cuantificación de o-xilosa en orina o en
Neuropatía autonómica sangre indica la ruta que debe seguirse. Una o-xilosa baja
Adaptado de Thomas PD, Forbes A, Green J, et al. Guidelines fer the investiga­ sugiere un problema de la pared intestinal y se requiere una
tion of chronic diarrhea. Gut. 2003;52:v1-v15. biopsia de intestino, en tanto que una normal sugiere diges­
tión deficiente y debe excluirse enfermedad pancreática.
b) Diarrea con sangre: puede presentarse con sangre oculta o
17. ¿Cuál es el primer paso en el abordaje diagnósti­
visible. En cualquier condición la colonoscopia con toma
co de una diarrea crónica?
de biopsia puede resolver el problema de diagnóstico. En
Lo primero que debe intentarse es identificar si la diarrea es algunos casos (síntomas anorrectales) una sigmoidoscopia
funcional u orgánica. Algunos datos que orientan para estable­ puede ser suficiente. En este grupo en particular el examen
cer el diagnóstico entre diarrea orgánica y funcional se mues­ en fresco de las heces proporciona información útil (leuco­
tran en el cuadro 11-8-5. citos, eritrocitos y parásitos) y debe solicitarse en todos los
casos.
18. ¿Cuáles exámenes de laboratorio son útiles para e) Diarrea acuosa: en estos casos la clave es la distinción en­
determinar la causa de la diarrea crónica? tre diarrea acuosa secretora y la osmótica. Aunque pocas
La utilidad de los métodos de diagnóstico varía según las ca­ veces utilizado, el método ideal es la determinación de la
racterísticas clínicas de la diarrea. Para fines prácticos la dia- brecha osmótica en heces fecales que se calcula al sustraer

722
SECCIÓN 11 GASTROENTEROLOGÍA @
a 290 müsm/kg el doble de la suma del sodio y del potasio c) Cuando el diagnóstico se ha establecido y no se cuenta con
medidos en una muestra de heces (290-2 [Na++ K+ ]). Una el tratamiento específico.
brecha mayor a 125 müsm/kg sugiere un componente os­
mótico, mientras que una menor a 50 müsm/kg tipifica a 21. ¿Cuál sería el tratamiento específico para las cau­
la diarrea secretora. sas más comunes de diarrea crónica?
Una medida indirecta es explorar si la diarrea cede con el a) Sobrepoblación bacteriana: uso de "ciclos" de antibióticos,
ayuno, característica de un componente osmótico, común en los cuales consisten en la administración de tetraciclina o
los problemas de absorción de carbohidratos (hipolactasia, con­ ciprofloxacino los primeros diez días del mes. El ciclo se re­
sumo exagerado de edulcorantes, sorbitol, fructosa entre otros), pite durante seis meses.
aceites (laxantes) y medicamentos o sus excipientes. b) Esprúe tropical: administración de ácido fólico y antibióti­
cos (tetraciclinas, sulfonamidas) por periodos prolongados
19. ¿Qué pruebas son indispensables para estable­ (más de seis meses). El esquema recomendado es tetracicli­
cer la causa del síndrome de absorción intestinal de­ na 100 mg cuatro veces al día durante seis semanas, junto
ficiente? con 10 mg de ácido fólico.
Existen al menos cuatro entidades que deben considerarse en c) Infección por Clostridium difficile: en aquellos pacientes con
el diagnóstico de diarrea crónica asociada con SAID, para las uso prolongado de antibióticos de amplio espectro es im­
cuales existen pruebas que permiten establecer el diagnóstico portante recordar que C. difficile puede ser el agente causal.
causal; éstas se muestran en el cuadro 11-8-6. El tratamiento de primera elección es metronidazol 500
mg oral cada seis a ocho horas, por siete a diez días. Dife­
20. ¿Es recomendable el uso de antidiarreicos en el rentes estrategias han sido recomendadas para el mane­
manejo de la diarrea crónica? jo de la recurrencia por Clostridium difficile; en estos casos
se recomienda el uso de vancomicina oral a dosis de 125 a
El tratamiento empírico con antidiarreicos para disminuir el
250 mg cada seis horas. Se debe recordar que los probióti­
número de las evacuaciones durante periodos cortos se debe
cos también son útiles.
considerar necesario en las siguientes situaciones:
d) Intolerancia a disacáridos: en el caso de documentarse hi­
a) Como medida adyuvante para evitar el desarrollo de com­ polactasia, se recomienda la suspensión de los lácteos y sus
plicaciones mientras se efectúa el diagnóstico. derivados.
b) Cuando en la evaluación inicial no se pueda determinar la e) Enfermedad celiaca y malabsorción: la restricción de gluten

1
causa de la diarrea. en la dieta.

CUADRO 11-8-6.
ENTIDADES Y PRUEBAS DIAGNÓSTICAS PARA ESTABLECER LA CAUSA DEL SÍNDROME DE ABSORCIÓN
INTESTINAL DEFICIENTE
FACTOR PREDISPONENTE Y/O
MANIFESTACIONES CLÍNICAS QUE
ENTIDAD ESTABLECEN LA SOSPECHA PRUEBA DIAGNÓSTICA HALLAZGOS
Sobrepoblación • Aclorhidria • Cultivo de aspirado duodenal • Presencia >100 000 UFC de
bacteriana • Diabetes • Prueba de hidrógeno en aire espirado anaerobios y Gram H
• Fístulas intestinales (aliento) con lactulosa • Elevación a más de 20 ppm en los
• Resección ileal primeros 90 minutos de la prueba
• lnmunosupresión
Absorción • Resección ileal • Medición directa de ácidos biliares en • Elevación por arriba del punto de corte
deficiente de • Enfermedad de Crohn heces con carbono 14 • Evidencia de retención del material
sales biliares • Enteritis por radiación • Gammagrafía con ácido homocólico­ marcado en colon
• Tuberculosis intestinal taurina marcado con selenio
Enfermedad • Historia familiar • Biopsia duodenal • Linfocitosis intraepitelial >25 linfocitos
celiaca • Autoinmunidad • Anticuerpos específicos como anti­ por campo
• Dermatitis herpetiforme transglutaminasa tisular (lgA-lgG) o • Atrofia vellositaria
• Osteoporosis anti-endomisio (lgA-lgG) • Positividad. Se considera el estándar de
• Anemia por deficiencia de hierro • Determinación de haplotipo HLA DQ2/ oro para el diagnóstico
de causa inexplicable DOS • Su positividad establece el riesgo
• Infertilidad genético
• Talla baja
Esprúe tropical • Viajar o residir en áreas • Biopsia duodenal • Atrofia vellositaria, eosinofilia duodenal
tropicales y subtropicales • Determinación de ácido fólico • Niveles bajos según el punto de corte
• Anemia megaloblástica

723
e MANUAL PARA EL EXAMEN NACIONAL DE RESIDENCIAS MÉDICAS

EXAMEN DE HECES sa diarrea intermitente o síntomas de varias semanas y es una


causa menos frecuente de diarrea aguda verdadera.
22. ¿Es útil el examen de leucocitos en heces?
Sí. La mayoría de los enteropatógenos que invaden la mucosa 27. ¿Para qué se usa la determinación de toxina A-8
produce diarrea inflamatoria que suele acompañarse de leuco­ de Clostridium difficile en heces?
citos en las heces. Los gérmenes que no la invaden producen Esta prueba debe realizarse en todo paciente con diarrea agu­
una diarrea acuosa en donde los leucocitos son pocos o nulos. da luego del empleo de antibióticos, en aquellos que han sido
Mediante su identificación es posible distinguir ambas varieda­ recientemente hospitalizados y en quienes presentan diarrea
des. Sin embargo, la decisión del empleo de antibióticos no de­ durante la hospitalización, ancianos e inmunodeprimidos.
be basarse sólo en esta prueba y deben considerarse los facto­ BIBLIOGRAFÍA
res de riesgo del enfermo y su estado de salud en ese momento. Aslam S, Musher DM. An update in diagnosis, treatment and prevention on
Clostiridum difficile associated-disease. Clin Gastroenterol North Am.
23. ¿Qué es la calprotectina y para qué se usa su de­ 2006;35:315-35.
terminación en heces? Bertomeu A, Ros E, Barragan V, Sachje L, Navarro S. Chronic diarrhea with
normal stool and colonic examinations: organic or functional? J Clin Gas­
La calprotectina es una proteína derivada de los neutrófilos y troenterol. 1991;13:531-6.
es quizás el marcador más sensible en heces para detectar in­ Eherer AJ, Fordtran JS. Fecal osmotic gap and pH in experimental diarrhea
flamación intestinal, con menor variabilidad interobservador ofvarious causes. Gastroenterology. 1992;103:545-51.
y mayor especificidad, ya que correlaciona con la presencia de Fine KD, Schiller LR. AGA technical review on the evaluation and manage­
ment ofchronic diarrhea. Gastroenterology. 1999; 116: 1464-86.
granulocitos o su migración a la mucosa.
Centro Nacional de Excelencia Tecnológica en Salud. Guía de práctica clíni­
ca. Atención, diagnóstico y tratamíento de la diarrea aguda en adultos en
24. ¿En qué pacientes y para qué debe solicitarse el el primer nivel de atención. México: CENETEC; 2011.
examen de cultivo de heces (coprocultivo)? Mery CM, Robles Díaz G. Tropical Sprue: a commonly overlooked diagnosis.
Rev Medicine ofthe Americas. 2001;2:29-36.
Los coprocultivos deben solicitarse en todo paciente con fie­
Pimentel M. Review ofrifax.imin as treatment for SIBO and IBS. Expert Opin
bre elevada (mayor a 38.5 ºC), disentería o leucocitos en mo­ Investig Drugs. 2009; 18:349-58.
co fecal, en enfermos inmunocomprometidos, con enfermeda­ Rana SV, Bhardwaj SB. Small intestinal bacteria] overgrowth. Scand J Gas­
des graves asociadas o debilitantes y para fines epidemiológicos troenterol. 2008;43: 1030-7.
durante brotes de diarrea aguda. Remes-Troche JM, Gómez-Escudero O, Bielsa-Fernández MV, Garrido Pal­
ma J, Méndez-Gutiérrez T, Vázquez-Ávila I. Clinical guidelines for diagno­
sis and treatment ofchronic diarrhea. Epidemiology, etiology and classifi­
25. ¿Para qué sirve el examen coprológico? cation. Rev Gastroenterol Mex. 2010 Apr-Jun;75(2):223-5.
El examen coprológico, además de identificar parásitos o hue­ Remes-Troche JM, Sagols-Méndez GA, Trujeque-Franco MA. Clínica! guide­
vecillos, informa sobre azúcares reductores (frecuentes en la lines for diagnosis and treatment of chronic diarrhea. Approach of the
patient with chronic diarrhea and special situations. Rev Gastroenterol
digestión deficiente de hidratos de carbono), ácidos grasos y Mex. 2010 Apr-Jun;75(2):231-6.
grasa neutra (sugerentes de esteatorrea), sangre oculta y medi­ Remes-Troche JM, Uscanga-Dominguez LF, Icaza-Chávez ME, Nogueira-de
ción del pH (absorción deficiente de carbohidratos). Rojas JR, Peláez-Luna M, Rivera-Ramos JF. Clinical guidelines for diag­
nosis and treatment of chronic diarrhea. Diagnosis. Rev Gastroenterol
26. ¿Es útil el análisis coproparasitoscópico? Mex. 2010 Apr-Jun;75(2):226-30.
Simko V. Fecal fat microscopy. Acceptable predictive value in screening for
La búsqueda de huevos, larvas y parásitos se emplea con fre­ steatorrhea. Am J Gastroenterol. 1981;75:204-8.
cuencia en el medio, pero su utilidad real en el paciente con Thomas PD, Forbes A, Green Jet al. Guidelines for the investigation ofchro­
diarrea aguda es limitada. La excreción intermitente de hueve­ nic diarrhea. Gut. 2003;52 Suppl 5:v l -v l5.
cilio y parásitos, el manejo especial que se debe tener con las Tysk C, Bohr J, Nyhlin N, Wickbom A, Eriksson S. Diagnosis and manage­
ment ofmicroscopic colitis. World J Gastroenterol. 2008; 14:7280-8.
muestras y la experiencia del personal que realiza el examen Uscanga L, Galván E, Robles G. Diagnóstico por laboratorio del síndrome de
son factores que contribuyen a que los resultados pocas veces absorción intestinal deficiente de origen intestinal. Rev Gastroenterol
sean confiables. Por otro lado, la mayoría de los parásitos cau- Mex. 1993;58:96-102.

724
SECCIÓN 11 GASTROENTEROLOGÍA @
CAPÍTULO 11.9
MALABSORCIÓN
Jesús Gerardo López Gámez • Brenda Patricia Rendón Martínez

DEFINICIÓN CUADRO 11-9-1.


MANIFESTACIONES CLÍNICAS POR DÉFICIT
1. ¿Qué es la malabsorción? DE NUTRIENTES
Se entiende por malabsorción a la alteración de la absorción
MANIFESTACIONES CLÍNICAS DÉFICIT DE NUTRIENTES
mucosa de los nutrientes, a diferencia de la maldigestión que
Debilidad, pérdida de peso Deficiencia proteico-calórica
se caracteriza por la hidrólisis defectuosa de nutrientes; esta
diferencia puede ser útil a nivel fisiopatológico, pero la presen­ Palidez, hiperqueratosis, Folatos, hierro, vitaminas 81 y 812 ,
petequias, alopecia vitaminas A, C y K, zinc
tación y las manifestaciones clínicas son similares.
Ceguera nocturna Vitamina A
2. ¿Cuáles son las fases de la absorción intestinal? Glositis, gingivitis, disgeusia, Vitaminas A, C, K, complejo 8,
queilosis zinc
a) Fase luminal, donde ocurre la solubilización de las grasas y
la hidrólisis a sustratos simples, en la que participan la ami­ Tetania, parálisis, demencia Complejo 8. vitamina E
lasa salival, la pepsina gástrica y las secreciones pancreato­ Anemia Hierro, vitamina 812 , ácido fólico
biliares. Osteoporosis Calcio, vitamina D
b) Fase mucosa, donde se produce la hidrólisis de carbohidra­
tos y péptidos facilitada por las disacaridasas, oligosacari­
dasas y peptidasas del borde en cepillo de los enterocitos. te examen tiene una sensibilidad de 100% y especificidad de
e) Fase absortiva, en donde se realiza el paso de nutrientes des­ 96%. La presencia de más de l00 glóbulos de grasa mayor a
de las microvellosidades a la circulación linfática y venosa. 6 µm de diámetro por campo de alto poder en las heces indica
un incremento en la excreción de grasa fecal, lo cual se corre­
3. ¿Dónde se realiza la absorción de hierro, ácido laciona con la malabsorción de grasas.
fálico y vitamina 8 12 ?
La absorción de grasas, hidratos de carbono, proteínas, mine­ CUADRO 11-9-2.

1
rales, calcio, magnesio, vitaminas y oligoelementos ocurre en CAUSAS DE MALABSORCIÓN
los dos primeros tercios del intestino delgado. El hierro se
absorbe sobre todo en duodeno. El ácido fólico se absorbe en FASE LUMINAL
yeyuno proximal. La absorción de vitamina B 12 y sales bilia­ a) Insuficiencia pancreática Pancreatitis crónica, fibrosis
res ocurre en el íleon terminal. El colon se encarga de la absor­ exocrina quística
b) Déficit de sales biliares Colestasis crónica, sobrecrecimiento
ción de agua, electrolitos y ácidos grasos de cadena corta. bacteriano, resección intestinal

FASE MUCOSA
4. ¿Cuáles son los hallazgos clínicos de la malabsor­
ción? a) Enfermedad difusa de la Enfermedad celiaca, esprúe tropical,
mucosa enfermedad de Crohn, enteritis por
El cuadro de malabsorción se caracteriza por pérdida ponde­ radiación
ral, distensión abdominal, presencia de evacuaciones volumino­ b) Infecciones intestinales Enfermedad de Whipple, enteropatía
sas, fétidas, postprandiales, con restos de alimentos (lientería) asociada con VIH (Cryptosporidium,
lsospora), giardiosis
y grasa (esteatorrea), así como manifestaciones secundarias a
c) lnfiltrativas Mastocitosis, gastroenteritis
déficit de nutrientes. Véase cuadro 11-9-1. eosinofílica, amiloidosis, linterna
intestinal
5. ¿Cuáles son las diferentes causas de malabsor­ d) Fármacos Colchicina, colestiramina, neomicina,
ción? AINE

La malabsorción puede deberse a problemas en la fase luminal, FASE ABSORTIVA (TRANSPORTE)


mucosa, absortiva o de transporte de la absorción intestinal, a) Defectos de transporte Cistinuria de Hartnup
cuyas causas se enumeran en el cuadro 11-9-2. b) Alteraciones en el Abetalipoproteinemia
procesamiento epitelial
c) insuficiencia vascular Ateromatosis, vasculitis
6. ¿Cuáles métodos son útiles para la detección de
d) Obstrucción linfática Linfangiectasia intestinal,
malabsorción? endometriosis intestinal,
El examen microscópico de las heces con tinción de Sudán es mesenteritis retráctil, tuberculosis
y sarcoidosis mesentérica
el mejor método para detectar la malabsorción de grasa. Es-

725
e MANUAL PARA EL EXAMEN NACIONAL DE RESIDENCIAS MÉDICAS

Una prueba de bajo costo, accesible y práctica es la cuanti­ y fluoresceína, que es absorbida en el intestino delgado, con­
ficación del j3-caroteno en suero. Su absorción es similar a la jugada en el hígado y eliminada en la orina. Los pacientes
de las grasas, por lo que requieren de la hidrólisis intraluminal con insuficiencia pancreática presentan concentraciones ba­
(digestión) y la integridad de la mucosa (absorción). Las con­ jas: menos de 20% de fluoresceína en orina de diez horas de
centraciones séricas de j3-caroteno varían según el género y el recolección.
estado nutricio. Los valores por arriba de 200 mg en mujeres
o de 160 mg en hombres se consideran normales, mientras que 10. ¿Qué es el síndrome de sobrecrecimiento bacte­
las concentraciones por debajo de 60 mg en mujeres o 50 mg riano?
en hombres apoyan el diagnóstico de malabsorción. En pa­ Es un síndrome que se caracteriza por malabsorción de nu­
cientes con niveles séricos de j3-caroteno "intermedios" es ne­ trientes, asociado con un excesivo número de bacterias en el
cesario realizar una cuantificación de grasa en materia fecal. intestino delgado, lo cual produce alteraciones en la digestión
y la absorción intestinal. Las bacterias que colonizan al intes­
7. ¿Cuál es el mejor examen cuantitativo para el diag­ tino delgado son capaces de hidrolizar a la D-xilosa, por lo que
nóstico de malabsorción? pacientes con sobrecrecimiento bacteriano presentan concen­
La prueba de Van de Kamer, que consiste en la recolección de traciones bajas de D-xilosa, aun cuando la pared intestinal es
grasa fecal en 72 horas tras una dieta consistente en 100 gra­ normal en términos estructurales.
mos de grasa durante los tres días. Por lo regular la absorción Dentro de las causas que favorecen el sobrecrecimiento bac­
debe ser aproximadamente de 93% de la grasa consumida, por teriano se encuentran las anormalidades anatómicas (diver­
lo que 7 gramos o menos de grasa deberían aparecer en el ex­ tículos, estenosis, asa ciega), los trastornos de motilidad (pseu­
cremento después de 24 horas. Si la cantidad es mayor a 7 gra­ doobstrucción intestinal, neuropatía autonómica por diabetes
mos se confirma la presencia de malabsorción. mellitus, colagenosis, etcétera) y la alteración de los mecanis­
mos de defensa (hipoclorhidria, aclorhidria, resección de la vál­
8. Si ya se confirmó la malabsorción, ¿cómo se pue­ vula ileocecal, inmunodeficiencia, malnutrición grave, etcé­
de diferenciar un trastorno intraluminal de una altera­ tera).
ción en la pared intestinal? En la clínica se caracteriza por diarrea acuosa con o sin es­
Con el test de la D-xilosa. La D-xilosa se absorbe por difusión teatorrea, borborigmos, meteorismo y flatulencia. Se pueden
pasiva y se utiliza para evaluar la capacidad de absorción de acompañar de pérdida de peso y anemia megaloblástica secun­
la pared intestinal, ya que no requiere la presencia de enzimas daria a deficiencia de cianocobalamina, ya que es fijada por
pancreáticas para su digestión. Los pacientes deben tomar las bacterias intestinales e impiden su absorción; así como de
una dosis de 25 gramos de xilosa disuelta en agua. Se extrae una hipocalcemia, tetania, osteomalacia, neuropatía periférica y
muestra de sangre una y dos horas después de su ingesta o se ataxia.
recolecta orina durante cinco horas. De existir alteración en la
pared intestinal se obtendrán concentraciones bajas de D-xilo­ 11. ¿Cómo se diagnostica el sobrecrecimiento bacte­
sa en sangre ( <20 mg/dL) u orina ( <5 gramos/5 horas). riano?
En estos casos es necesario investigar las alteraciones ana­ El estándar de oro para el diagnóstico de sobrecrecirniento bac­
tómicas mediante estudios radiológicos, endoscópicos e his­ teriano es el cultivo cuantitativo del aspirado del intestino delga­
tológicos para determinar la presencia de lesión en la mucosa do, con la demostración de altas concentraciones ( > 10 5 UFC/
intestinal o el sobrecrecimiento bacteriano. mL) de bacterias en el fluido intestinal.
Con el fin de evitar la intubación intestinal se han propuesto
9. ¿Cómo se interpreta una prueba de D-Xilosa nor­ métodos diagnósticos alternativos, como el test del hidrógeno
mal? espirado con lactulosa, en el que un alza en el nivel de hidróge­
La normalidad en los resultados de D-xilosa indica la presencia no espirado de 12 ppm por sobre el valor base es tomada como
de un trastorno intraluminal de maladigestión secundario a in­ diagnóstico de sobrecrecimiento bacteriano; dada la capacidad
suficiencia pancreática exocrina o por déficit de sales biliares. bacteriana para desconjugar las sales biliares, el uso de ácido
Aunque en algunos casos el diagnóstico de insuficiencia pan­ glicocólico marcado con 14C es utilizado para la realización de
creática exócrina, como en la pancreatitis crónica, es práctico, la prueba del aliento con ácidos biliares como un método alter­
en otros es necesario recurrir a pruebas de función pancreáti­ nativo.
ca. Las pruebas de mayor sensibilidad son las pruebas directas
mediante la canulación del ámpula de Vater y la estimulación 12. ¿Cuál es el tratamiento del sobrecrecimiento bac­
con secretina o colecistocinina para cuantificar la produc­ teriano?
ción de bicarbonato y enzimas pancreáticas. El tratamiento de estos pacientes se enmarca en tres pilares
Las pruebas indirectas emplean sustratos que requieren la básicos: evaluar y tratar las deficiencias nutricionales, corregir
hidrólisis intraluminal. El dilaurato de fluoresceína, por acti­ la enfermedad adyacente asociada y erradicar la proliferación
vidad de las enzimas pancreáticas, se separa en ácido !áurico bacteriana.

726
SECCIÓN 11 GASTROENTEROLOGÍA @
El tratamiento antibiótico es en su mayoría empírico y de­ 15. ¿Cuál es el tratamiento de la enfermedad celia­
be cubrir la flora aeróbica y anaeróbica. Los antibióticos más ca?
utilizados son ciprofloxacino, metronidazol, doxiciclina, tetra­ El tratamiento actual para la enfermedad celiaca es una dieta
ciclina, neomicina y rifaximina. El tratamiento por siete días estricta libre de gluten de por vida. Por ello se debe evitar trigo,
puede mejorar los síntomas en 46 a 90% de los pacientes. De­ cebada y centeno, así como todos sus derivados. Debe aconse­
bido a que ciertas condiciones adyacentes favorecedoras de so­ jarse a los pacientes el consumo de alimentos altos en hierro y
brecrecimiento bacteriano no son reversibles, los pacientes pre­ folatos, en especial si se documenta una deficiencia de estos
sentarán cuadros recurrentes, por lo que necesitarán repetir el minerales. El arroz y el maíz pueden ser parte de una dieta li­
tratamiento antibiótico o recibir un tratamiento continuo; en bre de gluten.
este caso se sugiere rotar los antibióticos para prevenir el desa­ La mayoría de los pacientes tiene una respuesta clínica rá­
rrollo de resistencia. pida a una dieta libre de gluten dentro de las dos primeras se­
manas, aunque la tasa de respuesta puede variar.
13. ¿Qué es la enfermedad celiaca? La razón más común en la persistencia de los síntomas en
Enteropatía que afecta el intestino delgado en personas pre­ pacientes con enfermedad celiaca casi siempre es provocada
dispuestas genéticamente, provocada por la activación de la res­ por la ingesta inadvertida de gluten, pero los pacientes que no
puesta inmunitaria humoral y celular, precipitada por la expo­ responden a una dieta libre de gluten, ni al inicio del diagnós­
sición al gluten, proteína contenida en diversos granos como tico ni después de volverse refractarios a la terapia de dieta,
trigo, centeno y cebada. podrían responder al uso de esteroides u otros inmunosupre­
La susceptibilidad genética está determinada por la alta con­ sores, como la azatioprina, la ciclofosfamida o la ciclosporina.
cordancia de hasta 70% que se encuentra entre gemelos mo­ Los pacientes que no responden a ningún tratamiento enfren­
nocigotos, además de su asociación con ciertos tipos de antí­ tan un mal pronóstico.
genos leucocitarios tipo 11 (HLA); 95% de los pacientes con El linfoma es la complicación maligna más común de la en­
enfermedad celiaca presenta HLA-DQ2, mientras que el resto fermedad celiaca. El diagnóstico requiere un alto índice de sos­
de los pacientes presenta HLA-DQ8. pecha puesto que la presentación puede ser insidiosa o abrup­
Clínicamente se caracteriza por diarrea crónica, pérdida de ta y la apariencia histológica puede ser indistinguible de la
peso y distensión abdominal; puede estar acompañada de ane­ enfermedad celiaca.
mia por deficiencia de hierro, dermatitis herpetiforme, neuro­
patía periférica, osteoporosis e infertilidad. 16. ¿Qué es la enfermedad de Whipple?
La dermatitis herpetiforme se considera la manifestación La enfermedad de Whipple es un padecimiento sistémico cró-
cutánea más común y específica de la sensibilidad al gluten en nico causado por Tropheryma whipplei, una bacteria gramposi-1
pacientes con enfermedad celiaca; se caracteriza por pápulas tiva de la familia deActinomyces; por lo regular considerada un
eritematosas y placas de urticaria coronadas por vesículas que trastorno gastrointestinal, la enfermedad de Whipple causa
pueden ser sustituidas a menudo por erosiones a causa del pru­ principalmente malabsorción, aunque puede afectar cualquier
rito intenso. parte del organismo, incluso el corazón, los pulmones, el cere-
bro, las articulaciones y los ojos.
La presentación más común incluye pérdida de peso(90%),
diarrea(> 70%) y artralgias(> 70%). El compromiso cardiaco
14. ¿Cómo se realiza el diagnóstico de enfermedad
incluye falla congestiva, pericarditis y enfermedad valvular
celiaca?
Se dispone de un panel de exámenes serológicos para detectar
del corazón(30%). La hematoquecia es rara, pero el sangrado
la enfermedad en aquellos pacientes con sospecha clínica de
oculto se ha detectado en alrededor de 80% de los pacientes
enfermedad celiaca: los anticuerpos antiendomisio (lgA) tie­
con enfermedad de Whipple. Las manifestaciones más comu­
nen una especificidad de 100% para enfermedad celiaca, mien­
nes en el sistema nervioso central(5%) son cefalea, ataxia, de­
tras su sensibilidad es de 85%; los anticuerpos antigliadina
mencia progresiva, cambios de la personalidad, meningitis y
(lgG) tienen una sensibilidad de 69 a 85% y una especificidad
neuropatías periféricas, así como los síntomas oculares y la
de 90%; el anticuerpo antitransglutaminasa(lgA) es una prue­
uveítis.
ba más sensitiva que detecta 98 a 100% de los pacientes con
enfermedad celiaca.
Los valores séricos de anticuerpos anti-transglutaminasa 17. ¿Cómo se diagnostica la enfermedad de Whipple?
tienen una sensibilidad y especificidad para el diagnóstico de El mejor medio de diagnóstico es el estudio de la biopsia duode­
enfermedad celiaca con lesión de las vellosidades intestinales nal por endoscopia. Los estudios endoscópicos pueden revelar
cercanas a 95%. Por tanto, debe realizarse biopsia duodenal hipertrofia de las vellosidades intestinales con alternación de
mediante una endoscopia digestiva alta. Sólo la endoscopia con áreas de mucosa denudada en duodeno, así como lesiones múl­
biopsia del intestino delgado más una serología positiva de la tiples con acumulación de placas y parches de vellosidades en
EC brindan un diagnóstico definitivo. el yeyuno e íleon proximal.

I 727
e MANUAL PARA EL EXAMEN NACIONAL DE RESIDENCIAS MÉDICAS

En la evaluación histopatológica de las biopsias obtenidas nes UI por día) o ceftriaxona (2 gramos diarios) por dos sema­
por endoscopia se observa a la lámina propia expandida por nas, para que el porcentaje de recurrencias sea menor.
infiltrados difusos de gran cantidad de macrófagos que contie­
BIBLIOGRAFÍA
nen partículas granulares citoplasmáticas, positivas a la tinción Dewar DH, Ciclitira PJ. Clínica] features and diagnosis of celiac disease. Gas­
con ácido peryódico de Schiff (PAS), y también puede ser loca­ troenterology. 2005; 128:S 19-S24.
lizada a través de ADN mediante reacción en cadena de poli­ Fenollar F, PuéchalX, RaoultD. Whipple's disease. N Engl J Med. 2007;356:55.
merasa (PCR), que en la actualidad es el estándar de oro. H6genauer C, Hammer HF. Mala digestión y malabsorción. 8a. ed. Fortran's
Gastrointestinal and Liver Disease. 9th ed. New York: Saunders; 2008. p.
2199-241.
18. ¿Cuál es el tratamiento para la enfermedad de Quera R, Quigley EM, Madrid AM. Small intestinal bacteria! overgrowth. An
Whipple? update. Rev Med Chile. 2005;133:1361-70.
Se han utilizado múltiples esquemas de antibióticos basados Rubio-Tapia A, Barton SH, Rosenblatt JE, Murray JA. Prevalence of small in­
testine bacteria! overgrowth diagnosed by quantitative culture of intestinal
sobre todo en cloranfenicol, estreptomicina, isoniazida y peni­
aspirate in celiac disease. J Clin Gastroenterol. 2009;43: 157.
cilina. Más adelante se observó que la tetraciclina era más efec­ Sands BE. From symptom to diagnosis: clinical distinctions among various
tiva; sin embargo, con el tiempo se presentaban recurrencias al forms of intestinal inflammation. Gastroenterology. 2004; 126: 1518-32.
suspender el tratamiento, en especial si había afección del sis­ Schiller LR. Chronic diarrhea. Gastroenterology. 2004; 127:287-93.
tema nervioso central. En la actualidad se utilizan antibióticos Swartz MN. Whipple's disease-past, present, and future. N Engl J Med. 2000;
342:648.
que cruzan la barrera hematoencefálica, como son trimetoprim/
Wildt S, Norby Rasmussen S, Lysgard Madsen J, Rumessen JJ. Bite acid malab­
sulfametoxazol (recomendado en 160/800 mg dos veces al día) sorption in patients with chronic diarrhea. Scand J Gastroenterol. 2003;38: 826.
por uno o dos años, pero suele iniciarse con estreptomicina in­ World Gastroenterology Organization Practice Guidelines: Enfermedad celía­
tramuscular (1 gramo diario) junto con penicilina G ( 1.2 millo- ca. 2001.

CAPÍTULO 11.10
ENFERMEDAD INFLAMATORIA INTESTINAL
Hairé Manzano Cortés

COLITIS ULCERATIVA CRÓNICA INESPECÍFICA ciado con CUCI con una evolución más agresiva y mayor ries­
go de cáncer colorrectal. Existen también sitios genéticos que
1. ¿Cuál es la edad de presentación de la EII y la po­ se comparten con otras enfermedades autoinmunitarias.
blación más afectada?
La enfermedad tiene dos picos de edad de los 20 a los 40 años 3. ¿En qué consiste el factor inmunológico en la etio­
y de los 60 a 80 años de edad, con mayor frecuencia en la eta­ logía de la EII?
pa productiva y con una edad promedio de presentación a los La respuesta innata es dada por las células del epitelio intesti­
30 años. Se presenta más a menudo en países industrializa­ nal y macrófagos, pero en la EII hay alteraciones en la barrera
dos y zonas urbanas, con mayor prevalencia e incidencia en epitelial con alteración en las uniones intercelulares con incre­
los países del norte de América y Europa; sin embargo, los ju­ mento en la permeabilidad de antígenos; de igual manera hay
díos ashkenazí son los más afectados. alteraciones en la fagocitosis y la acción lisosomal.
Las células dendríticas, los monocitos y los macrófagos ac­
2. ¿Cuáles son los genes causantes de la EII? tivados denominados células presentadoras de antígenos li­
Los factores genéticos en la EII se relacionaron en un inicio beran diferentes interleucinas que activarán a los linfocitos T,
con el gen NOD 2 (CARO 15), localizado en el cromosoma generando mayor cantidad de citocinas e interleucinas proin­
16 que condiciona reclutamiento y activación celular, así co­ flamatorias, como el factor de necrosis tumoral alfa (TNF-a),
mo disminución de la apoptosis celular. En la actualidad por el interferón gamma (IFN-y ), las interleucinas 17, 22 y 23, con
estudios genéticos de asociación se reconocen 71 sitios ge­ un desequilibrio con las interleucinas antiinflamatorias deter­
néticos que confieren susceptibilidad a EC en 17 cromoso­ minadas por los linfocitos T reguladores (LTreg).
mas. En la colitis ulcerativa crónica inespecífica (CUCI) se Existe también un incremento de las moléculas de adhesión
han identificado 47 sitios genéticos y 17 sitios se comparten en endotelial (ICAM-1 y MadCAM-1), lo cual aumenta la migra­
ambas enfermedades; el haplotipo HLA DRB*0103 se ha aso- ción leucocitaria. La respuesta adquirida se manifiesta ante to-

728
SECCIÓN 11 GASTROENTEROLOGÍA @
do por linfocitos T helper o colaboradores (LTh). Se considera CUADRO 11-10-1.
que los LTh 1 y LTh17 son predominantes en la enfermedad CRITERIOS DE TRUELOVE Y WITTS

---·M·iUHM--
de Crohn y los LTh2 para CUCI; sin embargo, están presentes
en ambas enfermedades.
Heces con sangre/día <4 Cuatro o más ?:6 y
si
4. ¿Cuáles son los factores ambientales que condi­
Frecuencia cardiaca <901pm :$901pm >901pm o
cionan la EII?
Temperatura <37.5 ºC :$37.8 ºC >37.8 ºC o
Dentro de los factores ambientales se ha considerado el taba­
Hemoglobina >11.5 g/dL ?:10.5 g/dL <10.5 g/dL o
quismo como factor protector para el desarrollo de CUCI, pe­
ro incrementa el riesgo para desarrollar enfermedad de Crohn. VSG <20 mm/h :$30 mm/h >30 mm/h o
También se ha mencionado que la apendicectomía disminuye PCR Normal :$30 mg/L >30 mg/L
el riesgo de CUCI y colectomía en estos pacientes, no así en la VSG: velocidad de sedimentación globular; PCR: proteína C reactiva.
enfermedad de Crohn. La teoría de la higiene o sanidad excesi­
va se ha considerado en países desarrollados por la exposición
CUADRO 11-10-2.
tardía a antígenos por la migración, no así en países en desa­
ESCALA DE MAYO
rrollo.El uso de hormonas orales incrementa el riesgo deEII;
de igual manera, la lactancia materna y la dieta baja en fibra o
rica en azúcares refinados pueden ser factores condicionantes
iiii·iiiif:M·I----
Frecuencia Normal 1 a 2/día > 3 a 4/día > 5/día >
deEII. de las de- normal normal normal
posiciones
5. ¿Cuáles son las manifestaciones extraintestinales Sangrado Ninguno Manchas Obvio Sangre en su
de la EII? rectal mayoría

Un tercio de los pacientes tiene una o más manifestaciones ex­ Mucosa Normal Friabilidad Friabilidad Sangrado es-
traintestinales, por lo cual se considera una enfermedad sisté­ leve, modera- pontáneo,
eritema, da, erite- úlceras
mica. Los síntomas extraintestinales se dividen en dos grupos: disminu- ma mar-
las manifestaciones extraintestinales propiamente y las com­ ción de cado,
plicaciones extraintestinales. Dentro de las manifestaciones se patrón ausencia
encuentran la artritis periférica, el eritema nodoso y el pioder­ vascular de patrón

1
vascular
ma gangrenoso que se relaciona con la actividad, la colangitis
esclerosante primaria (más asociada con CUCI) y la espondili­ Evaluación Normal Leve Moderada Grave
tis anquilosante independientes de la actividad de la enferme­ global del
médico
dad, así como uveítis, epiescleritis y tromboembolismo venoso.
Las complicaciones extraintestinales son causadas por la en­
fermedad o los tratamientos establecidos, como deficiencia de CUADRO 11-10-3.
nutrientes, anemia, artropatías, nefrotoxicidad, fibrosis pulmo­ CLASIFICACIÓN DE MONTREAL DE CUCI
nar y enfermedad hepática grasa relacionadas con fármacos.
ii!Miiii·ll·iHlil1=M!i·iii DESCRIPCIÓN
6. ¿Cuáles son los índices de actividad clínica y en­ E1 Proctitis Afectación limitada al recto (esto es, la
extensión proximal de la inflamación
doscópica utilizados en la CUCI?
es distal a la unión rectosigmoidea)
Los criterios de Truelove y Witts, establecidos en 1995, permi­
E2 Lado Afectación limitada a la proporción del
ten determinar la gravedad clinica de la enfermedad y se confor­ izquierdo colon distal hasta el ángulo esplénico
man de parámetros clínicos y de laboratorio (cuadro 11-10-1 ). (colitis "distal")
LaEscala de Mayo también determina la gravedad sin embar­ E3 Extensa La afectación se extiende más allá de la
go, también incluye el aspecto endoscópico y en la actualidad flexión esplénica, incluso pancolitis
es la más utilizada en estudios clínicos (cuadro 11-10-2).

• Reactantes de fase aguda, como la PCR y la VSG; sin


7. ¿Cuál es la clasificación de Montreal de la CUCI?
embargo, son inespecificos y su elevación correlaciona
Véase cuadro 11-10-3. con los síntomas gastrointestinales.
• Anticuerpos: anti-Saccharomyces cereviciae (ASCA) que
8. ¿Cuáles son los principales biomarcadores de la se presentan en enfermedad de Crohn y los anticuerpos
EII? antineutrófilos perinucleares (pANCA) en CUCI; no
a) Marcadores serológicos. obstante, tienen baja sensibilidad y especificidad.

729
e MANUAL PARA EL EXAMEN NACIONAL DE RESIDENCIAS MÉDICAS

b) Marcadores fecales: lactoferrina fecal y calprotectina fecal. 13. ¿Qué significa colitis refractaria o dependiente a
• Calprotectina fecal con mayor evidencia científica y que esteroides?
no sólo correlaciona con la actividad sino también per­ En pacientes que inician tratamiento convencional con este­
mite diferenciar la EII de un trastorno funcional, como roides debe considerarse refractariedad a los mismos cuan­
predictor de recaída y respuesta al tratamiento. do tienen enfermedad activa, a pesar de la prednisolona hasta
0.75 mg/kg/día en un periodo de cuatro semanas, aplicable
9. ¿Cuál es el manejo de una proctitis y una colitis también en pacientes con EC. Sin embargo, en la actualidad
izquierda? se considera que un paciente con PCR mayor de 45 y más de
En los pacientes con proctitis puede usarse mesalazina tópi­ ocho evacuaciones al tercer día no está respondiendo y requie­
ca y, ante mayor intensidad, considerar terapia sistémica que re cambio de terapia.
puede manejarse de manera ambulatoria. Si la actividad es Los pacientes con colitis dependiente de esteroides son in­
moderada a grave, considerar esteroides orales. En una colitis capaces de reducir los esteroides por debajo del equivalente
izquierda con actividad leve están indicados los esteroides ora­ de prednisolona de 10 mg/día dentro de tres meses de esteroi­
les y la mesalazina oral o enemas, pero en estadios graves des iniciales o tienen una recidiva tres meses después de haber
requiere esteroides Ny mesalazina, aunque debe considerarse interrumpido los esteroides. Este criterio aplica también en la
inicio de inmunomoduladores, ya que la acción es lenta y con enfermedad de Crohn.
efecto hasta dos meses después. En los casos moderados a
graves que no respondan al tratamiento con esteroides está in­ ENFERMEDAD DE CROHN
dicada la ciclosporina.
14. ¿Cuál es el género más afectado por la enferme­
dad de Crohn?
10. ¿Cuál es el manejo de CUCI con actividad gra­
Aunque se considera que afecta a hombres y mujeres por igual
ve?
con una relación 1: 1, hay artículos que reportan ligero predo­
En pacientes con actividad grave, sin tomar en cuenta la lo­
minio en mujeres.
calización, debe considerarse el uso de esteroides No terapia
biológica, la cual contempla estrategias dirigidas a inhibir la
inflamación con la subsecuente liberación de citocinas. En el 15. ¿Cuál es el sitio anatómico más afectado en la en­
medio sólo se cuenta con anti-TNF, como adalimumab; sin em­ fermedad de Crohn?
bargo, existen otros fármacos que inhiben interleucinas y mo­ La región ileocolónica es la porción más afectada en 45 a 55%
léculas de adhesión. de los casos y los síntomas característicos son de malabsor­
Dado que estos pacientes tienen riesgo de tromboembolis­ ción. En menor proporción ocurre en el intestino delgado y
mo y presentan trombocitosis, requieren tratamiento con he­ ofrece un cuadro similar. En el colon el cuadro puede semejar
parina profiláctica. a CUCI. El sitio menos afectado es el tracto digestivo alto y
los síntomas dependen del grado de afectación, con náusea,
vómito o un cuadro franco de obstrucción.
11. Según la clasificación de la FDA para medicamen­
tos usados durante el embarazo, ¿a cuál categoría
pertenecen los anti-TNF? 16. ¿Las fístulas son una forma de presentación ha­
bitual?
Los anti-TNF son categoría B; es decir, no hay evidencia de
riesgo en humanos, a diferencia del metotrexato que está con­ Alrededor de 15% de los casos aparece con un cuadro perfo­
traindicado en el embarazo por su teratogenicidad. rante que puede ser con perforación intestinal o fístulas; las
más frecuentes son las perineales o perirrectales.
12. ¿Cuáles son los factores de mal pronóstico que
17. ¿Cuál es el gen predominante en la enfermedad
presentan la mayoría de los pacientes?
de Crohn?
La edad menor de 40 años constituye un factor de mal pro­
nóstico en pacientes con EII. En pacientes con CUCI se ha El gen NOD 2 se observa con mayor frecuencia en esta enfer­
relacionado con riesgo de recaídas y colectomía; sin embargo, medad, aunque existen diferentes sitios genéticos y cromoso­
también los criterios de Truelove y Witts y los hallazgos endos­ mas en la fisiopatogenia. Este gen se ha asociado con Crohn
ileal complicado.
cópicos con ulceraciones profundas, grandes o con zonas am­
plias de erosiones de la mucosa o denudación de la mucosa son
factores de riesgo a colectomía. Entre otros factores de mal pro­ 18. ¿Cuál es el comportamiento evolutivo de la en­
nóstico están la hipoalbuminemia menor de 2.8 g/dL y la hi­ fermedad de Crohn?
popotasemia al ingreso, PCR mayor a 45 mg/L y más de seis Se consideran tres patrones evolutivos de la enfermedad: infla­
semanas de evolución del brote. matorio, estenosante y penetrante.

730
SECCIÓN 11 GASTROENTEROLOGÍA @
a) Inflamatorio: consiste en la presencia de úlceras, las cuales CUADRO 11-10-4.
pueden ser aftosas, pero conforme avanza la enfermedad CARACTERÍSTICAS DE LA ENFERMEDAD DE CROHN
son grandes, profundas e irregulares, con características ser­ Y LA COLITIS ULCERATIVA CRÓNICA INESPECÍFICA
piginosas que alternan con mucosa normal y dan aspecto
ENFERMEDAD
de empedrado. Las lesiones pueden estar en diferentes par­ '
CARACTERÍSTICA DE CROHN CUCI
tes del tracto digestivo. Alrededor de 80% de los pacientes
Localización Cualquier área Lesiones continuas
se aprecia con un patrón inflamatorio.
del tracto que inician en el
b) Estenosante: puede presentarse desde el inicio de la enfer­ gastrointestinal recto
medad, pero el riesgo incrementa con el tiempo de evolu­
Afección de pared Transmural Sólo mucosa y
ción y los síntomas se presentan según el sitio afectado; la intestinal submucosa
estenosis puede ser corta o larga (mayor a 10 cm), única o
Hallazgos Lesiones en parches, Pseudopólipos, áreas
presentarse en diferentes segmentos intestinales. De acuer­
colonoscópicos en empedrado, continuas de
do con el grado de estenosis puede requerir manejo quirúr­ ulceraciones, inflamación
gico urgente; sin embargo, si la estenosis no se acompaña de estenosis
fibrosis responde de manera satisfactoria a la terapia bioló­ Anemia + ++
gica.
Dolor abdominal ++ +
e) Perforación: puede presentarse como cuadro inicial y reque­
rir resección intestinal o tener fistulas entéricas o perianales. Rectorragia + ++
Cáncer colorrectal ++ ++++
Conforme evoluciona la enfermedad predomina el patrón
penetrante sobre el patrón inflamatorio, lo cual condiciona ma­ + Más común o prevalente
yor agresividad y complicación de la enfermedad.
considerando remisión con una puntuación <150, actividad
19. ¿Cuáles son las fístulas más frecuentes en pacien­ leve con 150 a 219, moderada con 220 a 450 y >450 indica
tes con enfermedad de Crohn? actividad grave; sin embargo, el índice de Harvey-Bradshaw
Las fistulas en la enfermedad de Crohn pueden extenderse del es similar pero se simplifica con la valoración de un día. Jun­
intestino a cualquier órgano vecino, sean en el mismo intestino to con la clasificación de Montreal se determina el trata­
o enteroentéricas, las cuales se manifiestan con frecuencia co­ miento.
mo síndrome de malabsorción cuando son de gasto alto hacia

• 1
vagina, vejiga o piel y las perianales son las más frecuentes, las
CUADRO 11-10-5.
cuales pueden clasificarse por clínica en simples o complejas.
CLASIFICACIÓN DE MONTREAL
Las que tienen más de un orificio se asocian con estenosis, abs­
cesos o fístulas rectovaginales. La clasificación de Parks las di­ EDAD AL MOMENTO
vide en interesfintéricas, transesfintéricas, supraesfintéricas y DEL DIAGNÓSTICO
extraesfintéricas. La presencia de fístulas determina un estado .::,16 años A1
grave de la enfermedad y su diagnóstico requiere ultrasonido en­ 17 a 40 años A2
doanal y examen bajo anestesia. En resumen, las manifesta­
>40 años A3
ciones clínicas diferenciales entre enfermedad de Crohn y
CUCI se muestran en el cuadro 11-10-4. Localización (L)

Íleon terminal L1 L1 + L4 (fleon + tracto


20. ¿Cuál es la clasificación fenotípica de la enferme­ digestivo superior)
dad de Crohn? Colon L2 L2 + L4 (colon + tracto
En los pacientes con enfermedad de Crohn se requiere la de­ digestivo superior)
terminación de la localización, ya que puede estar afectado el lleocólica L3 L3 + L4 (ileocólica + tracto
tracto digestivo bajo a la par del tracto digestivo superior, así digestivo superior)
como el patrón evolutivo para establecer el tratamiento, lo cual Tracto digestivo superior L4
se efectúa mediante la Clasificación de Montreal aunada a la Patrón clínico (B)
edad de presentación, ya que a menor edad, mayor riesgo de
inflamatorio B1 B1p (inflamatorio con afección
complicaciones y menor respuesta terapéutica. Véase cuadro
perianal asociada)
11-10-5.
Los índices de actividad clínica de la enfermedad de Crohn Estenosante B2 B2p (estenosante con afección
perianal asociada)
son el índice de actividad clínica o CDAI, que comprende la
valoración de siete días previos de evacuaciones, estado gene­ Fistulizante B3 B3p (fistulizante con afección
perianal asociada)
ral, uso de antidiarreicos y complicaciones extraintestinales,

731
e MANUAL PARA EL EXAMEN NACIONAL DE RESIDENCIAS MÉDICAS

21. Si el paciente presenta fístulas perianales com­ Cosnes J, Cattan S, Blain A, Beaugerie L, Carbonnel F, Pare R, Gendre JP.
Long-term evolution of disease behavior of Crohn's disease. Inflamm
plejas con dos orificios, ¿cuál tratamiento médico o Bowel Dis. 2002 Jul;8(4):244-50.
quirúrgico indicaría? D'Haens Geert et al. Early combined immunosuppression or conventional
La presencia de fistulas complejas determina la gravedad de management in patients with newly diagnosed Crohn's disease: an open
la enfermedad, con o sin afección en otra parte del tracto di­ randomised tria!. Lancet. 2008;371 :66()..7.
Danese S, Fiocchi C. Ulcerative Colitis. N Engl J Med. 2011;365:1713-25.
gestivo, por lo que el manejo óptimo debe ser con anti-TNF y Dignass A, Eliak.im R, Magro F, Maaser C, Chowers Y, Geboes K et al. Se­
antibióticos. Es preciso observar que el ciprofloxacino incre­ cond European evidence-based consensus on the diagnosis and manage­
menta la cicatrización de fístulas cuando se une a un anti-TNF. ment ofulcerative colitis part 1: definitions and diagnosis. J Crohns Coli­
También se indica la aplicación de setones con el retiro opor­ tis. 2012 Dec;6(10):965-90. doi: 10.1016/j.crohns.2012.09.003
tuno, lo cual depende de la evolución y la respuesta al trata­ Dignass A, Van Assche G, Lindsay JO, Lémann M, Siiderholm J, Colombel
JF et al. The second European evidence-based Consensus on the diagno­
miento médico. sis and management ofCrohn's disease: Current management. J Crohns
Colitis. 2010 Feb;4(1 ):28-62. doi: 10.1016/j.crohns.2009.12.002.
22. ¿Cuáles son los factores de mal pronóstico en Geremia A, Biancheri P, Allan P, Corazza GR, Di Sabatino A. Innate and
un paciente con enfermedad de Crohin? adaptive immunity in inflammatory bowel disease. Autoimmun Rev. 2014
Se considera enfermedad de Crohn invalidante si el paciente Jan;l3(1):3-10. doi: 10.1016/j.autrev.2013.06.004. Epub 2013 Jun 15.
Hovde 0, Moum BA. Epidemiology and clínica! course of Crohn's disease:
ha tenido uno o más de estos factores: 2::2 cursos de esteroides, results from observational studies. World J Gastroenterol. 2012 Apr 21;
dependencia a éstos, requiere hospitalización, síntomas cróni­ 18(15): 1723-31. doi: 10.3748/wjg.v l8.i 15.1723.
cos(> 12 meses), inmunosupresores o cirugía. Langan RC, Gotsch PB, Krafczyk MA, Skillinge DD. Ulcerative colitis: diag­
Se considera enfermedad grave cuando hay resección > 70 nosis and treatment. Am Fam Physician. 2007 Nov 1;76(9): 132330.
cm, >2 resecciones, colectomía, estomas y enfermedad peri­ Lewis JD. The utility of biomarkers in the diagnosis and therapy of inflam­
matory bowel disease. Gastroenterology. 2011 May;l40(6):18171826.e2.
anal compleja. doi: 10.1053/j.gastro.2010.l 1.058.
Se han considerado factores de gravedad independientes al Ott C, Schiilmerich J. Extraintestinal manifestations and complications
diagnóstico: edad <40, estenosis o enfermedad fistulizante in­ in IBD. Nat Rev Gastroenterol Hepatol. 2013 Oct; 10(10):585-95. doi:
traabdominal, fiebre, pérdida de peso >5 kg y trombocitosis. 10.1038/nrgastro.2013.l l 7
Patil S, Cross RK. Update in the management of extraintestinal manifesta­
23. ¿Cuándo se indica la cirugía en la enfermedad tions ofinflammatory bowel disease. Curr Gastroenterol Rep. 2013 Mar;
15(3):314. doi: 10.1007/s l l894-013-0314-8.
de Crohn? Satsangi J et al. The Montreal classification of Inflammatory Bowel Disease:
La cirugía está indicada cuando no hay respuesta al tratamien­ controversies, consensus and implications. Gut. 2006;55:74953.
to farmacológico y, en el caso de iniciar con terapia biológica, Tibble JA, Sigthorsson G, Bridger S, Fagerhol MK, Bjarnason l. Surroga­
si no hay respuesta en dos a seis semanas. En cuadros obstruc­ te markers ofintestinal inflammation are predictive ofrelapse in patients
with inflammatory bowel disease. Gastroenterology. 2000 Jul; l l9(1):l5-
tivos o perforación la mejor alternativa es la cirugía laparoscó­ 22.
pica; sin embargo, la afección extensa (> 100 cm) tiene alto Van Assche G, Dignass A, Bokemeyer B, Danese S, Gionchetti P, Moser G et
riesgo de síndrome de intestino corto. al. European Crohn's and Colitis Organisation. J Crohns Colitis. 2013 Feb;
7(1 ): 1-33. doi: 10.1016/j.crohns.2012.09.005. Epub 2012 Oct 3.
BIBLIOGRAFÍA Van Assche G, Dignass A, Bokemeyer B, Danese S, Gionchetti P, Moser G et
Baumgart DC, Sandborn WJ. Crohn's disease. Lancet. 2012 Nov 3;380 al. Second European evidence-based consensus on the diagnosis and ma­
(9853):159()..1605. doi: 10.1016/S0J4()..6736(12)60026-9. Epub 2012 Aug nagement ofulcerative colitis part 3: special situations. J Crohns Colitis.
20. Review. Erratum in: Lancet. 2013 Jan 19;381(9862):204. 2013 Feb;7(1 ): 1-33. doi: 10.1016/j.crohns. 2012.09.005.
Beaugerie L, Seksik P, Nion-Larmurier I, Gendre JP, Cosnes J. Predictors of Van Assche G, Dignass A, Panes J, Beaugerie L, Karagiannis J, Allez Metal.
Crohn's disease. Gastroenterology. 2006 Mar; l30(3):65()..6. The second European evidence-based Consensus on the diagnosis and
Bennike T, Birkelund S, Stensballe A, Andersen V. Biomarkers in inflamma­ management ofCrohn's disease: Definitions and diagnosis. J Crohns Co­
tory bowel diseases: current status and proteomics identification strate­ litis. 2010 Feb;4(1):7-27. doi: 10.1016/j.crohns.2009.12.003.
gies. World J Gastroenterol. 2014 Mar 28;20(12):3231-44. doi: 10.3748/ Wilkins T, Jarvis K, Patel J. Diagnosis and management ofCrohn's disease.
wjg.v20.i l2.3231. Am Fam Physician. 2011 Dec 15;84(12):1365-75.
Burisch J, Jess T, Martinato M, Lakatos PL. The burden ofinflammatory bowel Yarur JA et al. Predictors of Aggressive lnflammatory Bowel Disease. Gas­
disease in Europe. ECCO-EpiCom. J Crohns Colitis. 2013 May;7(4):322- troenterol Hepatol (NY). Oct 2011;7( 10):652-9.
337. doi: 10.1016/j.crohns.2013.01.010.

732
SECCIÓN 11 GASTROENTEROLOGÍA @
CAPÍTULO 11.11
SÍNDROME DE INTESTINO IRRITABLE
Fidel Mendoza Vare/a

1. ¿Qué es el síndrome de intestino irritable (Sil)? CUADRO 11-11-2.


Es un trastorno funcional intestinal en el que se presenta do­ DATOS DE ALARMA EN SIi
lor o molestia abdominal asociado con cambios en el hábito a) Inicio de síntomas o cambio de hábito intestinal en pacientes ma-
intestinal o alteraciones en la defecación. Presenta un curso yores de 50 años de edad
benigno y crónico que afecta la calidad de vida de manera va­ b) Pérdida ponderal involuntaria de más de 10% en seis meses
riable. Es más frecuente en el sexo femenino y se presenta de c) Historia familiar de malignidad gastrointestinal
cf) Fiebre
forma predominante en la edad adulta.
e) Hematoquecia
f) Diarrea asociada con deshidratación grave
2. ¿Cómo se establece el diagnóstico del Sil? g) Asociación con artritis, lesiones cutáneas, linfadenopatía, masa
abdominal, síntomas nocturnos, anemia, leucocitosis, velocidad
El diagnóstico de SIi no es de exclusión. Se debe considerar el
de sedimentación globular prolongada o sangre oculta en heces
diagnóstico de SIi en las personas que presentan dolor abdo­ positiva
minal y alteraciones en la defecación (diarrea, estreñimiento o
alternancia de ambos), con una evolución de doce semanas.
En pacientes con síntomas sugestivos el cumplimiento de los En presencia de datos de alarma se recomienda la realización
criterios diagnósticos (cuadro 11-11-1) es suficiente para esta­ de biometría hemática completa, velocidad de sedimentación
blecer el diagnóstico de SIi. globular, sangre oculta en heces y estudios para escrutinio de
Los datos de alarma del síndrome se mencionan en el cua­ neoplasias colorrectales, como colon por enema, rectosigmoi­
dro 11-11-2. doscopia flexible y/o colonoscopia, según disponibilidad. En
� http://www.cenetec.salud.gob.mx/descargas/gpc/Catalogo los casos en los que el síntoma predominante sea la diarrea se
Maestro/042_GPC_Colonirritable/lMSS_042_08_EyR.pdf justifica una evaluación más detallada para excluir parasitosis.
En los pacientes con persistencia de síntomas se recomienda
llevar a cabo estudios adicionales, como ultrasonido abdomi­
3. ¿Con cuáles patologías deberá realizarse un diag­

1
nal, pruebas tiroideas, panel celiaco, pruebas de aliento y quí­
nóstico diferencial? micas sanguíneas para excluir otras enfermedades.
El diagnóstico diferencial se realiza con todas aquellas enfer­
medades que presenten los síntomas predominantes de SIi, � http://www.cenetec.salud.gob.mx/descargas/gpc/Catalogo
como dolor abdominal y cambio de hábito intestinal o altera­ Maestro/042_GPC_Colonirritable/lMSS_042_08_EyR.pdf
ción en las evacuaciones (cuadro 11-11-3).
5. ¿Cuál es el tratamiento para el Sil?
4. ¿Cuál es el estudio de elección para el paciente Se recomienda un tratamiento integral encaminado a la mejo­
con Sil? ría de los síntomas y la calidad de vida del paciente. El trata­
En los pacientes que cumplen los criterios clinicos del SIi no miento va dirigido al síntoma predominante, ya sea diarrea,
existe justificación para realizar otras pruebas diagnósticas, estreñimiento, dolor o distensión abdominal. La modificación
siempre y cuando no tengan síntomas y signos de alarma. del estilo de vida y de los factores psicológicos, así como el uso

CUADRO 11-11-1. CUADRO 11-11-3.


CRITERIOS DE ROMA 111 DIAGNÓSTICO DIFERENCIAL PARA SIi

Dolor o molestia abdominal recurrente durante por lo menos tres a) Enfermedades parasitarias
días al mes y en los últimos tres meses, asociados con dos o más de b) Intolerancia a carbohidratos
los siguientes: c) Alergias alimentarias
cf) Colitis microscópicas
a) Mejoría de los síntomas con la defecación
e) Endocrinopatías
b) Está asociado con un cambio en la frecuencia de las evacuaciones
f) Enfermedad celiaca
c) Está asociado con un cambio en la forma (apariencia) de las de
g) Sobrecrecimiento bacteriano
las evacuaciones
h) Enfermedad inflamatoria intestinal
Estos criterios deben estar presentes en los últimos tres meses y los 1) Neoplasias colorrectales
síntomas deben haber comenzado mínimo seis meses antes del J) Neoplasias ginecológicas
diagnóstico k) Medicamentos

733
e MANUAL PARA EL EXAMEN NACIONAL DE RESIDENCIAS MÉDICAS

de medicamentos para el manejo de los síntomas digestivos, significativas en comparación con pacientes que sólo utilizan
son fundamentales para un tratamiento integral efectivo. tratamiento farmacológico.
En pacientes que no han respondido a ningún tratamiento
6. ¿Qué terapias se utilizan para tratar el SIi? después de doce meses se recomienda el envío a psicoterapia,
Para el estreñimiento se recomienda iniciar con laxantes for­ terapia cognitivo-conductual o hipnoterapia.
madores de masa o laxantes con efecto osmótico. En caso de
� http://www.cenetec.salud.gob.mx/descargas/gpc/Catalogo
diarrea el medicamento de elección es la loperamida, que me­ Maestro/042 GPC Colonirritable/lMSS_042_08_EyR.pdf
jora la consistencia y disminuye la frecuencia de las evacuacio­
nes. Para el dolor abdominal se recomiendan los anticolinér­
BIBLIOGRAFÍA
gicos con efecto antiespasmódico, como la butilhioscina. En
casos de dolor también se recomienda el uso de los inhibidores López Colombo A, Rivera Ramos JF, Sobrino Cossío S, Suárez Morán E..
Gastroenterology diagnosis and treatment guidelines of irritable bowel
de la recaptura de serotonina, como fluoxetina, sertralina, cita­
syndrome; epidemiology and physiopathology. Rev Gastroenterol Mex.
lopram y paroxetina. En caso de ansiedad y depresión se reco­ 2009; 74(1):56-7.
miendan los antidepresivos tricíclicos, como la amitriptilina y Remes Troche JM, Berna! Reyes R, Valladares Lepine MA, Alonso Lárraga O,
la imipramina. Para el manejo de la distensión abdominal por Gómez Escudero O, Meléndez Mena D. Gastroenterology diagnosis and
exceso de gas o meteorismo se utilizan agentes surfactantes, treatment guidelines of irritable bowel syndrome; clinical features and diag­
nostic criteria. Rev Gastroenterol Mex. 2009;74(1):58-62.
como la dimeticona y la simeticona, probióticos y antibióticos
Schmulson MJ, Noble Lugo A, Valenzuela de la Cueva VM, De Ariño Suárez
de acción luminal, como la rifaximina. M, Guillermo Denis LA, Ramos Narváez FA. Gastroenterology diagnosis
También existen las terapias alternativas (herbolaria, acu­ and treatment guidelines of irritable bowel syndrome; treatment. Rev Gas­
puntura y yoga); sin embargo, no han demostrado diferencias troenterol Mex. 2009;74(1):63-70.

CAPÍTULO 11.12
ESTUDIO DEL PACIENTE CON ENFERMEDAD HEPATOBILIAR
María Del Carmen Lomas Bautista

PRUEBAS DE FUNCIÓN HEPÁTICA 3. ¿Cuáles son las consideraciones que se deben co­
nocer al valorar el tiempo de protrombina?
1. ¿Cuál es la utilidad de las pruebas de síntesis he­ La coagulación es el resultado final de una compleja serie de
pática? reacciones enzimáticas que implican factores de coagulación,
Las pruebas de función hepática proporcionan información todos los cuales se producen en el hígado excepto el factor
de la síntesis y el metabolismo hepático, así como la excreción VIII, que es producido por las células endoteliales vasculares. El
biliar. En estas pruebas se incluyen los niveles séricos de albú­ tiempo de protrombina es una medida de la velocidad a la que
mina y el tiempo de protrombina (TP), bilirrubinas, gammag­ la protrombína se convierte en trombina, lo que refleja la vía
lutamil transpeptidasa (GGT), fosfatasa alcalina (FA), aspar­ extrínseca de la coagulación y es interpretado como marcador
tato aminotransferasa (AST), alaníno aminotransferasa (ALT), de síntesis del higado. El diagnóstico diferencial de un TP anor­
deshidrogenasa láctica (DHL) y S'nucleotidasa, entre otras malmente prolongado también incluye el déficit de vitamina K,
poco utilizadas. La evaluación clínica de los pacientes en quie­ la terapia de anticoagulación y la coagulopatía por consumo.
nes se sospecha enfermedad hepática involucra la adecuada La vitamina K es necesaria por la gamma-carboxilación y
interpretación de las anormalidades en las pruebas bioquími­ la función normal de los factores de coagulación 11, VII, IX
cas del hígado, en el contexto de una cuidadosa historia clíni­ y X. La deficiencia de vitamina K puede asociarse con malnu­
ca y una exploración física completa. El reconocimiento de las trición, malabsorción o uso de antibióticos que pueden con­
fluctuaciones en los parámetros bioquímicos del hígado pue­ ducir a prolongación del tiempo de protrombina (TP). Cuan­
de facilitar la evaluación clínica, que incluye pruebas bioquími­ do la prolongación del tiempo de protrombína es causada por
cas y marcadores específicos de enfermedad, estudios de ima­ enfermedad hepática los niveles de factor VIII son normales o
gen y biopsia hepática. incrementados, mientras que en el caso de la coagulación íntra­
vascular diseminada (CID) los niveles de factor VIII son dismi­
2. ¿Cómo se dividen para su estudio las pruebas de nuidos. Las medidas de niveles séricos de factor V y administra­
función hepática? ción de vitamina K pueden usarse para diferenciar enfermedad
a) Valores que indican función o síntesis hepática, y hepática y déficit de vitaminas K como causas de prolonga­
b) Marcadores de enfermedad hepática o del sistema biliar. ción de TP. Los niveles de factor V son disminuidos en enfer-

734
SECCIÓN 11 GASTROENTEROLOGÍA @
medad hepática, pero permanecen no afectados por las vita­ ferencia de un grupo amino desde el L-aspartato al 2-oxogluta­
minas K. La administración de 10 mg subcutáneos resulta en rato, formándose L-glutamato y oxaloacetato. Esta enzima uti­
la corrección del TP en 24 horas al menos en 30% de los pa­ liza el piridoxal 5'-fosfato como cofactor
cientes con déficit de vitamina K, pero no en aquellos con en­ Alanina aminotransferasa (ALT) o transaminasa glutámi­
fermedad hepática. co pirúvica (TGP): cataliza la transferencia de un grupo amino
La función de síntesis hepática puede establecerse con ma­ desde la alanina al alfa-cetoglutarato; los productos de esta
yor precisión en pacientes con falla hepática aguda al medir el transaminación reversible son piruvato y glutamato.
TP, ya que el factor VII tiene una vida media de 6 horas, lo
cual es mucho más corto que la albúmina. 7. ¿Dónde se localizan principalmente ALT y AST y
cuál es su valor normal?
4. ¿Cuáles son las consideraciones que se tienen ALT se localiza sobre todo en el hígado y se limita al citoplas­
que conocer al valorar la albúmina? ma, mientras que AST se puede aislar del hígado y una am­
Cuantitativamente es la proteína plasmática más importante, plia variedad de sitios extrahepáticos, incluso el miocardio,
representa 75% de la presión oncótica del plasma y es sinteti­ el músculo esquelético, el riñón, el cerebro, el páncreas y las
zada de manera exclusiva por los hepatocitos. El adulto pro­ células de la sangre, tanto en el citoplasma como en las mi­
medio produce alrededor de 15 g/día y tiene entre 300 y 500 g tocondrias. Las aminotransferasas se comportan como otras
de albúmina distribuidos en los fluidos corporales. El híga­ proteínas del suero. Se distribuyen en el plasma y el líquido.
do tiene la capacidad de duplicar la velocidad de síntesis en el Los valores normales de transaminasas en suero varían en gran
ajuste de la pérdida de albúmina rápida o una disminución por medida entre los laboratorios, pero los valores que han gana­
dilución en la concentración de albúmina de suero. La vida do aceptación general son <30 U/L. La variación entre labo­
media de la albúmina es de 14 a 21 días. La síntesis de albúmina ratorios en el rango normal es el resultado del rango de refe­
está regulada por cambios en el estado nutricional, la presión rencia normal establecido para la población de referencia; el
osmótica, la inflamación sistémica y los niveles de hormonas. rango normal se define como la media de la población de refe­
Por tanto, el diagnóstico diferencial de la hipoalbuminemia en rencia más 2 desviaciones estándar; alrededor de 95% de una
suero, además de la disfunción hepatocelular, incluye desnutri­ población distribuida de manera uniforme caerá dentro de este
ción, enteropatía con pérdida de proteínas o síndrome nefró­ rango "normal".
tico, condiciones inflamatorias sistémicas crónicas y desequi­
librios hormonales. 8. ¿Cómo se interpreta la elevación de las amino­
La larga vida media en suero de la albúmina contribuye a su transferasas?

1
falta de fiabilidad como un marcador de la función de síntesis Los niveles elevados de aminotransferasas indican daño o ne­
hepática en la lesión hepática aguda. Niveles de albúmina sé­ crosis hepatocelular; la filtración o fuga de aminotransferasas
rica inferiores a 3 g/dL en un paciente con hepatitis recién diag­ de los hepatocitos, como resultado de daño hepatocelular, se
nosticada deben plantear la sospecha de un proceso crónico. puede relacionar con varios tipos de enfermedad hepática, in­
La albúmina sérica es un excelente marcador de la función de cluso hepatitis viral, daño isquémico y hepatotoxicidad induci­
síntesis hepática en pacientes con enfermedad hepática cróni­ da por drogas.
ca y cirrosis, con excepción de los pacientes con cirrosis y as­ Ante una elevación aislada de AST es importante confirmar
citis. La albúmina tiene utilidad como prueba de tamizaje en el origen hepático correlacionándolo con los niveles de ALT.
pacientes que están bajo sospecha de enfermedad hepática. En pacientes con elevaciones de AST aisladas o desproporcio­
La prealbúmina, como es sintetizada por el hígado, tiene nadas en relación con ALT, la causa de la elevación puede ser
una vida media corta. Los niveles séricos de prealbúmina pue­ resultado de un origen extrahepático, como daño de músculo
den verse influidos por diversos factores y, por tanto, no se uti­ esquelético o miocárdico que puede ser resultado de actividad
lizan en gran medida como marcadores de disfunción hepática. física rigurosa, rabdomiólisis, distrofia muscular y polimiosi­
tis, entre otros.
5. ¿Cuáles son las consideraciones que se deben La elevación de los niveles de aminotransferasas correlacio­
conocer al valorar las aminotransferasas? na muy poco con la gravedad de la necrosis hepatocelular y es
Las aminotransferasas catalizan la transferencia de grupos poco predictor de resultados en pacientes con enfermedad he­
amino a partir del ácido aspártico o alanina a a-cetoglutarato, pática subyacente, ya que los niveles séricos de aminotransfe­
con lo que dan lugar a la formación de ácido oxaloacético y rasas pueden estar en rangos normales en paciente con cirrosis
ácido pirúvico, respectivamente, durante la gluconeogénesis. o fibrosis avanzada y por lo general suelen ser de mayor utili­
dad en enfermedad hepática aguda.
6. Cuales son las funciones específicas de la AST y El grado de elevación puede orientar al diagnóstico y se ca­
la ALT? tegoriza en leve, moderado e intenso. En general la mayoría de
Aspartato aminotransferasa (AST) o transaminasa glutámica las elevaciones son de leves a moderadas ( <300 U/L) y en Oc­
oxaloacética (TGO): esta enzima cataliza la reacción de trans- cidente el hígado graso no alcohólico (NASH) es la causa más

735
e MANUAL PARA EL EXAMEN NACIONAL DE RESIDENCIAS MÉDICAS

CUADRO 11-12-1. b) La fase aguda de la enfermedad de Wilson; AST es despro­

-
CATEGORIZACIÓN DE ELEVACIÓN DE LAS ENZIMAS porcionadamente elevada en relación con ALT.
HEPÁTICAS e) Enfermedad de Wilson fulminante: puede ser >4.
d) Hígado graso no alcohólico (NASH): relaciones � 1 en au­
ELEVACIÓN

. GRAVE O
sencia de cirrosis.
ELEVACIÓN LEVE MARCADA Es importante enfatizar que las anormalidades de la rela­
AST o ALT <2 a 3 veces 2 a 3 veces >10 a 20 veces ción AST/ALT deben interpretarse en el contexto de la infor­
Fosfatasa <1.5 a 2 veces 1.5 a 2 y hasta >5 veces
mación clínica y los estudios bioquímicos adicionales, y sólo
alcalina 5 veces debe considerarse como coadyuvante en el estudio de la enfer­
>10 veces
medad hepática.
GGT <2 a 3 veces 2 a 3 a 10
veces
11. ¿Cuáles son las consideraciones que se deben
conocer al valorar la deshidrogenasa láctica (DHL)?
común. En pacientes con hepatitis alcohólica los niveles de Puede ser aislada de una amplia variedad de tejidos, incluso
aminotransferasas son menores a 300 U/L y en el caso de obs­ hígado, músculo esquelético, eritrocitos, cerebro y riñones; por
trucción biliar suelen ser menores, a pesar de que la elevación ello, es de poca sensibilidad para enfermedad hepática y su
de las aminotransferasas puede ser evidente justo después de medición rara vez se usa como primera linea para establecer
la obstrucción biliar aguda. Estos niveles son inespecíficos la función hepática.
en la mayoría de los casos. La DHL puede ser fraccionada en isoenzimas para deter­
minar la fuente de la elevación de los niveles séricos, pero en
9. ¿Cuál es la causa más común de la elevación de la práctica clínica esto rara vez se realiza. En algunas circuns­
las aminotransferasas? tancias los niveles de DHL pueden ser útiles para establecer
La causa más común de elevación grave es la hepatitis viral algún diagnóstico; por ejemplo, la marcada elevación de DHL
aguda, seguida en frecuencia por la hepatotoxicidad inducida puede observarse en pacientes con hepatitis isquémica o bien
por paracetamol; ambos desórdenes pueden resultar en eleva­ en algunos casos de infiltración tumoral al hígado relaciona­
ciones de transaminasas a niveles > l 000 U/L; sin embargo, dos con FA elevada de forma persistente.
la característica de esta elevación es una relación AST/ALT
menor a 1, la cual permanece constante. PRUEBAS DE EXCRECIÓN BILIAR
El diagnóstico diferencial de elevaciones marcadas de los
niveles de aminotransferasas (> l 000 U/L) incluye hepatitis 12. ¿A qué se le llama fosfatasa alcalina (FA) y dónde
viral (Aa E), toxicidad o lesión hepática inducida por drogas se encuentra en el organismo?
o fármacos, hepatitis isquémica y, con menos frecuencia, he­ La fosfatasa alcalina es un término que por lo general se aplica
patitis autoinrnunitaria, síndrome de Budd-Chiari, enfermedad a un grupo de isoenzimas distribuido en gran medida en todo
de Wilson fulminante y obstrucción aguda de la vía biliar. el cuerpo. Las isoenzimas de mayor importancia clínica en los
Las hepatitis donde los niveles de aminotransferasas decli­ adultos se encuentran en el hígado y los huesos; estos órga­
nan con rapidez, seguido de un continuo empeoramiento de los nos son las principales fuentes de fosfatasa alcalina sérica.
niveles de bilirrubinas y prolongación del tiempo de protrom­ Otras isoenzimas se originan a partir de la placenta, el intesti­
bina, indican un mal pronóstico y el desarrollo de falla hepáti­ no delgado y los riñones. En el hígado, la fosfatasa alcalina se
ca aguda. encuentra en la membrana canalicular de los hepatocitos; su
función exacta no está definida, tiene una vida media en suero
10. Adicional a la valoración individual, ¿qué informa­ de aproximadamente siete días y, aunque no se conocen los
ción se puede obtener al evaluar la relación AST/ALT? sitios de degradación, el aclaramiento de la fosfatasa alcalina
La relación AST/ALT puede proporcionar orientación en el del suero es independiente de la permeabilidad de la vía biliar
diagnóstico de pacientes con necrosis hepatocelular. La re­ o la capacidad funcional del hígado. La enfermedad hepatobi­
lación es igual o menor a l en la mayoría de los casos de daño liar conduce a un aumento de los niveles séricos de fosfatasa
hepatocelular agudo, con la notable excepción de la hepatitis alcalina a través de la síntesis inducida de la enzima y las fugas
alcohólica en donde la relación suele ser >2. en el suero, un proceso mediado por los ácidos biliares.
La elevada relación AST/ALT se asocia con deficiencia de
piridoxina y mal estado nutricional. Ambas aminotransferasas 13. ¿Los valores de fosfatasa alcalina (FA) son siem­
requieren como coenzima al 5'piridoxal fosfato, y AST tiene pre constantes?
mucho mayor afinidad por esta coenzima que la ALT. Los valores de fosfatasa alcalina en suero varían con la edad:
a) Paciente con cirrosis secundaria a enfermedad hepática cró­ los adolescentes varones y mujeres tienen niveles de fosfatasa
nica también pueden presentar una relación AST/ALT > l. alcalina en suero dos a tres veces el nivel observado en los adul-

736
SECCIÓN 11 GASTROENTEROLOGÍA @
tos; así como los adultos mayores de 65 años de edad presen­ narse con consumo de alcohol, aunque esto no está bien des­
tan elevaciones de más de 50%; el nivel se correlaciona con el crito.
crecimiento y el metabolismo óseo. En una persona con eleva­
ciones aisladas de la fosfatasa alcalina, la GGT en suero o la 17. ¿Cuál es la utilidad de la 5' nucleotidasa (5'NT)?
5' nucleotidasa (5'NT) se utiliza para distinguir un origen he­
Se encuentra en varios tejidos como hígado, miocardio, pán­
pático de la elevación de la fosfatasa alcalina.
creas, cerebro y vasos sanguíneos. La 5 'NT hepática puede ser
aislada de los sinusoides y la membrana plasmática canalicu­
14. ¿Cómo afecta la falla renal los niveles de FA? lar. Un valor sérico elevado por lo regular se asocia con enfer­
La falla renal crónica puede resultar en una elevación anormal medad hepática y colestasis extrahepática. Sin embargo, los
de la isoenzima de FA intestinal. La isoenzima hepática puede niveles pueden durar varios días después de la obstrucción.
ser aislada del lado canalicular de la membrana plasmática del
hepatocito y de la superficie laminar del epitelio del dueto bi­ BILIRRUBINAS
liar, por lo que en pacientes con obstrucción biliar es elevada
por el incremento en la síntesis y liberación de la enzima hacia 18. ¿De dónde proviene la bilirrubina?
el suero, por el deterioro en la secreción biliar.
La bilirrubina es un pigmento, producto final del catabolismo
de la hemoglobina (ferroprotoporfirina IX). Cerca de 4 mg/
15. ¿Cómo se afecta la FA durante las diferentes for­ kg de peso corporal se producen cada día; casi 80% proviene
mas de colestasis? de la degradación de la hemoglobina en los glóbulos rojos se­
Durante la colestasis los ácidos biliares se acumulan en el he­ nectos y de células eritroides destruidas de forma prematura
patocito e incrementan la solubilidad de la membrana plasmá­ en la médula ósea; el resto de la cifra es resultado de hemopro­
tica, lo que resulta en liberación de FA al suero. Los niveles en teínas como la mioglobina y los citocromos que se distribuyen
el suero después de obstrucción biliar por lo general retornan por todo el cuerpo.
a la normalidad una vez resuelta la obstrucción. Una elevación
dos o tres veces sobre lo normal de FA no es específica y pue­ 19. ¿Cuáles son los pasos dentro del metabolismo de
de resultar de una amplia variedad de procesos patológicos que la bilirrubina?
involucran al hígado.
Los pasos iniciales del metabolismo de la bilirrubina se produ­
De manera característica las elevaciones anormales de FA
cen en las células reticuloendoteliales, principalmente del bazo.
sérica se originan del hígado y pueden observarse en pacientes

1
El hemo se transforma en biliverdina por la enzima hemooxi­
con obstrucción biliar intrahepática (cirrosis biliar primaria, co­
genasa microsomal. La biliverdina se convierte a continuación
langitis esclerosante primaria) o extrahepática (coledocolitia­
en la bilirrubina por la enzima reductasa citosólica de biliver­
sis, estenosis), así como en desórdenes hepáticos infiltrativos
dina.
(enfermedad granulomatosa) y tumores primarios o metastá­
Esta bilirrubina no conjugada es soluble en lipidos y prác­
sicos como resultado de compresión, invasión maligna o ambas
ticamente insoluble en agua, y para ser transportada en sangre
a nivel de los conductos intrahepáticos pequeños o extrahepá­
debe ser solubilizada uniéndose a la albúmina. El complejo de
ticos. Pacientes con obstrucción focal intrahepática por tumor
bilirrubina-albúmina no conjugada pasa fácilmente a través del
pueden presentar niveles séricos de FA de forma aislada sin
endotelio que reviste los sinusoides hepáticos en el espacio de
elevación de bilirrubinas. En contraste, niveles séricos de FA
Disse, donde la bilirrubina se disocia de la albúmina y es ab­
pueden permanecer en rangos normales en metástasis hepáti­
sorbida por los hepatocitos a través de un proceso facilitado
ca extensa o de vías biliares extrahepáticas, por lo que los nive­
mediado por proteínas.
les séricos de FA no son un indicador certero de la gravedad de
En el citoplasma del hepatocito es ligada a un gran número
la enfermedad hepática subyacente.
de proteínas, incluida la superfamilia de la glutatión S-transfe­
rasa, lo que reduce el flujo de salida de la bilirrubina de nuevo
16. ¿Qué se debe saber sobre la gammaglutamil trans­ en el suero y la presenta para la conjugación.
peptidasa (GGT) para su correcta evaluación? La enzima uridin-5'-difosfato glucuronil transferasa (UDP)
La gammaglutamil transpeptidasa (GGT) puede ser aislada en el retículo endoplásmico la solubiliza mediante la conjuga­
de los hepatocitos y el epitelio biliar. La GGT es una enzima ción a ácido glucurónico para producir monoglucurónido y
microsomal que puede ser inducida por estimulantes como el diglucurónido de bilirrubina, la cual es hidrófila y difunde a la
alcohol y los fármacos como los anticonvulsivantes y la warfa­ membrana canalicular para la excreción en la bilis por los ca­
rina. nalículos.
Está presente en varios órganos extrahepáticos incluyen­ La bilirrubina conjugada pasa inalterada hasta el íleon dis­
do el riñón, el bazo, el páncreas, el corazón y el cerebro. Puede tal y el colon, donde las bacterias que contienen [3-glucuronida­
ser útil en elevaciones de FA para distinguir entre causas hepá­ sas hidrolizan la bilirrubina conjugada (BD) a bilirrubina no
ticas y óseas. La relación GGT/FA mayor a 2.5 puede relacio- conjugada (BI) y se reduce aún más a urobilinógeno incoloro.

737
e MANUAL PARA EL EXAMEN NACIONAL DE RESIDENCIAS MÉDICAS

El urobilinógeno se excreta oxidado o como urobilina (anaran­ FIGURA 11-12-1. METABOLISMO DE LA BILIRRUBINA.
jado) o es absorbido de forma pasiva por el intestino hacia el
sistema portal como urobilinógeno. La mayoría del urobilinó­
geno absorbido es reexcretado por el hígado. Un pequeño por­
Eritrocito
centaje pasa a través de los filtros del glomérulo renal y se ex­
creta en la orina.
La bilirrubina no conjugada no se encuentra en orina por su
unión a albúmina; su presencia indica una hiperbilirrubinemia
conjugada y enfermedad hepatobiliar. Véase figura 11-12-1. Globina

20. ¿Cuáles son los valores normales de bilirrubina Hemooxigenasa


conjugada y no conjugada?
Los niveles séricos normales son menos de 1 mg/dL ( 18 mrnol/ --
Hemooxigenasa �
Grupo hemo

L). Sin embargo los parámetros de laboratorio normales pue­ -- _ �


� Biliverdina
� '._________,
den ser desde 1.0 a 1.5 mg/dL. La bilirrubina sérica puede ser
fraccionada en dos tipos: directa o conjugada (BD) y bilirrubi­ � Bilirrubina indirecta
na indirecta o no conjugada (BI), esto derivado de la reacción
original de Van den Bergh. La bilirrubina directa es soluble al se transporta a través de la
bilis hasta
agua en su forma conjugada y la bilirrubina indirecta es solu­
ble en lípidos en su forma no conjugada. En adultos sanos más
de 90% de la bilirrubina es no conjugada; los valores de bilirru­
bina conjugada pueden ser elevados como resultado de enfer­ se degrada a

medad del parénquima hepático y/u obstrucción biliar, debido


a que la bilirrubina conjugada puede ser filtrada a través de los
glomérulos y excretada en la orina. En la enfermedad hepa­ se oxida a
tobiliar la ictericia se asocia con la presencia de bilirrubina en
orina. La forma no conjugada de la bilirrubina es unida a la al­
búmina, lo que resulta en una macromolécula que no puede ser
filtrada a través del glomérulo.

21. ¿Cómo se debe evaluar la ocurrencia de hiperbi­


lirrubinemia no conjugada (bilirrubina indirecta)? pasa a la se transforma en
Los mecanismos patológicos de la elevación anormal de bili­
Sangre
rrubina no conjugada incluyen: incremento de la producción Estercobilina
(circulación general)
y defectos (congénitos o adquiridos) en la conjugación hepá­
tica por procesos como la hemólisis, historia de anemia y trans­
fusiones de sangre previas, así como el uso de ciertos medica­
mentos.
Las pruebas que sugieren el diagnostico de hemólisis inclu­
yen conteo completo de células sanguíneas, frotis de sangre pe­ y se expulsa mediante la y se expulsa mediante las
riférica, recuento de reticulocitos y medición de niveles séricos
de DHL, así como niveles de haptoglobina. La causa específi­ 1
t
Orina 1 Heces fecales
ca de hemólisis puede confirmarse por prueba de Coombs, glu­
cosa 6 fosfato deshidrogenasa (G6PD) y electroforesis de hemo­
globina; la hemólisis crónica suele causar niveles persistentes
de hiperbilirrubinemia ( <5 mg/dL) en paciente con función más de 50% del total de las bilirrubinas. El paso lirnitante es la
hepática normal (cuadro 11-12-2). excreción de bilirrubinas por el hepatocito y la secreción de
bilirrubina conjugada de los hepatocitos en el suero. Es nota­
22. ¿Cómo se debe evaluar la ocurrencia de hiperbi­ da de forma temprana, ya que la bilirrubina conjugada es solu­
lirrubinemia conjugada (bilirrubina directa)? ble en agua y puede ser excretada por los riñones; los niveles
Los valores de bilirrubina conjugada pueden ser anormalmen­ séricos de bilirrubina pueden ser tan altos o más de 30 mg/dL
te elevados como resultado de enfermedad hepatobiliar, en los en pacientes con enfermedad hepatobiliar en el marco de he­
cuales los niveles séricos de bilirrubina conjugada constituyen molisis o enfermedad renal (cuadro 11-12-2).

738
SECCIÓN 11 GASTROENTEROLOGÍA @
CUADRO 11-12-2. mún es el síndrome de Gilbert, una condición congénita que
PRINCIPALES CAUSAS DE HIPERBILIRRUBINEMIA resulta de un defecto en la conjugación de bilirrubina en una
CONJUGADA Y NO CONJUGADA EN ADULTOS actividad reducida de la uridin difosfato glucuronil transferasa.
HIPERBILIRRUBINEMIA HIPERBILIRRUBINEMIA Se manifiesta con moderadas e intermitentes elevaciones de
INDIRECTA DIRECTA bilirrubina no conjugada (2 a 5 mg/dL); suele asociarse con
DESÓRDENES HEMOLÍTICOS CONGÉNITAS eventos de estrés, como ayuno, trauma, cirugía o infección.
, Deficiencia de glucosa 6 , Síndrome de Dubin-Johnson • BD predominante: puede ser por alcohol, viral o drogas,
fosfato deshidrogenasa • Síndrome de Rotor
• Anemia de células falciformes
obstrucción biliar extrahepática o en raras ocasiones por
• Esferocitosis un desorden como sindrome de Dubin-Johnson y sindrome
ENFERMEDAD ADQUIRIDA ENFERMEDAD HEPÁTICA
de Rotor, incluso una infección bacteriana extrahepática, y
, Toxicidad por drogas • Toxicidad hepática puede asociarse con elevación de FA.
• Hiperesplenismo , Colangitis esclerosante
, Hemoglobinuria paroxística primaria 24. ¿Cuál es la utilidad de la determinación de áci­
nocturna • Cirrosis biliar primaria
dos biliares en suero?
• Trauma con daño vascular • Hepatitis autoinmunitaria
Los ácidos biliares se sintetizan a partir del colesterol en los
ERITROPOYESIS INEFECTIVA OBSTRUCCIÓN BILIAR
• Deficiencia de cobalamina EXTRAHEPÁTICA hepatocitos conjugado con glicina o taurina y se secretan en la
• Deficiencia de folatos • Litiasis de la vía biliar bilis; la mayoría se reabsorbe de forma activa en el intestino a
, Talasemia , Tumores de la vía biliar través de la circulación portal.
• Deficiencia de hierro , Infiltración tumoral El mantenimiento de las concentraciones de ácidos biliares
FÁRMACOS FÁRMACOS normales en suero depende del flujo sanguíneo hepático, la
• Rifampicina y probenecid , Esteroides
captación hepática, la secreción de ácidos biliares y el tránsito
, Estrógenos
, Antipsicóticos
intestinal.
• Antibióticos La medición de los ácidos biliares es un indicador sensible
CONGÉNITAS OTRAS CAUSAS pero no específico de disfunción hepática y permite cierta
, Síndrome de Crigler-Najjar , Hígado graso cuantificación de la reserva funcional hepática.
tipos lyll , Daño hepático por alcohol Se elevan en enfermedades hepáticas colestásicas pero son
, Síndrome de Gilbert , Enfermedad renal normales en pacientes con síndrome de Gilbert y síndrome de
OTRAS CAUSAS Dubin-Johnson; se pueden utilizar para hacer la distinción en­

1
• Hematoma
tre ambas entidades pero su uso es poco frecuente, ya que no
es altamente sensible.
ESTUDIO DEL PACIENTE CON UN
NIVEL ELEVADO DE BILIRRUBINA 25. ¿Cuáles son los patrones de daño hepático y su
grado de intensidad?
23. ¿Cómo se inicia el estudio de un paciente con bi­ Los patrones de daño e intensidad más comunes son:
lirrubina elevada? • Patrón de necrosis hepatocelular.
La presencia de la ictericia conjuntiva! sugiere un nivel de bi­ • Aminotransferasas con elevación leve a moderada persis­
lirrubina total en suero de al menos 3.0 g/dL, pero no permite tente.
la diferenciación entre la hiperbilirrubinemia conjugada y no • Elevaciones marcadas de aminotransferasas.
conjugada. La coloración oscura de la orina puede indicar la • Patrones colestásicos.
presencia de hiperbilirrubinemia conjugada. Las anormalidades en las pruebas de función hepática pue­
La evaluación del paciente con una elevación aislada de los den aportar importantes pistas en el diagnóstico diferencial de
niveles de bilirrubina en suero es bastante diferente de la eva­ enfermedad hepática; sin embargo, estos patrones de enferme­
luación del paciente con una bilirrubina elevada asociada con dad hepática no siempre resultan específicos o, bien, pueden
los niveles de enzimas hepáticas elevadas. tener estados de mezclas de patrones, por lo que la cuidadosa
El primer paso en la evaluación de un paciente con una historia clínica y la exploración física deben ser integradas a la
elevación aislada de los niveles de bilirrubina en suero es de­ interpretación anormal de estudios bioquímicos del hígado.
terminar si es conjugada (BD) o no conjugada (BI).
• BI predominante: BD < 15% se debe descartar: una sobre­ 26. ¿Cuál es la característica de un patrón de necro­
producción de bilirrubina por hemólisis que puede ocurrir sis hepatocelular?
en un gran número de trastornos hereditarios o adquiridos Resulta de forma predominante de una elevación anormal de
(cuadro 11-12-2). aminotransferasas, lo cual debe establecerse según los rangos
Si no se identifica una causa, descartar una deficiencia en­ de elevación de AST, ALT y a la relación AST /ALT correlacio­
zimática genética por alteración de la conjugación; la más co- nada con la clínica.

739
e MANUAL PARA EL EXAMEN NACIONAL DE RESIDENCIAS MÉDICAS

27. ¿Cuál es la característica de un patrón de amino­ La hepatitis isquémica demuestra elevación abrupta y sú­
transferasas con elevación leve a moderada persis­ bita con aminotransferasas > 2 000 U/L con una rápida norma­
tente? lización (menos de cinco a siete días), seguida de una mode­
• Leve ( <300 U/L): es inespecífica y puede resultar de cual­ rada pero persistente prolongación de tiempo de protrombina,
quier tipo de enfermedad hepática. Las causas en pacientes FA y bilirrubinas, con elevaciones de DHL de más de 5 000 U/
asintomáticos varían según el orden geográfico y el tipo de L y marcada falla renal.
población; en Estados Unidos el hígado graso (NASH, por El diagnóstico de hepatotoxicidad inducida por drogas in­
sus siglas en inglés) es la causa más común de pacientes cluye historia de fármacos consumidos por pacientes. La gra­
asintomáticos con transaminasas elevadas en rango de leve vedad de la duración de la transaminasemia varía en relación
a moderado; otras causas comunes son hepatitis por alcohol, con la causa, pero las elevaciones suelen ser más prolongadas
virales y otras razones metabólicas del hígado, así como to­ que en la hepatitis isquémica. La causa más frecuente de hepa­
xicidad hepática inducida por fármacos. topatía asociada con fármacos es el paracetamol; la dosis re­
• Moderada (elevaciones de 300 a l 000 U/L): tiene causas querida para daño hepático extenso es más de 10 a 12 gramos;
más comunes: virus hepatotróficos de hepatitis A-E; Epstein­ sin embargo, dosis menores pueden causar grave daño hepá­
Barr virus, citomegalovirus y virus del herpes simple, toxi­ tico en pacientes que tienen un periodo prolongado de inges­
cidad inducida por fármacos y/o drogas, la enfermedad de ta de alcohol o quienes tienen enfermedad hepática subya­
Addison y el esprúe celiaco. Durante el diagnóstico y la eva­ cente y/o cirrosis. En estos casos la exploración física puede
luación deben excluirse causas conocidas para descartar proporcionar datos de ayuda diagnóstica, como son: fiebre,
diagnósticos susceptibles de tratamiento en enfermedades exantema y eosinofilia. Otros agentes asociados con hepato­
silentes; por tanto, deben obtenerse estudios serológicos de toxicidad que condiciona marcada elevación de aminotrans­
hepatitis B y C en pacientes con factores de riesgo para he­ ferasas son la inhalación de anestésicos, solventes y carbono
patitis viral, así como cinética de hierro en pacientes con tetraclorhídrico.
historia familiar de hemocromatosis y enfermedad de Wil­
son, y marcadores de autoinmunidad hepática en pacientes
29. ¿Cuál es la característica de un patrón de coles­
femeninos jóvenes. tasis?
• Persistente y de origen incierto: deben suspenderse todos los La colestasis se refiere al deterioro del flujo normal de bilis
fármacos con potencial de daño hepático, así como los su­ caracterizado bioquímicamente por una elevación de forma
plementos alimenticios y evitar el consumo de alcohol. predominante de fosfatasa alcalina, gammaglutamil transpep­
tidasa, bilirrubina conjugada y a menudo es asociado con pru­
Para enfermedad hepática alcohólica se necesitan varias se- rito e ictericia.
manas para disminuir los niveles a la normalidad. En NASH La elevación de los niveles séricos de FA y BD no son espe­
la pérdida de peso es importante. cíficos de obstrucción ductal biliar, ya que pueden ser asocia­
La biopsia es viable cuando las medidas de intervención han dos con una amplia variedad de enfermedades, incluso hepatitis
sido inefectivas y los niveles de aminotransferasas persisten ele­ alcohólica, hepatitis A y estadios terminales de la enfermedad
vados por encima de 1.5 veces el limite normal por más de seis hepática, así como consumo de drogas y fármacos.
a doce meses.
30. ¿Cuáles son las diferencias entre colestasis in­
28. ¿Cuál es la característica de un patrón de eleva­ trahepática y extrahepática?
ción marcada de aminotransferasas? Es importante diferenciar la colestasis intrahepática de la extra­
El diagnóstico diferencial de elevada transaminasemia (niveles hepática. La historia clínica, la exploración física y los resul­
mayores de 1 000 U/L) incluye en primer lugar a la hepatitis tados de laboratorio son complementarios para el diagnóstico,
viral, la hepatotoxicidad por drogas y la hepatitis isquémica. así como los estudios de imagen de la via biliar, y en algunos
El daño hepático asociado con isquemia resulta en marca­ casos la biopsia hepática es necesaria para confirmar el diag­
da elevación de aminotransferasas séricas de forma abrupta y nóstico o establecer la gravedad de la enfermedad.
rápida (pocos días), comparado con las elevaciones resultado En el proceso de asociación típica de elevaciones FA aso­
de disfunción hepática inducida por virus o drogas en la cual ciada con obstrucción parcial de la vía biliar es recomenda­
la elevación es más lenta (semanas). Por tanto, la historia clí­ ble pensar en coledocolitiasis, estenosis de conductos biliares
nica y el tiempo de evolución tienen particular relevancia en intrahepáticos como colangitis esclerosante primaria, cirrosis
elevaciones graves y agudas; la hepatitis isquémica (choque he­ biliar primaria, colangitis, obstrucción de la vía biliar por tu­
pático) suele ocurrir en pacientes que experimentan un descen­ mores o enfermedad infiltrantes (colangiocarcinoma, amiloi­
so abrupto en la tensión arterial o un volumen circulatorio dosis, enfermedades granulomatosas e infección por micobac­
ínefectivo, lo cual casi siempre sucede después de cirugía, falla terias). Los fármacos que producen daño hepático con patrón
cardiaca, infarto del miocardio, arritmias, sepsis, quemaduras colestásico son: esteroides anabólicos, estrógenos y antipsi­
extensas, trauma grave, choque cardiaco o falla multiorgánica. cóticos.

740
SECCIÓN 11 GASTROENTEROLOGÍA @
31. ¿Cuáles son los estudios de elección en un pa­ dios complementarios, incluso recurrir a la biopsia hepática
ciente con dilatación de la vía biliar? para confirmar el diagnóstico.
Ante un patrón colestásico es necesario realizar estudios de
imagen del hígado y las vías biliares, como ultrasonido o tomo­ 33. ¿Cuáles son y cuándo están indicadas las prue­
grafía. En pacientes con dilatación de la vía biliar es mejor bas de función hepática cuantitativas?
realizar estudios selectivos para evaluación de la vía biliar co­ Las limitaciones de las pruebas de función hepática para pre­
mo colangiorresonancia, colangiopancreatografía retrógrada cisar la gravedad de la enfermedad hepática requieren cuida­
endoscópica o colangiografía transhepática percútanea. dosa interpretación de los resultados. Las pruebas cuantitati­
vas en teoría serían más sensibles para predecir intensidad de
32. ¿Qué estudios están indicados en una colesta­ enfermedad hepática; sin embargo, esto es controversia! aun­
sis sin obstrucción evidente? que hay múltiples estudios sobre su uso como marcadores de
En ausencia de obstrucción biliar ductal o lesión focal hepáti­ daño o disfunción hepática. Entre las pruebas cuantitativas
ca, una prueba de anticuerpos antimitocondriales u otros estu- del hígado, las más relevantes son: depuración de indocianina

FIGURA 11-12-2. ABORDAJE DE PACIENTES CON ALTERACIÓN DE LAS PRUEBAS DE FUNCIÓN HEPÁTICA.

PACIENTE CON
HISTORIA EXPLORACIÓN
PRUEBAS DE FUNCIÓN
CLÍNICA FÍSCA
HEPÁTICA ANORMAL

Pruebas anormales, Historia familiar o personal Fiebre/ictericia/dolor/exantema/


relación entre ellas de enfermedades hepáticas obesidad/hepatomegalia/
AST/ALT FA/GGT y autoinmunitarias, toxicomanías, signos en relación con daño
fármacos, factores de riesgo hepático crónico u otras
Tiempo de evolución enfermedades
Enfermedad concomitante

INTEGRAR UN PATRÓN
DE DAÑO HEPÁTICO
1
Diagnósticos posibles Diagnósticos posibles Posibles etiologías Posibles etiologías

• Hepatitis viral • Obstrucción biliar • Hepatitis isquémica • Hepatitis viral crónica


• Hepatitis alcohólica (litos, tumor, entre otros) • Hepatitis viral aguda • Hepatitis autoinmunitaria
• Toxicidad por drogas • Infiltración tumoral • Falla hepática fulminante • Hígado graso no alcohólico
o fármacos • Enfermedad hepática con • Falla multiorgánica • Toxicidad por drogas
• Hígado graso no colestasis • Enfermedades por
alcohólico Cirrosis biliar primaria depósito de hierro y cobre
Colangitis autoinmunitaria (hemocromatosis y
Toxicidad por drogas enfermedad de Wilson)
Enfermedades • Enfermedades
granulomatosas granulomatosas
• Trastorno congénito del • Hepatitis alcohólica
metabolismo. • Cirrosis
Dubin Johnson/Rotor • Infiltración hepática
Cligler-Najjar I y 11 • Esprúe celiaco
• Trastorno de eritropoyesis • Deficiencias enzimáticas

741
e MANUAL PARA EL EXAMEN NACIONAL DE RESIDENCIAS MÉDICAS

-
CUADRO 11-12-3.
PRINCIPALES PRUEBAS DE FUNCIÓN HEPÁTICA Y SUS CARACTERÍSTICAS

1
OTROS SITIOS DE SÍNTESIS FUNCIÓN ASOCIACIÓN CON ENFERMEDAD
ALT Hígado principalmente Marcador de daño tísular Hepatitis viral, tóxica y ++++ 10 a 55 UI
autoinmunitaria
AST Músculo esquelético, Marcador de daño tisular, Hepatitis viral, tóxica y ++ 10 a 40 UI
eritrocitos, riñón, pán- hepático y otros órganos autoinmunitaria
creas, cerebro y miocardio
FA Hueso, tumores, placenta e Enfermedad hepática Colestasis +++ 5 a 15 U/L
intestino (elevaciones moderadas) Infiltración tumoral a hígado
Colestasis difusa o
enfermedad infiltrante
GGT Hepatocitos y epitelio biliar Alteración de la excreción Colestasis y alteración de +++ O a 30 U/L
se alteran por consumo biliar a daño canalicular excreción biliar
de alcohol y fármacos
Bilirrubinas Sólo se producen en hígado Enfermedades del hígado, Hemólisis +++ O.O a 1.0 mg/dl
pero su depuración puede obstrucción de conductos Daño muscular
verse afectada por biliares, virus, alcohol Hematoma
alteración renal Eritropoyesis anormal
TP Hígado/se afecta por uso de Alteración de la síntesis Falla hepática aguda o crónica ++++ 10.9 a 12.5
anticoagulantes hepática y déficit de vitamina K segundos
Obstrucción biliar
Albúmina Hígado Falla en la síntesis hepática Hepatopatía aguda o crónica ++++ 3.5 a 5.0 g/dL
Catabolismo incrementado Desnutrición
5'Nucleotidasa Hígado y otros tejidos Déficit en la depuración Colestasis, enfermedades ++++ O a 11 U/L
hepáticas

verde, capacidad de eliminación de galactosa, prueba de ami­ Jost G, Wahllander A, van Mandach U et al. Overnight salivary caffeine clea­
nopirina, depuración de antitripsina y cafeína, entre otras; sin rance: A liver function test suitable far routine use. Hepatology. 1987;7:
338-44.
embargo, su utilidad aún se encuentra bajo estudio de ensayos Kaplan MM. Serum alkaline phosphatase: another piece is added to the
clinicos controlados. puzzle. Hepatology. 1986;6:526.
Kunde SS, Lazenby AJ, Clements RH et al. Spectrurn ofNAFLD and diag­
34. Mencione de manera resumida las principales nostic irnplications of the proposed new normal range for serurn ALT in
obese women. Hepatology. 2005;42:650-6.
pruebas de función hepática y sus características.
Lester R, Schrnid R. Bilirubin metabolism. N Engl J Med. 1964;270:77986.
Véanse figura 11-12-2 y cuadro 11-12-3. Levi AJ, Gatmaitan Z, Arias IM. Two hepatic cytoplasmic protein fractions,
Y and Z, and their possible role in the hepatic uptake ofbilirubin sulfobro­
mophthalein, and other anions. J Clin Invest. 1969;48:215667.
BIBLIOGRAFÍA Maddrey WC. Drug-induced hepatotoxicity. J Clin Gastroenterol. 2005;39:
Addario L, Scaglione G, Tritto G et al. Prognostic value of quantitative liver S83-9.
function tests in viral cirrhosis: A prospective study. Eur J Gastroenterol Pateria P, de Boer B, MacQuil!an G. Liver abnormalities in drug and substan­
Hepatol. 2006; 18:713-20. ce abusers. Best Pract Res Clin Gastroenterol. 2013 Aug;27(4):577596.
Armutcu F, Akyol S, Ucar F, Erdogan S, Akyol O. Markers in nonalcoholic doi: 10.1016/j.bpg.2013.08.001. Epub 2013 Aug 22.
steatohepatitis. Adv Clin Chem. 2013;61:67-125. Pitan A, Poynard T, lrnbert-Bismut F et al. Factors associated with serurn
Bani-Sadr F, Miailhes P, Rosenthal E et al. Risk factors far grade 3 or 4 gam­ alanine transaminase activity in healthy subjects: Consequences for the
ma-glutamyl transferase elevation in HN/hepatitis C virus coinfected pa­ definition ofnorrnal values, for selection ofblood donors, and for patients
tients. AIDS. 2008;22: 1234-6. with chronic hepatitis C. Hepatology. 1998;27: 1213-9.
Bellest L, Eschwege V, Poupon R et al. A modified international normalized Poland RL, Odell GB. Physiologic jaundice: The enterohepatic circulation of
ratio as an effective way ofprothrombin time standardization in hepatolo­ bilirubin. N Engl J Med. 1971;284:l-6.
gy. Hepatology. 2007;46:528-34. Prati D, Taioli E, Zanel!a A et al. Updated definitions of healthy ranges for
Clark JM, Brancati FL, Diehl AM. The prevalence and etiology of elevated serum alanine aminotransferase levels. Ann Intern Med. 2002;137:I-IO.
aminotransferase levels in the United States. Am J Gastroenterol. 2003; Rothschild MA, Oratz M, Schreiber SS. Serurn albumin. Hepatology. 1988;
98:960. 8:385-401.
Giannini E, Botta F, Fasoli A et al. Progressive liver functional irnpairment is Rothschild MA, Oratz M, Zimmon D et al. Albumin synthesis in cirrhotic
associated with an increase in AST/ALT ratio. Dig Dis Sci. 1999;44:1249-53. subjects studied with carbonate 14C. J Clin Invest. !969;48:344-50.
Giannini E, Botta F, Fasoli A et al. Progressive liver functional impairment is Seifert CF, Anderson DC. Acetaminophen usage patterns and concentrations
associated with an increase in AST/ALT ratio. Dig Dis Sci. 1999;44: 1249- of glutathione and gamma-glutamyl transferase in alcoholic subjects.
53. Pharmacotherapy. 2007 Nov;27( 11): 1473-82.

742
SECCIÓN 11 GASTROENTEROLOGrA @
Tripodi A, Chantarangkul V, Prirnignani M et al. The international normali­ Yang MH, Chen TH, Wang SE et al. Biochemical predictors for absence of
zed ratio calibrated for cirrhosis normalizes prothrombin time results for common bile duct stones in patients undergoing laparoscopic cholecys­
model for end-stage liver disease calculation. Hepatology. 2007;46:520-7. tectomy. Surg Endose. 2008;22: 1620-4.

CAPÍTULO 11.13
HEPATITIS VÍRICAS
Mayra Virginia Ramos Gómez

ASPECTOS GENERALES DE LAS HEPATITIS una media de 28 días. Se presenta un cuadro prodrómico que
AGUDAS VÍRICAS corresponde a la fase de incubación básicamente asintomáti­
ca, pero con excreción viral alta en las heces; la fase preictéri­
1. ¿Qué es la hepatitis? ca tiene una duración aproximada de uno a siete días, con
La hepatitis es la inflamación y la necrosis de los hepatocitos, malestar general, anorexia, febrícula o fiebre, dolor abdomi­
causadas por diferentes agentes que pueden ser tóxicos, como nal, náusea, vómito, en ocasiones diarrea, disgeusia, alteracio­
medicamentos, sustancias (alcohol, drogas ilícitas), bacterias, nes en el olfato (hiposmia, cacosmia) y repulsión por el ciga­
virus sistémicos, hongos y enfermedades autoinmunitarias, en­ rro. En la fase ictérica se acompaña de acolla y coluria, con
tre otras. También existen las hepatitis producidas por virus he­ una duración de tres a cuatro semanas. En los cuadros de co­
patotróficos. En esta sección se abordarán virus de hepatitis lestasis hay prurito y el resto de los síntomas va desaparecien­
A, B, C, D yE. do en la fase de convalecencia que tiene una duración de seis
a doce semanas.
INFECCIÓN POR VHA Una variante es la hepatitis recurrente o bifásica, que se ca­
racteriza porque, tras la mejoría de los síntomas y de los mar­
2. ¿Cuáles son las características de virus de la he­ cadores bioquímicos, donde las aminotransferasas pueden es­
patitis A? tar casi normales o normales, hay una nueva elevación de las

1
Es un enterovirus de la familia picornavirus, con un genoma aminotransferasas y la bilirrubina.
ARN, carece de envoltura, tiene siete genotipos (I-VII), con
una variación geográfica en su presentación. Sobrevive al am­ 5. ¿Cómo se efectúa el diagnóstico?
biente por periodos prolongados, es resistente al calor y esta­ La elevación de las aminotransferasas (ALT y AST), con pre­
ble a pH de l. Es directamente citopático. La replicación viral domino de la ALT, mayor de cinco veces el valor normal, has­
puede efectuarse en los hepatocitos y las células intestinales. ta cifras de 5 000 UI o más; las bilirrubinas se elevan a expen­
Altamente contagioso, pero deja inmunidad permanente. sas de la bilirrubina directa y la fosfatasa alcalina es normal o
con mínima elevación; cuando es muy alta se asocia con cua­
3. ¿Cómo se adquiere la enfermedad? dros de colestasis.
La transmisión principal es fecal-oral; las causas son: falta de La serología viral para el diagnóstico diferencial se hace
higiene y hacinamiento; se transmite de persona a persona, por con las fracciones de inmunoglobulinas lgM anti-VHA que se
agua y por alimentos contaminados (leche, mariscos); de for­ encuentran presentes en la fase aguda de la enfermedad, con
ma excepcional con agujas contaminadas, como entre usua­ un descenso progresivo con duración de tres a seis meses del
rios de drogas o bien por transfusiones, y a través de contacto inicio de la infección, mientras que la lgG anti-VHA se detecta
sexual ( oral-anal). Existen grupos de riesgo para contraer la durante la fase aguda con una elevación progresiva que alcan­
infección. za títulos altos durante la fase de convalecencia y persiste de
por vida, lo cual otorga inmunidad permanente; véase figura
11-13-1.
4. ¿Cuál es la historia natural de la enfermedad?
La edad más frecuente de presentación es la preescolar y esco­
lar, con un pico entre los cinco y nueve años. Se presenta en 6. ¿Existe vacuna contra la hepatitis A?
forma aguda, por lo general subclínica o anictérica, sobre todo En 1995 la FDA aprobó la vacuna de virus inactivos (HA­
en la infancia (alrededor de l0% de los niños presenta cuadro VR.IX; hay otras, como la VAQTA) para uso en personas ma­
ictérico); el adulto suele tener más sintomatología, incluso yores de dos años de edad, con una efectividad de 94 a l00%,
cuadro colestásico y con evolución más prolongada de 12 a 18 intramuscular en deltoides, y aplicaciones posteriores a los
semanas. Tiene un periodo de incubación de 15 a 50 días, con seis y doce meses.

1 743
e MANUAL PARA EL EXAMEN NACIONAL DE RESIDENCIAS MÉDICAS

FIGURA 11-13-1. PERFIL SEROLÓGICO DE LA HEPATITIS A. uno de ellos. Se replica en el citoplasma de los hepatocitos. En
México el genotipo 2 es el más frecuente. Tiene una distribu­
ción mundial y es endémico en algunos países en desarrollo.
Marcadores
Se han descrito epidemias en México (estado de Morelos),
Etapa Medio Oriente, Asia central y África. Los brotes epidémicos
Estado se han descrito después de la temporada de lluvias.

Síntomas
9. ¿Cómo se contrae la infección por virus E?
Oa14 La vía de transmisión es fecal-oral (por lo general por agua o
15 a 45 días días 3 a 6 meses Años alimentos contaminados). Sus partículas están presentes en
las heces y la bilis desde el final de la incubación hasta dos o
:;:,
ro tres semanas después del inicio de la enfermedad. La transmi­
'º'üe Anti-VHA sión entre personas es mínima. No existe evidencia de trans­
total misión por vía sexual. Puede ocurrir transmisión vertical; se
ha observado el virus en leche materna, pero no se han docu­
ue mentado casos de transmisión al lactante. Se ha sugerido la
o
u transmisión sanguínea y con menos frecuencia por medio de
anti-VHA lgM-
trasplante de órganos. Se ha considerado como zoonosis.
Grupos de riesgo: habitantes de zonas endémicas, viajeros
a las áreas mencionadas, trabajadores con animales conside­
Tiempo desde exposición al VHA
rados como reservorios de virus de hepatitis E y los refugiados
en campamentos construidos como consecuencia de desastres
naturales.
En 2001 la FDA aprobó la vacuna TWINRIX que combina
las vacunas contra las hepatitis A y B para personas mayores 10. ¿Cuál es el cuadro clínico de la hepatitis E?
de 18 años de edad.
Periodo de incubación de dos a nueve semanas con una media
de 45 días. La edad en que se observa con más frecuencia es
7. ¿Cuál es el tratamiento para la hepatitis A?
entre los quince y cuarenta años. Los síntomas son similares a
No hay evidencia para una dieta rica en carbohidratos; la dieta los de hepatitis de otras etiologías; otros síntomas menos co­
baja en grasas es útil en pacientes que presenten sintomatolo­ munes son: artralgia, diarrea, prurito y urticaria; 50% de los
gía digestiva. En casos no complicados no se justifica la res­ pacientes presenta vómito y dolor abdominal, este último pue­
tricción de proteínas en la dieta. Se debe evitar la ingestión de de persistir en algunos pacientes de dos a cuatro semanas más
bebidas alcohólicas. en el periodo posictérico. La fase de ictericia dura doce a
El tratamiento con medicamentos será sólo de soporte, se­ quince días. En general se sabe que se autolimita; sin embar­
gún la sintomatología. No hay un tratamiento específico con­ go, se ha reportado que el genotipo 3 puede evolucionar a la
tra la infección. No se justifica, por falta de evidencia, el uso cronicidad. Esto se ha observado en pacientes inmunosupri­
de "hepatoprotectores" ni vitaminas; en lo posible deben sus­ midos postrasplante de órganos. La mortalidad varía de 0.4 a
penderse los medicamentos potencialmente hepatotóxicos, 4%. La muerte por hepatitis fulminante ocurre en embaraza­
como los tratamientos anticonceptivos. das en 20 a 30% de los casos y es mayor en el tercer trimestre;
No es necesario el reposo en cama y la restricción del ejer­ existen muchas muertes fetales y partos prematuros, con alta
cicio estará en función de la sintomatología a causa de la adi­ mortalidad de alrededor de 33%. El periodo de convalecencia
namia. dura alrededor de dos meses.
La hospitalización se requiere si el paciente da indicios de En la exploración física se encuentra hepatomegalia dolo­
falla hepática aguda, en donde será preciso tener un cuidado rosa hasta en 60% de los pacientes en la fase preictérica y casi
intensivo y, de ser necesario, valorar la necesidad de trasplan­ en todos en la fase ictérica. La esplenomegalia es poco fre­
te hepático. cuente.
INFECCIÓN POR VHE
11. ¿Qué estudios son relevantes para hacer el diag­
8. ¿Cuáles son las características del virus E? nóstico de hepatitis E?
Clasificado en el género hepevirus de la familia Hepiviridae, Elevación de aminotransferasas, GGT y bilirrubinas; lgG an­
contiene un genoma ARN de cadena simple, compuesto por ti-VHE e lgM anti-VHE. Los anticuerpos pueden detectarse
la única cadena estructural de sentido positivo. Se conocen antes de que inicie la sintomatología. Se ha identificado ARN
cinco genotipos con diversos subtipos, de acuerdo con cada VHE al inicio del cuadro clínico y desaparece en diez días. La

744
SECCIÓN 11 GASTROENTEROLOGÍA @
primera que aparece es la lgM y desaparece después de tres a 90 a 95%; antes de los seis meses de edad en 80%; antes de los
seis meses de la fase aguda. cinco años de edad entre 25 y 50%; mayores de cinco años
entre 5 y 10%.
12. ¿Existe alguna vacuna contra la hepatitis E?
18. ¿Cuál es la manifestación clínica de la hepatitis B?
No se desarrolla comercialmente en forma universal; la Admi­
nistración del Estado de Alimentos y Drogas de China autori­ Periodo de incubación de 45 a 180 días, con un rango de 60 a
zó la HEV 239, vendida como Hecolin, en 2012. 90 días. La aparición del HBsAg es desde dos semanas des­
pués de adquirir la infección y de forma excepcional de seis a
13. ¿Cuál es el tratamiento para la hepatitis E? nueve meses; estas variaciones están relacionadas con diferen­
tes factores, como la cantidad de inóculo, la vía de transmi­
Medidas de soporte para la fase aguda. Al parecer la ribaviri­
sión y las características del huésped.
na, con o sin peginterferón, está dando buenos resultados en
La fase aguda es semejante a otros tipos de hepatitis, desde
la fase crónica.
cuadros subclinicos o anictéricos o cuadro clásico de hepati­
tis, con una etapa preictérica con duración de dias a una se­
INFECCIÓN POR VHB
mana con fiebre baja, anorexia, mialgias, exantema, artralgias,
náusea y vómito que preceden a la ictericia, coluria y acolia.
14. ¿Cuáles son las características del virus de la he­
Puede haber pérdida de peso y dolor en el cuadrante superior
patitis B?
derecho.
El virus de la hepatitis B pertenece a la familia hepadnavirus,
con ácido nucleico ADN. La partícula viral completa, conoci­ 19. ¿Cuáles son las manifestaciones extrahepáticas
da como partícula Dane, posee una núcleo-cápside core (Agc­ de la hepatitis B?
VHB) que contiene el ADN viral y la enzima polimerasa viral, Exantema, artralgias y artritis, poliarteritis nodosa con com­
rodeada por otra capa de lipoproteínas que contiene a los an­ plejos inmunitarios que contienen AgsVHB y AgeVHB en las
tígenos core (c) y e, y en la parte exterior el antígeno de super­ lesiones vasculares, glomerulopatía por complejos inmunita­
ficie (AgsVHB). Se han identificado nueve genotipos (A a 1). rios con los mismos antígenos; manifestaciones neurológicas
El virus se encuentra en la mayoría de los fluidos corporales de (polineuropatía, convulsiones y síndrome de Guillain-Barré).
pacientes con enfermedad aguda y crónica o en portadores. Este virus se considera oncogén (el ADN se integra al hepato­
cito) y el paciente puede desarrollar carcinoma hepatocelular,
15. ¿Cómo se adquiere la hepatitis B?

1
con o sin cirrosis.
Los modos de transmisión son:
20. ¿Cómo se diagnostica la hepatitis B y cuál es su
a) Perinatal (transmisión vertical).
comportamiento serológico?
b) Contacto con una persona infectada (transmisión horizon­
tal), puede ser familiar. AgsVHB: en la hepatitis B aguda la presencia del AgsVHB
e) Sexual, cuyos grupos de riesgo son: hombres que tienen precede a la elevación de las aminotransferasas y hasta la con­
sexo con hombres, heterosexuales promiscuos, sexoservi­ valecencia (cuatro a seis meses); si dura más de seis meses se
dores. considera hepatitis crónica. Se puede detectar en sangre y
d) Parenteral: transfusión de sangre y hemoderivados, recep­ otros fluidos corporales (calostro, saliva, secreciones vagina­
tores de órganos sólidos, material contaminado en consu­ les, semen, exudados, leche materna, etcétera).
mo de drogas tanto inyectadas como inhaladas (cocaína), Anti-AgcVHB: aparece con la elevación de las aminotrans­
pacientes en hemodiálisis, punciones corporales por acu­ ferasas y persiste de por vida. El AgsVHB desciende de forma
puntura, tatuajes o piercings y heridas con material conta­ progresiva mientras aparece el anti-AgsVHB.
minado como los utensilios caseros (tijeras, cortauñas, ce­ AgsVHB y AgeVHB: aparecen al mismo tiempo. Ambos
pillos de dientes) y dentro de la práctica médica. indican replicación viral, así como infectividad. En los casos
de curación o depuración del virus, el AgeVHB desaparece
antes que el AgsVHB.
16. ¿Se transmite la infección durante el embarazo
lgM anti-AgcVHB: aparece desde el inicio de la hepatitis
de madre a hijo?
hasta tres a doce meses.
El VHB de madres AgsVHB positivas es transmitido a 90% de lgG anti-AgcVHB: sugiere exposición pasada al virus de
los recién nacidos (vertical). hepatitis B. La vacunación no lo desarrolla.
Anti-AgeVHB: se detecta cuando el AgeVHB desaparece;
17. ¿Con qué frecuencia evoluciona a la cronicidad es un marcador de resolución espontánea, aun cuando el Ags­
la hepatitis B? VHB esté detectable. La pérdida del AgeVHB y la presencia
De acuerdo a la edad a la que se adquiere la infección, el ries­ de anti-AgeVHB se asocian con resolución bioquímica e histo­
go de evolucionar a la cronicidad es el siguiente: al nacimiento lógica de la enfermedad.

745
e MANUAL PARA EL EXAMEN NACIONAL DE RESIDENCIAS MÉDICAS

-
--
ADN VHB: se detecta durante las fases aguda y crónica. CUADRO 11-13-2.
Véanse cuadros 11-13-1 y 11-13-2 y figura 11-13-2 DIAGNÓSTICO DE FASE CRÓNICA

21. ¿Cuáles son los criterios de resolución o curación INFECCIÓN CRÓNICA


de hepatitis B?
AgsVHB + 6 meses + 6 meses + 6 meses
Criterios para establecer hepatitis resuelta: antecedente de he­
AgeVHB +!-
patitis B aguda o crónica o presencia de anti-AgcVHB, an­
ti-AgsVHB, AgsVHB negativo, ADN VHB indetectable, enzi­ Anti-AgsVHB

mas normales. Anti-AgeVHB +!- + +


Infección oculta por VHB: ADN VHB detectable, ausencia Anti-AgcVHB + + +
de AgsVHB, con o sin anti-VHB fuera del periodo de ventana. ADN VHB >2 000 a 20 000 con >20 000 52 000
sérico AgeVHB-
22. ¿Es recomendable la vacuna contra la hepatitis B? ADN VHB 2'.20 000 con AgeVHB +
Esta vacuna está incluida en la vacuna pentavalente en Méxi­ sérico
co; se recomienda la vacunación en población joven a la que ADN VHB + + +
no se le administró dicha vacuna y a grupos de riesgo, perso­ tejido
nas con enfermedad hepática crónica y personas con infec­ Enzimas Elevadas, persistentes o Normales Normales
ción por VlH. Se aplica intramuscular en deltoides en tres hepáticas intermitentes
dosis: al inicio ( día O), al mes y a los seis meses de la primera. Biopsia Hepatitis crónica con Presente Ausente
Pacientes con inmunosupresión requieren dosis mayor. hepática necroinflamación (90%) (>50%)
moderada a grave

23. ¿Cuál es el tratamiento para la hepatitis B?


El tratamiento se recomienda en la fase crónica de la enfer­
1 y, de él, el subtipo 1b. El genotipo 3 se relaciona con mayor
medad y depende de si el paciente es AgeVHB positivo o Age­
riesgo de hepatitis avanzada. Se estima que 5% de los portado­
VHB negativo, así como el nivel de aminotransferasas, la carga
res de AgsVHB están coinfectados con el VHD; existen alre­
viral y el estadio histológico. Ha sido aprobado para el trata­
dedor de 15 000 000 de personas infectadas por este virus en
miento: interferón alfa 2a y 2b, peg-interferón alfa 2a y 2b;
el mundo. Véase figura 11-13-4.
análogos nucleótidos: lamivudina, adefovir, entecavir, teno­
fovir, telbivudina y emtricitabina en monoterapia o combi­
25. ¿Cuál es el cuadro clínico de la hepatitis D?
nados.
La presentación de la infección por el virus D o delta puede
ser: aguda, que se resuelve, fulminante o crónica. El periodo
INFECCIÓN POR VHD
de incubación delta es de tres a siete semanas; en la fase preic­
24. ¿Qué características tiene el virus de hepatitis D? térica se puede presentar anorexia, fatiga, letargo y náusea,
dura tres a siete días y se elevan las aminotransferasas. Existen
Es un virus ARN de una cadena, de la familia Deltaviridae; es
dos formas de presentación:
un virus ARN incompleto que necesita el antígeno de superfi­
cie del virus de hepatitis B para transmitir su genoma de célu­ a) La coinfección, en la que el paciente se infecta al mismo
la a célula. Tiene variaciones genéticas conocidas como cuasi­ tiempo por el virus de hepatitis B y por el D; se presenta he­
especies T. Se reconocen tres genotipos, el más frecuente es el patitis aguda que por lo general se autolirnita y puede evo-

CUADRO 11-13-1.
DIAGNÓSTICO DE FASE AGUDA

MW1'41+
ANTl·AGSVHB ANTl·AGCVHB
lgM
IM@fri
+
ANTl·AGEVHB IGM ANTl·VHD FASE CLÍNICA
Infección aguda, infectividad elevada

+ lgG + Infección crónica, infectividad elevada

+ lgG + Infección crónica, infectividad baja

+ lnmun[zación (vacunación)

+ lgG + Recuperación

+ igM +!­ +!­ Infección aguda

igM +/- +!- Periodo de ventana

+ igM + Coinfección virus hepatitis D

746
SECCIÓN 11 GASTROENTEROLOGÍA @
FIGURA 11-13-2. DIAGRAMA DE HEPATITIS B AGUDA.

Ictericia

Síntomas

Ventana
HBsAg I
I

----
/ Anti-HBc
I
I
I
I
.._
....,I I....

,,,
I .._
I .._
...........
I
I
I
I Anti-HBe
I
I
I
I Enzimas hepáticas

2 3 4 5 6 12 24
Meses después de la exposición

lucionar de leve a grave o fulminante. Cuando la infección 26. ¿Cómo se elabora el diagnóstico de hepatitis D?
aguda se autolimita, la convalecencia inicia con la desapa­ a) Infección aguda: AgVHD y ARN VHD aparecen de forma
rición de los síntomas y el que más persiste es la fatiga. temprana, y más adelante la lgM anti-VHD; el anti-VHD
b) La hepatitis crónica casi siempre ocurre por sobreinfección. puede tardar entre 30 y 40 días en aparecer. Presencia del

1
La falla hepática aguda es diez veces más frecuente en la anti-AgsVHB.
sobreinfección, con una elevada mortalidad. El VHD pue­ b) Coinfección: anti-lgM VHD y anti-IgG VHD detectables
de producir cirrosis en 70% de los casos y al parecer tam­ durante el curso de la infección. La presencia de lgM anti­
bién aumenta la probabilidad de carcinoma hepatocelular. VHD junto con AgsVHB y el lgM anti-core establecen el

FIGURA 11-13-3. DIAGRAMA DE HEPATITIS B CRÓNICA.

Síntomas
Anti-HBc

HBsAg

g Anti-HBe
� /
.....
/
/

/
/
Diseminación /
viral
/ Enzimas hepáticas

2 3 4 5 6 2 3 4 5 6 7 8 9
Meses Años

Tiempo después de la exposición

1 747
e MANUAL PARA EL EXAMEN NACIONAL DE RESIDENCIAS MÉDICAS

FIGURA 11-13-4. PERFILES SEROLÓGICOS DE LA COINFECCIÓN DE VHB Y VHD.

Hepatitis B - Portador crónico Hepatitis D


Sin seroconversión Sobreinfección
Duración incubación Infección aguda Infección crónica Duración Infección aguda (Años)
(4 a 12 semanas) (6 meses) (años) (6 meses)

"'> anti-HBc total


e
'0
·u AgHBe
/ AgHBs

anti-HD
e --- anti-HBc lgM AgHD
Qj
u
e
o
u Tiempo Tiempo

Hepatitis B - Portador crónico Hepatitis O


Seroconversión tardía Coinfección
Duración incubación Infección aguda Infección crónica Duración Infección aguda (Años)
(4 a 12 semanas) (6 meses) (años) (6 meses)

"'> "'
> anti-HBc total
"'
·.¡:;

� �
'º·ue AgHBc
anti-HBc total 'º·ue
� �
eQj AgHBs -anti-HB e
u � AgHD --anti-HBc lgM
e e
o o
u Tiempo u Tiempo

diagnóstico de coinfección. En alrededor de 15% de los pa­ 28. ¿Cuál es el tratamiento para el virus de hepati­
cientes la única evidencia de la hepatitis D puede ser por tis D?
detección temprana de anti-VHD JgM durante el inicio del Sólo hay evidencia de tratamiento para la fase crónica con in­
periodo agudo o anti-VHD IgG sólo durante la convalecen­ terferón.
cia. Por lo general el anti-VHD disminuye a valores indetec­
tables después de la resolución de la infección. El AgVHD
se detecta en suero sólo en 25% de los pacientes; cuando el INFECCIÓN POR VHC
AgVHD es detectable suele desaparecer cuando el Ags­
29. ¿Cuáles son las características del virus de he­
VHB desaparece.
patitis C?
e) Sobreinfección: los títulos de AgsVHB disminuyen al tiem­
po que el AgVHD aparece. El AgVHD y el ARN VHD El virus de hepatitis C tiene cadena ARN simple, de la familia
permanecen detectables en la enfermedad crónica. En la Flaviviridae. Se han identificado seis genotipos y varios subti­
fase crónica se tiene tanto IgM anti-VHD como IgG an­ pos y tiene una gran tasa de mutaciones. La OMS estima que
ti-VHD. En la resolución de la infección desaparece el IgM hay 170 millones de portadores crónicos, alrededor de 3% de
anti-VHD. El ARN VHD es muy útil en el diagnóstico, so­ la población mundial. En México la tasa de prevalencia es va­
bre todo en los resultados falsos negativos, en el caso de riable, siendo mayor en el norte del país con una media de
que los anticuerpos se encuentren secuestrados por com­ 1.4%. Predominan los genotipos l a y el lb y en menor propor­
plejos inmunitarios con el antígeno. ción 2a y 3b. La mortalidad varía de 0.1 a 0.3% y en adultos
mayores de 50 años de edad es de alrededor de 1.8%.
27. ¿Existe alguna vacuna contra el virus de la hepa­
titis D? 30. ¿Cuáles son los factores de transmisión de la
No existe una vacuna específica para el virus D, pero la vacu­ hepatitis C?
nación contra la hepatitis B y evitar factores de riesgo para la Parece ser la hepatitis postransfusión más común; en pacien­
infección de hepatitis B disminuyen la coinfección por D, tes multitransfundidos presenta una frecuencia de alrededor
pues necesita del VHB para generar infección en humanos. de 7 a 10%. Receptores de órganos sólidos, Influyen otros fac-

748
SECCIÓN 11 GASTROENTEROLOGÍA @
tores como promiscuidad, consumo de drogas ilícitas tanto de 33. ¿Cuál es el genotipo más frecuente en México
uso intravenoso como inhalado (cocaína), instrumental y uten­ de hepatitis C?
silios domésticos infectados o no esterilizados, tatuajes, acu­ El genotipo I b es el más frecuente en el medio. El genotipo 1
puntura y piercings. La transmisión vertical madre-hijo ocurre tiene menor porcentaje de respuesta al tratamiento y el geno­
en cerca de 6% de los casos y aumenta si la madre está coin­ tipo 2 reporta mejor tasa de respuesta.
fectada con VIH y/o hepatitis B. La hepatitis C no se transmi­
te por abrazos, besos, compartir utensilios de alimentos ni por
34. ¿Cómo se trata la hepatitis C?
alimentación al seno materno, pero habría que tener cuidado
cuando existe lesión del pezón. Grupos de riesgo similares a la Se informa en diferentes estudios clínicos que en el tratamien­
hepatitis B. to de la fase aguda para evitar la cronicidad se ha usado tanto
interferón estándar como pegilado. Para la hepatitis C crónica
� http://www.cenetec.salud.gob.mx/descargas/gpc/ los tratamientos recomendados en la actualidad en México
CatalogoMaestro/336_IMSS_10_Hepatitis_Cronica_C/EyR_ son en terapia doble o triple: la primera con interferón pegila­
IMSS_336_10.pdf
do alfa 2a o 2b combinado con ribavirina; la triple agrega el
boceprevir a los medicamentos anteriores.
31. ¿Cuáles son las manifestaciones clínicas de la
hepatitis C? 35. ¿Cuál puede ser un efecto adverso del uso de
Sólo alrededor de 15% de los pacientes presenta una hepatitis interferón pegilado?
aguda sintomática. Las manifestaciones clínicas son semejan­ El uso de interferón pegilado se ha asociado con alteraciones
tes al resto de las hepatitis y suelen resolverse en dos a doce de la función tiroidea, tanto hiper como hipotiroidismo. La
semanas. Los casos de insuficiencia hepática aguda son raros, tasa de aparición de estas alteraciones es de sólo 6% de todos
< 1 %. Entre 70 y 85% de los pacientes evolucionará a la cro­ los pacientes que la reciben. El riesgo es mayor en mujeres, en
nicidad y 20 a 30% a cirrosis. El tiempo de evolución varía de especial aquellas que tienen anticuerpos antitiroideos preexis­
20% a 30 años por diferentes factores asociados, como consu­ tentes al recibir el tratamiento. Por tanto se deben hacer mo­
mo de alcohol, tabaquismo, más de 40 años de edad al adqui­ nitoreos de pruebas tiroideas basales y a las doce semanas de
rir la infección, obesidad, coinfección con VIH o hepatitis B, la aplicación del interferón.
género masculino y depósito de hierro en el hígado. Cada año
5% de los pacientes se descompensará y 2 a 7% desarrollará 36. ¿Cuáles son los estudios altamente recomenda­
carcinoma hepatocelular. De éstos, alrededor de 25% morirá

1
dos en la evaluación pretratamiento de los pacien­
por insuficiencia hepática o requerirá trasplante hepático. tes con virus de hepatitis C?
Los pacientes pueden cursar con manifestaciones extrahe­
Los estudios altamente recomendados en la evaluación pretra­
páticas como: crioglobulinemia mixta esencial, glomerulonefri­
tamiento de los pacientes con virus de hepatitis C son: biopsia
tis membranoproliferativa, úlceras corneales, eritema nodoso,
hepática para estadiaje, en especial en aquellos con genotipo
poliarteritis nodosa, porfiria cutánea tarda, vasculitis necro­
1, examen oftalmológico en pacientes que además son diabéti­
sante y liquen plano, entre otras.
cos e hipertensos y determinación de ferritina y saturación de
hierro.
32. ¿Cómo se efectúa el diagnóstico de hepatitis C?
En la fase aguda existe elevación de aminotransferasa y bilirru­ � http://www.cenetec.salud.gob.mx/descargas/gpc/
CatalogoMaestro/336_IMSS_10_Hepatitis_Cronica_C/EyR_
binas, como en cualquiera otra hepatitis. El primer marcador
IMSS_336_10.pdf
serológico que se solicita es anticuerpos anti-VHC, que son de­
tectados en 50 a 70% de los casos al iniciar los síntomas y en
90% a los tres meses. Los anticuerpos pueden ser indetec­ BIBLIOGRAFÍA
tables después de ocho semanas de la infección. Este estudio Aguilera Guiaro A. Romero Yuste S, Refueiro BJ. Epidemiología y manifesta­
no identifica si la infección es aguda o crónica, mientras que el ciones clínicas de las hepatitis virales. Enferm lnfecc Microbio! Clin.
ARN VHC se detecta entre una y dos semanas, por lo que es 2006;24(4 ):264-76.
Centro Nacional de Excelencia Tecnológica en Salud. Diagnóstico y trata­
el mejor método de diagnóstico de hepatitis C aguda. El ARN
miento de hepatitis C. Evidencias y recomendaciones. México: CENE­
VHC medido por el método de PCR (reacción en cadena de TEC; 2011.
polimerasa) puede ser cualitativo o cuantitativo y refleja la re­ Centro Nacional de Excelencia Tecnológica en Salud. Diagnóstico y trata­
plicación viral, que habla de la presencia de infección; asimis­ miento de la infección crónica por virus de hepatitis B. Evidencias y reco­
mo es necesario solicitar el genotipo viral. mendaciones. México: CENETEC; 2011.
Conde González C, Torres-Poveda K, Madrid-Marina Y. Presentación hepati­
Si se observó un cuadro agudo pasados seis meses se deter­
tis virales. México: Secretaria de Salud Pública. 2011 :53(1 ):S4-S6.
mina la cronicidad de la enfermedad; pero si el paciente estu­ Dirección General de Epidemiología. Manual de procedimientos estandariza­
vo asintomático, por el grado de fibrosis hepática se presupo­ dos para la vigilancia epidemiológica de las hepatitis virales. México: Se­
ne la cronicidad. cretaria de Salud; 2012.

749
e MANUAL PARA EL EXAMEN NACIONAL DE RESIDENCIAS MÉDICAS

Government of Alberta. Public health notifiable disease management guideli­ Protection Agency. 2009. Disponible en: https://www.gov.uk/governrnent/
nes hepatitis A [monografia en internet]. Alberta Health. 2013. Disponible uploads/system/uploads/attachment_data/file/363023/Guidance_for_
en: http://www.health.alberta.ca/documents/Guidelines- Hepatitis-A-2013. the_Prevention_and_Control_of_ Hepatitis_A_lnfection.pdf
pdf Toro Montoya Al, Restrepo Gutiérrez J C. La clínica y el laboratorio hepatitis
Guida M, Mangia A, Faa G. Chronic viral hepatitis: The histology report. B. Medicina & Laboratorio. 2011; 17(7-8):311-29.
Digestive and Liver Disease. 2011;43S:S331-43. Vázquez Campuzano R. Hepatitis virales. Departamento de Microbiología y
Peña A. Hepatitis viral aguda. Rev. Chil. Pediatr. 2002;73(2): 173-5. Parasitología. Recursos en Virología. México: Universidad Nacional Au­
Sánchez Partidas DA, Gutiérrez García CR. Virus de la hepatitis E. Caracte­ tónoma de México; 2014.
rísticas biológicas y epidemiológicas. Revista de la Sociedad Venezolana World Health Organization. Department of Communicable Disease Survei­
de Microbiología. 2012;32:6-12. llance and Response. Hepatitis A. monografía en internet] WHO. 2007.
Tanno H, Fay O. Hepatitis viral en América Latina. Acta Gastroenterol Lati­ Disponible en: http://www.who.int/csr/disease/hepatitis/HepatitisA_who
noam. 2005;35: 169-82. cdscsredc2000_ 7.pdf
Thomas L, the Hepatitis A Guidelines Group. Guidance for the Prevention and
Control ofHepatitis A Infection [monografia en internet] London: Health

CAPÍTULO 11.14
FÁRMACOS E HÍGADO
Fobia/a Maely González Ortiz

MECANISMOS DE TOXICIDAD tivo por la propensión de los hepatocitos a reducir el oxígeno,


en particular en las mitocondrias, pero también en los sistemas
1. ¿Qué nivel de elevación de enzimas hepáticas ex­ microsomales de transporte de electrones (como el sistema
presa daño tóxico para los hepatocitos? CYP2E l ).
ALT cinco veces su valor normal.
6. ¿Cuáles son los mecanismos antioxidantes del hí­
2. ¿Qué medicamento se utiliza para prevenir la fa­ gado?
lla hepática en la intoxicación por paracetamol? Micronutrientes como las vitamínas E y C, proteínas ricas en
La administración de donantes de cisteína estimula la sintesis grupos tiol (metalotioneína), proteínas secuestradoras de me­
de glutatión (N-acetilcisteina) y es el fármaco de elección. Es tales (ferritina) y enzimas que metabolizan los metabolitos
preferible la vía oral para su administración a dosis de 140 reactivos (epóxido hidrolasas).
mg/kg, seguida de la admínistración de 70 mg/kg cada cuatro
horas hasta 72 horas.
7. ¿Qué células se ven implicadas en el daño hepáti­
3_ ¿Cuáles son las características clinicopatológicas co tóxico a través de la respuesta inmunitaria innata?
de la hepatopatía de origen medicamentoso? Las células de Kupffer funcionan como macrófagos residentes
Véase cuadro 11-14-1. y como células presentadoras de antígenos. Algunos de los
efectos tóxicos pueden estar mediados por la liberación de ci­
4. ¿En qué situaciones se aumenta la síntesis de tocínas, como factor de necrosis tumoral (TNF) y Fas-L. Las
glutatión por los hepatocitos? células estrelladas del hígado (células de lto) son el principal
tipo de célula hepática implicada en el depósito de la matriz
Los hepatocitos son el lugar exclusivo de la síntesis de gluta­
en la fibrosis hepática.
tión; las concentraciones hepáticas van de 5 a 10 mmol/L y su
síntesis aumenta si se potencia el sumínistro de sustancias
prooxidantes (cisteína), como cuando se sobreexpresa el siste­ 8. ¿Cuál es el mecanismo más frecuente por el cual
ma CYP2E l. Este mecanismo es el pilar del tratamiento con se produce la hepatopatía de origen medicamentoso?
antídotos tiol para el envenenamiento por paracetamol. La mayoría de los casos de hepatopatía de origen medicamen­
toso representa reacciones adversas a los fármacos o reaccio­
5. ¿Qué es el estrés oxidativo? nes del hígado ante los fármacos. Estas reacciones son efectos
Es un estado de desequilibrio entre los componentes prooxi­ nocivos y no deseados que se producen con la dosis recomen­
dantes y los oxidantes. El hígado está expuesto al estrés oxida- dada como profilaxis o tratamiento.

750
SECCIÓN 11 GASTROENTEROLOGÍA @
CUADRO 11-14-1.
CLASIFICACIÓN CLINICOPATOLÓGICA DE LA HEPATOLOGÍA DE ORIGEN MEDICAMENTOSO

EJEMPLOS DE FÁRMACOS
CATEGORÍA DESCRIPCIÓN IMPLICADOS
Adaptación hepática Sin síntomas; elevación de las concentraciones séricas de GGT y FA (en Fenitoína
ocasiones, elevación de ALT) Warfarina
Hepatotoxicidad Síntomas de hepatitis; necrosis zonal, confluyente y masiva; concentración Paracetamol
dependiente de la dosis sérica de ALT aumentada >5 veces, a menudo >2 000 U/L Rifampicina
Ácido nicotínico
Amodiaquina
Esteatosis aguda Esteatosis microvesicular, difusa o zonal, parcialmente dependiente de la dosis; Ácido valproico
características de toxicidad mitocondrial Didanosina
Algunos productos de hierbas
Esteatohepatitis Esteatosis, necrosis focal, cuerpos hialinos de Mallory, fibrosis pericelular Amiodarona
Floxuridina intraarterial
Perhexilina
Hepatitis aguda Síntomas de hepatitis; necrosis focal, confluente y masiva; concentración sérica lsoniacida
de ALT >5 veces; características extrahepáticas de alergia a fármacos en Nitrofurantoína
algunos casos Halotano
Fenitoína
Disulfiram
Ketoconazol
Hepatitis crónica Duración >3 meses; hepatitis de interfase, necrosis confluente, fibrosis, cirrosis; Minociclina
características clínicas y analíticas de hepatopatía crónica Nitrofurantoína
Diclofenaco
Nefazodona
Hepatitis granulomatosa Granulomas hepáticos con hepatitis y colestasis variables; elevación de las Alopurinol
concentraciones séricas de ALT, GGTP y FA Carbamazepina
Hidralazina
Colestasis sin hepatitis Colestasis sin inflamación; concentración sérica de FA >2 veces lo normal Anticonceptivos orales

1
Andrógenos

Hepatitis colestásica Colestasis con inflamación; síntomas de hepatitis; elevación de las Clorpromacina
concentraciones séricas de ALT y FA Antidepresivos tricíclicos
Amoxicilina-ácido clavulánico

Síndrome de vías biliares Escasez de vías biliares pequeñas; se parece a una cirrosis biliar primaria pero Clorpromacina
evanescentes con AMA negativos Trimetoprim-sulfametoxazol

Colangitis esclerosante Estenosis de vías biliares grandes Floxuridina intraarterial

Trastornos vasculares Síndrome de obstrucción sinusoidal, hiperplasia nodular regenerativa Azatioprina


Mitomicina
GGTP, gammaglutamil transpeptidasa; FA, fosfatasa alcalina; ALT, aminotransferasa alanina; AMA, anticuerpos antimitocondriales.

FAC TORES DE RIESGO PARA DAÑO HEPÁTICO inmunitaria, la respuesta al estrés de la célula y las vias de
POR FÁRMACOS muerte celular.

9. ¿Qué factores son determinantes para el desarro­ 11. Es un ejemplo de predisposición a reacción ad­
llo de reacciones idiosincráticas? versa hepática:
En la mayoría de las reacciones idiosincráticas los determi­ Hepatitis por ácido valproico y por fenitoína.
nantes del huésped son fundamentales para que se produzca
la lesión hepática. 12. ¿A qué edad son más frecuentes las reacciones
hepáticas adversas a medicamentos?
10. ¿Qué factores del huésped son los más importan­ La mayoría de las reacciones hepáticas adversas son más fre­
tes para el desarrollo de una reacción idiosincrática? cuentes en adultos que en niños. Esto es consecuencia funda­
Los factores genéticos determinan la actividad de las vías que mental del aumento de la exposición y del uso de varios fár­
activan fármacos y las vias antioxidantes, codifican las vias de macos, así como de la alteración en la distribución de los
la secreción biliar en los canalículos y modulan la respuesta mismos.

751
e MANUAL PARA EL EXAMEN NACIONAL DE RESIDENCIAS MÉDICAS

13. ¿Existe algún predominio de sexo para presentar 21. ¿Cuáles son las enzimas responsables de las re­
reacciones hepáticas adversas a medicamentos? acciones de fase 2?
Sí. Las mujeres están en particular predispuestas a la hepatitis Las reacciones de fase 2 implican la formación de uniones
de origen medicamentoso. La hepatitis crónica causada por ésteres con el compuesto original o un metabolito del fárma­
nitrofurantoína, sulfonamidas y minociclina predomina de co. Las enzimas responsables son las glucuronil transferasas,
forma pronunciada en las mujeres. las sulfotransferasas, las glutatión S-transferasas y las acetilo y
aminoácido N-transferasas.
14. ¿La cirrosis alcohólica o la colestasis predispo­
nen a los pacientes a padecer reacciones hepáticas 22. ¿Cuáles son las vías de eliminación de fase 3?
adversas? Las vías por las que los fármacos, sus metabolitos o sus conju­
En general estas enfermedades no predisponen a los pacientes gados se excretan desde el hígado (fase 3) implican las proteí­
afectados a padecer reacciones hepáticas adversas. Como ex­ nas de transporte de unión a ATP (ABC). Estas proteínas in­
cepciones se pueden citar algunos fármacos antineoplásicos o cluyen el regulador de conductancia transmembrana de la
metotrexato. fibrosis quística (CFTR) y los transportadores de cobre cana­
liculares e intestinales.
15. ¿De qué forma el alcohol predispone a daño he­
pático por medicamentos? 23. ¿Cómo se transportan los fármacos catiónicos
La ingestión crónica de una cantidad excesiva de alcohol dis­ en el hígado?
minuye el umbral de dosis, potencia la gravedad de la hepato­ La proteina de resistencia a múltiples fármacos (MDR) se ex­
toxicidad inducida por paracetamol y aumenta el riesgo de
presa sobre todo en la membrana plasmática apical de los he­
gravedad de la hepatitis por isortiazida.
patocitos, donde transporta fármacos catiónicos, en particu­
lar antineoplásicos, hacia la bilis.
TIPOS DE REACCIONES FARMACOLÓGICAS

16. ¿Cuáles son las vías de metabolismo de fárma­ DIAGNÓSTICO, PREVENCIÓN Y TRATAMIENTO
cos de fase 1?
24. ¿Cuáles son ejemplos de fármacos que causan
En éstas se incluyen reacciones de oxidación, reducción e hi­ hepatotoxicidad dependiente de dosis?
drólisis. Los productos pueden conjugarse o excretarse sin
Paracetamol y algunos preparados de hierbas, toxinas de plan­
modificaciones.
tas y hongos, amodiaquina, vitamina A, metotrexato, ciclofos­
17. ¿En dónde se llevan a cabo las reacciones de famida y metales (hierro, cobre y mercurio) causan hepato­
toxicidad dependiente de dosis.
fase 1?
La mayoría se cataliza mediante oxidasas microsomales para
fármacos, cuyo principal componente es la hemoproteína de 25. ¿Cuál es ta dosis diaria máxima de paracetamot
la superfamilia de genes CYP. Se conocen más de veinte enzi­ para pacientes adultos sanos?
mas CYP en el hígado humano. De 1 a 4 gramos al día.

18. ¿En qué zona del ácino hepático es mayor la con­ 26. ¿Cuál es ta dosis diaria de paracetamot reco­
centración de proteínas CYP? mendada para bebedores crónicos?
El contenido hepático de las proteínas CYP es mayor en la Dosis de 2 gramos por día.
zona 3 de los acinos hepáticos; esta localización explica en
parte la zonalidad de las lesiones hepáticas producidas por
fármacos y por toxinas como paracetamol y tetracloruro de 27. ¿Cuál es el metabolito que causa daño en la in­
carbono, que se convierten en metabolitos reactivos. gestión por paracetamot?
La necrosis hepática se produce sólo cuando la concentración
19. ¿Qué se entiende por inducción enzimática? de glutatión cae por debajo del nivel crítico, momento en el
La exposición a sustancias lipofílicas da lugar a una respuesta cual se permite que el NAPQI produzca lesión hepática.
de adaptación que suele implicar la síntesis de una nueva pro­
teina CYP. 28. ¿Cuáles son tos factores de riesgo para desarro­
llar hepatitis por halotano?
20. ¿Cuáles son los fármacos que pueden realizar Edad avanzada (>40 años), sexo femenino, obesidad y predis­
una inducción enzimática? posición familiar son factores de riesgo reconocidos para de­
Rifampicina e isoniazida. sarrollo de hepatitis por halotano.

752
SECCIÓN 11 GASTROENTEROLOGÍA @
29. ¿Cuáles son las características asociadas con 33. ¿Cuál es la diferencia entre paracetamol y AINE?
hepatotoxicidad por cocaína? El paracetamol no es considerado un AINE; actúa en el siste­
Rabdomiólisis, hipertensión, hiperpirexia, coagulación intra­ ma nervioso central para inhibir síntesis de prostaglandinas.
vascular diseminada e insuficiencia renal. Los AINE inhiben la misma vía pero en tejidos periféricos.

34. ¿Cuál es el mecanismo de acción del paracetamol?


30. ¿Cuál es la dosis mortal de ingestión de setas
que producen hepatotoxicidad? Los ácidos grasos componen la membrana celular y uno de
ellos es el ácido araquidónico. Cuando la membrana es daña­
La ingestión de una única dosis de 50 gramos de setas es sufi­
da, el ácido araquidónico utiliza dos vías metabólicas: COX y
ciente para ser mortal.
LOX. La vía metabólica COX produce prostaglandinas y la vía
LOX produce leucotrienos. Las prostaglandinas son molécu­
31. ¿Qué son las aflatoxinas? las que median inflamación, dolor y fiebre. El paracetamol
Son una familia de micotoxinas que se encuentran en Asper­ inhibe de forma reversible esta vía metabólica (COX) a nivel
gillus jlavus y hongos relacionados, que son ubicuos en las del sistema nervioso central.
regiones tropicales y subtropicales. Estas toxinas contami­
nan cacahuates y otros cereales y son hepatotóxicas y carcinó­ 35. ¿Cuándo se solicita una biopsia hepática?
genas. La biopsia hepática se solicita cuando sigue en duda el diag­
nóstico. Sin embargo, no hay ningún signo patognomónica
32. ¿Cuál es el síndrome clínico relacionado con el histológico de lesión hepática causada por un fármaco.
consumo de aflatoxinas? BIBLIOGRAFÍA
Fiebre, malestar general, anorexia, vómito, ictericia, seguido Narci T, Shivakumar C, Geoffrey F. Liver disease caused by drugs. En: Feld­
de la presencia de hipertensión portal con esplenomegalia y man M, Friedman LS. Sleisenger and Fordtran's Gastrointestinal and Li­
ascitis. th
ver Disease. 9 ed. New York: Saunders; 2010. pp. 1413-39.

CAPÍTULO 11.15
HEPATITIS CRÓNICAS
Víctor Hugo García y García 1
CLÍNICA Y DIAGNÓSTICO CUADRO 11-15-1.
CAUSAS DE LA HEPATITIS CRÓNICA
1. ¿Cuál es la definición de hepatopatía crónica?
HEPATOPATÍA ETIOLOGÍA
Se define como la lesión hepática que dura al menos seis me­
Enfermedad grasa Hepatitis alcohólica y no alcohólica (esteatosis
ses. La cirrosis se define como el proceso difuso de fibrogéne­
del hígado y esteatohepatitis)
sis y formación de nódulos en el parénquima hepático. Se re­
Hepatitis tóxica Por medicamentos, por herbolaria
conoce como la etapa final de todas las hepatopatías crónicas.
Hepatitis virales Virus de las hepatitis B y C

2. ¿Cómo se identifican las lesiones hepáticas? Hepatopatía Hepatitis autoinmunitaria, cirrosis biliar primaria,
autoinmunitaria colangitis autoinmunitaria
En términos bioquímicos se considera lesión hepatocelular a
aquella que condiciona hipertransaminasemia; lesión por co­ Metabólicas o por Enfermedad de Wilson, hemocromatosis genéti-
atesoramiento ca, deficiencia de alfa1-antitripsina, porfiria
lestasis a aquella donde predomina la elevación de fosfatasa
alcalina, gammaglutamil transpeptidasa y puede o no existir Por obstrucción Cirrosis biliar secundaria, fibrosis quística,
biliar atresia biliar
hiperbilirrubinemia de predominio directa; por último, lesión
infiltrativa donde existe una elevación desproporciona! de fos­
fatasa alcalina y gammaglutamil transpeptidasa. CLASIFICACIÓN ANATOMOPATOLÓGICA

3. ¿Cuáles son las causas más frecuentes de hepati­ 4. ¿Cuál es la clasificación histopatológica de las he­
tis crónica? patopatías crónicas?
Véase cuadro 11-15-1. Véase cuadro 11-15-2.

753
e MANUAL PARA EL EXAMEN NACIONAL DE RESIDENCIAS MÉDICAS

CUADRO 11-15-2. 8. ¿Cómo se realiza el diagnóstico de hepatitis au­


CLASIFICACIÓN HISTOPATOLÓGICA DE LAS toinmunitaria?
HEPATOPATÍAS CRÓNICAS El diagnóstico se basa en una combinación de alteraciones his­
ETIOLOGÍA HALLAZGO ESPECÍFICO tológicas, clínicas, bioquimicas y serológicas. Existe un siste­
ma de puntaje para determinar la probabilidad diagnóstica que
Hepatitis C Esteatosis, lesión ductular, infiltrado por
linfocitos
categoriza a la enfermedad como probable o definitiva. Véase
cuadro 11-15-3.
Hepatitis B lnmunohistoquímica para core y Ag de
superficie
9. ¿Qué importancia tiene la biopsia hepática en es­
Hepatitis autoinmunitaria Hepatitis de interfase, infiltrado de
ta patología?
células plasmáticas
Se recomienda biopsia hepática a todos los pacientes en la pre­
Hepatitis alcohólica/no Esteatosis en zona 3 panacinar, baloniza­
alcohólica ción, cuerpos de Mallory-Denk, fibrosis sentación de la enfermedad porque aclara dudas diagnósticas
en zona 3, colestasis canalicular

Deficiencia alfa1 Glóbulos periportales positivos en tinción CUADRO 11-15·3.


antitripsina de PAS
SISTEMA ORIGINAL DE PUNTAJE DEL GRUPO
Hemocromatosis Gránulos de hierro en hepatocitos, colan­ INTERNACIONAL DE HEPATITIS AUTOINMUNITARIA
giocitos, con gradiente en zona 1 a 3

Enfermedad de Wilson Cobre en hepatocitos Género Femenino +2

Cirrosis biliar primaria Colestasis, lesión ductular florida, Relación FNAST >3 -2
granulomas <1.5 +2
Colangitis esclerosante Fibrosis periductal, obliteración ductular Gammaglobulina o lgG >2 +3
primaria (en capas de cebolla) elevada
1.5 a 2 +2
1 a 1.5 +1
HEPATITIS AUTOINMUNITARIA <1 o
ANA, AML, anti-LkM1 >1:80 +3

5. ¿Cuál es la definición de hepatitis autoinmunita­ 1:80 +2


ria? 1:40 +1

Inflamación sin resolver del hígado, de causa desconocida. Se <1:40 o


reconocen factores ambientales y falla en la inmunotolerancia AMA Positivo -4
combinada con predisposición genética, que condicionarán Marcadores virales Positivo -4
un proceso necroinflamatorio en el hígado. Negativo +3
Drogas Positivo -4
6. ¿Cuáles son las características epidemiológicas Negativo +1
más importantes? Alcohol <25 g/día +2

En la hepatitis autoinmunitaria las mujeres son afectadas con >60 g/día -2


más frecuencia en una relación 3: l. No hay diferencia entre HLA DR3 o DR4 +1
grupos étnicos ni en edades. Entre los caucásicos la inciden­ Enfermedad autoinmunitaria Cualquiera +2
cia es de uno a dos por cada 100 000 habitantes. De no recibir Otros A-SLA, A-actina, +2
tratamiento, 40% de los pacientes con hepatitis grave fallecerá A-LC1, pANCA
en los primeros seis meses. El resto desarrollará cirrosis y sus Histología Hepatitis interfase +3
complicaciones. Plasmocitos +1
Rosetas +1
7. ¿Cuál es la presentación clínica de esta enferme­ Ninguna -5
dad?
Cambios biliares -3
La presentación tiene un espectro muy amplio de característi­ Otras -3
cas clínicas, desde el paciente asintomático con sólo hallazgos Respuesta a tratamiento Completa +2
en la bioquímica (transaminasemia persistente), pasando por Recaída +3
sintomatología vaga e inespecífica como fatiga, ictericia, náu­
Diagnóstico probable 10 a 15 pre-tx 12 a 17 pos-tx
sea, dolor abdominal y artralgias. La presentación con hepati­
Diagnóstico definitivo >15 pre-tx >17 pos-tx
tis aguda (40%) o incluso falla hepática aguda no es rara.

754
SECCIÓN 11 GASTROENTEROLOGÍA @
y determina el tratamiento. El hallazgo característico es la he­ gradual 5 mg por semana hasta alcanzar 10 mg/día y poste­
patitis de interfase y la infiltración por células plasmáticas. riormente 2.5 mg por semana hasta lograr 5 mg diarios. El tra­
tamiento debe mantenerse hasta resolver la enfermedad o sus­
10. ¿Qué función desempeñan los marcadores sero­ penderse por falla o intolerancia al mismo.
lógicos en la hepatitis autoinmunitaria?
En la hepatitis autoinmunitaria 96% de los pacientes con HA1 15. ¿Qué esquema se recomendaría a una paciente
tiene los ANA y antimúsculo liso positivos al diagnóstico; y embarazada?
4 % tendrá los antimicrosomales hígado/riñón tipo 1 (LKM-1) Se reconoce que la terapia combinada es bien tolerada por el
y anticitosol/hígado (LC-1) positivos. Sin embargo, no son binomio; sin embargo, la azatioprina tiene categoría D de
específicos debido a que existen fluctuaciones séricas durante acuerdo con la FDA, por lo que la recomendación en general
la enfermedad. En pacientes seronegativos de anticuerpos con­ es la monoterapia a base de esteroides y con la menor dosis
vencionales, los anticuerpos contra el antígeno hepático solu­ posible.
ble (SLA) y los clásicos pANCA pueden concluir el diagnós­
tico. HEPATITIS AGUDA ALCOHÓLICA
11. ¿Cómo se clasifica la HAI según sus anticuerpos?
16. ¿Cómo se diagnostica la hepatitis aguda alcohó­
Se reconocen dos tipos de HAI. El tipo 1 se caracteriza por la lica?
presencia de ANA, AML o ambos y representa 80% de casos;
Es un síndrome clínico caracterizado por la aparición reciente
suelen ser mujeres de entre 16 y 30 años de edad con otras en­
de ictericia y/o ascitis en pacientes con historia de consumo
fermedades autoinmunitarias asociadas. El tipo 2 se caracteri­
abusivo de alcohol y donde se han descartado otras causas de
za por la presencia de anticuerpos anti-LKMl y/o anti-LC l ; la
hepatopatía, que condiciona una alta mortalidad a seis meses
mayoría son niños.
de su presentación. Sin embargo, se prefiere utilizar el térmi­
12. ¿Qué enfermedades autoinmunitarias suelen aso­ no hepatopatía alcohólica crónica agudizada, puesto que se
ciarse con HAI? requiere consumo de más de 100 gramos diarios de alcohol por
semanas o meses.
Tiroiditis autoinmunitaria, enfermedad de Graves, sinovitis,
CUCI, DMI y vitiligo son las enfermedades concurrentes de
17. ¿Cuáles son las manifestaciones clínicas y bio­
mayor reporte en la literatura internacional.
químicas más frecuentes en la hepatitis alcohólica?

1
13. ¿Cuáles son las indicaciones de tratamiento? ¿A En la hepatitis alcohólica la manifestación cardinal es la apa­
todos los pacientes con HAI hay que tratarlos? rición súbita de ictericia. Otros datos son: fiebre, ascitis, ence­
Las indicaciones de tratamiento se clasifican en absolutas, re­ falopatía hepática, hemorragia digestiva, hepatomegalia blan­
lativas y contraindicaciones (véase cuadro 11-15-4), por lo cual da o dura y una asociación muy frecuente con el síndrome de
es necesario entender que no todos los pacientes con HA1 de­ respuesta inflamatoria sistémica. En la bioquímica es carac­
ben recibir tratamiento médico. terística la elevación de AST ( <300 U/mL), leucocitosis, neu­
trofilia, hiperbilirrubinemia e hipoprotrombinemia. La fun­
14. ¿Cuál es el mejor régimen de tratamiento en HAI? ción renal alterada es un signo de mal pronóstico, incluso de
Dos esquemas tienen la misma eficacia: prednisona 60 mg mortalidad.
al día o prednisona 30 mg al día con azatioprina 50 mg al día
(1 a 2 mg/kg/día). La prednisona se deberá reducir de forma 18. ¿Qué importancia tiene la biopsia hepática en la
hepatitis alcohólica?
No es imprescindible para llegar al diagnóstico, aunque ayu­
CUADRO 11-15-4. da como pronóstico al reconocer si existe cirrosis. La lesión
INDICACIONES DE TRATAMIENTO histológica típica es esteatosis con focos de necrosis hepato­
INDICACIONES INDICACIONES
celular y hepatocitos abalonados, asociados con infiltrado in­
ABSOLUTAS RELATIVAS CONTRAINDICACIONES flamatorio agudo. Algunos hepatocitos tienen en su interior
• AST > 10 VLS. • Asintomáticos • Cirrosis no activa
un material eosinofílico (cuerpos de Mallory).
• AST >5 VLS + con alteraciones (Burned out cirrhosis).
gammaglobulina leves en la bioquí- • Riesgo aumentado de 19. ¿Cuál es el pronóstico para los pacientes con he­
>2VLS. mica o histología. intolerancia: osteopo- patitis alcohólica?
• Necrosis en • Resolución es- rosis, compresión ver-
puente o multi- pontánea 12%. tebral, inestabilidad La gravedad y la mortalidad son muy variables. Se recono­
acinar. • Valorar riesgo emocional, hiperten- cen como factores de mal pronóstico la encefalopatía hepáti­
• Síntomas inca- beneficio de sión mal controlada. ca, la bilirrubina > 10 mg/dL, el tiempo de protrombina <50%,
pacitantes. esteroides.
la falla renal y las infecciones bacterianas. Se han aceptado

755
e MANUAL PARA EL EXAMEN NACIONAL DE RESIDENCIAS MÉDICAS

CUADRO 11-15-5. FIGURA 11-15-1. ALGORITMO TERAPÉUTICO EN HEPATITIS


ÍNDICES PRONÓSTICOS EN HEPATITIS AGUDA ALCOHÓLICA.

1 1
PUNTO DE CORTE HISTORIA DE
SISTEMA PARÁMETROS SEVERIDAD
ALCOHOLISMO
Índice discriminatorio Bilirrubina y tiempo de �32
de Maddrey protrombina
SOSPECHAR HEPATITIS
Chil-Turcotte-Pugh Bilirrubina, tiempo �7
de protrombina, ALCOHÓLICA
albúmina, ascitis y Deterioro rápido de función
encefalopatía hepática (BT).
inicio reciente de
MELD Edad, bilirrubina, INR, �21 descompensación clínica.
creatinina
Glasgow Edad, BUN, leucocitos, �9
bilirrubina e INR VALORACIÓN PRONÓSTICA
Lille Edad �0.45 Maddrey. MELD.
Día O: bilirrubina, ABIC. Glasgow.
albúmina, tiempo de
protrombina
Día 7: cambios en BAJO RIESGO
bilirrubina. ALTO RIESGO Valoración nutricia.
Considerar biopsia hepática Tratamiento de complicaciones
ABi Edad, bilirrubina, INR, >I= 9.
transyugular. de cirrosis.
creatinina.

Prednisolona Pentoxifilina 400 mg/ocho


diversos índices pronósticos que identifican a pacientes graves, 40 mg/día/una semana. horas/cuatro semanas.
para quienes el beneficio del tratamiento con esteroides está
justificado. Así, la función discriminante de Maddrey es el siste­
1 En caso de contraindicación
a esteroides o falla renal.

ma más ampliamente utilizado e indica que todo paciente con


más de 32 puntos deberá recibir manejo con esteroides. Aque­
llos con menos puntaje no se benefician con este tratamien­
1 LILLE 1
to. A los siete días de manejo se recomienda el uso del índice
>I= 0.45.
de Lille, el cual define el tipo de respuesta a esteroides. Todo
<0.45. DETENER TRATAMIENTO
aquel paciente con un Lille de menos de 0.45 deberá conti­ Considerar trasplante
Continuar tratamiento
nuar con corticoterapia por tres semanas más; aquellos con por tres semanas más. hepático temprano.
más de 0.45 de Lille deberán suspender prednisona y, en casos
seleccionados, pasar a trasplante hepático, debido a que estos
pacientes tienen pronóstico fatal en 80% de los casos. Véase 22. ¿Cuál es la función de la pentoxifilina en HA?
cuadro 11-15-5. Algunos estudios sugieren que 400 mg cada ocho horas vía
oral de pentox.ifilina tendría la misma eficacia que la predniso­
20. ¿Qué manejo médico integral deben recibir los na, por lo cual en pacientes con contraíndicación de corticote­
pacientes con hepatitis alcohólica? rapia (sepsis, hemorragia digestiva o síndrome hepatorrenal)
La piedra angular, obviamente, es la abstinencia alcohólica. se sugiere como una excelente alternativa. La combinación de
Los pacientes con HA grave se deberán hospitalizar para hi­ esteroides y pentox.ifilína no conlleva a efecto sinérgico ni au­
dratación y soporte calórico suficiente para conseguir un ba­ menta la eficacia. Asimismo, en pacientes no respondedores a
lance nitrogenado positivo ( 1.5 g de proteínas por kilogramo esteroides la pentox.ifilina tampoco ha demostrado ser útil.
al día). Es frecuente el déficit vitamínico, por lo que se acon­
seja administrar preparados vitamínicos de complejo B (ence­ 23. ¿Cuál es el algoritmo terapéutico para pacientes
falopatía de Wernicke), ácido fólico y vitamina K. El uso de con hepatitis alcohólica?
antibióticos profilácticos aún es controversia!. Véase figura 11-15-1.

21. ¿Cuál es el tratamiento específico de elección pa­ BIBLIOGRAFÍA


ra la HA? Boyer TD, Manns MP, Sanyal AJ. Zakim and Boyer's hepatology: a textbook
of liver disease. 6th ed. P hiladelphia: Saunders; 2012.
El uso de corticosteroides en la HA es controversia!. Se acepta Dooley JS, Lok ASF, Burroughs AK, Heathcote EJ. Sherlock's Diseases of
que la prednisona 40 mg diarios en pacientes con HA grave the Liver and Biliary System. 12th ed. New York: Blackwell Publishing Ltd;
(Maddrey >32) mejora la sobrevida a 28 días. 2011.

756
SECCIÓN 11 GASTROENTEROLOGÍA @
Manns MP et al. Diagnosis and Management of Autoimmune Hepatitis. He­ Schiff ER, Maddrey WC, Sorrel MF. Schiff's Diseases of the Liver. 11 th ed.
patology. 2010;51(6):1-31. Philadelphia: Wiley-Blackwell; 2012.
Mathurin P et al. EASL Clinical Practica! Guidelines: Management of Alco­ Singa! AK, Shah VH. Alcoholic Hepatitis: Prognostic Models and Treatment.
holic Liver Disease. Journal of Hepatology. 2012;57:399-420. Gastroenterol Clin N Am. 2011;40:611-39.
O'Shea, Darasathy, McCullough. Alcoholic Liver Disease. Hepatology. 2010;
51( 1 ):307-28.

CAPÍTULO 11.16
CIRROSIS
Azalia Yuriria Ruiz Torres

1. ¿Qué es la cirrosis hepática? e) Tratamiento especifico de las complicaciones.


Es la fase avanzada de múltiples padecimientos inflamatorios d) Escrutinio de hepatocarcinoma.
crónicos que afectan al hígado, los cuales producen destruc­ Para más detalles, véase la figura 11-16-1.
ción del tejido hepático con la posterior evolución a cirrosis.
Los pacientes pueden desarrollar múltiples complicaciones 5. ¿Cuáles son las principales causas de cirrosis he­
graves, como hipertensión portal, hemorragia variceal, ascitis, pática?
falla renal, encefalopatía, infecciones y hepatocarcinoma. En Alrededor de 90% de los casos de cirrosis hepática en países
México la mortalidad por cirrosis hepática ocupa el quinto occidentales es secundario al consumo excesivo de alcohol,
lugar general (tercero en hombres en edad productiva y octavo seguido por la enfermedad por hígado graso no alcohólico
en mujeres), por lo que se considera un importante problema (EHNA) y la hepatitis crónica vírica. En México el alcohol
de salud pública. aún es la causa más frecuente de cirrosis. A escala mundial la

1
hepatitis crónica por el virus de la hepatitis B (VHB) y C
2. ¿Cuáles son los estudios iniciales para corroborar (VHC) con más de 400 millones de enfermos infectados repre-
el diagnóstico de cirrosis hepática? senta la etiología más importante. La causa de la cirrosis per­
En primer lugar, pruebas de laboratorio que sugieran inflama­ manece desconocida en cerca de LO% de los casos (cirrosis
ción hepática y/o disminución de la reserva hepática, es decir, criptogénica) y en alrededor de 70% de estos casos se cree que
pruebas de función hepática completas. Se buscará hiperbili­ en la actualidad se relacionan con la EHNA dentro del contex-
rrubinemia, transaminasemia, hipoalbuminemia y/o inversión to de resistencia a la insulina y síndrome metabólico, mientras
de la relación proteínas totales/albúmina, biometría hemáti­ que el resto puede estar vinculado con mecanismos autoinmu­
ca para descartar leucopenia, anemia y/o trombocitopenia y nitarios. Varios factores causales como hemocromatosis y al-
tiempo de protrombina (TP), pues puede haber prolongación. cohol o alcohol y hepatitis C pueden acelerar la progresión a
Además de ultrasonido hepático convencional o en caso de cirrosis. Véase cuadro 11-16-1.
dudas con este resultado, realizar ultrasonido Doppler.
6. ¿Cuál es la cantidad de consumo de alcohol que
3. ¿Cuál es el estudio de imagen más indicado para se considera como causa de cirrosis?
diagnóstico de cirrosis? El umbral para desarrollar una enfermedad hepática alcohóli­
El ultrasonido convencional y Doppler pueden tener sensibili­ ca grave en los varones se calcula en un consumo superior a
dad y especificidad de 91.1 y 93.5% y son los estudios de pri­ 60 a 80 g de alcohol diario durante diez años, mientras que las
mera elección para diagnóstico. mujeres tienen riesgo de desarrollarla al consumir menos can­
Cuando la paracentesis es mayor a 5 L debe usarse albúmi­ tidad y por menos tiempo.
na humana a razón de 8 a 10 g por litro extraído.
� http://www.cenetec.salud.gob.mx/descargas/gpc/ 7. ¿Cuál es la segunda causa de cirrosis en México?
CatalogoMaestro/087_GPC_VaricesesofagicasNarices_ La infección crónica por virus de la hepatitis C (VHC) y vi­
esofAgicasRR_CENETEC.pdf rus de la hepatitis B (VHB) en conjunto son la segunda cau­
sa de cirrosis en México; debe sospecharse en gran medida
4. ¿Cuál debe ser el abordaje en pacientes con diag­ en pacientes hepatópatas con historia de transfusión sanguí­
nóstico de cirrosis hepática? nea antes de 1991, en pacientes con conductas de alto riesgo,
a) Determinar la etiología. aquellos con infección de VlH o en parejas o hijos de pacien­
b) Establecer la progresión de la enfermedad. tes infectados; pero en 30% de estos individuos no se logra

757
e MANUAL PARA EL EXAMEN NACIONAL DE RESIDENCIAS MÉDICAS

FIGURA 11-16-1. ALGORITMO DIAGNÓSTICO DE PACIENTES CON CIRROSIS HEPÁTICA.

Paciente cirrótico de
reciente diagnóstico

1. Determinar etiología 2. Establecer progresión 3. Tratamiento específico 4. Escrutinio de


de la enfermedad de las compilaciones hepatocarcinoma

•Alcohol
• Hepatitis B o C Escala de Child-Pugh Várices y hemorragia
autoinmune
variceal
• Hígado graso
• Daño por fármacos
Grado de
• Otras
descompensación Ascitis y falla renal

Encefalopatía e
infecciones

determinar la causa de la transmisión. Es importante determi­ ción, mayor es el riesgo de descompensación, complicaciones
nar esta causa, ya que iniciar un tratamiento temprano puede y mortalidad (cuadro 11-16-2).
evitar la progresión a cirrosis.
La enfermedad grasa no alcohólica del hígado ocupa un 10. Después de conocer la causa y de que se ha de­
lugar cada vez más importante como causa de cirrosis en Mé­ terminado el grado de progresión de la enfermedad,
xico debido al aumento de la incidencia de la obesidad. Es la ¿qué otros estudios diagnósticos se requieren?
principal causa de cirrosis en pacientes con diabetes mellitus Todo paciente debe evaluarse con endoscopia al momento del
de larga evolución, pacientes obesos o muchos de los casos diagnóstico para identificar la presencia de várices esofágicas
que antes eran catalogados como idiopáticos. y clasificarlas como pequeñas o grandes. Si no se encuentran
Otras causas menos frecuentes de cirrosis son las patolo­ várices al inicio, debe realizarse control endoscópico cada dos
gías autoinmunitarias, como la cirrosis biliar primaria o las años. Si se encuentran várices pequeñas debe realizarse con­
hepatitis autoinmunitarias, que deben sospecharse sobre todo trol endoscópico cada año. Si se visualizan várices grandes
en mujeres jóvenes con antecedentes familiares o personales debe iniciarse profilaxis.
de autoinmunidad, sin consumo de alcohol y con panel viral
negativo. 11. ¿Cuál es el tratamiento de la cirrosis hepática?
En causas específicas como la hepatitis C, los pacientes con
8. ¿Cuál es el segundo paso en la valoración de la cirrosis Child-Pugh A todavía pueden beneficiarse del trata­
cirrosis hepática? miento específico y disminuir la progresión de la enfermedad.
También la hepatitis autoinmunitaria Child A puede requerir
Determinar la progresión de la enfermedad, ya que el grado de
insuficiencia hepática y el nivel de descompensación tienen tratamiento con esteroides en aquellos pacientes que presen­
tan datos de inflamación activa. Los pacientes con cirrosis
alto valor pronóstico para determinar el riesgo de complica­
biliar primaria requieren tratamiento con ácido ursodesoxicó­
ciones y de mortalidad a corto y largo plazos, lo que permite
lico a dosis de 15 mg/kg. En estadios más avanzados los pa­
identificar a los pacientes que requieren tratamiento y vigilan­
cientes ya están fuera de tratamiento especifico y sólo se les
cia más estrecha, así como el envío temprano a un especialista
apropiado o a un centro de referencia. dan medidas generales para la insuficiencia hepática.

12. ¿Todos los pacientes cirróticos requieren diuréti­


9. ¿Cuál es la escala más utilizada para determinar cos?
el grado de insuficiencia hepática? No, sólo los pacientes con ascitis. El manejo del paciente am­
La escala Child-Pugh-CPS; la suma de los puntos entre las di­ bulatorio con ascitis incluye la restricción en la ingesta de so­
versas variables de la escala determinará si el paciente se en­ dio a 1 a 2 gramos/día y en los casos que no responden sólo
cuentra en etapa A, B o C; entre más elevada esté la puntua- con esta medida se iniciarán diuréticos y se ajustará la dosis

758
SECCIÓN 11 GASTROENTEROLOGÍA @
CUADRO 11-16-1. CUADRO 11-16-2.
ETIOLOGÍA DE LA CIRROSIS HEPÁTICA ÍNDICES PRONÓSTICOS USADOS EN LA PRÁCTICA
CLÍNICA EN CIRROSIS. PUNTUACIÓN DE CHILD-PUGH
Metabólica-tóxica

■1;11111•■■1;1111H■■1;1111H■
Y MORTALIDAD ASOCIADA
Alcohol
Enfermedad de hígado graso no alcohólico (resistencia a la insulina,
síndrome metabólico)
Cirrosis infantil de la India Bilirrubina (mg/dl) <2 2a 3 >3

Infecciosa Albúmina (g/dl) >3.5 2.8 a 3.5 <2.8


Virus de las hepatitis B, C y D INR <1.7 1.7a 2.3 >2.3
Esquistosomiasis
Ascitis Ausente Responde a Ascitis
Autoinmunitaria diuréticos refractaria
Hepatitis autoinmunitaria
Cirrosis biliar primaria Encefalopatía Ausente Grados 1-11 Grados 11I-IV
Colangitis autoinmunitaria La puntuación de Child-Pugh (5 a 15 puntos) es el resultado de la suma de la
Síndromes de solapamiento o superposición puntuación de cada una de las cinco variables. De esa forma se determina:
Child A: 5 a 6 puntos, mortalidad 0% a un año y 15% a los dos años; Child B: 7 a
Inducido por fármacos 9 puntos, mortalidad 20% a un año y 40% a los dos años; Child C: 10 a 15 pun­
Arsénico, metotrexato, isoniazida, amiodarona, a-metildopa, CCl 4 tos; mortalidad 55% a un año y 65% a los dos años.
Genético-hereditaria
Hemocromatosis hereditaria
Enfermedad de Wilson
Déficit de a,-antitripsina
CUADRO 11-16-3.
Porfiria cutánea tarda
ÍNDICE DE MELD Y MORTALIDAD EN LA CIRROSIS
Enfermedades por depósito de glucógeno
Galactosemia
ÍNDICE DE MELD MORTALIDAD A LOS TRES MESES
Tirosinemia
Abetalipoproteinemia <9 1.9%
Fibrosis quística
10a 19 6%
Enfermedades biliares 20a 29 19%
Cirrosis biliar secundaria (obstrucción biliar por estenosis, litiasis de
larga evolución, etcétera) 30a 39 52.3%
Colangitis esclerosante primaria >40 71.3%

1
Colangitis asociada con lgG 4
Colangiopatía isquémica
MELD score = 9.57 Ln (creat, mg/dl) + 3.78 Ln (Sil, mg/dl) + 11.2 Ln (INR) + 6.43.
Ductopenia Datos tomados de Wiesner R, et al.
Atresia de vías biliares
Síndrome de Alagille

Vascular
Insuficiencia cardiaca crónica derecha ("cirrosis cardiaca") ral. Guia de referencia rápida. [Monografía en interne!.] México; 201 l .
Pericarditis constrictiva crónica pp. l -1 l. Disponible en: http://www.cenetec.salud.gob.mx/descargas/gpc/
Síndrome de Budd-Chiari CatalogoMaestro/087_GPC_Varicesesofagicas/Vari ces_esofAgicasRR_
Síndrome de obstrucción sinusoidal (enfermedad venooclusiva) CENETEC.pdf
Telangiectasia hemorrágica hereditaria (enfermedad Rendu-Osler- Centro Nacional de Excelencia Tecnológica en Salud. Diagnóstico y tra­
Weber) tamiento de várices esofágicas. Consejo de Salubridad General. Guia
de referencia rápida. [Monografía en interne!.] México; 20 l l. pp. l-1 l.
Criptogénica
Disponible en: http://www.cenetec.salud.gob.mx/descargas/gpc/Catalo­
Alrededor de 70% de las cirrosis criptogénicas se desarrolla en el
goMaestro/087_GPC_Varicesesofagicas/Varices_esofAgicasRR_CENE­
contexto de resistencia a la insulina y síndrome metabólico
T EC.pdf
Conn HO, Lieberthal MM. T he hepatic coma syndromes and lactulose. Bal­
timore: Williams and Wilkins; 1979.
hasta lograr el control de la misma. Lo más recomendable es Contreras Omaña R. Manual práctico de gastroenterología. México: Ed. Zar­
iniciar tratamiento con diuréticos ahorradores de potasio, co­ Pra; 2011. pp. 114-22.
García Bueya L, González Mateas F, Moreno-Otero R. Cirrosis hepática.Me­
mo la espironolactona a dosis de 100 mg/día, y después, en
dicine. 2012; l l ( l l ):625-33.
caso de requerirlo, agregar furosemida. http://www.cenetec.salud.gob.mx/descargas/gpc/CatalogoMaestro/lMSS_
Todas las demás complicaciones de la cirrosis, como la ence­ 978-607-8270-02-6_Diagnóstico y Tratamiento de la Insuficiencia Hepáti­
falopatía hepática, las infecciones, la falla renal y el desarrollo ca Crónica/978GPC.pdf; Guía de referencia rápida. K74.X Fibrosis y Ci­
de hepatocarcinoma, se comentarán en siguientes apartados. rrosis del hígado, GPC; Diagnóstico y Tratamiento de la Insuficiencia
Hepática Crónica; México: Secretaria de Salud Pública. ISBN: 978-607-
8270-02-6.
BIBLIOGRAFÍA Wiesner R, Edwards E, Freeman R, Harper A, Kim R, Kamath P et al. Model
Centro Nacional de Excelencia Tecnológica en Salud. Diagnóstico y trata­ for end-stage liver disease (MELD) and allocation of donar livers. Gastro­
miento de la insuficencia hepática crónica. Consejo de Salubridad Gene- enterology. 2003 Jan; 124( l ):91-6.

759
e MANUAL PARA EL EXAMEN NACIONAL DE RESIDENCIAS MÉDICAS

CAPÍTULO 11.17
COMPLICACIONES DE LA CIRROSIS
Sara Aquino Pérez

PATOGENIA DE LA HIPERTENSIÓN PORTAL 7. ¿Cuál es la complicación letal más común de la


cirrosis con hipertensión portal?
1. ¿A qué se refiere el término hipertensión portal?
La hemorragia aguda variceal es la complicación letal más
La hipertensión portal se debe al incremento patológico de la común de la cirrosis y causa entre 60 y 70% de todos los epi­
presión venosa portal, de manera que el gradiente de presión sodios de hemorragia. Su frecuencia es de 5 a 10% al año. Son
entre la vena porta y la vena cava inferior aumenta por encima características predictoras de sangrado: la tensión de la pared
de sus valores normales. El gradiente de presión portal está de la várice, sus características morfológicas, así como el gra­
determinado por el producto entre el flujo sanguíneo y la resis­ do de insuficiencia hepática.
tencia vascular en el sistema venoso portal.
8. ¿Cuál es el factor de riesgo más importante para
2. ¿A qué se debe el aumento en la resistencia al el desarrollo de várices esofágicas?
flujo en la cirrosis?
El aumento del gradiente de la presión venosa portal.
a) Alteración en la arquitectura vascular hepática.
b) Contracción activa de las células musculares lisas vascula­
res, miofibroblastos y células estrelladas hepáticas. 9. ¿Cuál es el mecanismo de generación de las vári­
e) Disminución en la producción del óxido nítrico (vasodila­ ces esofágicas?
tador). Primero tiene que existir hipertensión portal, la cual puede
d) Incremento en la liberación de vasoconstrictores endóge­ ocurrir por daño hepático avanzado o trombosis de la vena es­
nos. plénica. El incremento en las resistencias hepáticas aunado a
un aumento del flujo venoso portal provocado por la dilata­
3. ¿Cuál es el valor normal del gradiente de presión ción esplácnica contribuyen al desarrollo de hipertensión por­
entre la vena porta y la vena cava inferior? tal y várices esofágicas.
Entre 1 y 5 mm Hg.
10. ¿Qué porcentaje de pacientes presenta várices
4. ¿Cómo se divide la hipertensión portal de acuer­ al momento del diagnóstico?
do con la localización? Cincuenta por ciento.
a) Prehepática.
b) Intrahepática. 11. ¿Cuál es la tasa del primer sangrado variceal al
e) Poshepática. año del diagnóstico?
De 12% (5% para várices pequeñas y 15% para várices gran­
5. ¿Cuál es la causa más frecuente de hipertensión des).
portal?
Intrahepática: cirrosis. 12. ¿Cuáles son los factores que se asocian con el
riesgo de hemorragia por várices?
CONSECUENCIAS DE LA HIPERTENSIÓN a) Tamaño de las várices.
PORTAL b) Gravedad de la función hepática expresada mediante la cla­
sificación de Child-Turcotte-Pugh.
6. ¿Cuáles son las consecuencias directas de la hi­ e) Presencia de marcas rojo púrpura (vasos de neoformación
pertensión portal? en las paredes de la várice).
a) La hemorragia secundaria a rotura de várices esofágicas
o gástricas es la complicación principal de la hipertensión 13. ¿En qué pacientes se debe sospechar la presen­
portal. cia de várices esofágicas?
b) Ascitis.
En aquellos con:
e) Síndrome hepatorrenal.
d) Síndrome hepatopulmonar. a) Diámetro de la vena porta> 13 mm.
e) Encefalopatía hepática. b) Plaquetas < 100 000.

760
SECCIÓN 11 GASTROENTEROLOGÍA @
14. ¿Cuál es el momento ideal para realizar la endos­ CUADRO 11-17-1.
copia en búsqueda de várices esofágicas? CLASIFICACIONES ENDOSCÓPICAS DE LAS VÁRICES
En todo paciente al momento del diagnóstico de cirrosis. CLASIFICACIÓN ENDOSCÓPICA DE LAS VÁRICES SEGÚN PAQUET
Grado I Protrusión escasamente perceptible
15. ¿Cuál es la recomendación de vigilancia para el
Grado 11 Protrusión hasta 1/4 de la luz esofágica
paciente con cirrosis compensada que no tiene vári­
ces o tiene várices pequeñas en la endoscopia inicial? Grado 111 Protrusión hasta 1/2 de la luz esofágica

Cada uno o dos años. Grado IV Protrusión mayor a la mitad de la luz esofágica

CLASIFICACIÓN DE LAS VÁRICES ESOFÁGICAS DE SOEHENDRA


16. Si el paciente no tiene várices y la hepatopatía
Grado I Repleción leve, diámetro inferior a 2 mm, apenas
está compensada, ¿en qué tiempo se recomienda la elevadas con esófago relajado
siguiente endoscopia?
Grado 11 Repleción moderada, curso serpenteante 3 a 4 mm de
A los dos o tres años. diámetro, limitadas a la mitad esofágica inferior

Grado III Repleción completa tensa, diámetro superior a 4 mm,


17. ¿Cuándo y cómo están indicados los análogos paredes finas, pasan al fondo gástrico
de somatostatina o la terlipresina en los eventos del Grado IV Repleción completa tensa, abarcan todo el esófago, se
sangrado variceal? combinan las gástricas y las duodenales
Los análogos de somatostatina u octapéptidos de somatostati­ CLASIFICACIÓN DE DAGRADI
na como el octreótido, el lanreótido o el vapreótido se inician
Grado I Trayectos rectos, rosados, visibles a través de la mucosa
sólo ante la comprobación endoscópica de sangrado variceal o poco ingurgitados
y se usan desde el evento de sangrado hasta tres a cinco días
Grado 11 Trayectos sinuosos, rosados, poco ingurgitados
posteriores al mismo.
Grado 111 Trayectos violáceos, rectos, ingurgitados

18. ¿Cuáles son las clasificaciones endoscópicas de Grado IV Trayectos violáceos, tortuosos, ingurgitados en racimos
las várices que correlacionan con el riesgo de san­ Grado V Presencia de manchas hematoquísticas (várice sobre
grado? várice)

1
Véase cuadro 11-17-1.

19. ¿Cuáles son las opciones de tratamiento que exis­


ten para la hemorragia variceal? 21. ¿Cuál es el mecanismo de acción de los betablo­
queadores no selectivos en las várices esofágicas?
a) Medicamentos que reducen la presión portal esplácnica: va­
sopresina, somatostatina y análogos (octreótido y vapreó­ Reducción de la presión portal mediante vasoconstricción es­
tido). plácnica debida al bloqueo de los receptores Pradrenérgicos y,
b) Betabloqueadores. por tanto, disminución de la presión en el interior de las vári­
e) Nitratos y simvastatina: incrementan el óxido nítrico en la ces esofágicas y reducción del gasto cardiaco secundario al
circulación intrahepática. bloqueo de receptores p, adrenérgicos.
d) Terapia local: sin efecto en la presión portal.
• Ligadura de várices: colocación de bandas elásticas en 22. ¿Cuáles son las desventajas del uso de betablo­
la várice. queadores?
• Escleroterapia variceal: inyección de sustancias esclero­ a) Fatiga, disnea (por lo general asociadas con bradicardia).
santes. b) Alteraciones del sueño.
• Obturación variceal: inyección de adhesivos tisulares. e) Sólo consiguen reducir el GPVH en una tercera parte de
• Sonda de balones y prótesis esofágicas temporales. los pacientes con cirrosis avanzada.
e) Terapia de rescate:
• Shunt porto cava. 23. ¿Cómo se lleva a cabo la prevención de un pri­
• TIPS. mer evento de sangrado (prevención primaria)?
En un paciente que cumple estas tres: diagnóstico endoscópi­
20. ¿En qué porcentaje de los pacientes con hiperten­ co de várices grandes o pequeñas + etapa Child B/C + estig­
sión portal están presentes también las várices gás­ mas de sangrado inminente. La primera elección son los be­
tricas? tabloqueadores; se deben administrar ya sea propranolol 20
Entre 5 y 33%, con una incidencia de hemorragia de 25% en mg dos veces al día o nadolol 40 mg una vez al día titulados
dos años. por efectos (bradicardia). La segunda línea de tratamiento

761
e MANUAL PARA EL EXAMEN NACIONAL DE RESIDENCIAS MÉDICAS

para los intolerantes a betabloqueadores es la ligadura vari­ GASTROPATÍA DE LA HIPERTENSIÓN PORTAL


ceal. Los betabloqueadores no se usan ante un evento de san­
grado agudo. 31. ¿Qué es la gastropatía hipertensiva portal?
Es el cambio en la mucosa gástrica en pacientes con hiperten­
24. ¿Cuál es el tratamiento de elección para preve­ sión portal. Se caracteriza por la presencia de múltiples áreas
nir el sangrado en pacientes con várices medianas a eritematosas rodeadas por una trama reticular blanquecina y
grandes que nunca han sangrado? manchas rojo cereza. La histología muestra dilatación de los
capilares y las vénulas de la mucosa gástrica.
Betabloqueo.

32. ¿Cuál es la prevalencia e incidencia de la GHP?


25. En el caso de hemorragia por várices, ¿cuál es el
blanco de hemoglobina deseado cuando se indica En el momento del diagnóstico inicial de la cirrosis la preva­
transfusión? lencia es de alrededor de 30% y la incidencia anual de cerca de
12%.
Entre 7 y 8 gramos; mayor nivel de hemoglobina puede incre­
mentar la presión portal.
33. ¿Qué caracteriza a la GHP?
Hemorragia de la mucosa gástrica crónica o manifiesta que se
26. ¿Cuál es el medicamento de elección para el tra­
presenta como anemia ferropénica crónica o como melena.
tamiento del sangrado variceal?
La terlipresina es el medicamento de elección y se usa bolo de 34. ¿Cuál es su principal causa?
2 mg IV seguido de 1 a 2 mg cada cuatro horas durante 48
La regurgitación del flujo sanguineo debido a la hipertensión
horas; la terapia se puede prolongar hasta cinco días a dosis
portal.
de 1 mg cada cuatro horas (seis veces al día).

35. ¿Cuál es el tratamiento para la gastropatía hi­


27. ¿Cuál es una complicación frecuente en pacien­
pertensiva portal?
tes que tienen sangrado variceal?
Los betabloqueadores no selectivos reducen la intensidad de la
Las infecciones de diverso tipo, como la peritonitis bacteriana gastropatía hipertensiva portal y previenen el resangrado. En
espontánea, son más frecuentes, por lo que está indicado ini­ el caso de la hemorragia aguda, la somatostatina y los análo­
ciar de manera profiláctica con quinolonas VO o ceftriaxona gos son el tratamiento de elección.
IV a dosis comunes por siete días.
36. ¿Cuál es el diagnóstico diferencial de la GHP?
28. ¿Cuál es el riesgo de resangrado y mortalidad La ectasia vascular antral, que se caracteriza por acumulación
después de un primer evento? de manchas rojizas que presentan distribución radial desde el
Los pacientes tiene un riesgo de resangrado de 60% y una píloro hasta el antro gástrico (estómago en sandía). La histo­
mortalidad de 33% en los siguientes dos años si no reciben logía se caracteriza por hiperplasia de las células musculares
tratamiento subsecuente con betabloqueadores o ligadura va­ lisas y de los miofibroblastos y fibrohialinosis.
riceal.
ASCITIS
29. ¿Qué es la profilaxis secundaria en hemorragia
variceal? 37. ¿Cuál es la causa más frecuente de ascitis?
Cirrosis en 85% de los casos.
Tratamiento indicado para prevenir hemorragia variceal recu­
rrente.
38. ¿Cuál es la prevalencia de la ascitis en pacientes
con cirrosis compensada?
30. ¿Qué tratamiento deben recibir los pacientes que
Cincuenta por ciento de los pacientes tiene ascitis.
sobreviven a un episodio de hemorragia variceal para
prevenir la recurrencia?
39. Mencione cuatro causas de ascitis diferente a la
Combinación de betabloqueador no selectivo más ligadura va­
cirrosis.
riceal endoscópica.
a) Insuficiencia cardiaca congestiva.
� http://www.cenetec.salud.gob.mx/descargas/gpc/ b) Cáncer (carcinomatosis peritoneal).
CatalogoMaestro/087_GPC_VaricesesofagicasNarices_ e) Peritonitis tuberculosa.
esofAgicasRR_CENETEC.pdf
d) Síndrome nefrótico.

762
SECCIÓN 11 GASTROENTEROLOGÍA @
40. ¿Qué métodos diagnósticos se requieren para 48. ¿Cuáles son las causas frecuentes de ascitis con
determinar el origen de la ascitis? gradiente de albumina <1.1 g/dl?
a) Historia clínica. Carcinomatosis peritoneal, tuberculosis, ascitis pancreática,
b) Examen físico. ascitis biliar, síndrome nefrótico y enfermedad del tejido co­
e) Análisis del líquido. nectivo con serositis.

41. ¿Cuáles son los datos clínicos que sugieren asci­ 49. ¿Cuáles son las opciones de tratamiento para la
tis? ascitis?
a) Aumento del perímetro abdominal. Se dividen en tres grupos:
b) Signo de la ola.
a) Primera línea:
e) Signo del témpano de hielo.
• Restricción de sodio: menos de 2 gramos al día u 88
d) Matidez cambiante.
mmol.
• Abstinencia alcohólica.
42. ¿Cuál es la clasificación de la ascitis?
• Suspender AINE.
a) Ascitis no complicada grados I, 11 y III. • Diuréticos: espironolactona-furosemida, darlos en una
b) Ascitis refractaria. sola toma.
(5) http://www.cenetec.salud.gob.mx/descargas/gpc/ b) Segunda línea:
CatalogoMaestro/087_GPC_VaricesesofagicasNarices_
esofAgicasRR_CENETEC.pdf
• Suspender inhibidores de la ECA, bloqueadores de los
receptores de angiotensina y betabloqueadores.
• Paracentesis terapéutica.
43. ¿Cuál es el sitio de elección para realizar la para­
• Trasplante hepático.
centesis?
• TIPS.
El sitio ideal para realizar una paracentesis es a nivel del cua­
drante inferior izquierdo, 3 cm cefálico y 3 cm medial a la es­ e) Tercera linea:
pina ilíaca anterosuperior. En este sitio la pared abdominal • Derivación peritoneovenosa.
parece ser más delgada.

1
50. ¿Cuál es el tratamiento de la ascitis según la gra­
vedad y el volumen extraído?
44. ¿Cuál es la sensibilidad del signo de matidez
• Ascitis grado I: diuréticos (furosemida, espironolactona) Y
cambiante al detectar ascitis?
verificar la pérdida de volumen mediante la determinación
Sensibilidad de 83% y especificidad de 56%.
seriada del peso.
• Ascitis grados II y III: tambien diuréticos pero puede ha­
45. ¿Qué volumen debe estar presente para detec­ cerse paracentesis evacuadora.
tar la matidez en flancos? • Si la paracentesis es menor a 5 L deben usarse expansores
Alrededor de l 500 ce. del plasma (hemacel, dextran) a razón de 125 mL por litro
extraído.
46. ¿Cuáles son las teorías que existen para expli­
car la formación de ascitis? 51. ¿Cuál es la dosis recomendada de diuréticos para
a) Falta de llenado intravascular por alteración en el balance el tratamiento de la ascitis?
oncótico-hidrostático que lleva a pérdida de fluido intravas­ Dosis única matutina de 100 mg de espironolactona y 40 mg de
cular hacia la cavidad peritoneal. furosemida; se pueden incrementar de acuerdo con la pérdida
b) Sobreflujo: hipervolemia por retención primaria renal de de volumen cada tres a cinco días hasta un máximo de 400/ 160
sodio, con escape de fluido a la cavidad peritoneal. mg de espironolactona y furosemida, respectivamente.
e) Vasodilatación periférica: la hipertensión portal lleva a va­ (5) http://www.cenetec.salud.gob.mx/descargas/gpc/
sodilatación, que causa disminución del volumen arterial CatalogoMaestro/087_GPC_VaricesesofagicasNarices_
efectivo, aumento en la estimulación neurohumoral y re­ esofAgicasRR_CENETEC.pdf
tención de sodio.
52. ¿Cuáles son las indicaciones para suspender los
47. ¿Cuál es la utilidad de determinar el gradiente al­ diuréticos y considerar tratamiento de segunda línea?
búmina suero-ascitis (GASA)? a) Encefalopatía no controlada o recurrente.
Detecta hipertensión portal con una exactitud de 97% cuando b) Sodio sérico menor a 10 mmol/L, a pesar de la restricción
el valor es> 1.1 g/dL. Se obtiene al sustraer el valor de la albú­ de liquido.
mina en el líquido de ascitis al valor de la albúmina sérica. e) Creatinina sérica mayor a 2.0 mg/dL.

I 763
e MANUAL PARA EL EXAMEN NACIONAL DE RESIDENCIAS MÉDICAS

53. ¿Por qué los pacientes con cirrosis deben evitar que son y no candidatos a trasplante hepático y que tienen múl­
los AINE? tiples cicatrices por paracentesis previas.
Porque reducen la excreción urinaria de sodio y pueden indu­
cir azoemia. PERITONITIS BACTERIANA ESPONTÁNEA

54. ¿Cuál es la recomendación para el uso de albú­ 60. ¿Qué es la peritonitis bacteriana espontánea?
mina en las paracentesis? Infección espontánea del líquido ascítico.
Cuando se extraen grandes volúmenes (más de cinco litros)
deben administrarse ocho gramos por cada litro extraído. 61. ¿Cuál es la prevalencia de la PBE?
Entre 10 y 30% de los pacientes hospitalizados con ascitis.
55. ¿Qué es la ascitis refractaria?
Es aquella que no puede ser movilizada debido a una falta de 62. ¿Cuál es la patogenia de la PBE?
respuesta a dosis máxíma de diuréticos (400/160 mg de espi­ a) Translocación bacteriana desde la luz intestinal hasta la
ronolactona y furosemida, respectivamente) y que recurre con circulación.
rapidez después de paracentesis terapéuticas. b) Alteración de la actividad fagocítica del sistema reticuloen­
dotelial, con bacteriemia sostenida.
� http://www.cenetec.salud.gob.mx/descargas/gpc/
CatalogoMaestro/087_GPC_VaricesesofagicasNarices_
e) Disminución de la actividad bacteriana del líquido ascítico.
esofAgicasRR_CENETEC.pdf
63. ¿Cómo se clasifica la infección del líquido de as­
56. ¿Qué es la ascitis intratable? citis de acuerdo con su celularidad y cultivo?
Aquella que no puede tratarse con dosis adecuada de diuréti­ a) Peritonitis bacteriana espontánea: polimorfonucleares
cos por la presencia de complicaciones: (PMN) �250/mm3, un único microorganismo cultivado.
b) Peritonitis neutrocítica con cultivo negativo: PMN �250/
a) No hay pérdida de peso o es mínima y no hay natriuresis
mm 3, cultivo negativo.
adecuada (78 mmol).
e) Peritonitis bacteriana secundaria: PMN �250/mm 3, por lo
b) Desarrollo de complicaciones significativas: encefalopatía,
regular múltiples microorganismos.
creatinina sérica mayor de 2.0 mg/dL, sodio sérico menor
d) Bacteriascitis monomicrobiana: PMN <250/mm 3 , único
a 10 mmol/L, potasio sérico mayor a 6.0 mmol/L.
microorganismo.
57. ¿Cuáles son dos alternativas de terapia de la as­ e) Bacteriascitis polimicrobiana: PMN <250/mm 3, múltiples
citis refractaria? microorganismos.
Las alternativas son: el shunt o cortocircuito transyugular
64. ¿En qué situaciones es conveniente administrar
porto-cava y el shunt peritoneo-venoso.
albúmina en el tratamiento de la PBE?
58. ¿A que se le llama Shunt o cortocircuito transyu­ a) Creatinina sérica >1 mg/dL.
gular porto-cava? b) BUN >30 mg/dL.
e) Bilirrubinas totales >4 mg/dL.
En inglés su acrónimo es TIPS (Transjugu/ar Intrahepatic Porto­
systemic Shunt) y consiste en una comunicación intrahepática El beneficio es disminuir la mortalidad de 29 a 10%, apro­
entre la porta y la vena hepática por abordaje yugular. Logra ximadamente.
conectar los sistemas hepático y portal derivando la alta pre­
sión portal a un sistema de baja presión en vena hepática. AJ 65. ¿Cuáles son los factores que se asocian con ries­
disminuir la presión portal a menos de 12 mm Hg y el sistema go de padecer PBE?
renina-angiotensina-aldosterona (con mejoría en la excreción a) Proteína en el liquido de ascitis menor a 1.5 g/dL.
renal de sodio) se logra corregir la ascitis. La tasa de éxito de b) Función renal alterada (creatinina �1.2, BUN 2".:25 o Na
este procedimiento es alta 93 a 100% con un rango de morta­ ::;130 mmol/L).
lidad de l a 2% debido a hemoperitoneo, hemobilia, hemólisis e) Child-Pugh >9.
y sepsis. d) Bilirrubina >3 mg/dL.
e) Hemorragia digestiva.
59. ¿A que se le llama shunt o cortocircuito peritoneo­
venoso? 66. ¿Cuál es el antibiótico de elección en el tratamien­
El shunt peritoneovenoso consiste en realizar una comunica­ to de la infección activa del líquido de ascitis?
ción que drene la ascitis de la cavidad peritoneal hacia la vena Cefotaxima IV 2 gramos cada ocho horas o una cefalosporina
yugular interna. Puede indicarse en pacientes que no son can­ de tercera generación (ceftriaxona I gramo IV dos por día, por
didatos a realizar una comunicación porto-cava transyugular, cinco días).

764
SECCIÓN 11 GASTROENTEROLOGÍA @
SÍNDROME HEPATORRENAL 76. ¿Cómo se realiza el diagnóstico de síndrome he­
patopulmonar?
67. ¿Cuál es la fisiopatología del SHR? Saturación de oxigeno de la hemoglobina inferior a 92%, deter­
El origen es una disfunción circulatoria. Ocurre por disminu­ minada mediante pulsioximetría, o la PaO 2 inferior a 70 mm
ción del flujo sanguíneo renal y de la tasa de filtración glome­ Hg mediante gasometría. La ecocardiografía tiene alta sensibi­
rular secundaria a una dilatación arterial marcada en la circu­ lidad para detectar la dilatación vascular pulmonar.
lación esplácnica. Esto lleva a uná reducción del volumen
arterial efectivo y de la presión arterial con activación compen­ 77. ¿Cuál es la evolución clínica del SHP?
satoria de los sistemas vasoconstrictores.
Aunque no está bien definida, una vez que se demuestra la hi­
68. ¿Cuál es el factor precipitante más frecuente del poxemia grave hay un empeoramiento a lo largo de meses o
SHR? años con una mortalidad aproximada de 41 % a los 2.5 años
del diagnóstico.
Peritonitis bacteriana espontánea.

69. ¿Cuáles son los criterios mayores para el diag­ ENCEFALOPATÍA HEPÁTICA
nóstico de síndrome hepatorrenal?
a) Cirrosis con ascitis. 78. ¿Cuál es la definición de encefalopatía hepática?
b) Creatinina sérica mayor a 1.5 mg/dL. Es un síndrome neuropsiquiátrico complejo que ocurre en pa­
e) Sin mejoría del nivel de creatinina después de suspender los cientes con disfunción hepática significativa y que es poten­
diuréticos y administrar albúmina. cialmente reversible.
d) Sin tratamiento actual o reciente con nefrotóxicos.
e) Ausencia de enfermedad renal. 79. ¿Cómo se elabora el diagnóstico de la encefalo­
70. ¿Cómo se clasifica el síndrome hepatorrenal patía hepática?
(SHR)? Por exclusión de otros trastornos orgánicos cerebrales y con
base en dos tipos de síntomas: alteración del estado mental y
En tipo l (falla renal rápidamente progresiva) y tipo 2 (falla
alteración de la función motora (hiperreflexia, rigidez, mioclo­
renal no rápidamente progresiva).
no y asterixis).

1
71. ¿Cuáles son los criterios para el SHR 1?
Incremento de la creatinina sérica mayor o igual a dos veces 80. ¿Cómo se clasifica a la encefalopatía hepática se­
su valor >221 micromoles/L (2.5 mg/dL) o disminución de gún las causas?
50% de la depuración de la creatinina a <20 mL/minuto en • Tipo A: encefalopatía hepática asociada con falla hepática
menos de dos semanas. aguda.
• Tipo B: asociada con bypass portosistémicos, sin enferme­
7 2. ¿Cuáles son los criterios para el SHR 2? dad hepática intrinseca.
Creatinina sérica > 1.5 mg/dL o depuración de creatinina <40 • Tipo C: encefalopatía en pacientes con enfermedad hepáti­
mL/minuto. ca crónica/cirrosis.
73. ¿Cuál es el tratamiento para el SHR?
81. ¿Cuáles son los subtipos de la encefalopatía ti­
a) Hemodiálisis.
po C?
b) Vasoconstrictores (terlipresina).
e) Albúmina. a) Episódica.
b) Precipitada.
SÍNDROME HEPATOPULMONAR
e) Espontánea.
d) Recurrente.
74. ¿Qué tríada caracteriza al síndrome hepatopul­ e) Persistente.
monar? j) Leve.
g) Grave.
a) Dilatación vascular pulmonar.
h) Dependiente de tratamiento.
b) Hipoxemia: PaO2 <70 mm Hg.
i) Minima.
e) Enfermedad hepática avanzada.

75. ¿Existe relación entre el grado de hipoxemia y la 82. ¿Cuáles son los criterios de West-Haven para en­
gravedad de la hepatopatía? cefalopatía hepática?
No hay relación. Véase cuadro 11-17-2.

I 765
e MANUAL PARA EL EXAMEN NACIONAL DE RESIDENCIAS MÉDICAS

CUADRO 11-17-2. 85. ¿Cómo se detecta la encefalopatía hepática mí­


CRITERIOS DE WEST-HAVEN PARA LA VALORACIÓN DEL nima?
ESTADO MENTAL EN LA ENCEFALOPATÍA HEPÁTICA
También denominada subclinica. Las manifestaciones neuro­
Grado O No se detecta ninguna anomalía lógicas no son obvias; en la exploración se detectan mediante
Grado 1 Falta trivial de alerta, euforia, ansiedad
pruebas neuropsicológicas.
Acortamiento del espacio de atención
Deterioro del rendimiento en la suma o en la resta 86. ¿Cuáles son los factores precipitantes de la en­
Grado 2 Letargia, apatía, desorientación temporoespacial cefalopatía?
Alteración evidente de la personalidad Sepsis, hemorragia digestiva, estreñimiento, consumo elevado
Comportamiento inapropiado
de proteínas, diuréticos, hipopotasemia, uremia y deshidra­
Grado 3 Somnolencia o estupor, pero con respuesta a estímulos tación.
Confusión

Grado 4 Coma, estado mental no evaluable 87. Mencione los factores que favorecen la acción
de las toxinas.
a) Factores precipitantes (sepsis, hemorragia digestiva, etcé­
� http://www.cenetec.salud.gob.mx/descargas/gpc/ Catalogo­ tera).
Maestro/087_GPC_VaricesesofagicasNarices_ b) Derivaciones portosistémicas (congénitas, espontáneas en
esofAgicasRR_CENETEC.pdf
la cirrosis y terapéuticas).
e) Aumento de la susceptibilidad cerebral.

83. ¿Cuáles son las teorías de la patogénesis de la 88. ¿Cómo se realiza el diagnóstico?
encefalopatía hepática?
Es clínico: manifestaciones neurológicas, presencia de un fac­
El denominador común patogénico es el hecho de que la HE tor precipitante, paciente con insuficiencia hepática grave o
es causada por sustancias que en circunstancias normales son con derivación portosistémica.
neutralizadas de forma eficaz por el hígado, más que por la
producción insuficiente de sustratos que podrían ser esencia­ 89. ¿Cuál es el tratamiento para la encefalopatía he­
les para la función neurológica. Véase cuadro 11-17-3. pática?
a) Eliminación del factor precipitante.
84. ¿Cuál es la presentación característica de la en­ b) Nutrición.
cefalopatía hepática? e) Disacáridos no absorbibles: reducen el amonio intestinal,
Episodio agudo de encefalopatía caracterizado por el inicio sú­ actúan como catárticos y mediante acidificación del lumen
bito de un estado confusional que puede evolucionar hacia el intestinal que inhibe el crecimiento de bacterias producto­
coma. ras de amonio.

CUADRO 11-17-3.
PRINCIPALES MECANISMOS PATOLÓGICOS EN LA ENCEFALOPATÍA HEPÁTICA

SUSTANCIAS ORIGEN EFECTOS ARGUMENTO A FAVOR


Amoniaco A partir del metabolismo de aminoácidos Agonista sobre la transmisión del GABA, Altas concentraciones en la
y sustancias nitrogenadas por parte de altera la inhibición postsináptica, interacción circulación sistémica y el LCR
las bacterias del colon y desaminación con neurotransmisores relacionados con en pacientes con
de glutamina en intestino delgado serotonina, edema cerebral encefalopatía hepática
Benzodiacepinas Diversos tejidos humanos en condiciones Agonista sobre el GABA, unión a receptores, Mejoría en el estado mental tras
normales benzodiacepina de tipo periférico administrar flumazenilo
Manganeso Resultado de la derivación portosistémica Manifestaciones parkinsonianas Aumento en la señal en el
y de la disminución de excreción biliar núcleo pálido
GABA Alteración de la transmisión GABAérgica Aumento en las concentracio­
nes plasmáticas en pacientes
con encefalopatía
Mercaptanos Efectos sinérgicos con el amoniaco

Ácidos grasos de Síntesis de neurotransmisores falsos Aumento de las concentracio­


cadenas cortas (octopamina feniletanolamina) y de nes en pacientes con EH
serotonina

766
SECCIÓN 11 GASTROENTEROLOGÍA @
d) Neomicina: aminoglucósido que reduce la flora intestinal lopathy and Nitrogen Metabolism consensus. Hepatology. 2013 Ju];
bacteriana que produce amonio y toxinas. 58:325.
Bari K, Garcia-Tsao G. Treatment of portal hypertension. World J Gastroen­
e) Rifaximína: antimicrobiano y bactericida con amplio espec­
terol. 2012;18(11):1166-75.
tro contra grampositivos y gramnegativos aerobios y anae­ Biecker E. Portal hypertension and gastrointestinal bleeding. World J Gas­
robios. troenterol. 2013; 19(31 ):5035-50.
f) Flumazenilo: antagonista de la benzodiacepina; ha mostra­ Blei AT, Córdoba J. Hepatic Encephalopathy. Am J Gastroenterol. 2001;
do beneficio en pacientes seleccionados con coma hepáti­ 96:1968-76.
Centro Nacional de Excelencia Tecnológica en Salud. Diagnóstico y trata­
co grave.
miento de la insuficencia hepática crónica. Consejo de Salubridad Gene­
g) Otras medidas: ral. Guia de referencia rápida. [Monografía en internet.] México; 2011.
pp. 1-11. Disponible en: http://www.cenetec.salud.gob.mx/descargas/gpc/
• Recomendaciones con evidencia fuerte:
CatalogoMaestro/087 _G PC_Varicesesofagicas/Varices_esofAgicasRR_
- Evaluar el estado nutricional de todos los pacientes. CENETEC.pdf
- Ingesta de 35 a 40 kcal/kg de peso ideal. Centro Nacional de Excelencia Tecnológica en Salud. Diagnóstico y tratamien­
- 1.2 a 1.5 gramos de proteínas. to de várices esofágicas. Consejo de Salubridad General. Guía de referen­
- Comidas pequeñas, fraccionadas a lo largo del día. cia rápida. [Monografía en internet.] México; 2011. pp. 1-11. Disponible
en: http://www.cenetec.saJud.gob.mx/descargas/gpc/CatalogoMaestro/087
- Cena de carbohidratos complejos.
_GPC_Varicesesofagicas/Varices_esofAgicasRR_CENETEC.pdf
- Corregir la hiponatremia de manera lenta. García-Tsao G, Bosch J. Management of Varices and Variceal Hemorrhage in
• Recomendaciones con evidencia moderada: Cirrhosis. N Engl J Med. 2010;362:823-32.
- Dieta rica en vegetales y proteínas. Garcia-Tsao G, Sanyal AJ, Grace ND, Carey W. Practice Guidelines Commi­
- Suplementos de aminoácidos de cadena ramificada ttee of the American Association for the Study of Liver Diseases; Practice
Parameters Committee of the American College of Gastroenterology. Pre­
para mantener la ingesta de nitrógeno recomendada vention and management of gastroesophageal varices and variceal hemo­
en pacientes con intolerancia a las proteínas. rrhage in cirrhosis. Hepatology. 2007 Sep;46(3):922-38.
- Tomar un multivitamínico durante dos semanas en Poh Z, Chang PEJ. A Current Review of the Diagnostic and Treatment Stra­
pacientes con cirrosis descompensada. tegies of Hepatic Encephalopathy. International Journal of Hepatology.
- Dieta diaria con 25 a 45 gramos de fibra. Evitar fór­ 2012:2012:480309.
Runyon BA; AASLD Practice Guidelines Committee. Management of adult
mulas nutricionales que contengan manganeso. patients with ascites due to cirrhosis: an update. Hepatology. 2009 Jun;
49(6):2087-107.
Schiff ER, Maddrey WC, Sorrel MF. Schiff's Diseases of the Liver. 11 th ed.
BIBLIOGRAFÍA Philadelphia: Wiley-Blackwell; 2012.

1
Amodio P et al. The nutritional management of hepatic encephalopathy Sola E, Gines P. Renal and circulatory dysfunction in cirrhosis: curren! mana­
in patients with cirrhosis: International Society for Hepatic Encepha- gement and future perspectives. J Hepatol. 2010;53: 1135-45.

CAPÍTULO 11.18
COLESTASIS CRÓNICAS
Claudia Isabel Martínez Camacho

CIRROSIS BILIAR PRIMARIA En fase avanzada de daño hepático hay elevación de ALT,
AST, bilirrubínas totales, disminución de albúmina y prolon­
1. ¿Qué es la cirrosis biliar primaria (CBP)? gación del tiempo de protrombina. Entre 90 y 95% de los pa­
Es una enfermedad autoinmunitaria, crónica y progresiva que cientes tiene anticuerpos antimitocondriales (AMA) positi­
afecta sobre todo a mujeres (90%); la edad promedio al diag­ vos > 1 :40, anticuerpos antinucleares (ANA) positivos; 85%
nóstico es a los 50 años, pero puede presentarse de los 30 a 60 de los pacientes tiene AMA negativos. Gabinete: el ultraso­
años de edad, en todo el mundo y sin predominio de raza o de nido abdominal excluye obstrucción biliar extrahepática. La
grupo étnico. Se caracteriza por la destrucción de los conduc­ tomografía computarizada (TC) está indicada para aquellos
tos biliares íntrahepáticos que conduce a colestasis crónic�. pacientes en quienes el ultrasonido no fue valorable. La colan­
fibrosis portal y cirrosis. giorresonancia está indicada para pacientes con AMA negati­
vos, para establecer diagnóstico diferencial con colangitis es­
2. ¿Qué exámenes de laboratorio y gabinete se re­ clerosante.
quieren para el diagnóstico de la CBP?
Los niveles séricos de fosfatasa alcalina (FA) son elevados, en 3. ¿Qué presentaciones clínicas tiene la CBP?
un inicio no hay alteraciones en el resto del perfil hepático, Tiene dos presentaciones: asintomática y sintomática. La asin­
hay elevación de lgM y grados variables de hiperlipidemia. tomática dura al menos nueve a diez años, hay incremento de

767
e MANUAL PARA EL EXAMEN NACIONAL DE RESIDENCIAS MÉDICAS

la FA, bilirrubinas normales, AMA positivos y ausencia de CUADRO 11-18-1.


signos físicos de alteración hepática. El diagnóstico se hace COMPLICACIONES DE LA CBP Y SU TRATAMIENTO
al investigar otra enfermedad autoinmunitaria. Sólo 5% de los­ ESPECÍFICO
pacientes permanece asintomático a 20 años. Ochenta por
Prurito Colestiramina, dosis inicial 4 mg/día
ciento se vuelve sintomático a los diez años de evolución de la (hasta 16 mg/día)
enfermedad. La supervivencia de una mujer de mediana edad Rifampicina 100 mg cada 24 horas
con prurito, fatiga, ictericia lentamente progresiva y elevación Antagonistas opioides. Naloxona, naltrexona o
discreta de fosfatasa alcalina, bilirrubinas y transaminasas es sertralina
El prurito refractario puede manejarse con MARS
de alrededor de siete a diez años. (sistema de recirculación molecular absorbente)
o trasplante hepático
4. ¿La CBP se asocia con otras enfermedades auto­ Osteoporosis Suplementar calcio 1 000 a 1 500 mg/día y
inmunitarias? vitamina D 800 a 1 000 U/día
La CBP puede asociarse con síndrome de Sjogren, tiroiditis, Bifosfonatos: el alendronato puede mejorar la
densidad ósea
esclerodermia, síndrome de CREST, artritis reumatoide, en­
fermedad celíaca y otras menos frecuentes, como enfermedad Hiperlipidemia Estatinas: fezafibrato, fenofibrato

inflamatoria intestinal. Malabsorción Reponer vitaminas A, D, E y K


de vitaminas
liposolubles
5. ¿Qué enfermedades deben considerarse para rea­
lizar diagnóstico diferencial de la CBP? Complicaciones La ascitis, el sangrado variceal, la peritonitis
de la bacteriana espontánea y la encefalopatía
Obstrucción de la vía biliar extrahepática (coledocolitiasis, es­ insuficiencia hepática se manejan según lo indicado en
tenosis inflamatorias o neoplasias), colangitis esclerosante pri­ hepática la hepatopatía crónica por cualquier causa
maria, colestasis inducida por fármacos (estrógenos, fenotia­
zidas), hepatitis granulomatosa, hepatitis autoinmunitaria,
hepatitis C crónica, hepatitis alcohólica, situaciones especia­ progresa a cirrosis biliar y sus complicaciones. Es probable que
les con lesión de la vía biliar (la enfermedad injerto contra sea de origen inmunitario; en 70% de los casos se presenta en
huésped, rechazo de aloinjertos de hígado, colangitis isquémi­ hombres de alrededor de 40 años de edad. Puede tener tres
ca, colangitis infecciosa en pacientes inmunocomprometidos) variantes: colangitis esclerosante de pequeños conductos, CEP
y sarcoidosis. con hepatitis autoinmunitaria (un síndrome de overlap) y co­
langitis esclerosante IgG4 positiva.
6. ¿Cuál es el tratamiento de elección para la CBP?
9. ¿Cuáles son los criterios diagnósticos para la CEP?
El ácido ursodesoxicólico (UDCA) es la primera línea de tra­
tamiento para la CBP, ya que retrasa la progresión de la CBP, Véase cuadro 11-18-2.
aunque no tiene un beneficio significativo en estadios avanza­
10. ¿Cuáles son las características colangiográficas
dos. La dosis es de 13 a 15 mg/kg/día, hay mejoría de los pa­
de la CEP?
rámetros de laboratorio e histológica de la colangitis, inflama­
ción y fibrosis hepática. Una gran variedad de fármacos se han Estenosis cortas, multifocales y anulares que alternan con seg­
probado sin éxito en los pacientes con CBP, incluso los inmu­ mentos normales o ligeramente dilatados, lo que produce una
nosupresores (prednisona, budesonida, azatioprina, ciclospo­ imagen "en cuentas" (arrosariado ).
rina, metotrexato y micofelonato) y los antifibróticos (colchi­
cina y D-penicilamina) con un efecto marginal e importantes CUADRO 11-18-2.
efectos secundarios que no justifican su uso clínico. CRITERIOS DIAGNÓSTICOS DE CEP
Anormalidades típicas que Colangiografía con estrecheces y
7. ¿Cuáles son sus principales complicaciones y cuál
involucren la vía biliar dilataciones
es su tratamiento?
Cuadro clínico y exámenes Síntomas de colestasis, historia de
Véase cuadro 11-18-1. de laboratorio enfermedad inflamatoria intestinal,
aumento de fosfatasa alcalina por
más de seis meses
COLANGITIS ESCLEROSANTE PRIMARIA
Excluir causas identificables Colangiopatía por sida,
8. ¿Qué es la colangitis esclerosante primaria (CEP)? de colangitis esclerosante colangiocarcinoma, cirugía o
secundaria trauma de conductos biliares,
Es una enfermedad hepática crónica, caracterizada por coles­ coledocolitiasis, estenosis
tasis que condiciona inflamación y fibrosis de los conductos isquémica de conductos biliares,
biliares intrahepáticos y extrahepáticos. Esto conduce a la for­ toxicidad intraarterial, enfermedad
injerto contra huésped
mación de estenosis biliares multifocales de los conductos y

768
SECCIÓN 11 GASTROENTEROLOGÍA @
11. ¿Se requiere realizar biopsias hepáticas para con­ langitis bacteriana recurrente se recomienda el uso a largo
firmar la CEP? plazo de antibióticos. Se recomienda evaluar a los pacientes
No, ya que los hallazgos histológicos de fibrosis periductal con colangitis bacteriana refractaria para trasplante hepático.
concéntrica en "piel de cebolla" pueden ser compatibles con
CEP pero en general no son específicos. Sólo está indicada 17. ¿Cuál es el riesgo de malignidad asociada con la
ante la sospecha de CEP de pequeños conductos o para eva­ CEP?
luar un posible síndrome de overlap. Hay alto riesgo de malignidad asociada con pólipos vesicula­
res, por lo que debe hacerse vigilancia anual y hasta colecistec­
12. ¿Cuáles son las principales enfermedades con tomia, inclusive si el pólipo vesicular es < 1 cm. Hay una ma­
las que se asocia la CEP? yor susceptibilidad para desarrollar cáncer colorrectal en los
La CEP se asocia con enfermedad inflamatoria intestinal, sín­ pacientes con colitis ulcerosa asociada. El riesgo de colangio­
drome de overlap: hepatitis autoinmunitaria/CEP, pancreatitis carcinoma es de 7 a 15%.
autoinmunitaria (lgG4 asociada con colangitis).
18. ¿Cuál es el escrutinio propuesto para el colangio­
carcinoma?
13. ¿Cómo se definen las estenosis dominantes en
Se ha propuesto como escrutinio una prueba de imagen (co­
la CEP?
langiorresonancia o tomografía por emisión de positrones) y
Son estenosis con un diámetro ::51.5 mm en el conducto biliar una determinación de CA 19-9 de forma anual (CA 19-9 > 130
común o ::51 mm en el conducto hepático. La presentan entre U/mL, niveles asociados con malignidad).
45 y 85% de los pacientes durante su evolución. Suelen ser
benignas, sólo entre 10 y 15% son malignas por un colangio­ 19. ¿Cuál es el tratamiento específico para la CEP?
carcinoma. El uso de ácido ursodesoxicólico a dosis de 13 a 15 mg/kg/día
para retrasar la progresión de la enfermedad hepática aún no
14. ¿Cuáles son las indicaciones para tratamiento en­ está claro, pues las dosis altas pueden ser perjudiciales. No
doscópico, percutáneo o quirúrgico de la CEP? existen datos consistentes sobre la utilidad de distintos fár­
Las estenosis dominantes causan colangitis, ictericia, prurito, macos inmunomoduladores o agentes antifibrosantes. El tra­
dolor en cuadrante superior derecho y empeoramiento de los tamiento sintomático de prurito, osteopenia y malabsorción
índices bioquímicos. El tratamiento endoscópico (CPRE) es intestinal es equivalente al de las demás enfermedades colestá­

1
de primera elección. El percutáneo se asocia con mayores com­ sicas.
plicaciones, pero su eficacia es similar y se reserva para pa­
cientes con estenosis dominantes proximales en quienes falló BIBLIOGRAFÍA
el endoscópico. Pueden realizarse esfinterotomía, dilatación Boonstra K, Beuers U, Ponsioen CY. Epidemiology of primary sclerosing cho­
con catéter o balón, con o sin colocación de prótesis biliar. El langitis and primary biliary cirrhosis: a systematic review. Journal of hepa­
tratamiento quirúrgico sólo se recomienda para las estenosis tology. 2012;56(5):1181-8.
Bowlus CL, Gershwin ME. The diagnosis of primary biliary cirrhosis. Au­
dominantes refractarias al tratamiento endoscópico o percutá­ toimmunity reviews. 2014;13(4):441-4.
neo en pacientes sin cirrosis. Duarte-Rey C, Bogdanos D, Yang CY, Roberts K, Leung PS, Anaya JM et al.
Primary biliary cirrhosis and the nuclear pore complex. Autoimmunity
15. ¿Qué consideraciones deben tenerse en cuenta reviews. 2012; 11 (12):898-902.
Karlsen TH, Vesterhus M, Boberg KM. Review article: controversies in the
para el manejo del prurito en la CEP? management of primary biliary cirrhosis and primary sclerosing cholangi­
En un inicio se debe considerar la presencia de una estenosis tis. Alimentary pharmacology & therapeutics. 2014;39(3):282-301.
dominante, ya que en este caso el tratamiento sería invasivo Othman MO, Dunkelberg J, Roy PK. Ursodeoxycholic acid in primary sclero­
(endoscópico o percutáneo). En ausencia de estenosis domi­ sing cholangitis: a meta-analysis and systematic review. Arab Journal of
Gastroenterology. 2012; 13(3): 103-10.
nante, el control del prurito es similar al de la cirrosis biliar Rudic JS, Poropat G, Krstic MN, Bjelakovic G, Gluud C. Ursodeoxycholic
primaria con fármacos. acid for prirnary biliary cirrhosis. Cochrane Database Syst Rev. 2012 Dec
12;12:CD00055I.
16. ¿Cuáles son las recomendaciones para el mane­ Selmi C, Bowlus CL, Gershwin ME, Coppel RL. Primary biliary cirrhosis.
The Lancet. 2011;377(9777):1600-9.
jo de la colangitis bacteriana en CEP? Tabibian J H, Lindor KD. Primary sclerosing cholangitis: a review and update
El uso de antibióticos asociados con la corrección de la obs­ on therapeutic developments. Expert Rev Gastroenterol Hepatol. 2013
trucción de la estenosis dominante. En los pacientes con co- Feb;7(2): 103-14.

769
e MANUAL PARA EL EXAMEN NACIONAL DE RESIDENCIAS MÉDICAS

CAPÍTULO 11.19
ENFERMEDADES HEPÁTICAS DE CAUSAS METABÓLICA Y CARDIACA
Miriam Gabriela Reyes Zermeño

HEMOCROMATOSIS PRIMARIA la segunda y tercera articulaciones metacarpofalángicas y cam­


bios físicos en relación con el daño hepático crónico.
1. ¿Qué es la hemocromatosis hereditaria?
Es un trastorno autosómico recesivo; sólo en individuos que 6. ¿Cómo se elabora el diagnóstico de hemocroma­
portan la mutación de los dos alelos del gen HFE permite su tosis?
expresión. Se caracteriza por depósitos de hierro secundarios Se debe estudiar a los pacientes con análisis de hierro anorma­
a un incremento en la absorción intestinal del mismo y su de­ les y síntomas o hallazgos compatibles en la exploración físi­
pósito en las células parenquimatosas. ca y a las personas con antecedentes familiares positivos de
hemocromatosis. En los exámenes bioquímicos se encuentra:
2. ¿Cuál es la mutación más frecuente en la hemo­ elevación en la saturación de transferrina >45 a 55%; elevación
cromatosis hereditaria? de ferritina > 1 000 ng/mL o >200 ng/mL en mujeres preme­
La mutación del gen HFE causa la hemocromatosis, es un gen nopáusicas. La saturación de transferrina es más sensible y
similar al complejo mayor de histocompatibilidad, clase I, lo­ específica que la ferritina (es un reactante de fase aguda). Si
calizado en el brazo corto del cromosoma 6. Esta mutación los estudios de hierro son anormales se debe buscar la muta­
produce la sustitución de una cisteína por tirosina (C282Y). ción del gen HFE. Si el paciente es homocigoto para la mu­
Esta mutación homocigota del C282Y está en 90 a 95% de los tación C282Y o heterocigoto compuesto (C282/H63D) y tie­
casos de HH. Una segunda mutación (H63D) puede presen­ ne menos de 40 años de edad, con enzimas hepáticas normales
tarse cuando el paciente es heterocigoto compuesto (C282Y/ (alanino aminotransferasa y aspartato aminotransferasa), no
H63D) y representa entre 5 y 7% de los casos de HH. Como es necesaria otra valoración.
resultado, HFE no puede interactuar con la microglobulina
beta2 (Beta2M); esta última suele unirse al receptor de trans­ 7. ¿Cuándo está indicada la biopsia hepática en pa­
ferrina, que es necesario para el ingreso de hierro por el recep­ cientes con hemocromatosis?
tor de transferrina dentro de las células. En pacientes mayores de 40 años de edad, con enzimas hepá­
ticas anormales o niveles de ferritina muy altos (> 1 000 ng/
3. ¿Cuáles son las manifestaciones clínicas en pa­ mL) se debe realizar biopsia hepática percutánea para obtener
cientes con hemocromatosis hereditaria? tejido para estudio histológico, realizar tinción con azul de
Es la presentación clásica de diabetes mellitus bronceada; se Prusia de Perls para los depósitos de hierro y determinación
diagnosticaba antes de 1960 y se asociaba con artritis, enfer­ de la concentración hepática de hierro (CHH). Este último es
medad hepática y falla cardiaca. Los síntomas más comunes necesario para calcular el índice de hierro hepático (concen­
son debilidad, fatiga, artralgias o artritis, dolor en hipocon­ tración de hierro hepático en micromol/gramo de peso seco
drio derecho no especifico, hepatoesplenomegalia, cardiomio­ dividido entre la edad en años); un valor superior a 1.9 es com­
patía dilatada, falla cardiaca congestiva, hiperpigmentación patible con hemocromatosis hereditaria homocigótica.
cutánea, atrofia testicular, hipogonadismo, pérdida de la libi­
do o impotencia. La cirrosis se presenta en 40% de los pacien­ 8. ¿Qué enfermedades pueden condicionar sobre­
tes no tratados. carga de hierro?
En eritropoyesis inefectiva, transfusiones repetidas, talasemia,
4. ¿Qué tipo de infecciones tienen mayor riesgo de anemia sideroblástica, anemias hemolíticas crónicas, enferme­
presentar los pacientes con hemocromatosis? dad hepática alcohólica, hepatitis crónica por virus B y C e
Los pacientes tienen mayor riesgo de padecer infecciones bac­ hígado graso no alcohólico. La biopsia hepática en sobrecarga
terianas, virales y micóticas. Infecciones bacterianas poco co­ de hierro secundaria muestra sobrecarga de hierro en las célu­
munes: Vibrio vulnificus, Yersinia enterocolitica, Yersinia pseu­ las de Kupffer y en las células reticuloendoteliales.
dotuberculosis, Listeria monocytogenes.
9. ¿Cuál es el tratamiento de elección para la hemo­
5. ¿Cuáles son los hallazgos en la exploración física cromatosis hereditaria?
de los pacientes con hemocromatosis? La flebotomía es la piedra angular, pues remueve 500 mL de
La enfermedad avanzada puede presentar pigmentación cutá­ sangre (alrededor de 250 mg de hierro) semanal o cada dos
nea grisácea o "bronceada" en zonas expuestas al sol, hepato­ semanas hasta alcanzar valores de ferritina sérica de 50 a 100
megalia, artropatía con edema y sensibilidad a la palpación de microgramos/L. Se debe revisar el hematocrito antes de cada

770
SECCIÓN 11 GASTROENTEROLOGÍA @
flebotomía y la ferritina sérica debe valorarse cada tres meses 14. ¿Cuál es la prevalencia de la enfermedad de Wil­
(después cada diez a doce flebotomías). Si el hematocrito es son?
menor de 32% la flebotomía debe ser pospuesta y se realizará Se estima una prevalencia de l en 30 000 personas.
cada dos semanas. Cuando se alcancen niveles de ferritina de
50 microgramos/L se debe revisar cada tres a cuatro meses y 15. ¿Cuál es la alteración genética de la enfermedad
se realizarán nuevas flebotomías para mantener la ferritina sé­ de Wilson?
rica en 50 a 100 microgramos/L. Los quelantes de hierro, co­
Es un trastorno autosómico recesivo, con una frecuencia de
mo la deferoxamina, no están indicados porque son menos
0.3 a 0.7%. Se localiza en el cromosoma 13. El gen se ha deno­
efectivos y tienen efectos adversos; sólo se contemplarán en
minado ATP7B y codifica una trifosfatasa de adenosina tipo
pacientes que no toleren la flebotomía o presenten anemia.
P, que es una proteína transmembrana del cobre. Se han iden­
tificado más de 269 mutaciones, por lo que las pruebas gené­
10. ¿Cuál es el pronóstico para los pacientes con he­
ticas son de poca utilidad.
mocromatosis hereditaria?
Los pacientes diagnosticados y tratados antes de desarrollar 16. ¿Cuál es el mecanismo fisiopatológico en la en­
cirrosis presentan una sobrevida normal. La mortalidad rela­ fermedad de Wilson?
cionada es de hasta 60% de los pacientes que desarrollan falla
La toxicidad del cobre desempeña una función fundamental.
hepática o hepatocarcinoma. El paciente que desarrolla cirro­
Existe un desbalance entre la absorción gastrointestinal y la
sis tiene un riesgo incrementado de 20 a 200 veces de desarro­
excreción biliar del cobre. La absorción es normal, pero la ex­
llar hepatocarcinoma.
creción biliar está reducida. El exceso de cobre es tóxico por
la generación de radicales libres, con peroxidación lipídica,
ENFERMEDAD DE WILSON depleción de antioxidantes y polimerización de tionina-Cu que
generan necrosis y apoptosis. Los depósitos de cobre en los
11. ¿Qué es la enfermedad de Wilson? ganglios basales del cerebro, el núcleo caudado y el putamen
La enfermedad de Wilson es un trastorno autosómico recesivo son el origen de los síntomas neurológicos y psiquiátricos. El
del almacenamiento del cobre que se caracteriza por un bajo depósito en la membrana de Descemet en la córnea produce
nivel sérico de cobre debido a la incapacidad de la ceruloplas­ los anillos de Kayser-Fleischer.
mina para combinarse con el cobre del plasma. Aparecen gran­
des cantidades de cobre en el hígado, el bazo, el cerebro, la 17. ¿Qué grupo de pacientes está principalmente

1
córnea y el riñón. Puede tener manifestaciones asociadas con afectado?
la falla de los órganos donde se deposita: neurológicas(ataxia, Aparece en la adolescencia o en adultos jóvenes. A la edad de
temblor, neurosis, deterioro del lenguaje), hepáticas (insufi­ 15 años 50% de los pacientes presentará síntomas clínicos.
ciencia hepática, cirrosis), oftalmológicas (catarata, anillo de
Kayser-Fleischer).
18. ¿Cómo se manifiesta la enfermedad de Wilson?
El espectro de daño hepático puede ser hepatitis crónica o ci­
12. ¿Cómo se elabora el diagnóstico de enfermedad
rrosis. La presencia de falla hepática fulminante es poco fre­
de Wilson?
cuente; en sus formas tempranas se manifiesta como hígado
Los niveles bajos de ceruloplasmina sérica y los anillos de graso. Menos de 50% de los pacientes no presenta los anillos
Kayser-Fleischer son suficientes para diagnosticar enfermedad de Kayser-Fleischer y no tiene síntomas neurológicos. Den­
de Wilson. La ceruloplasmina sérica es< 10 mg/dL en más de tro del espectro de falla hepática fulmínante por lo general se
95% de los casos. Si esta prueba es positiva debe solicitarse co­ incluyen pacientes jóvenes que presentan hemólisis intravascu­
bre en orina de 24 horas(> 80 mg/24 horas). Los pacientes con lar, esplenomegalia y anillos de Kayser-Fleischer. Estos pa­
falla hepática fulminante pueden presentar cifras mayores de cientes no sobreviven semanas, a menos que sean trasplan­
1 000 microgramos/24 horas. Con ambas pruebas está indica­ tados. Los pacientes con falla hepática fulminante presentan
do realizar biopsia hepática para determinación cuantitativa elevación de aminotransferasas leve a moderada, incremento
de cobre en el tejido hepático. La determinación cuantitati­ marcado de bilirrubina, fosfatasa alcalina baja, Coombs nega­
va de cobre en biopsia hepática es > 50 microgramos/gramo de tivo, la ceruloplasmina sérica puede ser normal y el cobre uri­
peso seco; los niveles van de 250 a 3 000 mícrogramos/gramo. nario de 24 horas está elevado. Las manifestaciones neuroló­
gicas son: rigidez, disartria, temblor, salivación y cambios en
13. ¿Cuáles son los hallazgos en la biopsia hepática el comportamiento. Los anillos de Kayser-Fleischer son una
en la enfermedad de Wilson? decoloración dorada-verdosa en área límbica y polos cornea­
Se han encontrado en la biopsia hepática: esteatosis hepática, les de Descemet. Renal: acidosis tubular renal proximal, ane­
hepatitis crónica o cirrosis. La tinción del tejido con rodamina mia de Fanconi, hematuria, proteinuria y nefrolitiasis. A nivel
no es sensible para el diagnóstico. musculoesquelético: artropatía, osteomalacia y osteoporosis.

771
e MANUAL PARA EL EXAMEN NACIONAL DE RESIDENCIAS MÉDICAS

19. ¿Cuál es el tratamiento para la enfermedad de una relación dosis-dependiente. La resistencia a la insulina con­
Wilson? diciona lipólisis y liberación de ácidos grasos a la circulación.
En el tratamiento de la enfermedad de Wilson se usa un agente La lipogénesis de novo, a partir de un exceso de carbohidratos
quelante del cobre, la D-penicilamina, con una dosis inicial de en la dieta, produce ácidos grasos libres (AGL). Los AGL libe­
1 a 2 gramos diarios y la dosis de mantenimiento de 1 gramo ran proteasas lisosómicas, como la catepsina B. La beta oxida­
diario. Se debe agregar piridoxina 25 mg/día. El efecto secun­ ción mitocondrial de los ácidos grasos está aumentada y gene­
dario más común es reacción de hipersensiblidad, con presen­ ra especies reactivas de oxigeno (ROS) que condicionan daño
cia de fiebre, mal estado general, exantema y linfadenopatía oxidativo al ADN mitocondrial, lo que aumenta el factor de
ocasional. La trientina es igual de eficaz con menores efectos necrosis tumoral alfa y la lipoperoxidación de las membranas
colaterales. Los pacientes que presentan insuficiencia hepáti­ mitocondriales y pérdida del citocromo C de las mitocondrias.
ca fulminante deben ser valorados para trasplante hepático. Esto inicia la apoptosis y la muerte celular. Otra vía alterada
es la activación del sistema citocromo P450.
20. ¿Está indicada la detección familiar?
24. ¿Cuál es el hallazgo histológico en EHGNA?
La enfermedad de Wilson es un trastorno autosómico recesi­
La esteatosis hepática es una acumulación de triglicéridos en
vo y todos los familiares en primer grado del caso índice de­
macro y microvesículas en >5% de los hepatocitos, en la región
ben someterse a detección.
perivenular. Se encuentra degeneración balonoide (los cuer­
pos hialinos de Mallory son similares a los de estatohepatitis
ENFERMEDAD HEPÁTICA GRASA
alcohólica), centrolobulillar (zona 3), infiltración de linfoci­
NO ALCOHÓLICA
tos, polimorfonucleares intraacínares, inflamación portal y ne­
crosis de los hepatocitos. Los pacientes con esteatohepatitis
21. ¿Cómo se define enfermedad hepática grasa no
pueden presentar diferentes grados de fibrosis perívenular, pe­
alcohólica, hígado graso no alcohólico y esteatohe­
risinusoidal y periportal con puentes de fibrosis focales o ex­
patitis no alcohólica?
tensos, en diferentes grados, y su expresión más grave presen­
La enfermedad hepática grasa no alcohólica (EHGNA) se ca­ ta datos compatibles con cirrosis.
racteriza por la presencia de esteatosis, demostrada por estu­
dios de imagen o evidencia histológica en pacientes con un 25. ¿Qué tipo de pacientes deben someterse a estu­
consumo <20 gramos de alcohol/día, en quienes se han des­ dios para descartar EHGNA?
cartado causas secundarias: consumo del alcohol, medicamen­
La mayoría de las veces no existen síntomas específicos; puede
tos esteatogénicos o enfermedades hereditarias. Se asocia con
presentarse fatiga, malestar y molestia abdominal. La presencia
factores metabólicos como son: resistencia a la insulina, obe­
de los siguientes datos justifica el estudio para descartar HGNA
sidad y síndrome metabólico, hipertrigliceridemia e hiperco­
o EHNA: obesidad, en especial con índice de masa corporal
lesterolemia (20 a 81 %). La EHGNA se divide en: hígado gra­
>35 kg/m2 con distribución de grasa central; presencia de dia­
so no alcohólico (HGNA), que es una acumulación excesiva
betes mellitus tipo dos, síndrome metabólico, apnea obstructi­
de grasa en forma de triglicéridos (esteatosis), en el hígado
va del sueño, resistencia a la insulina, elevación ALT�30 Ul/L
>5% de los hepatocitos, sin daño hepatocelular; y esteatohe­
en hombres y� 19 Ul/L en mujeres de forma persistente duran­
patitis no alcohólica (EHNA), que es la presencia de esteato­
te más de seis meses; mujeres con ovarios poliquísticos, candi­
sis hepática e inflamación con daño balonizante de los hepa­
datos a cirugía bariátrica o en los ya sometidos a cirugía bariá­
tocitos, con o sín fibrosis.
trica que condicionó malabsorción elevada; un consumo menor
a 20 g/día de alcohol en mujeres y <30 g/día en hombres.
22. ¿Cuál es la prevalencia del hígado graso no al­
cohólico y de la esteatohepatitis? 26. ¿Qué estudios de laboratorio son útiles para el
La prevalencia mundial de HGNA en promedio se estima en estudio y el diagnóstico diferencial en EHGNA?
20% y de EHNA entre 3 y 5%. En México se ha estimado una Perfil hepático completo, biometría hemática, tiempo de pro­
prevalencia de 17.05% en población asintomática. Entre 70 y trombína, anticuerpos contra el virus de la hepatitis C, antígeno
90% de los pacientes tiene esteatosis simple. Entre 10 y 30% de superficie de hepatitis B, pruebas de hierro, ceruloplasmína
de los sujetos con hígado graso no alcohólico tiene esteatohe­ en pacientes jóvenes, anticuerpos antínucleares, alfa-1-anti­
patitis. tripsína y anticuerpos antimitocondriales.

23. ¿Cuáles son los puntos importantes en la fisio­ 27. ¿Qué resultados reportan los exámenes de labo­
patología de la EHGNA? ratorio en pacientes con EHGNA?
La esteatosis hepática se genera por disfunción de diversas De los pacientes con EHGNA, 10% presenta ALT y AST nor­
vías metabólicas. Existe un incremento de la concentración de males, sobre todo en esteatosis simple. Niveles de ferritína
ácidos grasos libres (AGL) y resistencia a la insulina; esta es anormales en presencia de saturación normal de la transferri-

772
SECCIÓN 11 GASTROENTEROLOGÍA @
na deben descartar EHGNA. Una razón AST/ALT < l (en rrolla hepatocarcinoma. Existe la teoría de que el estrés oxida­
origen etílico es >2). tivo desempeña una función predominante en la progresión a
formas más graves de la enfermedad. Son predictores indepen­
28. ¿Cuál es la función de los estudios de imagen en dientes para la progresión de la fibrosis: edad >45 a 50 años,
el diagnóstico de EHGNA? IMC >28 a 30 kg/m2, grado de resistencia a la insulina y pre­
El ultrasonido hepático identifica esteatosis traducida como sencia de diabetes e hipertensión.
incremento de la ecogenicidad o brillantez del parénquima, bo­
rramiento de la pared de los vasos hepáticos y mayor atenua­ 32. ¿Cuál es el tratamiento para la EHGNA?
ción acústica. El ultrasonido tiene una sensibilidad de 64% y El objetivo del tratamiento es mejorar los cambios histológi­
especificidad de 97% y es operador dependiente; en pacientes cos, la resistencia a la insulina y los niveles de enzimas hepá­
con obesidad mórbida disminuye su utilidad. La tomografía ticas. El tratamiento benéfico y seguro actual es la pérdida de
computarizada identifica la esteatosis con una sensibilidad de peso, pues reduce la esteatosis hepática, basada en una dieta
43 a 95% y especificidad de 90%; se encuentra densidad del hipocalórica y ejercicio. Una pérdida de peso de 3 a 5% mejo­
parénquima hepático <40 UH o densidad del hígado 10 UH ra la esteatosis; las pérdidas mayores a 10% mejoran la activi­
menor que la del bazo. La resonancia magnética cuantifica la dad necroinflamatoria. En cuanto a los medicamentos, la met­
esteatosis por diferencias de señal entre la grasa y el agua. Es formina no tiene un efecto significativo en la modificación
una cuantificación dinámica de la grasa hepática, pero es cara histológica, por lo que no está recomendada en adultos con
y no está disponible en la mayoría de los centros. EHGNA. Se ha utilizado la pioglitazona en pacientes con evi­
dencia histológica de esteatohepatitis; sin embargo, no se ha
29. ¿Qué pacientes pueden verse beneficiados por establecido su seguridad y eficacia en pacientes con EHNA.
una biopsia hepática en el estudio de EHGNA? Estará indicada en pacientes que sean candidatos para dicho
La biopsia hepática es el estándar de oro para caracterizar his­ tratamiento por cursar con diabetes mellitus tipo 2. La vitami­
tológicamente la enfermedad, pues establece el diagnóstico y na E (alfa tocoferol), el ácido ursodesoxicólico y el omega 3 se
estadifica la EHGNA. Es un método invasivo, de alto costo, han utilizado en protocolos de investigación, pero aún no exis­
con riesgos potenciales (morbilidad, mortalidad), variaciones te evidencia suficiente para generalizar su uso.
en el muestreo y variaciones interobservador. Los pacientes que
pueden verse beneficiados son aquellos con riesgo elevado de 33. ¿Cuál es el pronóstico en la EHGNA?
padecer esteatohepatitis y fibrosis avanzada con síndrome me­ La esteatosis del HGNA no incrementa la morbilidad o mor­
tabólico y EHGNA. También pacientes con EHGNA y con talidad a corto plazo, pero su progresión a EHNA aumenta el
probable coexistencia de enfermedad hepática crónica distin­
ta a la enfermedad hepática grasa no alcohólica.

30. ¿Qué pruebas serológicas existen para evaluar


riesgo de cirrosis, falla hepática y carcinoma hepatocelular. La
mortalidad en esteatosis simple es de 0%; la mortalidad en en­
fermedad hepática grasa no alcohólica es de 1.6 a 6.8%. Se
considera que la EHNA es la expresión hepática del sindrome
1
la esteatosis? metabólico.
El FibroTest determina el grado de fibrosis y el ActiTest deter­
mina la actividad necroinflamatoria (sensibilidad de 91 % y 34. ¿Qué factores tienen un impacto negativo en la
especificidad de 89%). Son un algoritmo patentado. El índice supervivencia en la EHGNA?
de fibrosis determina alfa 2 macroglobulina, apolipoproteina Diabetes mellitus, cifras séricas elevadas de alanina amino­
A 1, haptoglobina, bilirrubina, bilirrubina total y gammagluta­ transferasa y aspartato aminotransferasa (ALT y AST); mayor
mil transpeptidasa. El índice de actividad necroinflamatoria edad y presencia de inflamación necrótica en la biopsia hepá­
combina los cinco marcadores y niveles de ALT. Existe otro tica inicial, alteración de la glucemia en ayunas y presencia de
algoritmo: el EsteatoTest, que combina seis componentes del cirrosis.
FibroTest-ActiTest ajustados a edad, género, IMC y niveles de
AST, glucosa, triglicéridos y colesterol. El NASH test combina CIRROSIS CARDIACA
edad, género, los seis componentes del FibroTest-ActiTest más
peso, talla, AST, glucosa, triglicéridos, colesterol y valores del 35. ¿Qué es la cirrosis cardiaca?
Esteato-Test. La especificidad es de 94% y la sensibilidad es de Es una disfunción hepática de origen cardiaco. Las causas pue­
33%. La desventaja es su costo; en la actualidad está disponi­ den ser pericarditis constrictiva, hipertensión pulmonar arte­
ble en México. rial grave, estenosis mitral, regurgitación tricuspídea, cor pulmo­
nale, miocardiopatía isquémica y procedimientos quirúrgicos
31. ¿Cuáles son las posibilidades de progresión en por atresia pulmonar. Conducen a una congestión por una
EHNA? presión elevada del ventrículo derecho, con falla cardiaca de­
Entre 3 y 5% de los sujetos con EHNA progresa a cirrosis he­ recha. Con el tratamiento actual para la falla cardiaca, la preva­
pática en un tiempo promedio de 20 años, y de éstos 8% <lesa- lencia de cirrosis cardiaca está en descenso.

773
e MANUAL PARA EL EXAMEN NACIONAL DE RESIDENCIAS MÉDICAS

36.¿Qué ocurre en el hígado por la insuficiencia car­ Chalasani N, Younossi Z, Lavine JE et a/. The diagnosis and management of
non-alcoholic fatty liver disease: practice guideline by the American Gas­
diaca crónica?
troenterological Association, American Association for the Study of Liver
Los hepatocitos de la zona 3 son sustituidos por reticulina y en Diseases, and American College of Gastroenterology. Gastroenterology.
los de las zonas 1 y 2 se genera colágeno y zonas esteatósicas. 2012 Jun;142(7): 1592-609.
Puede desarrollarse una fibrosis alrededor de las zonas porta­ Festi D, Schiumerini R, Marzi L, Di Biase AR, Mandolesi D, Montrone L et
a/. Review article: the diagnosis of non-alcoholic fatty liver disease-availa­
les conservadas con aspecto de nuez moscada y un patrón de bility and accuracy of non-invasive methods. Aliment Pharmacol Ther.
hemorragia y necrosis de la zona 3. La vena central puede es­ 2013 Feb;37(4):392-400.
tar prominente. La congestión no tratada de larga evolución Fouad YM, Yehia. Hepato-cardiac disorders. R. World J Hepatol. 2014 Jan
progresa a cirrosis cardiaca. 27;6(1 ):41-54.
La cirrosis cardiaca es rara y los síntomas predominantes Kanwar P, Kowdley KV. Metal storage disorders: Wilson disease and hemo­
chromatosis. Med Clin North Am. 2014 Jan;98(l ):87-102.
son por insuficiencia cardiaca derecha. La cirrosis se debe sos­ Lizardi-Cervera J, Laparra DI, Chávez-Tapia NC, Ostos ME, Esquive! MU.
pechar en pacientes con estenosis mitral grave, pericarditis P revalence of NAFLD and metabolic syndrome in asymptomatic sub­
constrictiva y falla cardiaca congestiva de larga evolución. jects. Rev Gastroenterol Mex. 2006;71:453-9.
Méndez Sánchez, Gutiérrez Grabe, Chávez Tapia et al. Hígado graso no alco­
37. ¿Cuáles son las alteraciones bioquímicas en la hólico y esteatohepatitis no alcohólica: conceptos actuales. Rev Gastroen­
cirrosis cardiaca? terol Mex. 2010;75(Supl.2): 143-8.
Myers RP, Cerini R, Sayegh R, Moreau R, Degott C, Lebrec D, Lee SS. Car­
Se puede observar aumento de la bilirrubina menor 4.5 mg/ diac hepatopathy: clinical, hemodynamic, and histologic characteristics
dL. Los niveles de aspartato aminotransferasa (AST) pueden and correlations. Hepatology. 2003 Feb;37(2):393-400.
ser mayores que la alanino aminotransferasa (ALT); la eleva­ Nascimbeni F, Pais R, Bellentani S, Day CP, Ratziu V, Loria P, Lonardo A.
ción de la fosfatasa alcalina es rara en falla cardiaca; el INR From NAFLD in clinical practice to answers from guidelines. Hepatol.
2013 Oct;59(4):859-71.
está elevado en más de 80% de los casos y puede haber hipoal­
Nogueira de Rojas, Dehesa Violante, González Huezo et a/. Guías clínicas de
buminemia. diagnóstico y tratamiento de hepatopatía grasa no alcohólica. Tratamien­
to. Rev Gastroenterol Mex. 2008;73:134-6.
38. ¿Cuál es el tratamiento de la cirrosis cardiaca? Roberts EA, Schilsky ML. A practice guideline on Wilson disease. Hepatolo­
Debe tratarse la patología primaria, que es la insuficiencia car­ gy. 2003;37:1475-92.
Roberts EA, Schilsky ML. Diagnosis and treatment of Wilson disease: an
diaca, así como la paracentesis en caso de ascitis refractaria a
update. Hepatology. 2008;47:2089-111.
tratamiento. La cirrosis cardiaca, con o sin falla hepática, pre­ Uscanga Domínguez, Bielsa Fernández, Huerta Iga et a/. Guías clínicas de
senta un riesgo incrementado en la mortalidad perioperatoria diagnóstico y tratamiento de hepatopatia grasa no alcohólica. Generalida­
que puede contraindicar la cirugía. Hay casos anecdóticos re­ des. Rev Gastroenterol Mex. 2008;73:126-8.
portados con reversión de la cirrosis cardiaca después de diez World Health Organization. Guías de la Organización Mundial de Gastroente­
rología. Enfermedad del hígado graso no alcohólico y esteatohepatitis no
años de trasplante de corazón.
alcohólica. [Monografía en internet] Ginebra. 2012. Disponible en: http://
www.worldgastroenterology.org/ as sets/ ex-port/userfiles/ 2013_NASH­
BIBLIOGRAFÍA NALD_SP _Final_long.pdf
Bosques-Padilla, Aguirre-García, Kershenobich-Stalnikowitz et al. Guías clíni­
cas de diagnóstico y tratamiento de hepatopatia grasa no alcohólica. Diag­
nóstico. Rev Gastroenterol Mex. 2008;73: 129-33.

CAPÍTULO 11.20
ABSCESOS HEPÁTICOS
Hairé Manzano Cortés • Samuel Belkotosky Villa

ABSCESO HEPÁTICO PIÓGENO absceso hepático amebiano, pero en países en vías de desa­
rrollo dicha incidencia es mayor, donde el ARA es más fre­
1. ¿Qué tipos de absceso hepático existen? cuente.
El absceso hepático tiene una incidencia variable en la pobla­
ción. Comprende el absceso hepático piógeno (AHP) y el 3. ¿Cuál es la presentación del absceso hepático pió­
amebiano (ARA). geno?
El AHP suele ocurrir en pacientes mayores de 50 a 60 años de
2. ¿Cuál es la incidencia de los abscesos hepáticos? edad, con ligero predominio en hombres. Por lo general son
Alrededor de 10% de la población mundial está crónicamente múltiples y se localizan en lóbulo hepático derecho. La mayo­
infectada por Entamoeba histolytica y cerca de 10% desarrolla ría de los abscesos múltiples corresponde a causa biliar o he-

774
SECCIÓN 11 GASTROENTEROLOGÍA @
matógena y en menor frecuencia son únicos por causa portal, 8. ¿Cuál es el método diagnóstico de elección en un
criptogénica o traumática. Los síntomas se establecen en días absceso hepático?
a semanas, por lo regular menos de dos a cuatro semanas, pe­ La tomografía computarizada tiene una mayor sensibilidad:
ro adultos mayores pueden presentar un cuadro clínico suba­ 95 a 100%. Se considera el método de elección para lesiones
gudo y si los abscesos son múltiples tienen mayor síntomato­ hepáticas y además es útil para identificar otras patologías in­
logía sistémica. traabdominales.
La radiografía simple de abdomen puede mostrar hepato­
4. ¿En qué consiste la tríada de Charcot? megalia con niveles hidroaéreos en la cavidad del absceso. La
En la mayoría de los casos de absceso hepático piógeno se pre­ radiología de tórax es a menudo patológica, sobre todo en los
senta la tríada de Charcot que incluye: hepatomegalia, hi­ abscesos subdiafragmáticos, y las alteraciones más frecuentes
pertermia y hepatalgia (dolor en hipocondrio derecho), que son las atelectasias y el derrame pleural.
también está descrita como manifestación de colangitis. Los La ecografía abdominal tiene una sensibilidad de 85 a 95%.
síntomas son dolor abdominal superior, fiebre de alto grado, En el medio es el método inicial debido a que no es invasivo y
náusea y vómito. La pérdida de apetito, la ictericia y los sínto­ tiene bajo costo, además puede ser usado para guiar la aspira­
mas respiratorios son menos frecuentes y dependen del tama­ ción y el cultivo del absceso.
ño del absceso, las comorbilidades y las complicaciones exis­
tentes. En abscesos subdiafragmáticos se presenta tos y dolor 9. ¿Cuál es el tratamiento para el AHP?
pleurítico irradiado a hombro derecho. El signo más común es
En el absceso hepático piógeno deben aplicarse de manera con­
dolor en hipocondrio derecho con resistencia a la palpación,
junta antibióticos y drenaje cuando el absceso es igual o ma­
así como hepatomegalia. Puede haber ictericia, ascitis o derra­
yor a 5 cm o en abscesos menores si no hay respuesta a anti­
me pleural.
bióticos. Los antibióticos por vía intravenosa son la primera
línea de tratamiento y deben ser de amplio espectro; incluyen
5. ¿Cuál es la causa más frecuente de AHP? cefalosporinas, aminoglucósidos, metronidazol y carbapené­
La causa más frecuente es biliar por una colangitis ascendente, micos. Al inicio el tratamiento es empírico, pero el régimen se
ya sea por litiasis, estenosis o neoplasia, seguida de la crip­ establece con el resultado de los cultivos y puede durar de dos
togénica. Con menor frecuencia su origen es portal (secun­ a cuatro semanas. El tratamiento antibiótico sin drenaje es con­
daria a apendicitis, diverticulitis o enfermedad inflamatoria trovertido y aplicable sólo en abscesos pequeños y siempre de
intestinal), por extensión directa de una estructura vecina (ve­ forma individualizada. Véase figura 11-20-1.

1
sícula, colon), bacteriemias de cualquier foco y traumatismos.
10. ¿Cuál es la técnica de drenaje preferida?
6. ¿Cuál es el agente infeccioso más frecuente en el
La técnica de elección es el drenaje percutáneo guiado por ul­
AHP?
trasonido, ya que es eficaz en abscesos múltiples o solitarios
La etiología más habitual son las enterobacterias, en especial menores de 10 cm y reduce la estancia hospitalaria. El drenaje
Escherichia coli y Klebsiella sp., seguidas de Staphyi!ococcus au­ quirúrgico abierto debe realizarse en casos de rotura, multi­
reus y anaerobios. Depende de la causa, ya que también se loculación, patología biliar o intraabdominal asociada. Véase
puede aislar Streptococcus sp. (enterococo, estreptococos anae­ figura 11-20-1.
robios como Peptococcus sp. y Peptostreptococcus sp., y estrep­
tococos microaerofílicos); entre ellos, Streptococcus milleri es
11. ¿Cuáles son las complicaciones más frecuentes?
la especie más frecuente.
Sospechar Candida sp. y Pseudomonas en pacientes con Entre 10 y 20% de los pacientes presenta complicaciones rela­
inmunosupresión por infección por VIH, quimioterapia y tras­ cionadas con la extensión a estructuras vecinas o con la rotu­
plante. ra del absceso. Las más frecuentes son las pleuropulmonares,
Existe un elevado porcentaje de abscesos estériles, pero como derrame pleural y/o empiema y las menos comunes
pueden deberse a un inadecuado procesamiento de cultivos son el absceso subfrénico, la peritonitis, la pericarditis y la he­
anaerobios. mobilia.

7. ¿Qué hallazgos de laboratorio presentan estos pa­ ABSCESO HEPÁTICO AMEBIANO


cientes?
Los que suelen presentarse son leucocitosis con desviación a la 12. ¿Qué es el absceso hepático amebiano?
izquierda, anemia, velocidad de sedimentación globular y pro­ El absceso hepático amebiano (AHA) es una complicación
teína C reactiva elevadas. Perfil hepático por lo general con poco común que pone en riesgo la vida; se presenta con mayor
elevación de fosfatasa alcalina, bilirrubinas y transaminasas, frecuencia en menores de 50 años de edad y su causa es la ín­
hipoalbuminemia. fección por el parásito Entamoeba histolytica.

775
e MANUAL PARA EL EXAMEN NACIONAL DE RESIDENCIAS MÉDICAS

La transmisión de Entamoeba histolytica es la vía fecal-oral bién es posible. Además de viajar a una zona endémica, otros
y, una vez que se establece en el colon, es capaz de invadir la factores de riesgo son hombres que tienen sexo con hombres
mucosa, donde ocasiona ulceración e inflamación. Después se y asilados. La transmisión heterosexual de ARA se ha descrito
disemina a sitios extraintestinales distantes, el más común de en fechas recientes. La supresión de la inmunidad mediada por
los cuales es el hígado. células aumenta el riesgo de enfermedad invasiva, que puede
La amebosis es una infección parasitaria causada por el deberse a uso de corticosteroides, malignidad, edades extre­
protozoario intestinal Entamoeba histolytica. Se estima que 40 mas y embarazo.
millones de personas son infectadas cada año. La enfermedad
por E. histolytica ocasiona 40 000 a 100 000 muertes cada año 15. ¿En qué género se presenta con más frecuencia
a partir de una colitis amebiana y de infección extraintestinal. el AHA?
El ARA es el sitio de infección extraintestinal más común y El ARA es diez veces más común en adultos varones que en
ocurre en menos de 1 % de las infecciones por E. histolytica; sin mujeres. La distribución de la infección colónica (sintomática
embargo, en países en vías de desarrollo esta frecuencia se in­ o asintomática) es casi idéntica entre hombres y mujeres, lo
crementa. que sugiere una mayor susceptibilidad en los hombres para una
enfermedad invasiva. Los factores predisponentes que contri­
13. ¿Cómo se produce la infección hepática?
buyen a esta disparidad incluyen daño hepatocelular por al­
La infección por E. histolytica ocurre cuando los quistes madu­ cohol en hombres.
ros son ingeridos, por lo regular por alimentos o agua contami­
nados con materia fecal, con más frecuencia en países en vías
16. ¿Cuáles son los síntomas característicos del AHA?
de desarrollo. Cuando los parásitos llegan al intestino delgado
se liberan los trofozoítos, los cuales penetran la mucosa coló­ El absceso hepático amebiano suele presentarse con una his­
nica. Ahí pueden ocasionar úlceras colónicas y tener acceso al toria aguda o subaguda de fiebre y dolor en el cuadrante su­
sistema portal venoso para infectar el hígado, el cerebro, los perior derecho, aunque en 20 a 50% de los casos tiene una pre­
pulmones, el pericardio y otros sitios metastásicos. En el híga­ sentación crónica con una historia de diarrea, pérdida de peso
do las amibas generan una reacción inflamatoria y causan ne­ y dolor abdominal. La duración promedio de los síntomas es
crosis de los hepatocitos, lo cual produce un absceso. de alrededor de catorce días.

14. ¿Cuáles son los factores condicionantes en la for­ 17. ¿Cuáles son las complicaciones del AHA?
mación del absceso hepático amebiano? Las complicaciones del ARA incluyen la rotura del absceso,
La mayoría de los casos de amebosis invasiva en países desa­ que provoca diseminación al peritoneo, espacio pleural o peri­
rrollados ocurre en inmigrantes de países endémicos para E. cardio. La rotura hacia el sistema pleuropulmonar ocurre en
histolytica, aunque la infección en viajeros de corto plazo tam- más de 40% de los pacientes con ARA, mientras que la rotura

FIGURA 11-20-1. ALGORITMO TERAPÉUTICO DE ABSCESO HEPÁTICO PIÓGENO.

Absceso piógeno confirmado


por imagen

Tratamiento antibiótico

Rotura

<5cm 25cm

Respuesta
Multilocular
patología biliar concomitante
s�
i
Antibióticos 1
No

Drenaje percutáneo Cirugía


Falla

776
SECCIÓN 11 GASTROENTEROLOGÍA @
peritoneal ocurre en sólo 7% de los pacientes. También se han 22. ¿Está indicado el drenaje del AHA?
descrito ictericia obstructiva, obstrucción de la vena cava infe­ El drenaje antes estaba indicado en abscesos mayores de 10
rior y superinfección bacteriana. En raros casos pueden ocu­ cm; en la actualidad está indicado para los pacientes que pre­
rrir abscesos extraintestinales vía hematógena que se disemi­ senten un gran riesgo de rotura; es decir, cavidad > 5 cm de
nan hacia cerebro, pulmón, bazo, vagina, útero y piel. diámetro, absceso del lóbulo izquierdo, por su mayor mortali­
dad y la posibilidad de rotura hacia el peritoneo o el pericar­
18. ¿Cuáles son los hallazgos de laboratorio en es­ dio, nula respuesta al tratamiento médico después de cinco a
tos pacientes? siete días o empeoramiento del dolor e ictericia, así como en
Las alteraciones de laboratorio incluyen leucocitosis en 75% abscesos de larga evolución o que, por el contrario, tengan
de los pacientes. Los niveles de transaminasas pueden estar una rápida progresión clínica.
elevados en más de 66% de los casos, mientras que alcalina es (5:) http://www.cenetec.salud.gob.mx/descargas/gpc/
más frecuente que la fosfatasa esté elevada. La anemia está CatalogoMaestro/432_GPC_Absceso_hepatico/GRR_
presente en más de la mitad de los casos y la hiperbilirrubine­ Absceso_hepxtico_amebiano.pdf
mia en una tercera parte de los pacientes. Comparado con el
absceso piógeno, es más frecuente que el ARA provoque hi­ BIBLIOGRAFÍA
poalbuminemia. Carrillo ÑL et al. Absceso hepático: características clínicas, imagenológicas y
manejo en el Hospital Loayza en 5 años. Rev Gastroenterol Perú. 2010;
30(1):46-51.
19. ¿Qué estudios de imagen deben realizarse ante
Centro Nacional de Excelencia Tecnológica en Salud. Diagnóstico y trata­
sospecha de AHA? miento del absceso hepático amebiano no complicado. [Monografia en
Se había considerado que el ultrasonido tenía una sensibilidad internet.] Mexico 2011; CENETEC. Disponible en: http://www.cenetec.
mayor a 90% para el ARA, y si está disponible es una prueba salud.gob.mx/descargas/gpc/CatalogoMaestro/432_GPC_Absceso_
hepatico/GRR_Absceso_hepxtico_ame biano.pdf
inicial razonable. Sin embargo, estudios recientes encontraron Chung YF et al. Management of pyogenic liver abscesses-percutaneous or
una menor sensibilidad (58%). Si se tiene alta sospecha y el open drainage? Singapore Med J. 2007 Dec;48( 12): 1158-65; quiz 1165.
ultrasonido inicial se reporta negativo, se recomienda el uso de Cosme A et al. Pyogenic versus amoebic liver abscesses. A comparative clinical
la tomografía computarizada, cuya sensibilidad es mayor y es study in a series of 58 patients. Rev Esp Enferm Dig (Madrid). 2010; 102:90-9.
considerada el método de elección en lesiones hepáticas. Com­ Dhaval O. Manguk.iya DO et al. A prospective series case study of pyogenic
liver abscess: recent trends in etiology and management. lndian J Surg.
parados con los abscesos piógenos, los ARA suelen ser solita­ September-October 2012;74(5):385-90.
rios y localizarse en lóbulo derecho, con un tamaño de 4 a 12

1
Ellen L, Stanley SL. Amebiasis. Gastroenterolo Clin North Am. 1996;25:417-92.
cm, pero en promedio son de 8 cm. Cabe mencionar que la Elzi L, Laifer G, Sendi P, Ledermann HP, Fluck.iger U, Bassetti S. Low sensi­
presencia de abscesos múltiples en hígado no descarta el ARA. tivity of ultrasonography for the early diagnosis of amebic liver abscess.
Am J Med. 2004; 117:519-22.
20. ¿Cuál es la prueba serológica que se utiliza? Espinosa-Cantellano M, Martínez-Palomo A. Pathogenesis of intestinal ame­
biasis. From molecules to disease. Clin Microbio! Rev. 2000; 13:318-31.
Se pueden realizar estudios de serología; la hemaglutinación García AG, Rebollar GRB, Trejo TR, Sánchez CRM. Abscesos hepáticos pió­
indirecta (HAI) es la técnica más sensible y es positiva en 90 genos. Rev Hosp Juá Mex. 2011;78(3): 156-63.
a 100% de los casos después de una o dos semanas. Las prue­ Haque R, Houston C, Hughes M, Houpt E, Petri W. Amebiasis. N Eng J Med.
bas de aglutinación con látex se correlacionan con la HAI y se 2003;348: 1565-73.
Kain KC, Boggild AK. Amebiasis. En: Rakel RE, Bope ET (ed). Conn's Cu­
llevan a cabo con mayor rapidez, pero pueden ser negativas rret Therapy. 56th ed. Philadelphia: WB Saunders Co; 2004. pp. 60-3.
durante la primera semana. Kimura K, Stoopen M, Reeder MM, Moneada R. Amebiasis: Modern diag­
nostic imaging with pathological and clinical correlation. Semin Roent.
21. ¿En qué consiste el tratamiento del AHA? 1997;4:250-75.
Los nitroimidazoles, sobre todo el metronidazol, han sido el Knobloch J, Manweiler E. Development and persistence of antibodies to En­
tamoeba histolytica in patients with amebic liver abscess: Analysis of 216
tratamiento estándar para ARA por más de 40 años y se man­ cases. Am J Trop Med Hyg. 1983;32:732-37.
tienen como el tratamiento de elección. El metronidazol se da Li E, Stanley SL Jr. Parasitic disease of the liver and intestines. Gastroenterol
oral a una dosis de 750 mg, tres veces al día por siete a diez Clin North Am. 1996;25:471-92.
días. Más adelante hay que administrar un agente luminal pa­ Maltz G, Knauer CM. Amebic liver abscess: A 15 year experience. Am J Gas­
ra asegurar la erradicación, como el tinidazol u ornidazol. En troenterol. 1991 ;86:704.
Ortiz SF et al. Absceso hepático amebiano: ¿tratamiento farmacológico o
los casos de intolerancia a imidazoles o como alternativa tera­ punción-aspiración? Gastroenterol Hepatol. 2007;30(7):399-401.
péutica se puede usar nitazoxanida. Salanta RA, Martínez-Palomo A, Murray HW et al. Patients treated for ame­
Algunos estudios concluyen que no existe una ventaja de­ bic liver abscess develop cell mediated immune responses effective in vi­
mostrable del drenaje percutáneo de un absceso no complica­ tro against Entamoeba histolytica. J lmmunol. 1986; 136:2633-9.
do por E. histolytica comparado con el tratamiento de metro­ Salit !E, Khairnar K, Gough K, Pillai DR. A possible cluster of sexually trans­
mitted Entamoeba histolytica: Genetic analysis of a highly virulent strain.
nidazol solo. Se han reportado indices de cura mayores a 90% Clin Infect Dis. 2009;49:346-53.
con terapia médica, con resolución de fiebre, dolor y anorexia World Health Organization. WHO/PAHO/UNESCO. Consultation of experts
entre 72 y 96 horas. on amoebiasis. Wk.ly Epidemiol Rec. 1997;72:97-100.

777
e MANUAL PARA EL EXAMEN NACIONAL DE RESIDENCIAS MÉDICAS

CAPÍTULO 11.21
PANCREATITIS AGUDA
José Luis Vega Fonseca

ETIOPATOGENIA Y CUADRO CLÍNICO CUADRO 11-21-1.


ETIOLOGÍA DE LA PANCREATITIS AGUDA
1. ¿Cuál es la incidencia de la pancreatitis aguda?
CAUSAS MÁS COMUNES CAUSAS POCO COMUNES
La pancreatitis aguda es un proceso inflamatorio del páncreas,
Litiasis vesicular (microlitiasis) Traumatismo
que con frecuencia afecta el tejido peripancreático y puede
Alcohol (agudo-crónico) Estado posoperatorio
involucrar órganos adyacentes o distantes. La incidencia de Pos-CPRE Hiperparatiroidismo
pancreatitis es de 5 a 35 por 100 000 habitantes (Estados Uni­ Hipertrigliceridemia (1 000 mg/dL), (hipercalcemia)
dos), la mortalidad es de 10% en el total de las pancreatitis fármacos (estrógenos, tacrolimús, Mecánicas (áscaris)
agudas, 1.5% en pancreatitis leves o moderadas y 17% en pan­ tetraciclinas, ácido valproico, Disfunción de esfínter de Oddi
sulfonamidas y antirretrovirales) ldiopáticas
creatitis grave.

2. ¿Cómo se clasifica la pancreatitis aguda? ingerido existen factores precipitantes que afectan la suscep­
a) Pancreatitis aguda leve: no hay complicaciones ni falla de tibilidad del individuo a desarrollar lesión del páncreas. La hi­
la glándula. pertrigliceridemia representa la tercera causa de pancreatitis
b) Pancreatitis aguda moderada: complicaciones transitorias aguda (entre 1.3 y 3.8%) cuando los valores de triglicéridos
que desaparecen en 48 horas. son superiores a 1 000 mg/dL. Algunos de estos individuos son
e) Pancreatitis aguda grave: presencia de necrosis y hemorra­ portadores de diabetes mellitus y receptores de fármacos que
gia, falla orgánica persistente. pueden contribuir a la enfermedad. Otras causas de inflama­
ción pancreática tienen que ver con la ingesta de medicamen­
3. ¿Cuáles son las fases de la etiopatogenia de la tos (diuréticos, inmunosupresores) y factores infecciosos y/o
pancreatitis aguda? secundarios a CPRE. Véase cuadro 11-21-1.
La primera fase se caracteriza por la activación de enzimas
digestivas y por la lesión de células acinares, como consecuen­ 5. ¿Cuál es el cuadro clínico característico?
cia de la activación del cimógeno. Los pacientes con pancreatitis aguda por lo regular se presen­
La segunda fase se caracteriza por la activación, la quimio­ tan con dolor abdominal en la región epigástrica o cuadrante
atracción y el secuestro de neutrófilos en el interior del pán­ superior izquierdo, el cual puede variar desde una molestia
creas, lo que origina una reacción inflamatoria intrapancreáti­ leve y tolerable hasta el dolor intenso, constante e incapacitan­
ca de intensidad variable. te, con irradiación hacia región dorsal, tórax y ambos flancos.
La fase 3 se debe al efecto de enzimas proteolíticas (tripsi­ El dolor suele ser más intenso en posición supina y mejorar
na, elastasa y fosfolípasa A2) y a mediadores activados por la con la posición en gatillo (tronco flexionado y rodillas encogi­
inflamación pancreática, los cuales originan proteólisis, ede­ das). Son frecuentes náusea, vómito y distensión abdominal,
ma, hemorragia intersticial, daño vascular y necrosis de tipo secundarios al íleo y a la peritonitis química. En la explora­
graso y coagulativa en el parénquima pancreático. ción física el paciente suele mostrarse angustiado e inquieto.
Como consecuencia se genera la producción de sustancias Por lo regular se asocia con febrícula, taquicardia e hipoten­
vasoactivas (bradicinina, histamina) que originan vasodilata­ sión. Hasta en 20% de los casos se pueden encontrar esterto­
ción, mayor permeabilidad y edema en múltiples órganos, lo res basales, atelectasias y derrame pleural (más frecuente iz­
cual condiciona síndrome de respuesta inflamatoria sistémica, quierdo).
síndrome de insuficiencia respiratoria aguda y falla orgánica � http://www.cenetec.salud_gob.mx/descargas/gpc/
múltiple. La pancreatitis aguda es una entidad multifactorial CatalogoMaestro/011_GPC_PancreatAguda/
condicionada por múltiples mecanismos que favorecen o des­ SSA_011_08_EyR.pdf
encadenan la inflamación pancreática.
6. ¿Cuáles son los signos sugestivos de pancreatitis
4. ¿Cuál es la principal causa de la pancreatitis aguda? necrosante grave?
En la mayoría de las series la litiasis vesicular continúa siendo En ocasiones se observa el signo de Cullen, que es una colora­
la causa más común de la enfermedad (entre 30 y 60%); la ción azulosa alrededor del ombligo debida a hemoperitoneo,
segunda causa se debe a la ingesta de alcohol (entre 15 y 30% y/o el signo de Grey Turner, que es una equimosis en ambos
de los casos en Estados Unidos). Sin embargo, se desconoce el flancos, poco frecuentes pero sugestivos de pancreatitis necro­
mecanismo de lesión, ya que además del volumen de alcohol sante grave.

778
SECCIÓN 11 GASTROENTEROLOGÍA @
Datos de abdomen agudo que resultan insignificantes en pancreatitis aguda lo constituye la determinación de niveles sé­
comparación con la intensidad del dolor o la peristalsis que se ricos de lipasa (sensibilidad entre 90 y 100%, especificidad de
encuentra disminuida o ausente también sugieren un cuadro 99%). La proteína C reactiva sigue siendo el estándar de oro
grave. en la valoración del pronóstico de gravedad.

DIAGNÓSTICO 9. ¿Cuáles son los estudios de imagen que deben


realizarse en un inicio en PA?
7. ¿Cómo se elabora el diagnóstico de pancreatitis
aguda? La radiografía de tórax y la placa simple de abdomen, aunque
son de baja sensibilidad y especificidad, son recomendables pa­
El diagnóstico de pancreatitis aguda se establece por la presen­ ra la orientación en el diagnóstico diferencial. Dado que la li­
cia de dos de los tres criterios siguientes: tiasis vesicular es la causa principal en la etiología de la pan­
a) Dolor abdominal compatible con la enfermedad. creatitis aguda, el ultrasonido abdominal es un procedimiento
b) Amilasa y/o lipasa sérica mayor de tres veces el limite su­ que debe realizarse en primera instancia en el paciente con
perior (niveles de amilasa dentro de rangos de normalidad sospecha de la enfermedad.
después de 72 horas no descartan pancreatitis o no signifi­
ca que el cuadro agudo haya remitido). 10. ¿Cuándo se indica la realización de tomografía
e) Hallazgos característicos en imagen abdorrtinal. en PA?
La edad mayor de 55 años, IMC �30, la falla orgánica en el La tomografía computarizada de abdomen debe realizarse
momento del ingreso y la presencia de derrame pleural cons­ cuando el diagnóstico de pancreatitis aguda no pueda estable­
tituyen factores de riesgo de intensidad que deben registrarse cerse con los datos clínicos y de laboratorio. La tomografía
en el momento del diagnóstico. Véase cuadro 11-21-2. computarizada tiene una sensibilidad y especificidad mayor
a 90% para el diagnóstico de pancreatitis; sin embargo, su uso
CRITERIOS DIAGNÓSTICOS rutinario no está justificado ya que el diagnóstico es clínico
y la mayoría de los pacientes tiene una evolución corta y no
8. ¿Cuál es la prueba de laboratorio con mayor sen­ complicada. Sin embargo, en un paciente que no mejora des­
sibilidad y especificidad diagnóstica de PA? pués de 48 a 72 horas se recomienda para la valoración de
En el momento de la evaluación clínica del paciente con sos­ complicaciones locales (necrosis pancreática).
pecha de pancreatitis aguda se recorrtienda solicitar de forma � http://www.cenetec.salud.gob.mx/descargas/gpc/

1
orientada y razonada: deterrrtinación de arrtilasa y lipasa, prue­ CatalogoMaestro/011_G PC_PancreatAguda/
bas de función hepática, biometría hemática completa, quími­ SSA_011_08_EyR.pdf
ca sanguinea, electrolitos séricos, proteínas séricas totales,
gasometría arterial, tiempos de coagulación y proteína C reac­ 11. ¿Cuáles son los criterios pronóstico que deben
tiva. El mejor parámetro bioquímico para el diagnóstico de la realizarse ante un paciente con PA?
Véase cuadro 11-21-3.
CUADRO 11-21-2.
AUXILIARES DIAGNÓSTICOS PARA PANCREATITIS 12. ¿Cuáles son los criterios pronósticos de Balthazar?
AGUDA Véase cuadro 11-21-4.
ESTUDIOS HALLAZGOS
13. ¿Con qué otras escalas puede valorarse la gra­
Radiografía simple de Íleo localizado a un segmento de
abdomen intestino delgado, asa centinela,
vedad en pacientes con PA?
signo del colon cortado, ascitis Es importante identificar a personas con pancreatitis aguda que
(pancreatitis grave), cálculos calci­ tienen riesgo de fallecer. Los sistemas de puntuación de múlti­
ficados en vesícula biliar, calcifica­
ciones en área pancreática
ples factores (Ranson, APACHE 11) son difíciles de utilizar y
además tienen escasa capacidad predictiva por parte del clíni­
Radiografía de tórax Elevación del hemidiafragma, derra­
me pleural, atelectasia basal
co, ya que no se han aceptado de forma unánime. Los indi­
cadores básicos de un ataque grave incluyen: edad > 70 años,
Ultrasonido de abdomen Litiasis biliar, dilatación del colédoco
(estudio de elección en y ascitis, calcificaciones intraducta­
IMC >30%, hematocrito >44%, PCR > 150 mg/L (ingreso).
las primeras 24 horas) les (pancreatitis crónica) Sin embargo, el pronóstico en la mayoría de los casos gra­
Tomografía computarizada Aumento de tamaño pancreático, co­
ves se rige por la falla multiorgánica, en donde predomina la
de abdomen (modalidad lecciones líquidas peripancreáticas, insuficiencia respiratoria (PO 2 <60 mm Hg). Otros factores
diagnóstica más precisa necrosis pancreática fundamentales para determinar la gravedad de la enfermedad
para establecer gravedad son la presencia de choque (TA <90 mm Hg, FC > 130 1pm),
y detectar complicaciones)
insuficiencia renal ( creatinina >2 mg/100 mL) y hemorragia

779
e MANUAL PARA EL EXAMEN NACIONAL DE RESIDENCIAS MÉDICAS

CUADRO 11-21-3. para prevenir complicaciones infecciosas. La nutrición paren­


CRITERIOS PRONÓSTICOS DE RANSON Y GLASGOW teral debe evitarse en lo posible, a menos que haya contraindi­
cación entera!.
48 HORAS
La analgesia es fundamental en el tratamiento de la pancrea­
. CRITERro�J titis aguda grave. Los fármacos deben elegirse de manera es­
CRITERIOS DE GLASGOWi
calonada en función de la intensidad del dolor, desde anal­
DE RANSON '. (Ptl �l�!A R(�
_ gésicos no opiáceos hasta la morfina. La succión gástrica a
Edad >55 >70años Baja de hematocrito Igual
través de sonda nasogástrica es innecesaria en pacientes con
años >10%
Leucocitos >18 000 Elevación de BUN >2mg/100 pancreatitis aguda, a menos que la enfermedad esté asociada
>16 000 >5mg/100m L ml con íleo y vómito frecuente.
DH L >250 >400 UI/L Calcio sérico Igual
UI/L <8mg/100ml
15. ¿Cuándo deben indicarse los antibióticos?
AS T(TGO) >250UI/L PaO2 <60mm Hg Igual
>250UI/L Exceso de base >5mEq/L No hay evidencia suficiente que sustente el uso rutinario de
Glucosa >220mg/100 >4mEq/L profilaxis antibiótica en el paciente con pancreatitis leve o mo­
>200mg/ ml Secuestro de >4 L derada. En caso de administrar antibiótico profiláctico, la du­
100ml líquidos >6 L
ración puede ser hasta de catorce días o más, si persisten las
CRITERIOS • • J • J t complicaciones locales o sistémicas no sépticas o si los niveles
Oa 2 1% de PCR son> 120 mg/dL. Los pacientes con síntomas persis­
3a 4 16% tentes y necrosis> 30%, así como aquellos con áreas pequeñas
5a 6 40% de necrosis y sospecha de sepsis, deben someterse a aspiración
7a 8 100%
con aguja fina por imagen, con la finalidad de enviar culti­
vo con antibiograma y tinción de Gram, dentro de un periodo de
siete a catorce días después de la presentación de la pancreatitis.
intestinal (pérdida >500 mL/24 horas). El elevado indice de En pacientes con necrosis infectada los antibióticos que
mortalidad en estos pacientes se debe en gran medida a la permean a la zona de necrosis, como carbapenémicos, quinolo­
sepsis y justifica la vigilancia radiográfica intensiva, el trata­ nas y metronidazol, pueden ser útiles para retrasar o evitar
miento en terapia intensiva y la intervención quirúrgica (ne­ intervención, con lo cual se disminuye la morbimortalidad.
crosectomía y drenaje).
16. ¿En qué casos se recomienda la colecistectomía?
TRATAMIENTO En pacientes con pancreatitis aguda leve que tienen colelitia­
sis se debe realizar colecistectomía antes del egreso, siempre y
14. ¿Cuál es el tratamiento de la PA? cuando haya remisión de los síntomas y los niveles de lipasa
Se inicia hidratación agresiva (250 a 500 mL/hora de solu­ hayan descendido a rangos de normalidad, con el fin de preve­
ciones cristaloides isotónicas), a menos que haya alguna con­ nir cuadros recurrentes.
traindicación cardiovascular o renal. Los requerimientos de En pacientes con pancreatitis aguda biliar necrosante, y a
líquidos deben reevaluarse en las primeras seis horas a partir fin de prevenir infección, la colecistectomía se difiere hasta que
del ingreso y en las siguientes 24 a 48 horas. la inflamación activa sea controlada y las colecciones líquidas
En la pancreatitis aguda leve la alimentación vía oral debe se resuelvan.
iniciarse de inmediato, en especial si no hay náusea, vómito o
dolor abdominal. El inicio de la alimentación con una dieta 17. ¿Cuándo se realiza la CPRE?
baja en grasa parece ser seguro, como la dieta de liquidas cla­ La CPRE se deberá efectuar dentro de las primeras 48 horas
ros. En las formas graves se recomienda la nutrición entera! a partir del aumento de las enzimas pancreáticas y del cuadro

CUADRO 11-21-4.
PRONÓSTICO TOMOGRÁFICO DE BALTHAZAR

HALLAZGO PUNTUACIÓN NECROSIS MORBILIDAD MORTALIDAD


Páncreas normal o Ausente(O) 8% 3%
Inflamación focal o difusa 30%(2) 8% 3%
Inflamación del páncreas y grasa peripancreática 2 30a 50%(4) 8% 3%
Grado C + 1colección peripancreática 3 >50%(6) 92% 17%
Grado C + 2o más colección peripancreática(liquido o gas ) 4 >50%(6) 92% 17%

780
SECCIÓN 11 GASTROENTEROLOGÍA @
de pancreatitis aguda para delimitar el cuadro clinico y evitar dolor abdominal, fiebre y leucocitosis. Como tratamiento re­
complicaciones debidas a la obstrucción y el edema del con­ quiere drenaje quirúrgico y antibioticoterapia adyuvante.
ducto pancreático por la presencia de microlitos. En paciente
estable con necrosis infectada, debe diferirse de preferencia BIBLIOGRAFÍA
más de cuatro semanas hasta la licuefacción del contenido y el Bakker OJ et al. Treatment options for acute pancreatitis. Nat Rev Gastroen­
desarrollo de una pared fibrosa alrededor de la necrosis. En terol Hepatol. 2014 Mar 25. doi: 10.1038/nrgastro.2014.39.
pacientes sintomáticos con necrosis infectada se recomienda Centro Nacional de Excelencia Tecnológica en Salud. Diagnóstico y referen­
la utilización de métodos minimamente invasivos de necrosec­ cia oportuna de la pancreatitis aguda en el primer nivel de atención. [Mo­
nografía en internet.] México; 2011. CENETEC. Disponible en: http://
tomia para drenar el material. www.cenetec.salud.gob.mx/descargas/gpc/CatalogoMaestro/011 _GPC_
PancreatAguda/SSA_011_08_EyR.pdf
Coronel E, Czul F, Gelrud A. Endoscopic management of the complications
COMPLICACIONES LOCALES of pancreatitis. Rev Gastroenterol Peru. 2013 Jul-Sep;33(3):237-45.
Cucher D, Kulvatunyou N, Green DJ, Jie T, Ong ES. Gallstone pancreatitis:
18. ¿Cuáles son las complicaciones de una PA? a review. Surg Clin North Am. 2014 Apr;94(2):257-80. doi: 10.1016/j.
suc.2014.01.006. Epub 2014 Feb 20.
Dentro del curso clínico de la pancreatitis aguda pueden sur­
Martín RF, Hein AR. Operative management of acute pancreatitis. Surg Clin
gir complicaciones como la necrosis pancreática, que por lo North Am. 2013 Jun;93(3):595-610. doi: 10.1016/j. suc.2013.02.007. Epub
general aparece durante la primera o segunda semana de evo­ 2013 Mar 29.
lución y se manifiesta con dolor abdominal, fiebre persistente Mylarappa P, Javali T, Prathvi, Ramesh D. Acute pancreatitis and develop­
y masa palpable, además de leucocitosis y de hiperamilasemia ment of pancreatic pseudo cyst after extra corporeal shock wave litho­
tripsy to a left renal calculus: A rare case with review of literature. Indian
prolongada. J Uro!. 2014 Jan;30( 1):110-112. doi: 10.4103/09701591.124219.
Un pseudoquiste es una acumulación de líquido y tejido ne­ Otsuki M et al. Criteria for the diagnosis and severity stratification of acute
erótico originado por la rotura del conducto pancreático; se pancreatitis. World J Gastroenterol. 2013 Sep 21;19(35):57985805. doi:
manifiesta por persistencia o reaparición del dolor abdominal e 10.3748/wjg.v 19.i35.5798.
hiperamilasemia hacia la tercera semana de evolución; 85% de Petrov M. Nutrition, inflamrnation, and acute pancreatitis. ISRN lnflamm.
2013 Dec 29;2013:341-410. doi: 10.1155/2013/341410. E Collection 2013.
los casos se resuelve de forma espontánea a las ocho semanas. Schepers NJ, Besselink MG, van Santvoort HC, Bakker OJ, Bruno MJ. Early
El absceso pancreático es una acumulación de pus mal de­ management ofacute pancreatitis. Best Pract Res Clin Gastroenterol. 2013
limitada que aparece hacia la segunda o tercera semana con Oct;27(5):727-743. doi: 10.1016/j.bpg.2013.08.007. Epub 2013 Sep 6.

CAPÍTULO 11.22
PANCREATITIS CRÓNICA
1
Rosa María Miranda Cordero • Brenda Patricia Rendón Martínez

CLÍNICA fermedad hepática por alcohol se presenta por lo menos en la


mitad de los pacientes con daño pancreático.
1. La presentación clínica de la pancreatitis crónica
se caracteriza por: DIAGNÓSTICO
Cincuenta por ciento de los pacientes se presenta con episo­
dios recurrentes de pancreatitis aguda; sin embargo, el dolor 3. ¿Qué datos hacen sospechar pancreatitis cróni­
también puede ser insidioso e intermitente en 35% de los ca?
afectados. Alrededor de 15% de los pacientes presenta diabe­ Factores de riesgo: alcohol y tabaco, dolor abdominal, pancrea­
tes, malabsorción e ictericia. titis recurrente.

2. ¿Qué hallazgos se encuentran en la pancreatitis 4. ¿Qué datos clínicos deben estudiarse en los cua­
crónica secundaria a consumo crónico de alcohol? dros de dolor por pancreatitis crónica?
En la necropsia los cambios morfológicos de la pancreatitis Los niveles de amilasa y lipasa pueden ser normales incluso
crónica se observan hasta en 45% de los pacientes con abuso en un cuadro agudo. La diabetes se presenta hasta en 30% de
de alcohol asintomáticos. El etilismo puede contribuir hasta los pacientes; sin embargo, la cetoacidosis o la nefropatía son
en 75% de los casos de pancreatitis crónica, sin importar el raras. Las calcificaciones pancreáticas se pueden observar has­
tipo de alcohol ingerido, pero sí la cantidad; asimismo, la en- ta en 30 a 60% de los pacientes afectados. De manera inicial

781
e MANUAL PARA EL EXAMEN NACIONAL DE RESIDENCIAS MÉDICAS

FIGURA 11·22·1. ALGORITMO DIAGNÓSTICO DE LA PANCREATITIS CRÓNICA.

Dolor abdominal
Esteatorrea
Pancreatitis recurrente

Historia clínica
Exploración física
Síntomas: pérdida de peso, -
1-- ---<
diarrea, diabetes, ictericia
Exámenes: función
hepática, Cr, glucosa

1
Elastasa fecal, tripsina
1
Tomografía de abdomen
1
Colangiorresonancia
1
Ultrasonido endoscópico
sérica • páncreas 2x, c. principal • Conducto principal anormal Anormalidades en el
Estimulación con dilatado 2-4 obstrucción, defectos de llenado, parénquima pancreático
secretina/colecistocinina Dilatación y estenosis del dilatación e irregularidad -hiperecoico

1
conducto pancreático -contorno lobular
conducto >3 mm en la cabeza,
>2 mm cuerpo,
CPRE >1 mm cola litos
• obstrucción y ramificaciones
• conducto principal anormal
• defectos de llenado
Recomendada para fines
terapéuticos.

debe realizarse un estudio de imagen para valorar el parénqui­ glándulas salivales y por un incremento en la absorción a nivel
ma y el conducto pancreático: TC/resonancia magnética nu­ de la mucosa, provocado por disminución en la destrucción
clear/colangiorresonancia. La CPRE debe reservarse para fi. proteolítica por las hidrolasas del borde en cepillo y la conver­
nes terapéuticos debido a que es un procedimiento invasivo y sión de carbohidratos no digeridos en ácidos grasos de cadena
en la actualidad se cuenta con otros métodos diagnósticos. La corta. Comparado con otros pacientes con malabsorción, el
figura 11-22-1 muestra un algoritmo diagnóstico. volumen fecal es menor debido a que la capacidad de absorber
líquidos no se encuentra tan comprometida.
COMPLICACIONES
TRATAMIENTO
5. La insuficiencia pancreática se asocia con malab­
sorción cuando la secreción pancreática es menor a: 7. ¿Qué debe incluir el manejo nutricional de los pa­
La malabsorción no ocurre hasta que la secreción es menor a cientes con pancreatitis crónica?
10% de lo normal. La malabsorción de grasas se presenta an­ El manejo nutricional requiere control del dolor para permitir
tes que la de proteínas o carbohidratos. la ingesta de alimentos. La administración de enzimas pan­
creáticas reduce los síntomas asociados con malabsorción y
6. ¿Cuáles son otras manifestaciones acompañantes mejora el estado nutricional; a pesar de ello, no se elimina la
de la pancreatitis crónica? esteatorrea y debe restringirse la ingesta de grasa a 50 g/día. El
A pesar de la esteatorrea y la deficiencia de proteasas, la defi­ consumo de ácidos grasos de cadena mediana no requiere en­
ciencia clínica de vitamina B 12 y otras vitaminas liposolubles zimas pancreáticas para su absorción, incrementando la in­
es rara. Una reducción en la secreción de amilasa pancreática gesta calórica; mientras que los ácidos grasos de cadena corta
reduce la absorción de carbohidratos; sin embargo, no es clíni­ (butirato, propionato) no tienen ninguna ventaja nutricional.
camente tan importante como para causar diarrea osmótica, La analgesia puede requerir incluso narcóticos. La inactiva­
debido a que la digestión se lleva a cabo por la amilasa de las ción de las enzimas por el ácido y la pepsina puede prevenirse

782
SECCIÓN 11 GASTROENTEROLOGÍA @
CUADRO 11-22-1.
CAUSAS DE PANCREATITIS CRÓNICA. SISTEMA DE CLASIFICACIÓN TIGAR-O
TÓXICO·METABÓLICAS
Alcohol
MHi-WMUW
Inicio temprano Hereditaria
GENÉTICAS AUTOINMUNITARIAS
Autoinmunitaria aislada
RECURRENTES
Secundaria a
OBSTRUCTIVAS
Páncreas divisum
necrosis
Tabaco Inicio tardío Tripsinógeno catiónico Asociado con síndrome Vascular-isquémica Disfunción del
de Sji:igren, colangitis esfínter de Oddi
esclerosante primaria (controversia!)
yDM1

Hipercalcemia Tropical Mutaciones CFTR Secundaria a Tumores


Regulador de conductancia radiación
transmembrana de la
fibrosis quística

Hiperlipidemia Mutaciones SPINK1 Quistes duodenales


lnhibidor Kasal tipo 1 de la preampulares
serin proteasa

con el uso concomitante de inhibidores de la bomba de pro­ FIGURA 11-22-2. TAC DE CONDUCTO PANCREÁTICO.
tones.

8. ¿Cuál es el mejor abordaje quirúrgico de la pan­


creatitis crónica?
El procedimiento de Puestow (pancreaticoyeyunostomía lon­
gitudinal) se realiza en pacientes con dolor intenso e intratable
que presentan un conducto pancreático irregular con esteno­
sis y dilataciones. En ocasiones se puede realizar una pancrea­
tectomia por persistencia del dolor; sin embargo, no es efec­

1
tiva en todos los casos, incluso en los pacientes que ya han
tenido resecciones parciales. La colangiopancreatografía retró­
grada endoscópica (CPRE) está indicada en los pacientes con
ascitis pleural o derrame pleural pancreático en quienes se sos­
peche fístula y puedan ser candidatos a tratamiento endoscó­
pico. Por otro lado, la cistoenterostomía es el método quirúr­
gico más común para drenar los pseudoquistes. Véase figura
11-22-2.

9. ¿Cuáles son las principales causas de pancreati­


tis crónica?
Véase cuadro 11-22-1.

BIBLIOGRAFÍA
Longo DL, Fauci AS. Harrison's Gastroenterology and Hepatology. rt• ed.
Nueva York: McGraw-Hill Medicar; 2013. pp. 490-7.
Conwell DL, Wu BU. Chronic Pancreatitis: Making the Diagnosis. Clin Gas­
troenter and Hepatology. 2012; JO: 1088-95.
Sleisenger, Fordtran. Gastrointestinal and Liver Disease. 6th ed. Philadelphia:
W.B Saunders Company; 1999.
Talukdar R, Saikia N. Chronic Pancreatitis: evolving paradigms. Pancreatolo­
gy. 2006:6(5):440-9.

783
e MANUAL PARA EL EXAMEN NACIONAL DE RESIDENCIAS MÉDICAS

-�
RESPUESTAS A CASOS CLÍNICOS CASO CLÍNICO 2
A. La paciente cursa con disfagia. En su abordaje requiere
determinar si es a sólidos o líquidos, lo cual sugiere altera­
CAPÍTULO 11.1 ción en la motilidad o inervación esofágica. La progresión
ESTRUCTURA DEL ESÓFAGO, SÍNTOMAS lenta descarta un proceso neoplásico. El primer estudio a
Y ANOMALÍAS realizar de manera precoz es la videofluoroscopia; si no está
disponible, realizar un trago de bario o serie esofagogastro­
CASO CLÍNICO 1 duodenal (SEGD). La videofluoroscopia permite valorar la
E. Dentro de las anomalías congénitas del esófago se en­ función oral y faringea en casos de disfagia alta, así como
cuentran la estenosis esofágica y la atresia esofágica (AE), detectar y analizar la alteración funcional de la deglución.
con o sin fístula traqueoesofágica (FTE). Es una malforma­ Proporciona evidencias de los cuatro datos clave de la dis­
ción congénita frecuente que afecta a cerca de uno de cada función orofaringea: imposibilidad o retraso en el inicio de
3 500 recién nacidos y consiste en la interrupción de la con­ la deglución faríngea, aspiración de alimento al pulmón, re­
tinuidad del esófago, que puede tener o no comunicación gurgitación nasofaríngea y presencia de residuo del material
con la tráquea. En más de 50% de los casos se asocia con ingerido en la cavidad faríngea tras la deglución. La serie
otras malformaciones congénitas, como cardiacas (29%), esofagogastroduodenal (SEGD) es más sensible que la en­
genitourinarias (14%), anorrectales (14%) y gastrointestina­ doscopia en la detección de pequeñas estenosis esofágicas,
les (13%). Sin embargo, se utiliza el acrónimo VACTERpor como aquellas producidas por anillos, y estenosis pépticas
las asociaciones de malformaciones: V = vertebrales, A = de más de 10 mm de diámetro. Además estos estudios per­
anales, TE = fístula TE, R == radiales y/o renales. También miten identificar alteraciones en la motilidad esofágica y
está asociada con enfermedades y/o síndromes genéticos co­ son en particular útiles en el diagnóstico de acalasia y es­
mo trisomía 21 y trisomía 18; no obstante, la mayoría de los pasmo esofágico difuso, que en estadios precoces pueden
casos de AE y de FTE son esporádicos. Dentro de los an­ ser difíciles de identificar por endoscopia. También identifi­
tecedentes principales se encuentran polihidramnios y pre­ can lesiones que pueden crear confusión para el endosco­
maturidad. pista, como un divertículo de Zenker o las hernias para­
El diagnóstico se establece al momento del nacimiento esofágicas.
por la imposibilidad de pasar un catéter o sonda para aspirar El uso de contraste baritado en Rx de urgencias está con­
el estómago. Ante la sospecha de malformación esofágica traindicado, ya que dificulta la realización de endoscopia
nunca debe iniciarse la vía oral, ya que el alimento provoca­ posterior en caso de que sea necesaria; de igual manera, an­
ría una neumonía por broncoaspiración (típica imagen ra­ te sospecha de perforación debe emplearse contraste hidro­
diológica de colapso y/o condensación en segmento poste­ soluble.
rior del LSD o en segmento 6 de LID). Se intenta pasar una En un paciente con datos de progresión rápida o datos de
sonda al estómago a través de un orificio nasal y, si la son­ alarma es preciso descartar un proceso neoplásico y pue­
da se detiene y no pasa, debe practicarse de urgencia una de realizarse endoscopia de primera intención.
radiografía de tórax con sonda radioopaca. En caso de atre­ Otras pruebas: la manometría esofágica es la técnica de
sia la sonda se detiene en el cabo o extremo superior y se elección para el estudio de los desórdenes de 1a motilidad
enrolla sobre él. Algunas veces, muy raras, está justificado esofágica y es en especial útil para establecer el diagnóstico
añadir una pequeña cantidad de contraste a la sonda para de acalasia y espasmo esofágico difuso, así como para de­
descartar una fistula del cabo superior. La existencia de aire tectar anomalías motoras esofágicas asociadas con enfer­
infradiafragmático demostrará la existencia de una fístula medades colágeno-vasculares. También puede estudiarse el
en el cabo inferior (puede originar distensión abdomínal más tránsito esofágico por escintigrafia (el paciente ingiere agua
marcada si se requirió insuflación con presión positiva co­ marcada con tecnecio-99).
mo medida de reanimación y, si es muy marcada, la gastros­
tomía puede aliviar dicha situación, aunque en ocasiones ha­ CAPÍTULO 11.2
ce que el soporte ventilatorio escape por esta vía, lo que hará TRASTORNOS MOTORES DEL ESÓFAGO
preciso sellar la fístula). En caso de no existir aire intestinal
se trata de una atresia tipo A sin fístula traqueoesofágica, CASO CLÍNICO 3
con depresión abdominal. Además se deben efectuar estu­ C. La manifestación clinica más frecuente en los trastornos
dios radiográficos y de ultrasonido para descartar otras mal­ motores del esófago es la disfagia xifoidea de presentación
formaciones. El tratamíento definitivo es quirúrgico con cie­ baja (retroesternal) y de larga evolución. El tránsito esofági­
rre de fístula y anastomosis término-termínal. En caso de una co con medio de contraste puede mostrar alteraciones espe­
separación mayor de los cabos esofágicos, el paciente puede cificas, pero la confirmación diagnóstica es mediante mano­
requerir transposición colónica. metría.

784
SECCIÓN 11 GASTROENTEROLOGÍA @
CASO CLÍNICO 4 CASO CLÍNICO 9
A. La manometría esofágica muestra una presión normal C. La endoscopia es el estudio de elección para evaluar el
o elevada de reposo. Tras la deglución, la relajación del esfín­ grado de intensidad y la extensión de las lesiones; de acuer­
ter esofágico inferior es incompleta ( <80%) o prácticamente do con ello se elaborará el pronóstico; sin embargo, antes
inexistente. En el cuerpo esofágico la presión basal se en­ de someter a la paciente a este procedimiento se debe deter­
cuentra anormalmente elevada y es igual o superior a la gás­ minar si no existe perforación esofágica o neumomediasti­
trica, con contracciones no peristálticas por lo general de no, así como el grado de inflamación y afección transmural
baja amplitud. del esófago, lo cual se evalúa de forma apropiada con una
radiografía PA de tórax y una tomografía de tórax simple y
CASO CLÍNICO 5 contrastada.
D. La endoscopia alta es el estudio de elección para la eva­
luación de la ERGE. Debe realizarse para descartar esofagi­ CASO CLÍNICO 10
tis, estenosis o malignidad; sin embargo, sólo entre 20 y 30% A. En un paciente con ingestión de cáusticos debe determi­
de los pacientes presentará esofagitis o enfermedad erosi­ narse la naturaleza de la ingestión, ya que la lesión por álca­
va. La especificidad de la endoscopia alta es de 97%. La se­ lis es diferente a la de los ácidos. La lesión por ácidos tiende
rie esofagogastroduodenal y el esofagograma tienen una a autolimitarse, ya que el daño es por coagulación y necro­
sensibilidad y especificidad de 26 y 37%, respectivamente, y sis y la formación de cicatriz limita el daño tisular. Por su
en la actualidad no se consideran pruebas diagnósticas de parte, los álcalis causan licuefacción y necrosis por saponi­
ERGE. ficación, que afectan la totalidad de la pared esofágica, pro­
ducen mayor lesión y penetración y el proceso de cicatri­
CASO CLÍNICO 6 zación produce estenosis esofágica en más de 80% de los
C. La cirugía antirreflujo está indicada ante el fracaso del casos.
tratamiento médico, con el uso de dosis altas de inhibidores También debe valorarse el tiempo de exposición y el
de bomba de protones, en pacientes jóvenes de entre 25 y motivo de ingestión, ya que si es accidental conlleva me­
40 años de edad. El procedimiento de elección, cuando no nor exposición que el intencional, que por lo general es sui­
existen alteraciones en la motilidad del cuerpo esofágico y cida.
del esfínter esofágico inferior, es la funduplicatura tipo Nis­
CAPÍTULO 11.3
sen. El monitoreo del pH intraesofágico, o pHmetría de 24

1
OTROS TRASTORNOS ESOFÁGICOS
horas, ayuda a determinar la presencia de reflujo patológico
y si es ácido; en la actualidad se sabe que el reflujo gastro­
CASO CLÍNICO 11
esofágíco puede ser ácido y no ácido, por lo que ha surgi­
l. B. La videofluoroscopia con bario es la prueba ideal para
do la pHmetría con impedancia, que ayuda a determinar
el diagnóstico de la disfagia alta. La proyección esofagogás­
mejor la naturaleza del reflujo. La pHmetría requiere la co­
trica lateral con videofluoroscopia ayuda a detectar defectos
locación de un catéter transnasal con uno o más sensores de
de llenado en el tercio proximal esofágico, para lo cual es
antimonio, que registra el pH y sus variaciones por encima
indispensable la administración del medio de contraste por
del EEI durante 24 horas.
vía oral.
2. C. El divertículo de Zenker es un divertículo adqui­
CASO CLÍNICO 7 rido que surge cuando la presión del bolo alimenticio au­
D. La esofagitis por virus herpes simple también es co­ menta de forma anormal durante la deglución y causa una
mún en pacientes con VIH/sida y CD4 menores o iguales a protrusión de la mucosa por una zona de debilidad anató­
150. La esofagitis por Candida fue adecuadamente tratada mica de la faringe, llamada triángulo de Killian. El triángulo
y hubo mejoría clínica. La esofagitis por CMV se caracte­ de Killian se sitúa donde las fibras transversas del esfínter
riza por eritema difuso de la mucosa esofágica y tanto por cricofaríngeo se cruzan con las fibras oblicuas del músculo
CMV como por CAi se presenta en pacientes con CD4 me­ constrictor de la faringe. Se considera que el divertículo eso­
nores a 50. fágico es más frecuente. Aparece entre la séptima y la octa­
va décadas de la vida, con una proporción hombre-mujer de
CASO CLÍNICO 8 2:1.
B. La esofagitis por Candida presenta síntomas recientes de
odinofagia y disfagia, de forma descendente. Al ser un co­ CASO CLÍNICO 12
mensal de mucosa oral, afecta sobre todo a los pacientes D. Los divertículos del cuerpo esofágico se localizan so­
inmunosuprimidos, tanto VIH positivos como con inmuno­ bre todo en su tercio medio inferior. Los divertículos por
supresión inducida por fármacos. tracción son más frecuentes en países en vías de desarrollo;

785
e MANUAL PARA EL EXAMEN NACIONAL DE RESIDENCIAS MÉDICAS

suelen relacionarse con la inflamación mediastínica asocia­ cual se transmite al esófago contra una glotis cerrada. Este
da con la tuberculosis (como en el paciente) o la histoplas­ aumento de presión suele ser producto de náusea y vómito,
mosis. Las linfadenopatías mediastínicas por neoplasias pero también puede ocurrir durante la defecación, el parto,
malignas de los pulmones también pueden ocasionar diver­ convulsiones, levantamíento de pesas o estatus asmático. La
tículos por tracción. causa principal descrita corresponde al vómito, presente en
más de 75% de los pacientes. Por lo general la perforación
CASO CLÍNICO 13 es en el esófago intratorácico, en el tercio inferior, 3 a 5 cm
A. Las hernias del hiato deslizantes (tipo 1) suceden cuan­ por arriba de la unión gastroesofágica y, en 90% de los casos
do la unión esofagogástrica y parte del estómago se despla­ adultos, en el lado izquierdo. Las causas de esto son: en esta
zan por arriba del diafragma (al mediastino). Se ignora la área la capa muscular es más delgada y las fibras están dis­
causa, pero su frecuencia aumenta con la edad. Se cree que tribuidas de forma longitudínal; la angulación anterior del
la hernia se origina por debilidad de la membrana frenoeso­ esófago al cruzar el pilar izquierdo; y la carencia de estruc­
fágica. La herniación fisiológica ocurre durante la peristal­ turas que den soporte. La longitud de la perforación varía
sis, la distensión esofágica y la relajación fisiológica del es­ de O. 75 a 7.5 cm. En 80% de los casos hay comunicación
fínter esofágico ínferior. Puede asociarse con la atrofia y la con el espacio pleural, por lo que los contenidos del estóma­
laxitud del ligamento frenoesofágico. La deglución, la disten­ go y el esófago pasan hacia el tórax por la presión negativa,
sión esofágica y la ínstrumentación esofágica se asocian con con inflamación, secuestro de líquidos y estado de choque
acortamiento esofágico y disociación de la unión esofagogás­ del paciente con sepsis.
trica y hiato en aproximadamente 2 centímetros. Un complejo sintomático muy característico es la tríada
de Mackler (vómitos, dolor torácico, enfisema subcutáneo)
que, sin embargo, aparece en muy pocos casos. El dolor to­
CASO CLÍNICO 14
rácico es muy intenso, se localiza en la región subesternal o
D. Los desgarros de Mallory-Weiss son laceraciones en
en el hemitórax izquierdo y puede irradiar a cuello, hom­
la región gastroesofágica que por lo general afectan a la mu­
bros o espalda; por lo general se precisan dosis elevadas de
cosa gástrica, aunque en 10 a 20% de los casos afectan a la
analgésicos para su control. En algunos casos puede estar
mucosa esofágica. La incidencia es aproximadamente cua­
ausente. En ocasiones, si la perforación es muy distal, pue­
tro casos por cada 100 000 personas, tiende a afectar más a
de presentarse como un abdomen agudo. Suele aparecer dis­
hombres que mujeres y pueden aparecer a cualquiera edad.
nea, por participación pulmonar o pleural, fiebre relaciona­
Consisten en desgarros no penetrantes, lineales, únicos o
da con la aparición de infección mediastínica y hematemesis,
múltiples y de menos de 2 cm de longitud de la mucosa eso­
por lo general de escasa cuantía.
fágica o gástrica en la proximidad de la unión esofagogás­
trica. Se producen por las arcadas forzadas y el vómito re­
petido. Se caracterizan por hematemesis, que pueden ser CASO CLÍNICO 16
desde muy leves hasta masivas después de violentos inten­ D. El hematoma íntramural esofágico es una patología poco
tos para vomitar. En otras ocasiones pueden presentarse frecuente, caracterizada por un compromiso de la capa sub­
como melena. mucosa del esófago. La mayoría de los casos reportados
compromete el tercio distal del esófago, tal vez por la au­
CASO CLÍNICO 15 sencia de musculatura estriada y el menor sustento del órga­
l. C. Ante la sospecha de perforación del tracto gastrointes­ no. Afecta con mayor frecuencia a mujeres de edad media y
tinal, la radiografía de tórax y abdomen brinda la mayor avanzada. El cuadro clínico se caracteriza por dolor retro­
cantidad de información: enfisema subcutáneo, ensancha­ esternal de inicio súbito, odinofagia y/o disfagia y puede
miento mediastínal, niveles hidroaéreos en el mediastino, acompañarse de hematemesis de aparición tardía. Esta tría­
derrame pleural, neumomediastino, hidroneumotórax y gas da sólo está presente en 35% de los pacientes. El dolor sú­
libre subdiafragmático. Más adelante se determína el sitio bito retroesternal es el síntoma más frecuente. Por ser una
de perforación y se realiza esofagograma con material hi­ patología poco común, el diagnóstico se retrasa y sólo se
drosoluble, ya que el bario se contraíndica. realiza una vez que otras condiciones graves han sido des­
2. E. La rotura esofágica espontánea o síndrome de Boer­ cartadas (ínfarto agudo del miocardio, disección aórtica o
haave es una condición muy poco frecuente, caracterizada perforación esofágica).
por una mortalidad elevada. Representa una forma de baro­
trauma al esófago con un gran potencial de complicaciones CASO CLÍNICO 17
que hacen necesario el reconocimiento temprano y un abor­ D. La esofagitis eosinofílica es una infiltración del esófago
daje oportuno. La rotura esofágica espontánea ocurre cuan­ por diez o más eosinófilos por campo y se manifiesta con
do existe aumento súbito de la presión intraabdominal, la síntomas como disfagia, impactaciones de alimento y piro-

786
SECCIÓN 11 GASTROENTEROLOGÍA @
sis. También puede asociarse con dermatitis atópica. La en­ CAPÍTULO 11.6
doscopia reporta anillos, sobre todo en el tercio medio y ENFERMEDAD ULCEROSA PÉPTICA
distal. El anillo de Schatzki se presenta en el esófago distal
y sólo en pocos casos causa disfagia o impactación. El sín­ CASO CLÍNICO 22
drome de Plummer-Vínson se acompaña de anemia y disfa­ B. El paciente con sangrado digestivo alto, evidenciado por
gia, y las membranas esofágicas son parte de dicho síndrome. pérdidas hemáticas macroscópicas (melena, hematemesis o
emesis en posos de café) o la sospecha del mismo por pre­
CASO CLÍNICO 18 sencia de sangre oculta en heces requiere una endoscopia
D. La edad en la cual es más frecuente la presentación es en digestiva alta como estudio de primera elección, ya que
la ínfancia, entre los seis meses y los seis años de edad. La aporta más datos tanto al diagnóstico como al pronóstico,
causa es accidental. En adultos los grupos vulnerables son incluso brinda posibilidad terapéutica. Los estudios radioló­
los pacientes con trastornos psiquiátricos o que busquen al­ gicos son obsoletos en la actualidad. La sangre oculta en
guna ganancia. heces se vuelve innecesaria, ya que hay evidencia de pérdi­
das macroscópicas.
CAPÍTULO 11.4
TUMORES ESOFÁGICOS CASO CLÍNICO 23
C. La edad, la comorbilidad con diabetes + ingesta de
CASO CLÍNICO 19 AINE y los datos de choque hipovolémico a su llegada dan
l. B. Ante un caso de disfagia el método de elección es el un riesgo alto de morbimortalidad por úlcera péptica aso­
esofagograma; sin embargo, los datos de alarma y la rápida ciada con AlNE. Es importante señalar que esta pacien­
progresión de los síntomas sugieren la presencia de neopla­ te, por escalas de puntaje alto tanto de Blatchford como de
sia, por lo cual debe realizarse endoscopia como primera Rockall, requiere endoscopia urgente, incluso de manera
elección. El ultrasonido endoscópico permite visualizar le­ ideal dentro de las primeras doce horas de establecida la sos­
siones de la pared esofágica, pero se realiza posterior a la pecha diagnóstica.
endoscopia. La tomografía es útil para determinar las eta­
pas de las lesiones neoplásicas. CAPÍTULO 11.7
2. C. La ingesta de cáusticos es un factor de riesgo para TUMORES GÁSTRICOS
cáncer de células escamosas y no para adenocarcinoma, CASO CLÍNICO 24

1
donde los factores de riesgo más importantes son el reflujo D. Pólipo gástrico antral. Los tumores benignos de estóma­
gastroesofágico y el esófago de Barrett. El leiomioma es el go tienen su origen en el tejido epiteHal, mesenquimático y
tumor esofágico benigno más frecuente, pero por lo general neural. Las lesiones más comunes benignas en el estómago
es asintomático. La disfagia, aunque se presenta en 50% de son pólipos de origen epitelial gástrico y constituyen 75% de
los casos, se asocia con el tamaño del tumor y no es rápida­ todos los tumores benignos de estómago. Los pólipos hiper­
mente progresiva. El lipoma es otro tumor benigno pero plásicos son los más frecuentes en el estómago con 70% de
poco frecuente y suele ser asintomático. los casos.
Los pólipos pueden encontrarse en cualquier parte del
CAPÍTULO 11.5 estómago, a menudo son únicos, pequeños, sésiles o pedicu­
GASTRITIS: AGUDA EROSIVA Y CRÓNICA lados. El adenocarcinoma gástrico y el tumor tipo MALT
son lesiones malignas que se acompañan de síntomas como
CASO CLÍNICO 20
pérdida de peso, anemia y sangrado digestivo. La gastritis
C. La pérdida de peso podría considerarse un dato de alar­ antral foveolar es un diagnóstico puramente histológico; y el
ma en esta paciente; sin embargo, no siempre hay una corre­ lipoma gástrico es una lesión benigna de tipo submucosa,
lación con los hallazgos endoscópicos. La baja ponderal no poco frecuente y localizada en antro.
justifica repetir endoscopia con biopsias, ya que sólo pre­
senta gastritis aguda asociada con Helicobacter pylori. CASO CLÍNICO 25
D. El adenocarcinoma gástrico es el tumor maligno más
CASO CLÍNICO 21 frecuente. El diagnóstico es histológico; la presentación tu­
C. La tríada de vómito, náusea y epigastralgia indica gastri­ moral puede ser en forma vegetante, polipoide, ulcerada o
tis. La asociación con una anemia macrocítica debe causar difusa y las características de la úlcera son la friabilidad y los
sospecha obligada de anemia de tipo autoinmunitario, ya bordes mal delimitados e irregulares. Los factores de riesgo
sea con anticuerpos anticélulas parietales o antifactor in­ son tabaco, alcohol, dieta rica en nitritos, infección por H
trínseco, lo que explica el déficit de cobalamina y, por ende, pylori, etcétera. Los síntomas de sospecha es un paciente ma­
los síntomas de anemia y glositis asociados. yor con historia de sangrado digestivo, anemia y pérdida de

I 787
e MANUAL PARA EL EXAMEN NACIONAL DE RESIDENCIAS MÉDICAS

peso, asociados con factores de riesgo. El estudio endoscó­ vo de aspirado duodenal es negativo, por lo que se descarta
pico es esencial para su diagnóstico y toma de biopsias. Por sobrecrecimiento bacteriano.
lo general las úlceras gástricas benignas son menores de 20
mm y con bordes bien delimitados, asociadas con consumo CAPÍTULO 11.10
crónico de AINE, infección por H. pylori, tabaco y alcohol. ENFERMEDAD INFLAMATORIA INTESTINAL

CAPÍTULO 11.8 CASO CLÍNICO 30


SÍNDROME DIARREICO 1. C. La enfermedad inflamatoria intestinal comprende dos
entidades: la enfermedad de Crohn y la colitis ulcerativa cró­
CASO CLÍNICO 26 nica inespecífica (CUCI). Ambas son de etiología multi­
C. Existen múltiples causas y mecanismos que producen factorial, con una respuesta inmunológica inadecuada a fac­
diarrea aguda. El paciente presenta fiebre, leucocitosis, pujo tores ambientales en pacientes genéticamente predispuestos,
y tenesmo rectal. Todos estos son datos de toxicidad sisté­ quienes cursan con cuadros de recaídas. Un tercio de los pa­
mica asociados con bacterias que con frecuencia producen cientes tiene manifestaciones extraintestinales. La colitis
inflamación de la mucosa colónica e intestinal, como Esche­ ulcerosa (UC) es una afección inflamatoria crónica que oca­
richia co!i enteroinvasiva, Shigella, Salmonella, Yersinia, en­ siona inflamación continua superficial de la mucosa colóni­
tre otras. ca (mucosa y submucosa) sin granulomas en la biopsia, que
afecta al recto y a una extensión variable del colon en con­
CASO CLÍNICO 27 tinuidad; en 10% de los casos afecta íleon terminal y se de­
B. El caso ilustra un paciente con diarrea crónica y síndro­ nomina ileítis por reflujo, que se caracteriza por una trayec­
me de absorción intestinal deficiente, quien tiene además toria de recidivas y remisiones. La CUCI se manifiesta ante
anemia megaloblástica y disminución de ácido fólico. Si bien todo con evacuaciones diarreicas con sangre, con o sin mo­
no se conoce cuál es la causa del esprúe tropical, el hecho co, tenesmo y en estadios graves con rectorragia o hema­
de haber viajado a un área subtropical es el principal fac­ toquecia, dolor, pérdida de peso y fiebre.
tor de riesgo en este caso. La Ell, según las guías ECCO (Organización Europea
CAPÍTULO 11.9 de Crohn y Colitis), contempla dos entidades. Ell no clasi­
MALABSORCIÓN ficada es el término más conveniente para la minoría de los
casos donde no puede hacerse una distinción definitiva en­
CASO CLÍNICO 28 tre UC, enfermedad de Crohn u otra causa de colitis después
D. La esteatorrea es la manifestación clínica pivote para es­ de que se han tomado en cuenta antecedentes, apariencias
tablecer el diagnóstico de malabsorción, pero no su origen. endoscópicas, histopatología de biopsias múltiples de la mu­
La D-xilosa es una pentosa que se absorbe por difusión pasi­ cosa y radiología adecuada. Colitis indeterminada es un tér­
va que no requiere de la hidrólisis intraluminal (digestión), mino reservado para que los patólogos describan una mues­
pero sí de la integridad de la mucosa, por lo cual se utiliza tra de colectomía que tiene características coincidentes de
para diferenciar entre una alteración de la pared o de la luz colitis ulcerosa y enfermedad de Crohn.
intestinal. La absorción de vitamina B 12 se lleva a cabo en 2. C. El abordaje diagnóstico ante un paciente con sospe­
el íleon terminal, por lo que la anemia no se debe a su mal­ cha de Ell es una exploración física completa que incluye
absorción por atrofia de las vellosidades duodenales. La de­ exploración anorrectal, así como hemograma completo, re­
terminación de niveles séricos de carotenos es una prueba actantes de fase aguda como PCR, VSG y electrolitos séri­
de bajo costo, accesible y práctica para el escrutinio de mal­ cos, creatinina, enzimas hepáticas, coprológico, coproculti­
absorción, aunque no es el estándar de oro. vo, coproparasitoscópico y toxina de C!ostridium diffici/e. Los
estudios baritados son útiles cuando no hay sospecha de
CASO CLÍNICO 29 gravedad y en la exploración no hay datos de hemorragia.
C. En el caso de esta paciente se tienen datos clínicos y En el caso de CUCI pueden encontrarse ulceraciones su­
radiológicos de malabsorción. La paciente presenta der­ perficiales o úlceras más profundas, denominadas "en botón
matitis herpetiforme, que es la manifestación cutánea más de camisa"; en estadios crónicos, imagen tubular de colon
común de la enfermedad celiaca. La paciente cuenta con an­ por pérdida haustral. Sin embargo, la hemorragia se consi­
ticuerpos antitransglutaminasa positivos que, junto con los dera un dato de alarma, por lo que debe realizarse colonos­
hallazgos de la biopsia de duodeno, tienen una sensibilidad copia con ileoscopia para toma de biopsias y además es útil
y especificidad para el diagnóstico de enfermedad celiaca para determinar la gravedad y la extensión de la enfermedad
cercanas a 95%. La paciente no presenta datos clínicos de (clasificación de Montreal, cuadro l l-10-3). Si se sospecha
enfermedad de Whipple ni datos imagenológicos, histoló­ megacolon tóxico no debe realizarse y en este caso sí está
gicos o séricos que sugieran enfermedad de Crohn. El culti- indicada la tomografía.

788
SECCIÓN 11 GASTROENTEROLOGÍA @
3. C. Dado que el paciente cumple con criterios clínicos, do de CUCI) que afecta desde la boca hasta el ano, con
bioquímicos y endoscópicos de gravedad, requiere manejo cursos de remisión y recaída. La afección intestinal es en
hospitalario y la decisión terapéutica depende no sólo de la parches, pero afecta todas las capas de la mucosa; es decir,
extensión sino de la disponibilidad de medicamentos y de es transmural y puede presentar granulomas en las biopsias.
factores pronósticos de gravedad y colectomía para determi­ Las manifestaciones son dolor abdominal que puede pre­
nar el tratamiento oportuno. Antes de iniciar el tratamiento ceder a los síntomas digestivos, diarrea que puede ser explo­
deben descartarse procesos infecciosos agregados, tanto in­ siva y acompañarse con moco y sangre, pero a menudo no
testinales como sistémicos, como tuberculosis latente, hepa­ es evidente y se manifiesta por anemia, así como pérdida de
titis B y C e infecciones oportunistas como citomegalovirus peso, fiebre y debilidad.
o Epstein-Barr, ya que el tratamiento tiene como objetivo dis­ 2. B. Un paciente con cuadro de diarrea abundante, ex­
minuir la respuesta inflamatoria mediante fármacos inmuno­ plosiva y con datos de malabsorción orienta a etiología de
moduladores o inmunosupresores. intestino delgado y el estudio de tránsito intestinal es útil
El tratamiento de la colitis ulcerativa depende de la ac­ porque puede demostrar datos de malabsorción, como frag­
tividad y la extensión de la enfermedad. Sin embargo, el mentación y floculación del medio de contraste, pero el en­
abordaje convencional incluye al inicio mesalazina tópica o grosamiento de las paredes es característico de la enferme­
sistémica y/o esteroides; si no hay respuesta, agregar inmu­ dad de Crohn o presencia de estenosis (signo de la cuerda)
nomoduladores, como los análogos de las purinas (azatio­ o fístulas. Hay datos de pérdida de peso, así como anemia
prina y 6-mercaptopurina) y ciclosporina; si no hay respuesta por pérdidas hemáticas ocultas, por lo que está indicado
se debe considerar terapia biológica y como último recurso realizar colonoscopia con ileoscopia dado el sitio de mayor
la cirugía, ya que no es curativa, al tratarse de una patología frecuencia. La toma de biopsias orienta el diagnóstico con
sistémica y porque tiene altos indices de complicaciones pos­ la presencia de granulomas epitelioides; sin embargo, éstos
quirúrgicas y recidiva. No obstante, debe considerarse una sólo se presentan en 30 a 40% de las biopsias.
proctocolectomía ante la ausencia de respuesta. Cuando se sospecha afección de intestino delgado, y pos­
4. D. El tratamiento de la EH comprende la inducción a terior a estudios endoscópicos y colonoscópicos negativos,
la remisión y el mantenimiento de la misma, entendiéndose puede indicarse la cápsula endoscópica, pero previo estu­
por remisión la completa resolución de los síntomas más dio baritado que descarte estenosis por el riesgo de reten­
cicatrización endoscópica de la mucosa. Los esteroides son ción de la cápsula; no obstante, tiene la limitación de no
eficaces para inducir la remisión; sin embargo, el tiempo de realizar biopsias, por lo que el estudio en estos casos es la

1
administración se relaciona con eventos adversos como sín­ enteroscopia que permita la visualización y toma de biop­
drome de Cushing, síndrome metabólico, osteoporosis y, en sias.
casos de retíro súbito, insuficiencia suprarrenal aguda, entre La TC permite determinar las zonas de estenosis (signo
otros; por ello se indican sólo para inducción a la remisión de la diana), así como la extensión de la enfermedad. Si no
y no para el mantenimiento. Los análogos de las purinas, es abordable por endoscopia, la resonancia magnética tam­
por su inicio lento de acción, se indican de manera conjunta bién valora el proceso inflamatorio (signo del peine) y la pre­
con esteroides, por lo que se utilizan para mantenimiento sencia de estenosis.
de la remisión con una cicatrización lenta de la mucosa; sin El ultrasonido puede utilizarse ante la sospecha de ma­
embargo, incrementan el riesgo de infecciones, pancreatitis sas abdominales.
y linfoma hepatoesplénico. El metotrexato no tiene efecto en 3. A. Los abscesos requieren drenaje quirúrgico y antibió­
la CUCI. La terapia biológica (anti-TNF) ha demostrado ser ticos; los más utilizados son ciprofloxacino y metronidazol,
útil para inducir y mantener la remisión al obtener la cicatri­ así como aplicación de setones de drenaje también conoci­
zación rápida de la mucosa, mejorando la calidad de vida y dos como sedales.
disminuyendo hospitalizaciones y cirugías, con un adecua­ 4. C. La cirugía en pacientes con EC debe evitarse, ya que
do perfil de seguridad, pero requiere descartar tuberculosis se ha demostrado que tiene altas tasas de morbimortalidad
latente e infecciones oportunistas y se contraíndica en hiper­ y recurrencia. En los pacientes con actividad moderada a
sensibilidad al anti-TNF, sepsis o insuficiencia cardiaca gra­ grave está indicada la terapia biológica; en el abordaje con­
dos III-IV. vencional se inicia con esteroides y, si no hay respuesta, se
inician inmunomoduladores, como los análogos de las pu­
CASO CLÍNICO 31 rinas (azatioprina y 6-mercaptopurina); sin embargo tienen
l. A. El cuadro clínico tiene datos de malabsorción intesti­ un inicio lento de acción (dos meses). Puede utilizarse me­
nal y, aunque las enfermedades enunciadas lo ocasionan, la totrexato y, si no hay respuesta, se inicia la terapia biológica,
enfermedad de Crohn pertenece al grupo de las enfermeda­ que en el medio sólo consiste en anticuerpos monoclonales
des inflamatorias intestinales, caracterizada por un proce­ contra el factor de necrosis tumoral (antiTNF), como el ada­
so crónico, de origen multifactorial (mencionado en aparta- limumab. En pacientes con factores de mal pronóstico en la

789
e MANUAL PARA EL EXAMEN NACIONAL DE RESIDENCIAS MÉDICAS

actualidad se considera iniciar con terapia biológica, ya que ciente cumpliría con las caracteristicas y desencadenantes.
ha demostrado que induce y mantiene la remisión, cicatriza Si fuese tóxica o viral deberia tener aumento en transamina­
la mucosa, mejora la calidad de vida, disminuye las hospita­ sas; la hemólisis de causa autoinmunitaria se descarta en
lizaciones y tiene menor riesgo de recurrencia. este caso, pues tiene un nivel de Hb completamente normal
y ningún antecedente de autoinmunidad. La hepatitis viral
CAPÍTULO 11.11 tiene que dar daño hepatocelular y la hiperbilirrubinemia
SÍNDROME DEL INTESTINO IRRITABLE sería más equilibrada entre directa o indirecta o incluso más
directa llegando hasta 3 mg/dL en algunos casos.
CASO CLÍNICO 32
2. A. La uridinadifosfato glucuronil transferasa (UGT)
D. Esta paciente presenta síntomas caracteristicos del sín­
es la enzima que se encarga de la conjugación de la bili­
drome de intestino irritable: dolor o molestia abdominal
rrubina y su producción se regula por la activación de la re­
con cambios en la mecánica intestinal. No hay datos de alar­
gión promotora del gen que la produce; la mutación de este
ma que sugieran una causa orgánica. Las parasitosis intes­
gen provoca una menor producción de esta enzima. La UGT
tinales crónicas se asocian con pérdida de peso y síntomas
se nota reducida en el síndrome de Gilbert a 30% del valor
constitucionales. El estreñimiento no se presenta en casos
normal, según análisis de muestras obtenidas por biopsia he­
de intolerancia a carbohidratos ni en colitis microscópicas.
pática. Esta enfermedad es hereditaria; se han sugerido pa­
CASO CLÍNICO 33
trones de herencia tanto autosómica dominante como auto­
C. En este caso los síntomas se presentan después de los 50 sómica recesiva.
años de edad, por lo que aunque cumple criterios para el CAPÍTULO 11.13
diagnóstico de síndrome de intestino irritable, se justifica la HEPATITIS VÍRICAS
realización de estudios diagnósticos por la presencia de da­
tos de alarma. CASO CLÍNICO 36
E. El paciente presenta cuadro de hepatitis aguda; no se re­
CAPÍTULO 11.12
fieren factores de riesgo para hepatitis de transmisión paren­
ESTUDIO DEL PACIENTE CON ENFERMEDAD
teral y quedan las opciones hepatitis A y E. Sobre ellas, la
HEPATOBILIAR
edad de presentación más frecuente de la hepatitis A es en
CASO CLÍNICO 34 la infancia y la hepatitis E en adolescentes y adultos, pero lo
l. B. La prednisona no provoca hemólisis, lo que descarta A. más significativo en el cuadro para el diagnóstico diferencial
Aunque tiene autoinmunidad, el daño hepatocelular provoca­ es la presencia de urticaria.
ria elevación de transaminasas de manera obligada y además
CASO CLÍNICO 37
es raro el daño de este tipo por lupus; son otras asociacio­
l. C. La principal forma de adquirir la hepatitis C es por
nes autoinmunitarias. La fragmentación mecánica tampoco
una transfusión sanguínea. La hepatitis B ocurre ante to­
es opción; es probable que el soplo cardiaco sea funcional
do por transmisión vertical o por contacto directo, y la A y
por la anemia aguda grave. La hemólisis da DHL muy eleva­
E son por transmisión fecal-oral.
da y la hiperbilirrubinemia con predominio es indirecta.
2. D. Con los datos que se tienen no se puede conocer la
2. B. De manera caracteristica en hemólisis autoinmuni­
etiología viral, por lo que es necesario practicar estudios pa­
taria la elevación de la DHL puede ser de hasta diez veces.
ra hacer el diagnóstico diferencial. Dado que es un hallazgo,
3. D. Al conjugarse, la bilirrubina se hace hidrosoluble y
sin elevación de las aminotransferasas más de cinco veces el
puede aparecer en orina; la bilirrubina no conjugada o indi­
valor normal ni ictericia, se descartan las hepatitis con pre­
recta es liposoluble, no aparece en orina y provoca el tinte
sentación aguda (A y E); el antígeno de núcleo contra el vi­
ictérico en los tejidos cuando está muy elevada. Al glucoro­
rus de hepatitis B sólo se encuentra en el tejido, por lo que
nizarse en el hígado la bilirrubina se vuelve directa o conju­
no es detectable en sangre. De inicio se descarta entre hepa­
gada, la bilirrubina indirecta se transporta unida a albúmina
titis B y C con anticuerpos contra hepatitis C y antígeno de
desde la sangre hasta el hígado.
superficie para el virus de hepatitis B.
CASO CLÍNICO 35 CAPÍTULO 11.14
l . C. El caso es caracteristico de síndrome de Gilbert que es FÁRMACOS E HÍGADO
una falla en la glucuronización. padecimiento benigno y cró­
nico en el que hay una ictericia leve sin otra alteración en CASO CLÍNICO 38
las pruebas hepáticas y que se agudiza en los portadores l. D. El más probable es el paracetamol, el cual produce
tras eventos estresantes como ayuno, deprivación de sueño, una hepatitis aguda. Un factor de riesgo para su presentación
infecciones o consumo de paracetamol, por lo que este pa- es el abuso de alcohol y el ayuno. El halotano produce hepa-

790
SECCIÓN 11 GASTROENTEROLOGÍA @
titis crónica y es más común en mujeres. La isoniazida es además de astenia, adinamia y síntomas generales, a los cua­
más común en personas mayores de 60 años de edad, al­ les muchas de las veces no se les da importancia, incluso sin
cohólicos y con uso de drogas concomitantes, como parace­ presentar otros síntomas agregados y con PFH y tiempo de
tamol. El metotrexato produce fibrosis hepática en pacien­ protrombina completamente normales.
tes con diabetes mellitus, falla renal y psoriasis. 2. C. El frotis de sangre periférica y el aspirado de médu­
2. D. El paracetamol se metaboliza en el hígado. La vía la ósea se solicitan para descartar causas hematológicas. Los
metabólica da un producto hepatotóxico. Según la dosis pue­ marcadores virales, así como los autoanticuerpos, son útiles
de presentarse necrosis hepática, en especial en pacientes cuando hay antecedentes de riesgo para contagio o cuando
alcohólicos. Los exámenes paraclínicos son pruebas de fun­ ya se identifica como cirrosis o hepatopatía crónica y se quie­
cionamiento hepático y pruebas de coagulación. re buscar la etiología. La endoscopia superior es un estudio
que se solicita para descartar várices esofagogástricas, que
CASO CLÍNICO 39 pueden ser una complicación de la cirrosis hepática, pero
A. El halotano es un medicamento conocido por causar ic­ no todos los pacientes con cirrosis tienen várices al momen­
tericia postoperatoria; se estima que su incidencia es de 2% to del diagnóstico ni todo los pacientes que tienen várices
en los pacientes que son expuestos al halotano. tienen cirrosis hepática. Por tanto, cuando se quiere descar­
tar cirrosis hepática, después de los laboratorios y con ultra­
CAPÍTULO 11.15 sonido de abdomen sin hallazgos específicos, el estudio más
HEPATITIS CRÓNICAS indicado es el ECO Doppler hepatoesplénico.
CASO CLÍNICO 40
3. C. La paciente presenta obesidad importante, así co­
l. A. El cuadro sugiere por clinica, bioquímica y serología la mo diabetes mellitus descontrolada y con síndrome meta­
presencia de hepatitis autoinmunitaria y se reconoce a la he­ bólico, así que lo más probable es que se trate de esteato­
patitis de interfase como el hallazgo histológico característi­ hepatitis no alcohólica, lo cual se tiene que corroborar con
co (aunque no patognomónico) de esta entidad. estudios de laboratorio.
2. C. El caso tiene indicaciones absolutas para iniciar
CASO CLÍNICO 43
terapia con esteroides. El tratamiento deberá ser finito de
acuerdo con remisión de la enfermedad, falla o intolerancia l. B. El etilismo crónico aún es la causa más frecuente de
al tratamiento. cirrosis en el medio. A pesar de que la cirrosis por alcohol

1
también produce esteatohepatitis en fases iniciales, el ante-
CASO CLÍNICO 41 cedente de alta ingesta de alcohol excluye que la causa sea
l. C. La prednisona en monoterapia es el manejo de elec­ esteatohepatitis no alcohólica. La hepatitis C no se puede
ción en las HA de alto riesgo, con valoración de respuesta a descartar, incluso se tiene que investigar por el anteceden-
tratamiento a los siete días con el índice de Lille. No se ha te de ser chofer, sobre todo porque entre choferes de carga
demostrado superioridad en eficacia de la terapia combina­ pesada es común el uso de drogas, por lo que aumenta el
da de esteroides con pentoxifilina. El trasplante hepático se riesgo de infecciones por VHC y VHB.
reserva para casos seleccionados de pacientes que no respon­ 2. E. En este caso son francos los datos de cirrosis hepá­
de al tratamiento inicial. tica, incluso con datos de dos complicaciones como la ascitis
2. C. El uso de pentoxifilina se reserva para los casos en y el sangrado de tubo digestivo manifestado por melena, por
los que existan contraindicaciones a los esteroides, como lo que el paciente requiere una complementación diagnósti­
pueden ser sepsis, hemorragia digestiva, fracturas patológi­ ca rápida para el adecuado control de las complicaciones.
cas o cuando el paciente presenta lesión renal asociada. 3. B. Corresponde a un Child B de 8 puntos, ya que albú­
mina, bilirrubinas y ascitis le dan dos puntos cada una, más
CAPÍTULO 11.16 uno de encefalopatía y uno más de TP suman un total de 8
CIRROSIS puntos. Esto puede mejorar con el tratamiento adecuado,
pero también puede progresar a Child C en caso de que no
CASO CLÍNICO 42 se controlen las complicaciones propias de la cirrosis.
l. E. La trombocitopenia, por sí sola, sí es un dato de cirro­
sis hepática. El valor normal de plaquetas es de 150 000 a CAPÍTULO 11.17
350 000 plaquetas, por lo que un valor fuera de este rango COMPLICACIONES DE LA CIRROSIS
debe ser estudiado. Como parte del estudio de pacientes con
trombocitopenia se deben descartar causas hematológicas, CASO CLÍNICO 44
como es el caso de la púrpura trombocitopénica idiopática C. El antecedente de alcoholismo y los estudios de labora­
(PTI), así como hepatopatías crónicas, ya que muchos pa­ torio orientan a sospechar que el paciente tiene cirrosis, ya
cientes inician su cuadro de cirrosis con trombocitopenia, con complicaciones como hipertensión portal y, por tanto,

791
e MANUAL PARA EL EXAMEN NACIONAL DE RESIDENCIAS MÉDICAS

várices esofágicas y gastropatía hipertensiva portal. Esta úl­ La prevalencia de Ell en CEP es de 60 a 80% y en la varie­
tima se caracteriza por hemorragia crónica o manifiesta. Si dad colitis ulcerosa es de 48 a 86%. Por el contrario, la CEP
bien la ectasia vascular antral puede manifestarse con ane­ se diagnostica entre 2.4 y 7.5% de los pacientes con colitis
mia ferropénica, es menos frecuente. ulcerosa.
2. D. En un paciente asintomático, con colangiografía
CASO CLÍNICO 45 normal pero con datos bioquímicos y serológicos sugestivos
D. En todos los pacientes con cirrosis es necesario detectar de CEP, se requiere realizar una biopsia hepática para esta­
la presencia de várices esofágicas en el momento del diag­ blecer CEP de pequeños conductos.
nóstico inicial, aunque varios estudios han señalado que las
pruebas no invasivas, como el recuento plaquetario y los CAPÍTULO 11.19
datos obtenidos mediante ecografía abdominal (hiperten­ ENFERMEDADES HEPÁTICAS DE CAUSAS
sión portal determinada mediante el diámetro de la porta), METABÓLICA Y CARDIACA
proporcionan una precisión satisfactoria (casi 80%). Ningu­
na prueba tiene una precisión suficiente como para omitir CASO CLÍNICO 50
con seguridad la endoscopia en pacientes con indicadores D. El diagnóstico es hemocromatosis primaria, en estudio
negativos. Aunque el riesgo de hemorragia por várices se por elevación de enzimas hepáticas con evidencia bioquími­
asocia de forma significativa con la clasificación de Child­ ca de alteración de transferrina 50%, ferritina l 000 ng/ mL,
Pugh, su sensibilidad es menor a 70%. con perfil viral para hepatitis B y C negativos, así como in­
munológicos negativos. En este caso está indicada la biop­
CAPÍTULO 11.18 sia hepática para determinar indice de hierro hepático, que
COLESTASIS CRÓNICAS se considera positivo por arriba de 1.9. En la clínica el pa­
CASO CLÍNICO 46
ciente cursa con hiperpigmentación de tegumentos y he­
patoesplenomegalia, que no son patognomónicas, pero con
C. Las enfermedades colestásicas autoinmunitarias más fre­
los hallazgos bioquímicos indican como primer diagnóstico
cuentes son cirrosis biliar primaria (CBP), 90% se presenta
hemocromatosis. Los pacientes con esteatohepatitis alcohó­
en mujeres; la colangitis esclerosante primaria (CEP), que
lica y no alcohólica no presentan niveles elevados de trans­
se presenta de forma predominante en hombres en 70%, y
ferrina > l 000 ng/mL. Los pacientes con virus de hepatitis
los síndromes overlap (sobreposición de enfermedades au­
B y C pueden mostrar alteración de la saturación de transfe­
toinmunitarias).
rrina, pero no niveles altos de ferritina como en este caso.
CASO CLÍNICO 47 La enfermedad de Wilson se caracteriza por alteraciones de
D. En esta paciente con marcadores de daño hepatocelu­ cobre urinario y ceruloplasmina.
lar y colestasis intensa aparentemente intrahepática a expen­
sas de directa se cumplen varios criterios que apuntan a CASO CLÍNICO 51
CBP como respuesta más correcta. Las asociaciones con D. El diagnóstico en este caso es esteatohepatitis no alco­
anticuerpos y anormalidades de estudios bioquímicos se hólica, pues la paciente no tiene antecedente de ingesta etí­
mencionan a detalle en la pregunta l. lica. Los factores de riesgo para esteatohepatitis es su IMC
35 kg/m2, que la ubica como obesa, así como una HOMA
CASO CLÍNICO 48 de 3.5 (normal 2.5), compatible con resistencia a la insulina
E. Ante un paciente que presenta datos de ictericia, fiebre y cifras de tensión arterial de 130/90 mm Hg, niveles eleva­
y colestasis, con un US que descarta proceso obstructivo de dos de colesterol y bajos de HDL, lo que la ubica en síndro­
las vías biliares, serología viral negativa, sin antecedentes me metabólico. El ultrasonido es compatible con hígado gra­
de hepatotoxicidad o cirugías previas a nivel hepatobiliar, so. La biopsia confirma el diagnóstico con fibrosis F2.
se requiere a la brevedad una colangiografía para evaluar el
estatus de la vía biliar intra y extrahepática. De primera elec­ CAPÍTULO 11.20
ción serán los métodos no invasivos, como la colangiorreso­ ABSCESOS HEPÁTICOS
nancia y, en caso de no contar con ella, se podrá realizar
una CPRE (colangiopancreatografía retrógrada endoscópi­ CASO CLÍNICO 52
ca), la cual puede ser un procedimiento diagnóstico y tera­ l. C. Es un paciente masculino menor de 50 años de edad
péutico. que tiene la tríada de Charcot característica de absceso
hepático. Una hepatitis viral se descarta, ya que las trans­
CASO CLÍNICO 49 arninasas tienen elevación mínima. Los datos clínicos no
l. E. Hay una fuerte asociación de la enfermedad inflamato­ corresponden a una pancreatitis ni a una colecistitis litiá­
ria intestinal en su variedad de colitis ulcerosa con la CEP. sica.

792
SECCIÓN 11 GASTROENTEROLOGÍA @
2. B. Si bien la tomografía es el estudio de elección por quiere un antibiótico con espectro más amplio y penetración
tener mayor sensibilidad, en el medio el estudio de menor hística. La repetición de la TC sólo serviría para retrasar el
costo y más accesible es el ultrasonido. diagnóstico y el tratamiento adecuado.
3. C. Por lo general el absceso hepático amebiano es úni­ 2. C. Aunque la amilasa se utiliza de forma invariable
co y se localiza en lóbulo derecho. El tratamiento de elec­ como apoyo para diagnóstico de pancreatitis aguda, no es
ción son los nitroimidazoles; sin embargo, cuando los abs­ específica por completo. Pueden observarse elevaciones en
cesos son localizados en lóbulo hepático izquierdo y son pacientes con insuficiencia renal, patología tuboovárica y
mayores de 5 cm debe realizarse drenaje por el riesgo eleva­ afectación de las glándulas salivales, entre una diversidad de
do de rotura. alteraciones intraabdominales. La lipasa es más especifica
que la amilasa para la pancreatitis aguda y tiende a perma­
CAPÍTULO 11.21 necer elevada más tiempo. En la pancreatitis aguda, la ami­
PANCREATITIS AGUDA Jasa puede ser normal, en particular en pacientes con pan­
creatitis de lípidos y la inducida por alcohol.
CASO CLÍNICO 53
l. C. Este paciente tiene un síndrome clínico y pruebas de CAPÍTULO 11.22
laboratorio compatibles con pancreatitis. Sin embargo, las PANCREATITIS CRÓNICA
características clinicas y las anormalidades de laboratorio
no son específicas de pancreatitis aguda y es necesario con­ CASO CLÍNICO 55
siderar diagnósticos alternativos. La ligera elevación de la 1. C. La pancreatitis crónica es el resultado de una serie de
amilasa y lipasa también puede aparecer en caso de úlcera eventos de necrosis, inflamación crónica y fibrosis que con
penetrante, isquemia u obstrucción gastrointestinal. Sería el tiempo provocan insuficiencia exocrina, endocrina e in­
prematuro instaurar tratamiento por pancreatitis aguda an­ flamación neural. El alcohol constituye una de las principa­
tes de establecer el diagnóstico. Una TC tiene la máxima les causas. La clasificación más aceptada según la causa se
posibilidad de confirmar el diagnóstico de pancreatitis agu­ describe en el sistema TIGAR-O (Tóxicas-metabólicas, Idio­
da, ayuda a definir su gravedad y puede detectar otros pro­ páticas, Genéticas, Autoinmunitarias, RecurrentesObstruc­
cesos que justifiquen los hallazgos clinicos y de laboratorio. tivas); véase cuadro 11-22-1.
2. C. La herencia es un factor que tiene mayor correla­ 2. A. Definitivamente el consumo de AINE o narcóticos
ción con episodios crónicos de pancreatitis. La litiasis biliar no provoca estos síntomas, por lo que C y D se descartan. La

1
es la principal causa de la afección aguda y el resto de las aparición de diabetes, diarrea y esteatorrea no implica que
respuestas son factores bien conocidos de la misma. el paciente ha desarrollado TFD y síndrome metabólico; son
sólo la expresión de la insuficiencia pancreática que ahora
CASO CLÍNICO 54 presenta. La diabetes por la deficiencia en la secreción de
l. B. Es probable que este paciente tenga necrosis infectada insulina (insuficiencia endócrina) y la falta de enzimas di-
y, aunque la presencia de gas intrapancreático es muy suges­ gestivas pancreáticas provoca diarrea crónica, esteatorrea y
tiva de infección, para confirmar el diagnóstico se requiere malabsorción, por lo que hay que sustituirlas farmacológica-
una aspiración con aguja fina de las áreas necróticas para mente. También deben considerarse como complicaciones a
cultivo y tinción de Gram. El octreótido no sería útil en este largo plazo el incremento en el riesgo de cáncer de páncreas,
contexto (pancreatitis crónica). Los antibióticos intraveno­ colangitis y cirrosis biliar. Después de 20 años de diagnós-
sos son componente importantes en el tratamiento; sin em­ tico el riesgo acumulativo para cáncer es de 4% y es diez
bargo, la elección de cefalosporina es inadecuada. Se re- veces mayor en los pacientes con pancreatitis hereditaria.

793

También podría gustarte